GR 10 Math STD - Different Schools-Vol 2

You might also like

Download as pdf or txt
Download as pdf or txt
You are on page 1of 517

Page 1 Sample Paper 01 CBSE Mathematics Class 10

Sample Paper 01
Class- X Exam - 2023-24
Mathematics - Standard

Time Allowed: 3 Hours Maximum Marks : 80


General Instructions :
1. This Question Paper has 5 Sections A-E.
2. Section A has 20 MCQs carrying 1 mark each
3. Section B has 5 questions carrying 02 marks each.
4. Section C has 6 questions carrying 03 marks each.
5. Section D has 4 questions carrying 05 marks each.
6. Section E has 3 case based integrated units of assessment (04 marks each) with sub-parts.
7. All Questions are compulsory. However, an internal choice in 2 Qs of 5 marks, 2 Qs of 3 marks and 2 Questions
of 2 marks has been provided.
8. Draw neat figures wherever required. Take π = 227 wherever required if not stated.

Section - A
Section A consists of 20 questions of 1 mark each.

1. Two APs have the same common difference. The first term of one of these is - 1 and that of the other is - 8. Then
the difference between their 4th terms is
(a) - 1 (b) - 8
(c) 7 (d) - 9

2. For the following distribution:

Marks Number of students


Below 10 3
Below 20 12
Below 30 27
Below 40 57
Below 50 75
Below 60 80
The modal class is
(a) 10-20 (b) 20-30
(c) 30-40 (d) 50-60

3. If one zero of the polynomial (3x2 + 8x + k) is the reciprocal of the other, then value of k is
(a) 3 (b) - 3
(c) 1 (d) - 1
3 3

Install NODIA App to See the Solutions.


Click Here To Install
Page 2 Sample Paper 01 CBSE Mathematics Class 10

4. ^x2 + 1h2 − x2 = 0 has


(a) four real roots (b) two real roots
(c) no real roots (d) one real root

5. The LCM of smallest two digit composite number and smallest composite number is
(a) 12 (b) 4
(c) 20 (d) 44

6. Which of the following statement is false?


(a) All isosceles triangles are similar. (b) All quadrilateral are similar.
(c) All circles are similar. (d) None of the above

7. If a regular hexagon is inscribed in a circle of radius r, then its perimeter is


(a) 3r (b) 6r
(c) 9r (d) 12r

8. If 4 tan θ = 3 , then c 4 sin q − cos q m is equal to


4 sin q + cos q
(a) 2 (b) 1
3 3

(c) 1 (d) 3
2 4

9. The top of two poles of height 20 m and 14 m are connected by a wire. If the wire makes an angle of 30c with the
horizontal, then the length of the wire is
(a) 12 m (b) 10 m
(c) 8m (d) 6 m

10. A ladder, leaning against a wall, makes an angle of 60c with the horizontal. If the foot of the ladder is 2.5 m away
from the wall, find the length of the ladder.
(a) 5 m (b) 5.5 m
(c) 2.5 m (d) 4.5 m

11. In the figure, PQ is parallel to MN. If KP = 4 and KN = 20.4 cm then find KQ.
PM 13

(a) 4.8 cm (b) 4.6 cm


(c) 4.4 cm (d) 4.2 cm

Install NODIA App to See the Solutions.


Click Here To Install
Page 3 Sample Paper 01 CBSE Mathematics Class 10

12. Ratio of lateral surface areas of two cylinders with equal height is
(a) 1 : 2
(b) H :h
(c) R :r
(d) None of these

13. In the formula x = a +


/ f i d i , for finding the mean of grouped data d ’s are deviation from a of
/ fi i

(a) lower limits of the classes


(b) upper limits of the classes
(c) mid-points of the classes
(d) frequencies of the class marks

14. If the zeroes of the quadratic polynomial ax2 + bx + c, where c ! 0 , are equal, then
(a) c and a have opposite signs
(b) c and b have opposite signs
(c) c and a have same sign
(d) c and b have the same sign

15. Two coins are tossed simultaneously. The probability of getting at most one head is
(a) 1 (b) 1
4 2

(c) 2 (d) 3
3 4

16. Someone is asked to take a number from 1 to 100. The probability that it is a prime, is
8 1
(a) 25 (b) 4

3 13
(c) 4 (d) 50

17. For which value(s ) of p , will the lines represented by the following pair of linear equations be parallel
3x - y - 5 = 0
6x - 2y - p = 0
(a) all real values except 10 (b) 10
(c) 5/2 (d) 1/2

18. The coordinates of a point A on y -axis, at a distance of 4 units from x -axis and below it are
(a) (4, 0) (b) (0, 4)
(c) (- 4, 0 ) (d) (0, - 4 )

Install NODIA App to See the Solutions.


Click Here To Install
Page 4 Sample Paper 01 CBSE Mathematics Class 10

In the question number 19 and 20, a statement of Assertion (A) is followed by a statement of Reason (R). Choose the
correction option.

19. Assertion : If a wire of length 22 cm is bent in the shape of a circle, then area of the circle so formed is 40 cm2 .
Reason : Circumference of the circle = length of the wire.
(a) Both assertion (A) and reason (R) are true and reason (R) is the correct explanation of assertion (A).
(b) Both assertion (A) and reason (R) are true but reason (R) is not the correct explanation of assertion (A).
(c) Assertion (A) is true but reason (R) is false.
(d) Assertion (A) is false but reason (R) is true.

20. Assertion : The slant height of the frustum of a cone is 5 cm and the difference between the radii of its two circular
ends is 4 cm. Then the height of the frustum is 3 cm.
Reason : Slant height of the frustum of the cone is given by l = ^R − r h2 + h2 .
(a) Both assertion (A) and reason (R) are true and reason (R) is the correct explanation of assertion (A).
(b) Both assertion (A) and reason (R) are true but reason (R) is not the correct explanation of assertion (A).
(c) Assertion (A) is true but reason (R) is false.
(d) Assertion (A) is false but reason (R) is true.

Section - B
Section B consists of 5 questions of 2 marks each.

21. In the figure, PQRS is a trapezium in which PQ || RS. On PQ and RS, there are points E and F respectively
such that EF intersects SQ at G . Prove that EQ # GS = GQ # FS.

22. In figure, O is the centre of the circle and LN is a diameter. If PQ is a tangent to the circle at K and +KLN = 30º
, find +PKL.

Install NODIA App to See the Solutions.


Click Here To Install
Page 5 Sample Paper 01 CBSE Mathematics Class 10

23. Evaluate : 3 tan2 30º + tan2 60º + cosec 30º − tan 45º
cot2 45º

24. One card is drawn from a well shuffled deck of 52 cards. Find the probability of getting :
(i) a non face card,
(ii) a black king.
 O
In a family of two children find the probability of having at least one girl.

25. In an equilateral triangle of side 24 cm, find the length of the altitude.
 O
In the given figure, PQR is a triangle right angled at Q and XY || QR. If PQ = 6 cm, PY = 4 cm and PX: XQ = 1: 2.
Calculate the length of PR and QR.

Section - C
Section C consists of 6 questions of 3 marks each.

26. Verify whether 2, 3 and 1


2 are the zeroes of the polynomial p ^x h = 2x3 − 11x2 + 17x − 6 .

27. Solve for x and y :


x + 1 + y − 1 = 9 ; x − 1 + y + 1 = 8.
2 3 3 2

28. In the given figure, if +ACB = +CDA , AC = 6 cm and AD = 3 cm, then find the length of AB .

Continue on next page....

Install NODIA App to See the Solutions.


Click Here To Install
Page 6 Sample Paper 01 CBSE Mathematics Class 10

 O
In the given figure, BC || PQ and BC = 8 cm, PQ = 4 cm, BA = 6.5 cm AP = 2.8 cm Find CA and AQ.

29. Prove that tan2 A + cosec2 A = 1


tan A − 1 sec A − cosec2 A 1 − 2 cos2 A
2 2

30. A toy is in the form of a cone radius 3.5 cm mounted on a hemisphere of same radius. If the total height of the
toy is 15.5 cm, find the total surface area of the toy. Use π = 227
 O
A well of diameter 4 m dug 21 m deep. The earth taken out of it has been spread evenly all around it in the shape
of a circular ring of width 3 m to form an embankment. Find the height of the embankment.

31. Given that 2 is irrational, prove that (5 + 3 2 ) is an irrational number.

Section - D
Section D consists of 4 questions of 5 marks each.

32. Solve for x : 1 + 2 = 4 x !- 1, - 2, - 4


x+1 x+2 x+4
 O
Solve for x : 4x + 4bx − ^a − b h = 0
2 2 2

33. A right triangle ABC , right angled at A is circumscribing a circle. If AB = 6 cm and BC = 10 cm, find the
radius r of the circle.

34. Daily wages of 110 workers, obtained in a survey, are tabulated below :

Daily Wages (in<) 100-120 120-140 140-160 160-180 180-200 200-220 220-240

Number of Workers 10 15 20 22 18 12 13

Compute the mean daily wages an modal daily wages of these workers.
 O
The mean of the following distribution is 18. Find the frequency of the class 19-21.

Class 11-13 13-15 15-17 17-19 19-21 21-23 23-25


Frequency 3 6 9 13 f 5 4

Install NODIA App to See the Solutions.


Click Here To Install
Page 7 Sample Paper 01 CBSE Mathematics Class 10

35. Two friends Seema and Aditya work in the same office at Delhi. In the Christmas vacations, both decided to go
to their hometown represented by Town A and Town B respectively in the figure given below. Town A and Town
B are connected by trains from the same station C (in the given figure) in Delhi. Based on the given situation
answer the following questions:

(i) Who will travel more distance, Seema or Aditya, to reach to their hometown?
(ii) Seema and Aditya planned to meet at a location D situated at a point D represented by the mid-point
of the line joining the points represented by Town A and Town B . Find the coordinates of the point
represented by the point D .

Section - E
Case study based questions are compulsory.

36. Cubic Coating : Frozen specimens are stored in a cubic metal box that is x inches on each side. The box is
surrounded by a 2 inch thick layer of foam insulation.
(i) Find a polynomial function V (x) that gives the total volume in cubic inches for the box and insulation.
(ii) Find the total volume if x is 10 inches.
(iii) Use the remainder theorem to find the total volume when x is 10 inches.

37. Wilt Chamberlain : Wilton Norman “Wilt” Chamberlain was an American basketball player, and played in the
NBA during the 1960s. At 7 feet 1 inch, he was the tallest and heaviest player in the league for most of his career,
and he was one of the most famous people in the game for many years. He is the first and only basketball player
to score 100 points in an NBA game.

Continue on next page....

Install NODIA App to See the Solutions.


Click Here To Install
Page 8 Sample Paper 01 CBSE Mathematics Class 10

In the 1961–1962 NBA basketball season, Wilt Chamberlain of the Philadelphia Warriors made 30 baskets. Some
of the baskets were free throws (worth 1 point each) and some were field goals (worth 2 points each). The number
of field goals was 10 more than the number of free throws.
(i) How many field goals did he make ?
(ii) How many free throws did he make?
(iii) What was the total number of points scored?
(iv) If Wilt Chamberlain played 5 games during this season, what was the average number of points per game?

38. Underground water tank is popular in India. It is usually used for large water tank storage and can be built
cheaply using cement-like materials. Underground water tanks are typically chosen by people who want to save
space. The water in the underground tank is not affected by extreme weather conditions. The underground
tanks maintain cool temperatures in both winter and summer. Electric pump is used to move water from the
underground tank to overhead tank.

Ramesh has build recently his house and installed a underground tank and overhead tank. Dimensions of tanks
are as follows :
Underground Tank : Base 2 m # 2 m and Height 1.1 m.
Overhead tank : Radius 50 cm and Height 175 cm
(i) What is the capacity of the underground tank ?
(ii) What is the ratio of the capacity of the underground tank to the capacity of the overhead tank?
(iii) If curved part of overhead tank need to be painted to save it from corrosion, how much area need to be
painted? If water is filled in the overhead tank at the rate of 11 litre per minute, the tank will be completely
filled in how much time?
 O
(iv) If the amount of water in the underground tank, at an instant, is 2400 litres, find then the water level in the
underground tank at that instant.

 ******

Install NODIA App to See the Solutions.


Click Here To Install
Page 1 Sample Paper 02 CBSE Mathematics Class 10

Sample Paper 02
Class- X Exam - 2023-24
Mathematics - Standard

Time Allowed: 3 Hours Maximum Marks : 80


General Instructions :
1. This Question Paper has 5 Sections A-E.
2. Section A has 20 MCQs carrying 1 mark each
3. Section B has 5 questions carrying 02 marks each.
4. Section C has 6 questions carrying 03 marks each.
5. Section D has 4 questions carrying 05 marks each.
6. Section E has 3 case based integrated units of assessment (04 marks each) with sub-parts.
7. All Questions are compulsory. However, an internal choice in 2 Qs of 5 marks, 2 Qs of 3 marks and 2 Questions
of 2 marks has been provided.
8. Draw neat figures wherever required. Take π = 227 wherever required if not stated.

Section - A
Section A consists of 20 questions of 1 mark each.

1. A letter of English alphabet is chosen at random, what is the probability that the letter so chosen is a consonant?
(a) 265 21
(b) 26 (c) 132 (d) 137

2. What is the HCF of smallest primer number and the smallest composite number?
(a) 2 (b) 4 (c) 6 (d) 8

3. If A ^5, 2h , B ^2, - 2h and C ^- 2, t h are the vertices of a right angled triangle with +B = 90º , then the value of t
will be
(a) 1 (b) 2 (c) 3 (d) 4

4. The sum and product of zeroes of a quadratic polynomial are 6 and 9 respectively. The quadratic polynomial
will be
(a) x2 + 9x − 6 (b) x2 + 6x + 9 (c) x2 − 6x + 9 (d) x2 + 6x − 9

5. Half the perimeter of a rectangular garden, whose length is 4 m more then its width, is 36 m. The dimensions of
garden will be
(a) 20 m by 16 m (b) 36 m by 10 m (c) 16 m by 30 m (d) 20 m by 16 m

6. The quadratic equation x2 + x − 5 = 0 has


(a) two distinct real roots (b) two equal real roots
(c) no real roots (d) more than 2 real roots

7. Which of the following equations has 2 as a root?


(a) x2 − 4x + 5 = 0 (b) x2 + 3x − 12 = 0 (c) 2x2 − 7x + 6 = 0 (d) 3x2 − 6x − 2 = 0

Install NODIA App to See the Solutions.


Click Here To Install
Page 2 Sample Paper 02 CBSE Mathematics Class 10

8. What happens to value of cos θ when θ increases from 0º to 90º.


(a) cos θ decreases from 1 to 0. (b) cos θ increases from 0 to 1.
(c) cos θ increases from 12 to 1 (d) cos θ decreases from 1 to 12

9. The times, in seconds, taken by 150 athletes to run a 110 m hurdle race are tabulated below

Class 13.8-14 14-14.2 14.2-14.4 14.4-14.6 14.6-14.8 14.8-15


Frequency 2 4 5 71 48 20
The number of athletes who completed the race in less than 14.6 second is :
(a) 11 (b) 71 (c) 82 (d) 130

10. For what value of k , the pair of linear equations kx − 4y = 3, 6x − 12y = 9 has an infinite number of solutions ?
(a) k = 2 (b) k ! 2 (c) k ! 3 (d) k = 4

11. The top of two poles of height 20 m and 14 m are connected by a wire. If the wire makes an angle of 30c with the
horizontal, then the length of the wire is
(a) 12 m (b) 10 m (c) 8 m (d) 6 m

12. Which term of an AP, 21, 42, 63, 84, ... is 210?
(a) 9th (b) 10th (c) 11th (d) 12th

13. The perimeters of two similar triangles TABC and TPQR are 35 cm and 45 cm respectively, then the ratio of
the areas of the two triangles is .........
(a) 29 (b) 79 (c) 49
81 (d) 34

14. tan2 θ + cot2 θ = ?


1 + tan2 θ 1 + cot2 θ
(a) 1 (b) 2 tan2 θ (c) 2 cot2 θ (d) 2 sec2 θ

15. It is proposed to build a single circular park equal in area to the sum of areas of two circular parks of diameters
16 m and 12 m in a locality. The radius of the new park would be
(a) 10 m (b) 15 m (c) 20 m (d) 24 m

16. If two solid hemispheres of same base radius r are joined together along their bases, then curved surface area of
this new solid is
(a) 4πr2 (b) 6πr2 (c) 3πr2 (d) 8πr2

17. If zeroes of the polynomial x2 + 4x + 2a are a and 2 , then the value of a is


a
(a) 1 (b) 2 (c) 3 (d) 4

18. If radii of two concentric circles are 4 cm and 5 cm , then the length of each of one circle which is tangent to the
other circle, is
(a) 3 cm (b) 6 cm (c) 9 cm (d) 1 cm

Install NODIA App to See the Solutions.


Click Here To Install
Page 3 Sample Paper 02 CBSE Mathematics Class 10

In the question number 19 and 20, a statement of Assertion (A) is followed by a statement of Reason (R). Choose the
correction option.

19. Assertion : Pair of linear equations : 9x + 3y + 12 = 0, 8x + 6y + 24 = 0 have infinitely many solutions.


Reason : Pair of linear equations a1 x + b1 y + c1 = 0 and a2 x + b2 y + c2 = 0 have infinitely many solutions, if
a1 = b1 = c1
a2 b2 c2
(a) Both assertion (A) and reason (R) are true and reason (R) is the correct explanation of assertion (A).
(b) Both assertion (A) and reason (R) are true but reason (R) is not the correct explanation of assertion (A).
(c) Assertion (A) is true but reason (R) is false.
(d) Assertion (A) is false but reason (R) is true.

20. Assertion : If n th term of an AP is 7 - 4n , then its common differences is - 4 .


Reason : Common difference of an AP is given by d = an + 1 − an .
(a) Both assertion (A) and reason (R) are true and reason (R) is the correct explanation of assertion (A).
(b) Both assertion (A) and reason (R) are true but reason (R) is not the correct explanation of assertion (A).
(c) Assertion (A) is true but reason (R) is false.
(d) Assertion (A) is false but reason (R) is true.

Section - B
Section B consists of 5 questions of 2 marks each.

21. If two positive integers p and q are written as p = a2 b3 and q = a3 b, where a and b are prime numbers than verify
LCM (p, q) # HCF (q, q) = pq

22. If the nth term of an AP - 1, 4, 9, 14, ..... is 129. Find the value of n.
 O
Write the nth term of the AP 1 , 1 + m , 1 + 2m , .....
m m m

23. If the mid-point of the line segment joining the points A (3, 4) and B (k, 6) is P (x, y) and x + y − 10 = 0 , find the
value of k .

24. In figure, AP , AQ and BC are tangents of the circle with centre O . If AB = 5 cm , AC = 6 cm and BC = 4 cm,
then what is the length of AP ?

 O
Two chords AB and CD of a circle intersect at E such that AE = 2.4 cm , BE = 3.2 cm and CE = 1.6 cm . What
is the length of DE ?

Install NODIA App to See the Solutions.


Click Here To Install
Page 4 Sample Paper 02 CBSE Mathematics Class 10

25. Two coins are tossed together. Find the probability of getting both heads or both tails.

Section - C
Section C consists of 6 questions of 3 marks each.

26. Find HCF and LCM of 378, 180 and 420 by prime factorization method. Is HCF × LCM of these numbers equal
to the product of the given three numbers?

27. In Figure, in TABC , DE z BC such that AD = 2.4 cm, AB = 3.2 cm and AC = 8 cm, then what is the length
of AE ?

28. Prove that : cot θ + cosec θ − 1 = 1 + cot θ


cot θ − cosec θ + 1 sin θ

29. From a point P , which is at a distant of 13 cm from the centre O of a circle of radius 5 cm, the pair of tangents
PQ and PR are drawn to the circle, then the area of the quadrilateral PQOR (in cm2).

30. In the given figure, a chord AB of the circle with centre O and radius 10 cm, that subtends a right angle at the
centre of the circle. Find the area of the minor segment AQBP . Hence find the area of major segment ALBQA
. (Use π = 3.14 )

Install NODIA App to See the Solutions.


Click Here To Install
Page 5 Sample Paper 02 CBSE Mathematics Class 10

 O
Find the area of shaded region shown in the given figure where a circular arc of radius 6 cm has been drawn with
vertex O of an equilateral triangle OAB of side 12 cm as centre.

31. A die is thrown once. Find the probability of getting a number which (i) is a prime number (ii) lies between 2
and 6.
 O
A die is thrown twice. Find the probability that
(i) 5 will come up at least once.
(ii) 5 will not come up either time.

Section - D
Section D consists of 4 questions of 5 marks each.

32. Solve the following pair of linear equations graphically:


x + 3y = 12, 2x − 3y = 12
Also shade the region bounded by the line 2x − 3y = 2 and both the co-ordinate axes.
 O
For what values of a and b does the following pair of linear equations have infinite number of solution ?
2x + 3y = 7, a ^x + y h − b ^x − y h = 3a + b − 2

33. Show that A ^- 1, 0h, B ^3, 1h, C ^2, 2h and D ^- 2, 1h are the vertices of a parallelogram ABCD.

34. Sides AB and AC and median AD of a triangle ABC are respectively proportional to sides PQ and PR and
median PM of another triangle PQR. Show that TABC~TPQR.
 O
Find the length of the second diagonal of a rhombus, whose side is 5 cm and one of the diagonals is 6 cm.

35. A solid iron pole consists of a cylinder of height 220 cm and base diameter 24 cm is surmounted by another
cylinder of height 60 cm and radius 8 cm. Find the mass of the pipe, given that 1 cm3 of iron has approximately
8 g mass. (Use π = 3.14 )

Install NODIA App to See the Solutions.


Click Here To Install
Page 6 Sample Paper 02 CBSE Mathematics Class 10

Section - E
Case study based questions are compulsory.

36. Auditorium, the part of a public building where an audience sits, as distinct from the stage, the area on which the
performance or other object of the audience’s attention is presented. In a large theatre an auditorium includes a
number of floor levels frequently designed as stalls, private boxes, dress circle, balcony or upper circle, and gallery.
A sloping floor allows the seats to be arranged to give a clear view of the stage. The walls and ceiling usually
contain concealed light and sound equipment and air extracts or inlets and may be highly decorated.

In an auditorium, seats are arranged in rows and columns. The number of rows are equal to the number of seats in
each row. When the number of rows are doubled and the number of seats in each row is reduced by 10, the total
number of seats increases by 300.
(i) If x is taken as number of row in original arrangement, write the quadratic equation that describes the
situation ?
(ii) How many number of rows are there in the original arrangement?
(iii) How many number of seats are there in the auditorium in original arrangement ? How many number of seats
are there in the auditorium after re-arrangement.
(iv) How many number of columns are there in the auditorium after re-arrangement?

37. Drawbridge : A drawbridge is a bridge that can be moved in order to stop or allow passage across it. Modern
drawbridges are often built across large, busy waterways. They can be lifted to allow large ships to pass or lowered
to allow land vehicles or pedestrians to cross.

A drawbridge is 60 metre long when stretched across a river. As shown in the figure, the two sections of the bridge
can be rotated upward through an angle of 30c .
(i) If the water level is 5 metre below the closed bridge, find the height h between the end of a section and the
water level when the bridge is fully open.
(ii) How far apart are the ends of the two sections when the bridge is fully opened, as shown in the figure?

Install NODIA App to See the Solutions.


Click Here To Install
Page 7 Sample Paper 02 CBSE Mathematics Class 10

38. Life insurance is a contract between an insurance policy holder and an insurer or assurer, where the insurer
promises to pay a designated beneficiary a sum of money upon the death of an insured person (often the policy
holder). Depending on the contract, other events such as terminal illness or critical illness can also trigger
payment. The policy holder typically pays a premium, either regularly or as one lump sum.

SBI life insurance agent found the following data for distribution of ages of 100 policy holders. Calculate the
median age, if policies are given only to persons having age 18 years onwards but less than 60 years.

Age (in years) Below Below Below Below Below Below Below Below Below
20 25 30 35 40 45 50 55 60
Number of 2 6 24 45 78 89 92 98 100
policy holders
(i) What is the median value of age ?
(ii) What will be the upper limit of the modal class? What is the mode value of age ?
(iv) Find the mean value of age using empirical relation.

 ******

Install NODIA App to See the Solutions.


Click Here To Install
Page 1 Sample Paper 03 CBSE Mathematics Class 10

Sample Paper 03
Class- X Exam - 2023-24
Mathematics - Standard

Time Allowed: 3 Hours Maximum Marks : 80


General Instructions :
1. This Question Paper has 5 Sections A-E.
2. Section A has 20 MCQs carrying 1 mark each
3. Section B has 5 questions carrying 02 marks each.
4. Section C has 6 questions carrying 03 marks each.
5. Section D has 4 questions carrying 05 marks each.
6. Section E has 3 case based integrated units of assessment (04 marks each) with sub-parts.
7. All Questions are compulsory. However, an internal choice in 2 Qs of 5 marks, 2 Qs of 3 marks and 2 Questions
of 2 marks has been provided.
8. Draw neat figures wherever required. Take π = 227 wherever required if not stated.

Section - A
Section A consists of 20 questions of 1 mark each.

1. What are the values of x and y for the following pair of linear equations ?
99x + 101y = 499 and 101x + 99y = 501
(a) 3 and 6 (b) 3 and 2
(c) 2 and 3 (d) 6 and 3

2. If a number x is chosen at random from the numbers - 3 , - 2 , - 1. 0, 1, 2, 3, then What is the probability of
x2 < 4 ?
4
(a) 7 (b) 73
1 2
(c) 7 (d) 7

3. The zeroes of polynomial p ^x h = ax2 + bx + c are reciprocal of each other if


(a) b = 2a (b) c = b
(c) b=a (d) c = a

4. If - 1 is a zero of the polynomial p ^x h = kx2 − 4x + k, the value of k is


(a) –4 (b) –2
(c) 2 (d) 4

5. If α and β are the roots of ax2 − bx + c = 0 (a ! 0), then value of α + β is


b
(a) a (b) ab
2a a
(c) b (d) 2b

Install NODIA App to See the Solutions.


Click Here To Install
Page 2 Sample Paper 03 CBSE Mathematics Class 10

6. Which of the following value of k should be selected so that the pair of equations x + 2y = 5 and 3x + ky + 15 = 0
has a unique solution ?
(a) k ! 5 (b) k ! 6
(c) k=5 (d) k = 6

7. The quadratic equation x2 + 3x + 2 2 = 0 has


(a) two distinct real roots (b) two equal real roots
(c) no real roots (d) more than 2 real roots

8. A ladder 10 m long reaches a window 8 m above the ground. The distance of the foot of the ladder from the base
of the wall is ................ m.
(a) 8 m (b) 2 m
(c) 6m (d) 4 m

9. If the sum of the circumferences of two circles with radii R1 and R2 is equal to the circumference of a circle of
radius R , then
(a) R1 + R2 = R (b) R1 + R2 > R
(c) R1 + R2 > R (d) R1 + R2 < R

10. The famous mathematician associated with finding the sum of the first 100 natural numbers is
(a) Pythagoras (b) Newton
(c) Gauss (d) Euclid

11. The value of x for which 2x, (x + 10) and (3x + 2) are the three consecutive terms of an AP, is
(a) 6 (b) - 6
(c) 18 (d) - 18

12. If points A (- 3, 12), B (7, 6) and C (x, 9) are collinear, then the value of x is ......... .
(a) 2 (b) 3
(c) 4 (d) 5

13. The value of sin2 41º + sin2 49º will be


(a) 1 (b) 2
(c) 2 (d) 3

14. The number 757 will have -


(a) non-terminating repeating decimal expansion.
(b) terminating decimal expansion.
(c) non-terminating non repeating decimal expansion.
(d) terminating non repeating decimal expansion

Install NODIA App to See the Solutions.


Click Here To Install
Page 3 Sample Paper 03 CBSE Mathematics Class 10

15. A tree casts a shadow 15 m long on the level of ground, when the angle of elevation of the sun is 45c. The height
of a tree is
(a) 10 m (b) 14 m
(c) 8m (d) 15 m

16. If the perimeter of one face of a cube is 20 cm, then its surface area is
(a) 120 cm2 (b) 150 cm2
(c) 125 cm2 (d) 400 cm2

17. sin2 60c − 2 tan 45c − cos2 30c = ?


(a) 2 (b) –2
(c) 1 (d) –1

18. If xi ’s are the mid-points of the class intervals of grouped data, f i ’s are the corresponding frequencies and x is the
mean, then / (f i xi - x ) is equal to
(a) 0 (b) - 1
(c) 1 (d) 2

In the question number 19 and 20, a statement of Assertion (A) is followed by a statement of Reason (R). Choose the
correction option.

19. Assertion : The values of x are - a2 , a for a quadratic equation 2x2 + ax − a2 = 0 .


Reason : For quadratic equation ax2 + bx + c = 0
2
x = − b ! b − 4ac
2a
(a) Both assertion (A) and reason (R) are true and reason (R) is the correct explanation of assertion (A).
(b) Both assertion (A) and reason (R) are true but reason (R) is not the correct explanation of assertion (A).
(c) Assertion (A) is true but reason (R) is false.
(d) Assertion (A) is false but reason (R) is true.

20. Assertion : The two tangents are drawn to a circle from an external point, then they subtend equal angles at the
centre.
Reason : A parallelogram circumscribing a circle is a rhombus.
(a) Both assertion (A) and reason (R) are true and reason (R) is the correct explanation of assertion (A).
(b) Both assertion (A) and reason (R) are true but reason (R) is not the correct explanation of assertion (A).
(c) Assertion (A) is true but reason (R) is false.
(d) Assertion (A) is false but reason (R) is true.

Section - B
Section B consists of 5 questions of 2 marks each.

21. Given that HCF (306, 1314) = 18. Find LCM (306, 1314)

Install NODIA App to See the Solutions.


Click Here To Install
Page 4 Sample Paper 03 CBSE Mathematics Class 10

22. If α and β are the zeroes of a polynomial x2 − 4 3 x + 3, then find the value of α + β − αβ .
 O
If one of the zeroes of the quadratic polynomial f ^x h = 14x2 − 42k2 x − 9 is negative of the other, find the value of
‘k ’.

23. The mid-point of the line-segment AB is P (0, 4), if the coordinates of B are (- 2, 3) then find the co-ordinates
of A.

24. In the given figure, G is the mid-point of the side PQ of TPQR and GH || QR. Prove that H is the mid-point of
the side PR or the triangle PQR.

 O
In the figure of TABC, the points D and E are on the sides CA, CB respectively such that DE || AB,
AD = 2x, DC = x + 3, BE = 2x − 1 and CE = x. Then, find x.

25. Two different dice are tossed together. Find the probability :
(i) that the number on each die is even.
(ii) that the sum of numbers appearing on the two dice is 5.

Section - C
Section C consists of 6 questions of 3 marks each.

26. Write the smallest number which is divisible by both 306 and 657.

27. TABC and TBDE are two equilateral triangle such that D is the mid-point of BC . Ratio of the areas of triangles
ABC and BDE is ................. .

Install NODIA App to See the Solutions.


Click Here To Install
Page 5 Sample Paper 03 CBSE Mathematics Class 10

28. Evaluate :
3 tan2 30º + tan2 60º + cosec 30º − tan 45º
cot2 45º

29. Prove that the rectangle circumscribing a circle is a square.

30. A solid is in the shape of a cone surmounted on a hemisphere. The radius of each of them being 3.5 cm and the
total height of the solid is 9.5 cm. Find the volume of the solid.
 O
A heap of rice is in the form of a cone of base diameter 24 m and height 3.5 m. Find the volume of the rice. How
much canvas cloth is required to just cover the heap?

31. The mean of the following distribution is 48 and sum of all the frequency is 50. Find the missing frequencies x
and y .

Class 20-30 30-40 40-50 50-60 60-70


Frequency 8 6 x 11 y

 O
The table below shows the daily expenditure on food of 25 households in a locality. Find the mean daily expenditure
on food.

Daily expenditure (in <) 100-150 150-200 200-250 250-300 300-350


Number of households 4 5 12 2 2

Section - D
Section D consists of 4 questions of 5 marks each.

32. Find the values of k for which the equation ^3k + 1h x2 + 2 ^k + 1h x + 1 has equal roots. Also find the roots.
 O
A person on tour has < 4200 for his expenses. If he extends his tour for 3 days, he has to cut down his daily
expenses by < 70. Find the original duration of the tour.

33. Prove that the point ^3, 0h , ^6, 4h and ^- 1, 3h are the vertices of a right angled isosceles triangle.

34. In figure, a circle with centre O is inscribed in a quadrilateral ABCD such that, it touches the sides BC , AB,
AD and CD at points P, Q, R and S respectively. If AB = 29 cm, AD = 23 cm, +B = 90c and DS = 5 cm, then
find the radius of the circle (in cm).

 O
Prove that opposite sides of a quadrilateral circumscribing a circle subtend supplementary angles at the centre of
the circle.

Install NODIA App to See the Solutions.


Click Here To Install
Page 6 Sample Paper 03 CBSE Mathematics Class 10

35. In Figure, a square OABC is inscribed in a quadrant OPBQ . If OA = 15 cm , find the area of the shaded region.
(Use π = 3.14 ).

Section - E
Case study based questions are compulsory.

36. Bequests to Charity : At the time our mother left this Earth, she gave Rs 90000 to her children of birth. This we
kept and each year added Rs 30000 more, as a lasting memorial from the children she bore. When Rs 4,20,000 is
thusly attained, all goes to charity that her memory be maintained.
(i) What was the balance in the sixth year?
(ii) In what year was the goal of Rs 420,000 met?

37. Eiffel Tower : The Eiffel Tower is a landmark and an early example of wrought-iron construction on a gigantic
scale. The lower section consists of four immense arched legs set on masonry piers. The legs curve inward until
they unite in a single tapered tower. Platforms, each with an observation deck, are at three levels; on the first is
also a restaurant.
The tower, constructed of about 7000 tons of iron, has stairs and elevators. A meteorological station, a radio
communications station, and a television transmission antenna, as well as a suite of rooms that were used by Eiffel
are located near the top of the tower.

Continue on next page....

Install NODIA App to See the Solutions.


Click Here To Install
Page 7 Sample Paper 03 CBSE Mathematics Class 10

(i) For a person standing 324 m from the center of the base of the Eiffel Tower, the angle of elevation to the
top of the tower is 45c . How tall is the Eiffel Tower?
(ii) A car is moving at uniform speed towards the Eiffel tower. It takes 15 minutes for the angle of depression
from the top of tower to the car to change from 30c to 60c . After how much time after this, the car will
reach the base of the tower?

38. Double-six Dominos : It is a game played with the 28 numbered tiles shown in the diagram.

The 28 dominos are placed in a bag, shuffled, and then one domino is randomly drawn. Give the following answer.
(i) What is the probability the total number of dots on the domino is three or less ?
(ii) What is the probability the total number of dots on the domino is greater than three ?
(iii) What is the probability the total number of dots on the domino does not have a blank half ?
(iv) What is the probability the total number of dots on the domino is not a “double” (both sides the same) ?

 ******

Install NODIA App to See the Solutions.


Click Here To Install
Page 1 Sample Paper 04 CBSE Mathematics Class 10

Sample Paper 04
Class- X Exam - 2023-24
Mathematics - Standard

Time Allowed: 3 Hours Maximum Marks : 80


General Instructions :
1. This Question Paper has 5 Sections A-E.
2. Section A has 20 MCQs carrying 1 mark each
3. Section B has 5 questions carrying 02 marks each.
4. Section C has 6 questions carrying 03 marks each.
5. Section D has 4 questions carrying 05 marks each.
6. Section E has 3 case based integrated units of assessment (04 marks each) with sub-parts.
7. All Questions are compulsory. However, an internal choice in 2 Qs of 5 marks, 2 Qs of 3 marks and 2 Questions
of 2 marks has been provided.
8. Draw neat figures wherever required. Take π = 227 wherever required if not stated.

Section - A
Section A consists of 20 questions of 1 mark each.

1. An event is very unlikely to happen. Its probability is closest to


(a) 0.0001 (b) 0.001
(c) 0.01 (d) 0.1

2. One equation of a pair of dependent linear equations − 5x + 7y = 2 The second equation can be
(a) 10x + 14y + 4 = 0 (b) − 10x − 14y + 4 = 0
(c) − 10x + 14y + 4 = 0 (d) 10x − 14y =− 4

3. The quadratic equation x2 + 3x + 2 2 = 0 has


(a) two distinct real roots (b) two equal real roots
(c) no real roots (d) more than 2 real roots

4. There are 60 terms is an AP of which the first term is 8 and the last term is 185. The 31 st term is
(a) 56 (b) 94
(c) 85 (d) 98

5. It is given that, TABC + TEDF such that AB = 5 cm, AC = 7 cm, DF = 15 cm and DE = 12 cm then the sum
of the remaining sides of the triangles is
(a) 23.05 cm (b) 16.8 cm
(c) 6.25 cm (d) 24 cm

6. If triangle ABC is similar to triangle DEF such that 2AB = DE and BC = 8 cm then find EF.
(a) 16 cm (b) 14 cm
(c) 12 cm (d) 15 cm

Install NODIA App to See the Solutions.


Click Here To Install
Page 2 Sample Paper 04 CBSE Mathematics Class 10

7. If radii of two concentric circles are 4 cm and 5 cm , then the length of each of one circle which is tangent to the
other circle, is
(a) 3 cm (b) 6 cm
(c) 9 cm (d) 1 cm

8. Observations of some data are x


5 , x, x
3 , 2x
3 , x
4 , 2x
5 and 3x
4 where x > 0. If the median of the data is 4, then the
value of x is
(a) 5 (b) 15
(c) 9 (d) 10

9. Two concentric circles of radii a and b where a > b, The length of a chord of the larger circle which touches the
other circle is
(a) a2 + b2 (b) 2 a2 + b2
(c) a2 - b2 (d) 2 a2 - b2

10. Which of the following equations has the sum of its roots as 3 ?
(a) 2x2 − 3x + 6 = 0 (b) − x2 + 3x − 3 = 0
(c) 2 x2 − 3 x + 1 = 0 (d) 3x2 − 3x + 3 = 0
2

11. If sin θ + cos θ = 2 cos θ , (θ ! 90c) then the value of tan θ is

(a) 2 -1 (b) 2 +1
(c) 2 (d) - 2

12. The angles of elevation of the top of a tower from the points P and Q at distance of a and b respectively from
the base and in the same straight line with it, are complementary. The height of the tower is
(a) ab (b) ab

(c) a (d) b
b a

13. In the given figure, the positions of the observer and the object are mentioned, the angle of depression is

(a) 30c (b) 90c


(c) 60c (d) 45c

Install NODIA App to See the Solutions.


Click Here To Install
Page 3 Sample Paper 04 CBSE Mathematics Class 10

14. In a circle of radius 14 cm, an arc subtends an angle of 45c at the centre, then the area of the sector is
(a) 71 cm2 (b) 76 cm2
(c) 77 cm2 (d) 154 cm2

15. During conversion of a solid from one shape to another, the volume of the new shape will
(a) increase (b) decrease
(c) remain unaltered (d) be doubled

16. In figure, AP , AQ and BC are tangents of the circle with centre O . If AB = 5 cm , AC = 6 cm and BC = 4 cm,
then the length of AP (in cm) is

(a) 15 (b) 10
(c) 9 (d) 7.5

17. If X , M and Z are denoting mean, median and mode of a data and X: M = 9 : 8 , then the ratio M : Z is
(a) 3 : 4 (b) 4 : 9
(c) 4:3 (d) 2 : 5

18. There are 1000 sealed envelopes in a box. 10 of them contain a cash prize of < 100 each, 100 of them contain a
cash prize of < 50 each and 200 of them contain a cash prize of < 10 each and rest do not contain any cash prize.
If they are well-shuffled and an envelope is picked up out, then the probability that is contains no cash prize is
(a) 0.65 (b) 0.69
(c) 0.54 (d) 0.57

In the question number 19 and 20, a statement of Assertion (A) is followed by a statement of Reason (R). Choose the
correction option.

19. Assertion : p ^x h = 14x3 − 2x2 + 8x 4 + 7x − 8 is a polynomial of degree 3.


Reason : The highest power of x in the polynomial p ^x h is the degree of the polynomial.
(a) Both assertion (A) and reason (R) are true and reason (R) is the correct explanation of assertion (A).
(b) Both assertion (A) and reason (R) are true but reason (R) is not the correct explanation of assertion (A).
(c) Assertion (A) is true but reason (R) is false.
(d) Assertion (A) is false but reason (R) is true.

Continue on next page....

Install NODIA App to See the Solutions.


Click Here To Install
Page 4 Sample Paper 04 CBSE Mathematics Class 10

20. Assertion : sin2 67c + cos2 67c = 1


Reason : For any value of θ, sin2 θ + cos2 θ = 1
(a) Both assertion (A) and reason (R) are true and reason (R) is the correct explanation of assertion (A).
(b) Both assertion (A) and reason (R) are true but reason (R) is not the correct explanation of assertion (A).
(c) Assertion (A) is true but reason (R) is false.
(d) Assertion (A) is false but reason (R) is true.

Section - B
Section B consists of 5 questions of 2 marks each.

21. Find the 21 st term of the AP - 4 12 , - 3, - 1 12 , ...

22. In Figure, common tangents AB and CD to the two circle with centres O1 and O2 intersect at E. Prove that
AB = CD.

23. If A ^5, 2h , B ^2, - 2h and C ^- 2, t h are the vertices of a right angled triangle with +B = 90º , then find the value
of t.

24. Show that any positive even integer can be written in the form 6q, 6q + 2 or 6q + 4, where q is an integer.
 O
Find the smallest natural number by which 1200 should be multiplied so that the square root of the product is a
rational number.

25. If five times the fifth term of an AP is equal to eight times its eighth term, show that its 13th term is zero.
 O
Find the values of a, b and c , such that the numbers a, 10, b, c, 31 are in AP

Section - C
Section C consists of 6 questions of 3 marks each.

26. For what value of p will the following system of equations have no solution ?
^2p − 1h x + ^p − 1h y = 2p + 1; y + 3x − 1 = 0

Install NODIA App to See the Solutions.


Click Here To Install
Page 5 Sample Paper 04 CBSE Mathematics Class 10

27. Solve the following quadratic equation for x :


x2 + b a + a + b l x + 1 = 0
a+b a

28. In figure, two tangents TP and TQ are drawn to circle with centre O from an external point T . Prove that
+PTQ = 2+OPQ .

 O

A circle is inscribed in a TABC, with sides AC, AB and BC as 8 cm, 10 cm and 12 cm respectively. Find the
length of AD, BE and CF.

29. A boy, flying a kite with a string of 90 m long, which is making an angle θ with the ground. Find the height of
the kite. (Given tan θ = 458 )

30. A wooden article was made by scooping out a hemisphere from each end of a solid cylinder, as shown in Figure.
If the height of the cylinder is 10 cm and its base is of radius 3.5 cm, find the total surface area of the article.

 O

A wooden toy was made by scooping out a hemisphere of same radius from each end of a solid cylinder. If the
height of the cylinder is 10 cm, and its base is of radius 3.5 cm, find the volume of wood in the toy. Use π = 227

31. A game consists of tossing a coin 3 times and noting the outcome each time. If getting the same result in all the
tosses is a success, find the probability of losing the game.

Install NODIA App to See the Solutions.


Click Here To Install
Page 6 Sample Paper 04 CBSE Mathematics Class 10

Section - D
Section D consists of 4 questions of 5 marks each.

32. If α and β are the zeroes the polynomial 2x2 − 4x + 5, find the values of
(i) α2 + β2 (ii) 1 + 1
α β

(iii) ^α − βh (iv) 12 + 12
2
α β
(v) α2 + β2

 O
If α and β are the zeroes of the polynomial p ^x h = 2x2 + 5x + k satisfying the relation, α2 + β2 + αβ = 21
4 , then
find the value of k .

33. In the right triangle, B is a point on AC such that AB + AD = BC + CD. If AB = x, BC = h and CD = d, then
find x (in term of h and d).

34. Find A and B if sin ^A + 2B h = 2


3
and cos ^A + 4B h = 0 , where A and B are acute angles.
 O
Evaluate the following :
2 cos2 60º + 3 sec2 30º − 2 tan2 45º
sin2 30º + cos2 45º

35. In figure, PQRS is square lawn with side PQ = 42 metre. Two circular flower beds are there on the sides PS and
QR with centre at O, the intersection of its diagonals. Find the total area of the two flower beds (shaded parts).

Install NODIA App to See the Solutions.


Click Here To Install
Page 7 Sample Paper 04 CBSE Mathematics Class 10

Section - E
Case study based questions are compulsory.
36. Salary Cut : Our personal and professional lives have taken quite a turn in the light of the coronavirus pandemic.
Worklife has been revamped across the globe, in that working from home has become the norm for those employees
and organizations that continue to carry on with their operations and functioning in the lockdown. Increments,
incentives, businesses have all been impacted.

The Covid-19 pandemic has stalled economic activity at an unprecedented scale globally, raising the spectre of job
losses and salary cuts. Most of companies decided to bring down the salary by upto 50 %.
The following table shows the salaries (in percent) received by 80 employee during lockdown.

Salary Received (in %) 40-50 50-60 60-70 70-80 80-90 90-100


Number of employee 4 10 14 20 24 8
Based on the above information, answer the following questions.
(i) What is the mean salary (in %) received ?
(ii) How many employee received more than 70 % salary ?
(iii) Find the total number of employee whose salary is reduced by more than 40 % ?
(iv) What is the lower limit of mode class of salary (in %) received?

37. To conduct sports day activities, in a rectangular shaped school ground ABCD , lines have been
drawn with chalk powder at a distance of 1 m each. 100 flower pots have been placed at a distance
of 1 m from each other along AB , as shown in figure. Nishtha runs 14 th the distance AB on the
2nd line and posts a green flag. Suman runs 15 th the distance AB on the 8th line and posts a red flag.

Continue on next page....

Install NODIA App to See the Solutions.


Click Here To Install
Page 8 Sample Paper 04 CBSE Mathematics Class 10

(i) What is the position of green flag ?


(ii) What is the position of red flag ?
(iii) What is the distance between both the flags?
(iv) What is the distance of red flag from point A ?

38. The Prime Minister’s Citizen Assistance and Relief in Emergency Situations Fund was created on 28 March 2020,
following the COVID-19 pandemic in India. The fund will be used for combating, and containment and relief
efforts against the coronavirus outbreak and similar pandemic like situations in the future.

The allotment officer is trying to come up with a method to calculate fair division of funds across various affected
families so that the fund amount and amount received per family can be easily adjusted based on daily revised
numbers. The total fund allotted for a village is x3 + 6x2 + 20x + 9 . The officer has divided the fund equally among
families of the village and each family receives an amount of x2 + 2x + 2 . After distribution, some amount is left.
(i) How many families are there in the village?
(ii) If an amount of < 1911 is left after distribution, what is value of x ?
(iii) How much amount does each family receive?
(iv) What is the amount of fund allocated?

 ******

Install NODIA App to See the Solutions.


Click Here To Install
Page 1 Sample Paper 5 Mathematics STD Class 10

Sample Paper 5
Class- X Exam - 2023-24
Mathematics - Standard

Time Allowed: 3 Hours Maximum Marks : 80


General Instructions :
1. This Question Paper has 5 Sections A-E.
2. Section A has 20 MCQs carrying 1 mark each.
3. Section B has 5 questions carrying 02 marks each.
4. Section C has 6 questions carrying 03 marks each.
5. Section D has 4 questions carrying 05 marks each.
6. Section E has 3 case based integrated units of assessment (04 marks each) with sub-parts of the values of 1, 1
and 2 marks each respectively.
7. All Questions are compulsory. However, an internal choice in 2 Qs of 5 marks, 2 Qs of 3 marks and 2 Questions
of 2 marks has been provided. An internal choice has been provided in the 2 marks questions of Section E.
8. Draw neat figures wherever required. Take π = 227 wherever required if not stated.

Section - A
Section A consists of 20 questions of 1 mark each.

1. If the distance between the points A (4, p) and B (1, 0) is 5 units then the value(s) of p is(are)
(a) 4 only (b) - 4 only
(c) ! 4 (d) 0

2. For which value(s ) of p , will the lines represented by the following pair of linear equations be parallel
3x - y - 5 = 0
6x - 2y - p = 0
(a) all real values except 10 (b) 10
(c) 5/2 (d) 1/2

3. In figure, on a circle of radius 7 cm, tangent PT is drawn from a point P such that PT = 24 cm. If O is the centre
of the circle, then the length of PR is

(a) 30 cm (b) 28 cm
(c) 32 cm (d) 25 cm

https://qrbook.page.link/app
Install NODIA App to See the Solutions.
Click Here To Install
Page 2 Sample Paper 5 Mathematics STD Class 10

4. Each root of x2 − bx + c = 0 is decreased by 2. The resulting equation is x2 − 2x + 1 = 0 , then


(a) b = 6, c = 9 (b) b = 3, c = 5
(c) b = 2, c =− 1 (d) b = − 4, c = 3

5. The centroid of the triangle whose vertices are (3, - 7), (- 8, 6) and (5, 10) is
(a) (0, 9) (b) (0, 3)
(c) (1, 3) (d) (3, 5)

6. If the sum of the zeroes of the quadratic polynomial kx2 + 2x + 3k is equal to their product, then k equals

(a) 1 (b) - 1
3 3
(c) 2 (d) - 2
3 3

7. The nth term of the AP a , 3a , 5a , ... is


(a) na (b) (2n - 1) a
(c) (2n + 1) a (d) 2na

8. In an AP, if a = 3.5 , d = 0 and n = 101, then an will be


(a) 0 (b) 3.5
(c) 103.5 (d) 104.5

9. If one zero of the quadratic polynomial x2 + 3x + k is 2, then the value of k is


(a) 10 (b) - 10
(c) -7 (d) - 2

10. In the adjoining figure, OABC is a square of side 7 cm. OAC is a quadrant of a circle with O as centre. The area
of the shaded region is

(a) 10.5 cm2 (b) 38.5 cm2


(c) 49 cm2 (d) 11.5 cm2

11. The value of the expression


cosec ^75c + qh − sec ^15c − qh − tan ^55c + qh + cot ^35c − qh is
(a) - 1 (b) 0
(c) 1 (d) 3
2

https://qrbook.page.link/appInstall NODIA App to See the Solutions.


Click Here To Install
Page 3 Sample Paper 5 Mathematics STD Class 10

12. If the angle of depression of an object from a 75 m high tower is 30c, then the distance of the object from the
tower is
(a) 25 3 m (b) 50 3 m
(c) 75 3 m (d) 150 m

13. The 2 digit number which becomes 56 th of itself when its digits are reversed. The difference in the digits of the
number being 1, then the two digits number is
(a) 45 (b) 54
(c) 36 (d) None of these

14. In the given figure, DE z BC . The value of EC is

(a) 1.5 cm (b) 3 cm


(c) 2 cm (d) 1 cm

15. If the radius of the sphere is increased by 100%, the volume of the corresponding sphere is increased by
(a) 200% (b) 500%
(c) 700% (d) 800%

16. The median and mode respectively of a frequency distribution are 26 and 29, Then its mean is
(a) 27.5 (b) 24.5
(c) 28.4 (d) 25.8

17. An event is very unlikely to happen. Its probability is closest to


(a) 0.0001 (b) 0.001
(c) 0.01 (d) 0.1

18. ^x2 + 1h2 − x2 = 0 has


(a) four real roots (b) two real roots
(c) no real roots (d) one real root

Continue on next page....

https://qrbook.page.link/app
Install NODIA App to See the Solutions.
Click Here To Install
Page 4 Sample Paper 5 Mathematics STD Class 10

In the question number 19 and 20, a statement of Assertion (A) is followed by a statement of Reason (R). Choose the
correction option.

19. Assertion : The value of y is 6, for which the distance between the points P ^2, - 3h and Q ^10, y h is 10.
Reason : Distance between two given points A ^x1, y1h and B ^x2, y2h is given,
AB = ^x2 − x1h2 + ^y2 − y1h2
(a) Both assertion (A) and reason (R) are true and reason (R) is the correct explanation of assertion (A).
(b) Both assertion (A) and reason (R) are true but reason (R) is not the correct explanation of assertion (A).
(c) Assertion (A) is true but reason (R) is false.
(d) Assertion (A) is false but reason (R) is true.

20. Assertion : 13 is a terminating decimal fraction.


3125
Reason : If q = 2m 5n where m, n are non-negative integers, then qp is a terminating decimal fraction.
(a) Both assertion (A) and reason (R) are true and reason (R) is the correct explanation of assertion (A).
(b) Both assertion (A) and reason (R) are true but reason (R) is not the correct explanation of assertion (A).
(c) Assertion (A) is true but reason (R) is false.
(d) Assertion (A) is false but reason (R) is true.

Section - B
Section B consists of 5 questions of 2 marks each.

21. In TABC, AD = BC, such that AD2 = BD # CD. Prove that TABC is right angled at A.

22. In figure, a circle touches all the four sides of a quadrilateral ABCD . If AB = 6 cm, BC = 9 cm and CD = 8 cm,
then find the length of AD .

23. If tan 2A = cot (A − 18c), where 2A is an acute angle, find the value of A.

24. Find the mean the following distribution :

Class 3-5 5-7 7-9 9-11 11-13


Frequency 5 10 10 7 8

Continue on next page....

https://qrbook.page.link/app
Install NODIA App to See the Solutions.
Click Here To Install
Page 5 Sample Paper 5 Mathematics STD Class 10

 O
Find the mode of the following data :

Class 0-20 20-40 40-60 60-80 80-100 100-120 120-140


Frequency 6 8 10 12 6 5 3

25. Explain why (7 # 13 # 11) + 11 and (7 # 6 # 5 # 4 # 3 # 2 # 1) + 3 are composite numbers.


 O
Explain whether 3 # 12 # 101 + 4 is a prime number or a composite number.

Section - C
Section C consists of 6 questions of 3 marks each.

26. The sum of four consecutive number in AP is 32 and the ratio of the product of the first and last term to the
product of two middle terms is 7 : 15. Find the numbers.

27. Prove that : cot θ + cosec θ − 1 = 1 + cot θ


cot θ − cosec θ + 1 sin θ

28. A road which is 7 m wide surrounds a circular park whose circumference is 88 m. Find the area of the road.
 O
In Figure, PQ and AB are two arcs of concentric circles of radii 7 cm and 3.5 cm respectively, with centre O . If
+POQ = 30c, then find the area of shaded region.

29. Compute the mode for the following frequency distribution:

Size of items (in cm) 0- 4 4- 8 8- 12 12-16 16-20 20-24 24-28


Frequency 5 7 9 17 12 10 6

Continue on next page....

https://qrbook.page.link/app
Install NODIA App to See the Solutions.
Click Here To Install
Page 6 Sample Paper 5 Mathematics STD Class 10

30. Find the ratio in which P (4, m) divides the segment joining the points A (2, 3) and B (6, - 3). Hence find m .
 O
In the given figure TABC is an equilateral triangle of side 3 units. Find the co-ordinates of the other two vertices.

31. Given that 5 is irrational, prove that 2 5 - 3 is an irrational number.

Section - D
Section D consists of 4 questions of 5 marks each.

32. For what value of k , which the following pair of linear equations have infinitely many solutions:
2x + 3y = 7 and ^k + 1h x + ^2k − 1h y = 4k + 1
 O
The cost of 2 kg of apples and 1kg of grapes on a day was found to be Rs. 160. After a month, the cost of 4kg of
apples and 2kg of grapes is Rs. 300. Represent the situations algebraically and geometrically.

33. Prove that opposite sides of a quadrilateral circumscribing a circle subtend supplementary angles at the centre of
the circle.

34. The angles of depression of the top and bottom of an 8 m tall building from top of a multi-storeyed building are
30º and 45º, respectively. Find the height of multi-storey building and distance between two buildings.
 O
Two poles of equal heights are standing opposite to each other on either side of a road, which is 80 m wide. From
a point between them on the road, angles of elevation of their top are 30c and 60c. Find the height of the poles
and distance of point from poles.

35. A solid is in the form of a cylinder with hemispherical end. The total height of the solid is 20 cm and the diameter
of the cylinder is 7 cm. Find the total volume of the solid. (Use π = 227 )

Continue on next page....

https://qrbook.page.link/app
Install NODIA App to See the Solutions.
Click Here To Install
Page 7 Sample Paper 5 Mathematics STD Class 10

Section - E
Case study based questions are compulsory.

36. Maximum Profit : An automobile manufacturer can produce up to 300 cars per day. The profit made from the sale
of these vehicles can be modelled by the function P (x) =− x2 + 350x − 6600 where P (x) is the profit in thousand
Rupees and x is the number of automobiles made and sold. Answer the following questions based on this model:
(i) When no cars are produce what is a profit/loss?
(ii) What is the break even point ? (Zero profit point is called break even) ?
(iii) What is the profit/loss if 175 cars are produced ?
 O
What is the profit if 400 cars are produced ?

37. Rohan is very intelligent in maths. He always try to relate the concept of maths in daily life. One day he is walking
away from the base of a lamp post at a speed of 1 m/s. Lamp is 4.5 m above the ground.

(i) If after 2 second, length of shadow is 1 meter, what is the height of Rohan ?
(ii) What is the minimum time after which his shadow will become larger than his original height?
 O
What is the distance of Rohan from pole at this point ?
(iii) What will be the length of his shadow after 4 seconds?

Continue on next page....

https://qrbook.page.link/app
Install NODIA App to See the Solutions.
Click Here To Install
Page 8 Sample Paper 5 Mathematics STD Class 10

38. Political survey questions are questions asked to gather the opinions and attitudes of potential voters. Political
survey questions help you identify supporters and understand what the public needs. Using such questions, a
political candidate or an organization can formulate policies to gain support from these people.

A survey of 100 voters was taken to gather information on critical issues and the demographic information
collected is shown in the table. One out of the 100 voters is to be drawn at random to be interviewed on the India
Today News on prime time.

Women Men Totals


Republican 17 20 37
Democrat 22 17 39
Independent 8 7 15
Green Party 6 3 5
Totals 53 47 100

(i) What is the probability the person is a woman or a Republican ?


 O
What is the probability the person is a Democrat ?
(ii) What is the probability the person is a Independent men ?
(iii) What is the probability the person is a Independent men or green party men ?

 ******

https://qrbook.page.link/app
Install NODIA App to See the Solutions.
Click Here To Install
Page 1 Sample Paper 06 CBSE Mathematics Class 10

Sample Paper 06
Class- X Exam - 2023-24
Mathematics - Standard

Time Allowed: 3 Hours Maximum Marks : 80


General Instructions :
1. This Question Paper has 5 Sections A-E.
2. Section A has 20 MCQs carrying 1 mark each
3. Section B has 5 questions carrying 02 marks each.
4. Section C has 6 questions carrying 03 marks each.
5. Section D has 4 questions carrying 05 marks each.
6. Section E has 3 case based integrated units of assessment (04 marks each) with sub-parts.
7. All Questions are compulsory. However, an internal choice in 2 Qs of 5 marks, 2 Qs of 3 marks and 2 Questions
of 2 marks has been provided.
8. Draw neat figures wherever required. Take π = 227 wherever required if not stated.

Section - A
Section A consists of 20 questions of 1 mark each.

1. There are 30 cards of the same size in a bag in which the numbers 1 to 30 are written. One card is taken out of
the bag at random. What is the probability that the number on the selected card is not divisible by 3?
(a) 151 (b) 23 (c) 101 (d) 13

2. If the mid-point of the line segment joining the points A (3, 4) and B (k, 6) is P (x, y) and x + y − 10 = 0 , the value
of k will be
(a) 4 (b) 5 (c) 6 (d) 7

3. In an AP, if a = 3.5 , d = 0 and n = 101, then an will be


(a) 0 (b) 3.5 (c) 103.5 (d) 104.5

4. In the given factor tree what is the composite number x ?

(a) 65 (b) 585 (c) 130 (d) 195

Install NODIA App to See the Solutions.


Click Here To Install
Page 2 Sample Paper 06 CBSE Mathematics Class 10

5. A tree is broken by the wind. The top struck the ground at an angle of 30c and at distance of 10 m from its root.
The whole height of the tree is ( 3 = 1.732)
(a) 10 3 m (b) 3 10 m (c) 20 3 m (d) 3 20 m

6. The zeroes of the polynomial p ^x h = 4x2 − 12x + 9 will be


(a) 32 and 32 (b) 23 and 13 (c) 2
3 and 1
3 (d) 1
3 and 1
3

7. If the equations kx − 2y = 3 and 3x + y = 5 represent two intersecting lines at unique point, then the value of k
is ........... .
(a) k =− 6 (b) k !- 6 (c) k = 4 (d) k ! 4

8. What do you say about the lines represented by ?


2x + 3y − 9 = 0 and 4x + 6y − 18 = 0
(a) lines are parallel (b) lines are coincident (c) lines are intersecting (d) can’t say anything

9. What are the values of x and y for the following system of equations.
21 + 47 = 110 , 47 + 21 = 162 , x , y ! 0
x y x y
1 1
(a) 3 and 2 (b) 13 and 1 (c) 12 and 13 (d) 1
2 and 1

ar (TABC)
10. If TABC + TPQR , and AB = 1 , then =?
PQ 3 ar (TPQR)
(a) 13 (b) 19 (c) 8
9 (d) 5
9

11. The quadratic equation x2 − 4x − 3 2 = 0 has


(a) two distinct real roots (b) two equal real roots (c) no real roots (d) more than 2 real roots

12. The quadratic equation x2 + 4x − 3 2 = 0 has


(a) two distinct real roots (b) two equal real roots (c) no real roots (d) more than 2 real roots

13. Two concentric circles are of radii 10 cm and 8 cm, then the length of the chord of the larger circle which touches
the smaller circle is
(a) 6 cm (b) 12 cm (c) 18 cm (d) 9 cm

14. Consider the following frequency distribution

Class 0-5 6-11 12-17 18-23 24-29


Frequency 13 10 15 8 11
The upper limit of the median class is
(a) 17 (b) 17.5 (c) 18 (d) 18.5

15. If sec θ $ sin θ = 0 , then value of θ will be


(a) 0 (b) 90º (c) 45º (d) 3

16. tan 4 θ + tan2 θ = ?


(a) sec2 q - 2 sec 4 q (b) 2 sec2 q - sec 4 q (c) sec2 q - sec 4 q (d) sec 4 q - sec2 q

Install NODIA App to See the Solutions.


Click Here To Install
Page 3 Sample Paper 06 CBSE Mathematics Class 10

17. Ratio of volumes of two cylinders with equal height is


(a) H : h (b) R : r (c) R2 : r2 (d) None of these

18. Which of the following are the zeroes of the polynomial p ^x h = 2x3 − 11x2 + 17x − 6 .
(a) 2 (b) 3 (c) 12 (d) Above all

In the question number 19 and 20, a statement of Assertion (A) is followed by a statement of Reason (R). Choose the
correction option.

19. Assertion : Common difference of the AP - 5 , - 1 , 3, 7, .......... is 4.


Reason : Common difference of the AP a, a + d, a + 2d, .......... is given by d = a2 − a1
(a) Both assertion (A) and reason (R) are true and reason (R) is the correct explanation of assertion (A).
(b) Both assertion (A) and reason (R) are true but reason (R) is not the correct explanation of assertion (A).
(c) Assertion (A) is true but reason (R) is false.
(d) Assertion (A) is false but reason (R) is true.

20. Assertion : If the circumference of a circle is 176 cm, then its radius is 28 cm.
Reason : Circumference = 2π # radius
(a) Both assertion (A) and reason (R) are true and reason (R) is the correct explanation of assertion (A).
(b) Both assertion (A) and reason (R) are true but reason (R) is not the correct explanation of assertion (A).
(c) Assertion (A) is true but reason (R) is false.
(d) Assertion (A) is false but reason (R) is true.

Section - B
Section B consists of 5 questions of 2 marks each.

21. Show that 5 6 is an irrational number.

22. Find the sum of first ten multiple of 5.

 O

If the sum of n terms of an AP is 2n2 + 5n, then find the 4th term.

23. If the points A (4, 3) and B (x, 5) are on the circle with centre O (2, 3), then what is the value of x ?

24. From an external point Q , the length of tangent to a circle is 12 cm and the distance of Q from the centre of circle
is 13 cm. What is the radius of circle?
 O

QP is a tangent to a circle with centre O at a point P on the circle. If TOPQ is isosceles, then find +OQR ?

25. A bag contains 3 red, 4 green and 5 white candles, one candle is drawn at random from the bag, find the
probability that candle is not red.

Install NODIA App to See the Solutions.


Click Here To Install
Page 4 Sample Paper 06 CBSE Mathematics Class 10

Section - C
Section C consists of 6 questions of 3 marks each.

26. Find HCF and LCM of 16 and 36 by prime factorization and check your answer.

27. In TABC, if X and Y are points on AB and AC respectively such that AX


XB = 34 , AY = 5 and YC = 9, then state
whether XY and BC parallel or not.

28. Prove that : 1 − cos A = cosec A − cot A


1 + cos A

29. In figure, two tangents TP and TQ are drawn to circle with centre O from an external point T . Prove that
+PTQ = 2+OPQ .

30. A road which is 7 m wide surrounds a circular park whose circumference is 88 m. Find the area of the road.
 O
Three horses are tied each with 7 m long rope at three corners of a triangular field having sides 20 m, 34 m and
42 m. Find the area of the plot which can be grazed by the horses.

31. An integer is chosen between 70 and 100. Find the probability that it is
(i) a prime number (ii) divisible by 7
 O
Find the probability that 5 Sundays occur in the month of November of a randomly selected year.

Section - D
Section D consists of 4 questions of 5 marks each.

32. Determine graphically the coordinates of the vertices of triangle, the equations of whose sides are given by
2y − x = 8 , 5y − x = 14 and y − 2x = 1.
 O
Aftab tells his daughter, ‘7 years ago, I was seven times as old as you were then. Also, 3 years from now, I shall
be three times as old as you will be.’ Represent this situation algebraically and graphically.

33. Show that the points ^a, a h , ^- a, - a h and `- 3 a, 3 a j are the vertices of an equilateral triangle.

Install NODIA App to See the Solutions.


Click Here To Install
Page 5 Sample Paper 06 CBSE Mathematics Class 10

34. In the given figure, DEFG is a square and +BAC = 90c. Show that FG2 = BG # FC .

 O
In Figure, if TABC + TDEF and their sides of lengths (in cm) are marked along them, then find the lengths of
sides of each triangle.

35. A toy is in the form of a cylinder of diameter 2 2 m and height 3.5 m surmounted by a cone whose vertical angle
is 90c. Find total surface area of the toy.

Section - E
Case study based questions are compulsory.

36. John and Priya went for a small picnic. After having their lunch Priya insisted to travel in a motor boat. The speed
of the motor boat was 20 km/hr. Priya being a Mathematics student wanted to know the speed of the current. So
she noted the time for upstream and downstream.

She found that for covering the distance of 15 km the boat took 1 hour more for upstream than downstream.
(i) Let speed of the current be x km/hr. What will be the speed of the motorboat in upstream ? What is the
relation between speed distance and time?
(ii) Write the correct quadratic equation for the speed of the current ?
(iii) What is the speed of current ? How much time boat took in downstream ?

Install NODIA App to See the Solutions.


Click Here To Install
Page 6 Sample Paper 06 CBSE Mathematics Class 10

37. Height of a Climber : Himalayan Trekking Club has just hiked to the south rim of a large canyon, when they spot a
climber attempting to scale the taller northern face. Knowing the distance between the sheer walls of the northern
and southern faces of the canyon is approximately 150 meter, they attempt to compute the distance remaining for
the climbers to reach the top of the northern rim.

Using a homemade transit, they sight an angle of depression of 60c to the bottom of the north face, and angles of
elevation of 30c and 45c to the climbers and top of the northern rim respectively.
(i) How high is the southern rim of the canyon?
(ii) How high is the northern rim?
(iii) How much farther until the climber reaches the top?

38. Electric scooters are plug-in electric vehicles with two or three wheels. The electricity is stored on board in a
rechargeable battery, which drives one or more electric motors. Leading manufacturer of electric scooter, Hero
Scooter Pvt Ltd wants to declare the mileage of their electric scooters. For this, they recorded the mileage (km/
charge) of 50 scooters of the same model. Details of which are given in the following table.

Mileage (km/charge) 100-120 120-140 140-160 160-180

Number of scooters 7 12 18 13

Based on the above information, answer the following questions.


(i) What is the average mileage.
(ii) What is the modal value of mileage ?
(iii) What is the median value of mileage ?
(iv) What about the mileage can be claimed by the manufacturer for his scooter ?

 ******

Install NODIA App to See the Solutions.


Click Here To Install
Page 1 Sample Paper 07 CBSE Mathematics Class 10

Sample Paper 07
Class- X Exam - 2023-24
Mathematics - Standard

Time Allowed: 3 Hours Maximum Marks : 80


General Instructions :
1. This Question Paper has 5 Sections A-E.
2. Section A has 20 MCQs carrying 1 mark each
3. Section B has 5 questions carrying 02 marks each.
4. Section C has 6 questions carrying 03 marks each.
5. Section D has 4 questions carrying 05 marks each.
6. Section E has 3 case based integrated units of assessment (04 marks each) with sub-parts.
7. All Questions are compulsory. However, an internal choice in 2 Qs of 5 marks, 2 Qs of 3 marks and 2 Questions
of 2 marks has been provided.
8. Draw neat figures wherever required. Take π = 227 wherever required if not stated.

Section - A
Section A consists of 20 questions of 1 mark each.

1. An observer, 1.5 m tall is 20.5 away from a tower 22 m high, then the angle of elevation of the top of the tower
from the eye of observer is
(a) 30c (b) 45c (c) 60c (d) 90c

2. What is the ratio in which the point P ^ 34 , 125 h divides the line segment joining the point A ^ 12 , 32 h and ^2, - 5h
(a) 4 : 7 (b) 3 : 7 (c) 1 : 5 (d) 2 : 5

3. If HCF(336, 54) = 6 , LCM(336, 54) will be


(a) 2024 (b) 3024 (c) 1012 (d) 1512

4. Consider the following distribution :

Marks obtained Number of students


More than or equal to 0 63
More than or equal to 10 58
More than or equal to 20 55
More than or equal to 30 51
More than or equal to 40 48
More than or equal to 50 42
the frequency of the class 30-40 is :
(a) 3 (b) 4 (c) 48 (d) 51

5. Zeroes of f ^x h = x2 − 2x are
(a) 2 and 4 (b) 1 and 3 (c) 0 and 2 (d) 0 and 4

Install NODIA App to See the Solutions.


Click Here To Install
Page 2 Sample Paper 07 CBSE Mathematics Class 10

6. Harpreet tosses two different coins simultaneously. What is the probability that she gets at least one head ?
(a) 14 (b) 24 (c) 34 (d) 1

7. For what value of p will the following system of equations have no solution ?
^2p − 1h x + ^p − 1h y = 2p + 1; y + 3x − 1 = 0
(a) p=2 (b) p ! 2 (c) p = 4 (d) p ! 4

8. The quadratic equation x2 + 3x + 2 2 = 0 has


(a) two distinct real roots (b) two equal real roots
(c) no real roots (d) more than 2 real roots

9. A right circular cylinder of radius r and height h (where, h > 2r ) just encloses a sphere of diameter
(a) r (b) 2r (c) h (d) 2h

10. If the common difference of an AP is 5, then what is a18 - a13 ?


(a) 5 (b) 20 (c) 25 (d) 30

11. For what value of k , the system of equations kx + 3y = 1, 12x + ky = 2 has no solution.
(a) k =− 6 (b) k !- 6 (c) k = 4 (d) k = − 4

12. If TABC + TPQR , AB = 1 , then arTABC will be


PQ 3 arTPQR
1 1 8 5
(a) 3 (b) 9 (c) 9 (d) 9

13. If A and B are acute angles and sin A = cos B, then the value of A + B is
(a) 60° (b) 180° (c) 90° (d) 120°

14. 1 + 1 =?
1 + sin θ 1 − sin θ
(a) 1 (b) 2 sec2 θ (c) 2 sin2 θ (d) 2 cos2 θ

15. Which of the following equations has the sum of its roots as 3 ?
(a) 2x2 − 3x + 6 = 0 (b) − x2 + 3x − 3 = 0 (c) 2 x2 − 3 x + 1 = 0 (d) 3x2 − 3x + 3 = 0
2
16. In figure, if +AOB = 125c, then +COD is equal to

(a) 62.5c (b) 45c (c) 35c (d) 55c

17. If the perimeter of a circle is equal to that of a square, then the ratio of their areas is
(a) 22: 7 (b) 14: 11 (c) 7: 22 (d) 11: 14

Install NODIA App to See the Solutions.


Click Here To Install
Page 3 Sample Paper 07 CBSE Mathematics Class 10

21 5
18. Select the quadratic polynomial whose sum and product of a the zeroes are 8 and 16 respectively
(a) 16x2 − 42x + 5 (b) 1
16 ^16x2 − 42x + 5h (c) 1
12 ^16x2 + 42x + 5h (d) 1
12 ^16x2 + 42x − 5h

In the question number 19 and 20, a statement of Assertion (A) is followed by a statement of Reason (R). Choose the
correction option.

19. Assertion : x + y − 4 = 0 and 2x + ky − 3 = 0 has no solution if k = 2 .


Reason : a1 x + b1 y + c1 = 0 and a2 x + b2 y + c2 = 0 are consistent if a1 ! k1 .
a2 k2
(a) Both assertion (A) and reason (R) are true and reason (R) is the correct explanation of assertion (A).
(b) Both assertion (A) and reason (R) are true but reason (R) is not the correct explanation of assertion (A).
(c) Assertion (A) is true but reason (R) is false.
(d) Assertion (A) is false but reason (R) is true.

20. Assertion : If sum of the first n terms of an AP is given by Sn = 3n2 − 4n. Then its n th term is an = 6n − 7 .
Reason : n th term of an AP, whose sum to n terms is Sn , is given by an = Sn − Sn − 1
(a) Both assertion (A) and reason (R) are true and reason (R) is the correct explanation of assertion (A).
(b) Both assertion (A) and reason (R) are true but reason (R) is not the correct explanation of assertion (A).
(c) Assertion (A) is true but reason (R) is false.
(d) Assertion (A) is false but reason (R) is true.

Section - B
Section B consists of 5 questions of 2 marks each.

21. Prove that 3 + 5 is an irrational number.

22. If Sn denotes the sum of n terms of an AP whose common difference is d and first term is a , find Sn − 2Sn − 1 + Sn − 2 .
 O
Find the sum of first 15 multiples of 8.

23. The ordinate of a point A on y-axis is 5 and B has co-ordinates ^- 3, 1h . Find the length of AB.

24. Two circles of radii 20 cm and 37 cm intersect in A and B . If O1 and O2 are their centres and AB = 24 cm, then
find the distance O1 O2 .
 O
In the figure, QR is a common tangent to given circle which meet at T . Tangent at T meets QR at P . If QP = 3.8
cm, then find length of QR .

Install NODIA App to See the Solutions.


Click Here To Install
Page 4 Sample Paper 07 CBSE Mathematics Class 10

25. One card is drawn from a well shuffled deck of 52 cards. Find the probability of getting :
(i) a non face card,
(ii) a black king.

Section - C
Section C consists of 6 questions of 3 marks each.

26. State Fundamental theorem of Arithmetic. Find LCM of numbers 2520 and 10530 by prime factorization.

27. In Figure +D = +E and AD = AE , prove that TBAC is an isosceles triangle.


DB EC

28. Prove that ^sin θ + cosec θh2 + ^cos θ + sec θh2 = 7 + tan2 θ + cot2 θ

29. If O is centre of a circle, PQ is a chord and the tangent PR at P makes an angle of 50c with PQ , find +POQ .

30. Find the area of minor segment of a circle of radius 14 cm, when its centre angle is 60c. Also find the area of
corresponding major segment. Use π = 227 .
 O
In the given figure, find the area of the shaded region, enclosed between two concentric circles of radii 7 cm and
14 cm where +AOC = 40c. Use π = 227 .

Install NODIA App to See the Solutions.


Click Here To Install
Page 5 Sample Paper 07 CBSE Mathematics Class 10

31. Two different dice are tossed together. Find the probability:
(i) of getting a doublet
(ii) of getting a sum 10, of the numbers on the two dice.

 O

An integer is chosen at random between 1 and 100. Find the probability that it is:
(i) divisible by 8.
(ii) not divisible by 8.

Section - D
Section D consists of 4 questions of 5 marks each.

32. Solve the following pair of linear equations graphically:


x - y = 1, 2x + y = 8
Also find the co-ordinates of the points where the lines represented by the above equation intersect y - axis.

 O

Find the value of p and q for which the system of equations represent coincident lines 2x + 3y = 7 ,
^p + q + 1h x + ^p + 2q + 2h y = 4 ^p + q h + 1

33. If P ^2, - 1h, Q ^3, 4h, R ^- 2, 3h and S ^- 3, - 2h be four points in a plane, show that PQRS is a rhombus but not
a square.

34. In given figure +1 = +2 and TNSQ ~TMTR, then prove that TPTS ~TPRO.

 O

Prove that in a right triangle, the square of the hypotenuse is equal to sum of squares of other two sides. Using
the above result, prove that, in rhombus ABCD, 4AB2 = AC2 + BD2 .

35. A solid toy is in the form of a hemisphere surmounted by a right circular cone. The height of the cone is 2 cm and
the diameter of the base is 4 cm. Determine the volume of the toy. If a right circular cylinder circumscribes the
toy, find the difference of the volume of the cylinder and toy. (Use π = 3.14 )

Install NODIA App to See the Solutions.


Click Here To Install
Page 6 Sample Paper 07 CBSE Mathematics Class 10

Section - E
Case study based questions are compulsory.

36. Raju and his classmates planned a picnic in zoo. The total budget for picnic was Rs 2000 but 5 students failed to
attend the picnic and thus the contribution for each student was increased by Rs 20.

The expanse of different item was as follows.

S. No. Article Cost per student


1 Entry ticket Rs 5
2 Coffee Rs 10
3 Food Rs 25
4 Travelling cost Rs 50
5 Ice-cream Rs 15
(i) If x is the number of students planned for picnic, find the quadratic equation that describe the situation.
(ii) What is the number of students planned for picnic ?
(iii) What is the number of students who attended the picnic? What is the total expanse for this picnic ?
(iv) How much money they spent for travelling ?

37. Water Tower : A water tower is a building that is used to hold and give out water. It is almost always built on a
high place. It works because a pump gives water to the tower, and gravity makes the saved water go out to the
places that need water. Those places are connected to the tower by pipes. A water tower is good when there is no
power because it uses gravity to send out the water.

A water tower is located 60 meter from a building (see the figure). From a window in the building, an observer
notes that the angle of elevation to the top of the tower is 60c and that the angle of depression to the bottom of
the tower is 30c .
(i) How tall is the tower?
(ii) How high is the window?

Install NODIA App to See the Solutions.


Click Here To Install
Page 7 Sample Paper 07 CBSE Mathematics Class 10

38. The Kendriya Vidyalaya Sangathan is a system of premier central government schools in India that are instituted
under the aegis of the Ministry of Education (MHRD), Government of India. As of October 2020, it has a total of
1239 schools. It is one of the world’s largest chains of schools. The system came into being in 1963 under the name
‘Central Schools’. Later, the name was changed to Kendriya Vidyalaya. Its schools are all affiliated to the Central
Board of Secondary Education (CBSE). The objective of KVS is to cater to the educational needs of the children
of transferable Central Government employees including Defence and Para-Military personnel by providing a
common programme of education.

Commissioner of Regional office Jaipur prepare a table of the marks obtained of 100 students which is given below

Marks obtained 0-20 20-40 40-60 60-80 80-100


Number of students 15 18 21 29 p
He was told that mean marks of a student is 53.
(i) How many students got marks between 80-100?
(ii) What is the lower limit of model class ? What is the value of model marks ?
(iii) What is the value of median marks ? What is the upper limit of median class ?

 ******

Install NODIA App to See the Solutions.


Click Here To Install
Page 1 Sample Paper 08 CBSE Mathematics Class 10

Sample Paper 08
Class- X Exam - 2023-24
Mathematics - Standard

Time Allowed: 3 Hours Maximum Marks : 80


General Instructions :
1. This Question Paper has 5 Sections A-E.
2. Section A has 20 MCQs carrying 1 mark each
3. Section B has 5 questions carrying 02 marks each.
4. Section C has 6 questions carrying 03 marks each.
5. Section D has 4 questions carrying 05 marks each.
6. Section E has 3 case based integrated units of assessment (04 marks each) with sub-parts.
7. All Questions are compulsory. However, an internal choice in 2 Qs of 5 marks, 2 Qs of 3 marks and 2 Questions
of 2 marks has been provided.
8. Draw neat figures wherever required. Take π = 227 wherever required if not stated.

Section - A
Section A consists of 20 questions of 1 mark each.

1. If x = a and y = b is the solution of the equations x − y = 2 and x + y = 4 , then the values of a and b are,
respectively
(a) 3 and 5 (b) 5 and 3
(c) 3 and 1 (d) - 1 and - 3

2. In the given figure, a circle touches all the four sides of quadrilateral ABCD with AB = 6 cm , BC = 7 cm and
CD = 4 cm , then length of AD is

(a) 3 cm (b) 4 cm
(c) 5 cm (d) 6 cm

3. The quadratic equation 5x2 − 3x + 1 = 0 has


(a) two distinct real roots
(b) two equal real roots
(c) no real roots
(d) more than 2 real roots

Install NODIA App to See the Solutions.


Click Here To Install
Page 2 Sample Paper 08 CBSE Mathematics Class 10

4. The area of a right angled triangle is 40 sq cm and its perimeter is 40 cm. The length of its hypotenuse is
(a) 16 cm (b) 18 cm
(c) 17 cm (d) data insufficient

5. In the figure, PQ is parallel to MN. If KP = 4 and KN = 20.4 cm then find KQ.


PM 13

(a) 4.8 cm (b) 4.6 cm


(c) 4.4 cm (d) 4.2 cm

6. A student noted the number of cars passing through a spot on a road for 100 periods each of 3 min and summarised
in the table give below.

Number of cars Frequency


0-10 7
10-20 14
20-30 13
30-40 12
40-50 20
50-60 11
60-70 15
70-80 08
Then, the mode of the data is
(a) 34.7 (b) 44.7
(c) 54.7 (d) 64.7

7. The zeroes of the quadratic polynomial x2 + 99x + 127 are


(a) both positive
(b) both negative
(c) one positive and one negative
(d) both equal

Install NODIA App to See the Solutions.


Click Here To Install
Page 3 Sample Paper 08 CBSE Mathematics Class 10

8. In figure, if +AOB = 125c, then +COD is equal to

(a) 62.5c (b) 45c


(c) 35c (d) 55c

9. In figure, on a circle of radius 7 cm, tangent PT is drawn from a point P such that PT = 24 cm. If O is the centre
of the circle, then the length of PR is

(a) 30 cm (b) 28 cm
(c) 32 cm (d) 25 cm

10. If cos A = 4 , then the value of tan A is


5
(a) 3 (b) 3
5 4

(c) 4 (d) 5
3 3

11. From a point on the ground, the angles of elevation of the bottom and the top of a transmission tower fixed at the
top of a 20 m high building are 45c and 60c respectively, then the height of the tower is
(a) 14.64 m (b) 28.64 m
(c) 38.64 m (d) 19.64 m

12. If the sum of the areas of two circles with radii R1 and R2 is equal to the area of a circle of radius R , then
(a) R1 + R2 = R (b) R12 + R22 = R2
(c) R1 + R2 < R (d) R12 + R22 < R2

13. A solid piece of iron in the form of a cuboid of dimensions 49 cm # 33 cm # 24 cm , is moulded to form a solid
sphere. The radius of the sphere is
(a) 21 cm (b) 23 cm
(c) 25 cm (d) 19 cm

Install NODIA App to See the Solutions.


Click Here To Install
Page 4 Sample Paper 08 CBSE Mathematics Class 10

14. If the mean of the squares of first n natural numbers is 105, then the first n natural numbers is
(a) 8 (b) 9
(c) 10 (d) 11

15. The P ^Ah denotes the probability of an event A, then


(a) P ^Ah < 0 (b) P ^Ah > 1
(c) 0 # P ^Ah # 1 (d) - 1 # P ^Ah # 1

16. A tree is broken by the wind. The top struck the ground at an angle of 30c and at distance of 10 m from its root.
The whole height of the tree is ( 3 = 1.732)
(a) 10 3 m (b) 3 10 m
(c) 20 3 m (d) 3 20 m

17. A fair die is thrown once. The probability of getting a composite number less than 5 is
(a) 1 (b) 1
3 6

(c) 2 (d) 0
3

18. If α and β are the zeroes of the polynomial 2x2 − 13x + 6 , then α + β is equal to
(a) - 3 (b) 3
(c) 13 (d) - 13
2 2

In the question number 19 and 20, a statement of Assertion (A) is followed by a statement of Reason (R). Choose the
correction option.

19. Assertion : If sum of the first n terms of an AP is given by Sn = 3n2 − 4n. Then its n th term is an = 6n − 7 .
Reason : n th term of an AP, whose sum to n terms is Sn , is given by an = Sn − Sn − 1
(a) Both assertion (A) and reason (R) are true and reason (R) is the correct explanation of assertion (A).
(b) Both assertion (A) and reason (R) are true but reason (R) is not the correct explanation of assertion (A).
(c) Assertion (A) is true but reason (R) is false.
(d) Assertion (A) is false but reason (R) is true.

20. Assertion : sin2 67c + cos2 67c = 1


Reason : For any value of θ, sin2 θ + cos2 θ = 1
(a) Both assertion (A) and reason (R) are true and reason (R) is the correct explanation of assertion (A).
(b) Both assertion (A) and reason (R) are true but reason (R) is not the correct explanation of assertion (A).
(c) Assertion (A) is true but reason (R) is false.
(d) Assertion (A) is false but reason (R) is true.

Install NODIA App to See the Solutions.


Click Here To Install
Page 5 Sample Paper 08 CBSE Mathematics Class 10

Section - B
Section B consists of 5 questions of 2 marks each.

21. If the sum of first n terms of an AP is n2 , then find its 10th term.

22. Two tangents PA and PB are drawn from an external point P to a circle inclined to each other at an angle of
70º, then what is the value of +PAB ?

23. Find the ratio in which the point P ^ 34 , 125 h divides the line segment joining the point A ^ 12 , 32 h and ^2, - 5h .

24. If two positive integers p and q are written as p = a2 b3 and q = a3 b, where a and b are prime numbers than verify
LCM (p, q) # HCF (q, q) = pq
 O
Prove that 3 + 5 is an irrational number.

25. The fifth term of an AP is 20 and the sum of its seventh and eleventh terms is 64. Find the common difference.
 O
For AP show that a p + a p + 2q = 2a p + q .

Section - C
Section C consists of 6 questions of 3 marks each.

26. Solve for x and y :


x + 2y =− 1
2 3
y
x- =3
3

27. Solve for x :


1 1 = 2 ; x ! 1 , 2, 3
^x − 1h^x − 2h ^x − 2h^x − 3h 3
+

28. In given figure, two circles touch each other at the point C . Prove that the common tangent to the circles at C ,
bisects the common tangent at P and Q .

Install NODIA App to See the Solutions.


Click Here To Install
Page 6 Sample Paper 08 CBSE Mathematics Class 10

 O
In the given figure, PA and PB are tangents to a circle from an external point P such that PA = 4 cm and +BAC
= 135º . Find the length of chord AB .

29. A 7 m long flagstaff is fixed on the top of a tower standing on the horizontal plane. From point on the ground,
the angles of elevation of the top and bottom of the flagstaff are 60º and 45º respectively. Find the height of the
tower correct to one place of decimal.(Use 3 = 1.73)

30. Water in a canal, 6 m wide and 1.5 m deep, is flowing with a speed of 10 km/hour. How much area will it irrigate
in 30 minutes; if 8 cm standing water is needed?
 O
The radii of two right circular cylinders are in the ratio of 2 | 3 and their height are in the ratio of 5 | 4 .
Calculate the ratio of their curved surface area and radio of their volumes.

31. A die is thrown once. Find the probability of getting a number which (i) is a prime number (ii) lies between 2
and 6.

Section - D
Section D consists of 4 questions of 5 marks each.

32. If α and β are zeroes of the polynomial p (x) = 6x2 − 5x + k such that α − β = 16 , Find the value of k.
 O
Polynomial x + 7x + 7x + px + q is exactly divisible by x2 + 7x + 12 , then find the value of p and q .
4 3 2

33. Prove that in a right triangle, the square of the hypotenuse is equal to sum of squares of other two sides.
 O
Prove that in a right triangle, the square of the hypotenuse is equal to sum of squares of other two sides. Using
the above result, prove that, in rhombus ABCD, 4AB2 = AC2 + BD2 .

34. Given that tan ^A + B h = tan A + tan B , find the values of tan 75º and tan 90º by taking suitable values of A
1 − tan A tan B
and B.
 O
Prove that : tan θ + cot θ = 1 + tan θ + cot θ.
1 − cot θ 1 − tan θ

Install NODIA App to See the Solutions.


Click Here To Install
Page 7 Sample Paper 08 CBSE Mathematics Class 10

35. In fig., two circular flower beds have been shown on two sides of a square lawn ABCD of side 56 m. If the centre
of each circular flower bed is the point of intersection O of the diagonals of the square lawn, find the sum of the
areas of the lawn and flower beds.

Section - E
Case study based questions are compulsory.

36. Traffic Management : A traffic enforcement camera is a camera which may be mounted beside or over a road or
installed in an enforcement vehicle to detect motoring offenses, including speeding, vehicles going through a red
traffic light. A worldwide review of studies found that speed cameras led to a reduction of 11% to 44% for fatal
and serious injury crashes. The British Medical Journal recently reported that speed cameras were effective at
reducing accidents and injuries in their vicinity and recommended wider deployment.

In order to monitor reckless driving on Mumbai road, special cameras have been installed at many traffic light.
The following table shows a frequency distribution table for the speed of 100 vehicles passing through a particular
spot on a day.

Speed (in km/h) 20-30 30-40 40-50 50-60 60-70 70-80 80-90 90-100
Number of Vehicles 1 3 7 16 35 29 7 2
Based on the above information, answer the following questions.
(i) Find the number of vehicles whose speed is more than 70 km/h and find the number of vehicles whose speed
is less than 50 km/h ?
(ii) What is the mode value of speed ?
(iii) What is the median value of speed ?

Continue on next page....

Install NODIA App to See the Solutions.


Click Here To Install
Page 8 Sample Paper 08 CBSE Mathematics Class 10

 O
(iv) Find the mean value of speed using empirical relation.

37. A garden is in the shape of rectangle. Gardener grew sapling of Ashoka tree on the boundary of garden at the
distance of 1 meter from each other. He want to decorate the garden with rose plants. He choose triangular region
inside the park to grow rose plants. On the above situation, gardener took help from the students of class 10th.
They made a chart for it which looks as the above figure.

(i) If A is taken as origin, What are the coordinates of triangle PQR ?


(ii) If C is taken as origin, what is the co-ordinate of point P ?
(iii) If B is taken as origin, what are the co-ordinate of P ?
(iv) What is distance between P and Q if origin is taken A?

38. Pyramid, in architecture, a monumental structure constructed of or faced with stone or brick and having a
rectangular base and four sloping triangular sides meeting at an apex. Pyramids have been built at various times
in Egypt, Sudan, Ethiopia, western Asia, Greece, Cyprus, Italy, India, Thailand, Mexico, South America, and on
some islands of the Pacific Ocean. Those of Egypt and of Central and South America are the best known.

Continue on next page....

Install NODIA App to See the Solutions.


Click Here To Install
Page 9 Sample Paper 08 CBSE Mathematics Class 10

The volume and surface area of a pyramid with a square base of area a2 and height h is given by
ha2
V = 3 and S = a 2 + 2a ^ a2 h + h2
2

A pyramid has a square base and a volume of 3y 3 + 18y2 + 27y cubic units.
(i) If its height is y , then what polynomial represents the length of a side of the square base ?
(ii) If area of base is 576 metre, what is the side of base?
(iii) What is the height of pyramid at above area of base ? What is the ratio of length of side to the height ?

 O

(iv) What is surface area of pyramid ?

 ******

Install NODIA App to See the Solutions.


Click Here To Install
Page 1 Sample Paper 9 Mathematics STD Class 10

Sample Paper 9
Class- X Exam - 2023-24
Mathematics - Standard

Time Allowed: 3 Hours Maximum Marks : 80


General Instructions :
1. This Question Paper has 5 Sections A-E.
2. Section A has 20 MCQs carrying 1 mark each.
3. Section B has 5 questions carrying 02 marks each.
4. Section C has 6 questions carrying 03 marks each.
5. Section D has 4 questions carrying 05 marks each.
6. Section E has 3 case based integrated units of assessment (04 marks each) with sub-parts of the values of 1, 1
and 2 marks each respectively.
7. All Questions are compulsory. However, an internal choice in 2 Qs of 5 marks, 2 Qs of 3 marks and 2 Questions
of 2 marks has been provided. An internal choice has been provided in the 2 marks questions of Section E.
8. Draw neat figures wherever required. Take π = 227 wherever required if not stated.

Section - A
Section A consists of 20 questions of 1 mark each.

1. The quadratic polynomial, the sum of whose zeroes is - 5 and their product is 6, is
(a) x2 + 5x + 6 (b) x2 − 5x + 6 (c) x2 - 5x - 6 (d) − x2 + 5x + 6

2. If the point P (k, 0) divides the line segment joining the points A (2, - 2) and B (- 7, 4) in the ratio 1 : 2, then
the value of k is
(a) 1 (b) 2 (c) - 2 (d) - 1

3. In a number of two digits, unit’s digit is twice the tens digit. If 36 be added to the number, the digits are reversed.
The number is
(a) 36 (b) 63 (c) 48 (d) 84

4. In the given figure, x is

(a) ab (b) ac
a+b b+c
(c) bc (d) ac
b+c a+c

https://qrbook.page.link/app
Install NODIA App to See the Solutions.
Click Here To Install
Page 2 Sample Paper 9 Mathematics STD Class 10

5. x and y are 2 different digits. If the sum of the two digit numbers formed by using both the digits is a perfect
square, then value of x + y is
(a) 10 (b) 11
(c) 12 (d) 13

6. If ½ is a root of the equation x2 + kx − 54 = 0 , then the value of k is


(a) 2 (b) - 2
(c) 1 (d) 1
4 2

7. If α and β are the zeroes of the polynomial x2 + 2x + 1, then α1 + 1β is equal to


(a) - 2 (b) 2
(c) 0 (d) 1

8. The real roots of the equation x2/3 + x1/3 − 2 = 0 are


(a) 1, 8 (b) - 1, - 8
(c) - 1, 8 (d) 1, - 8

9. Which term of an AP, 21, 42, 63, 84, ... is 210?


(a) 9th (b) 10th
(c) 11th (d) 12th

10. If the height and length of the shadow of a man are equal, then the angle of elevation of the sun is,
(a) 45c (b) 60c
(c) 90c (d) 120c

11. The area of a circular ring formed by two concentric circles whose radii are 5.7 cm and 4.3 cm respectively is (Take
π = 3.1416 )
(a) 44 sq. cm. (b) 66 sq. cm.
(c) 22 sq. cm. (d) 33 sq. cm.

12. A sphere is melted and half of the melted liquid is used to form 11 identical cubes, whereas the remaining half is
used to form 7 identical smaller spheres. The ratio of the side of the cube to the radius of the new small sphere is
(a) b 3 l (b) b 3 l
4 1/3 8 1/3

(c) (3) 1/3 (d) 2

13. The P ^Ah denotes the probability of an event A, then


(a) P ^Ah < 0 (b) P ^Ah > 1
(c) 0 # P ^Ah # 1 (d) - 1 # P ^Ah # 1

14. If sin θ = a , then cos θ is equal to


b
(a) b (b) b
b - a2
2 a

(c) b2 - a2 (d) a
b b2 - a2

https://qrbook.page.link/app
Install NODIA App to See the Solutions.
Click Here To Install
Page 3 Sample Paper 9 Mathematics STD Class 10

15. In figure, AP , AQ and BC are tangents of the circle with centre O . If AB = 5 cm , AC = 6 cm and BC = 4 cm,
then the length of AP (in cm) is

(a) 15 (b) 10
(c) 9 (d) 7.5

16. The cumulative frequency table is useful in determining


(a) Mean (b) Median
(c) Mode (d) All of these

17. The ratio in which the point (2, y) divides the join of (- 4, 3) and (6, 3), hence the value of y is
(a) 2 : 3, y = 3 (b) 3 : 2, y = 4
(c) 3 : 2, y = 3 (d) 3 : 2, y = 2

18. If the points A (4, 3) and B (x, 5) are on the circle with centre O (2, 3), then the value of x is
(a) 0 (b) 1
(c) 2 (d) 3

In the question number 19 and 20, a statement of Assertion (A) is followed by a statement of Reason (R). Choose the
correction option.

19. Assertion : 34.12345 is a terminating decimal fraction.


Reason : Denominator of 34.12345, when expressed in the form qp , q ! 0 , is of the form 2m # 5n , where m and
n are non-negative integers.
(a) Both assertion (A) and reason (R) are true and reason (R) is the correct explanation of assertion (A).
(b) Both assertion (A) and reason (R) are true but reason (R) is not the correct explanation of assertion (A).
(c) Assertion (A) is true but reason (R) is false.
(d) Assertion (A) is false but reason (R) is true.

20. Assertion : Common difference of the AP - 5 , - 1 , 3, 7, .......... is 4.


Reason : Common difference of the AP a, a + d, a + 2d, .......... is given by d = a2 − a1
(a) Both assertion (A) and reason (R) are true and reason (R) is the correct explanation of assertion (A).
(b) Both assertion (A) and reason (R) are true but reason (R) is not the correct explanation of assertion (A).
(c) Assertion (A) is true but reason (R) is false.
(d) Assertion (A) is false but reason (R) is true.

https://qrbook.page.link/app
Install NODIA App to See the Solutions.
Click Here To Install
Page 4 Sample Paper 9 Mathematics STD Class 10

Section - B
Section B consists of 5 questions of 2 marks each.

21. In an equilateral triangle of side 24 cm, find the length of the altitude.

22. In the given figure, from a point P , two tangents PT and PS are drawn to a circle with centre O such that
+SPT = 120c, Prove that OP = 2PS .

23. Evaluate :
3 tan2 30º + tan2 60º + cosec 30º − tan 45º
cot2 45º

24. The mode of the following frequency distribution is 36. Find the missing frequency f .

Class 0- 10 10-20 20-30 30-40 40-50 50-60 60-70


Frequency 8 10 f 16 12 6 7

 O
Find the median for the given frequency distribution :

Class 40-45 45-50 50-55 55-60 60-65 65-70 70-75


Frequency 2 3 8 6 6 3 2

25. Given that HCF (306, 1314) = 18. Find LCM (306, 1314)
 O
n
Check whether 4 can end with the digit 0 for any natural number n .

Section - C
Section C consists of 6 questions of 3 marks each.

26. The sum of the first 7 terms of an AP is 63 and that of its next 7 terms is 161. Find the AP.

27. If sin θ + cos θ = 2 prove that tan θ + cot θ = 2

28. Three horses are tied each with 7 m long rope at three corners of a triangular field having sides 20 m, 34 m and
42 m. Find the area of the plot which can be grazed by the horses.

https://qrbook.page.link/app Install NODIA App to See the Solutions.


Click Here To Install
Page 5 Sample Paper 9 Mathematics STD Class 10

 O
In the given figure, AOB is a sector of angle 60c of a circle with centre O and radius 17 cm. If AP = OB and
AP = 15 cm, find the area of the shaded region.

29. The mean of the following frequency distribution is 18. The frequency f in the class interval 19-21 is missing.
Determine f .

Class interval 11-13 13-15 15-17 17-19 19-21 21-23 23-25


Frequency 3 6 9 13 f 5 4

30. If the point C (- 1, 2) divides internally the line segment joining A (2, 5) and B (x, y) in the ratio 3 :4 find the
coordinates of B .
 O
Prove that the diagonals of a rectangle ABCD, with vertices A ^2, - 1h, B ^5, - 1h, C ^5, 6h and D ^2, 6h are equal
and bisect each other.

31. Given that 2 is irrational, prove that (5 + 3 2 ) is an irrational number.

Section - D
Section D consists of 4 questions of 5 marks each.

32. Find c if the system of equations cx + 3y + ^3 − c h = 0; 12x + cy − c = 0 has infinitely many solutions?
 O
Solve for x and y :
2x − y + 3 = 0
3x − 5y + 1 = 0

33. Prove that tangent drawn at any point of a circle perpendicular to the radius through the point contact.

34. The angle of elevation of an aeroplane from a point on the ground is 60º. After a flight of 30 seconds the angle of
elevation becomes 30º. If the aeroplane is flying at a constant height of 3000 3 m, find the speed of the aeroplane.
 O
Amit, standing on a horizontal plane, find a bird flying at a distance of 200 m from him at an elevation of 30c.
Deepak standing on the roof of a 50 m high building, find the angle of elevation of the same bird to be 45c. Amit
and Deepak are on opposite sides of the bird. Find the distance of the bird from Deepak.

https://qrbook.page.link/app
Install NODIA App to See the Solutions.
Click Here To Install
Page 6 Sample Paper 9 Mathematics STD Class 10

35. The weight of two spheres of same metal are 1 kg and 7 kg. The radius of the smaller sphere is 3 cm. The two
spheres are melted to form a single big sphere. Find the diameter of the new sphere.

Section - E
Case study based questions are compulsory.

36. Model Rocketry : A model rocket is a small rocket designed to reach low altitudes and be recovered by a variety
of means. Flying model rockets is a relatively safe and inexpensive way for person to learn the basics of forces and
the response of a vehicle to external forces. Like an airplane, a model rocket is subjected to the forces of weight,
thrust, and aerodynamics during its flight.

Shalvi is a member of first rocket club of India named STAR Club. She launches her latest rocket from a large
field. At the moment its fuel is exhausted, the rocket has a velocity of 240 ft/sec and an altitude of 544 ft. After
t sec, its height h (t) above the ground is given by the function h (t) = − 16t2 + 240t + 544 .
(i) How high is the rocket 5 sec after the fuel is exhausted?
(ii) How high is the rocket 10 sec after the fuel is exhausted?
(iii) What is the maximum height attained by the rocket?
 O
How many seconds was the rocket airborne after its fuel was exhausted?

37. The law of reflection states that when a ray of light reflects off a surface, the angle of incidence is equal to the
angle of reflection.

https://qrbook.page.link/app
Install NODIA App to See the Solutions.
Click Here To Install
Page 7 Sample Paper 9 Mathematics STD Class 10

Ramesh places a mirror on level ground to determine the height of a pole (with traffic light fired on it). He stands
at a certain distance so that he can see the top of the pole reflected from the mirror. Ramesh’s eye level is 1.5 m
above the ground. The distance of Ramesh and the pole from the mirror are 1.8 m and 6 m respectively.

(i) Which criterion of similarity is applicable to similar triangles?


(ii) What is the height of the pole?
(iii) If angle of incidence is i , find tan i .
 O
Now Ramesh move behind such that distance between pole and Ramesh is 13 meters. He place mirror
between him and pole to see the reflection of light in right position. What is the distance between mirror
and Ramesh ?

38. Eight Ball : This is a game played on a pool table with 15 balls numbered 1 through 15 and a cue ball that is solid
white. Of the 15 numbered balls, 8 are a solid (nonwhite) color and numbered 1 through 8, and seven are striped
balls numbered 9 through 15.

The fifteen numbered pool balls (no cueball) are placed in a large bowl and mixed, then one is drawn out.
(i) What is the probability of drawing the eight ball ?
(ii) What is the probability of drawing a number greater than fifteen ?
(iii) What is the probability of drawing an even number ?
(iv) What is the probability of drawing a multiple of three ?
 O
What is the probability of drawing a solid color and an even number ?

 ******

https://qrbook.page.link/appInstall NODIA App to See the Solutions.


Click Here To Install
Page 1 Sample Paper 10 CBSE Mathematics Class 10

Sample Paper 10
Class- X Exam - 2023-24
Mathematics - Standard

Time Allowed: 3 Hours Maximum Marks : 80


General Instructions :
1. This Question Paper has 5 Sections A-E.
2. Section A has 20 MCQs carrying 1 mark each
3. Section B has 5 questions carrying 02 marks each.
4. Section C has 6 questions carrying 03 marks each.
5. Section D has 4 questions carrying 05 marks each.
6. Section E has 3 case based integrated units of assessment (04 marks each) with sub-parts.
7. All Questions are compulsory. However, an internal choice in 2 Qs of 5 marks, 2 Qs of 3 marks and 2 Questions
of 2 marks has been provided.
8. Draw neat figures wherever required. Take π = 227 wherever required if not stated.

Section - A
Section A consists of 20 questions of 1 mark each.

1. If α and β are the zeroes of the polynomial x2 + 2x + 1, then α1 + 1β is equal to


(a) - 2 (b) 2
(c) 0 (d) 1

2. The roots of the quadratic equation x2 − 0.04 = 0 are


(a) ! 0.2 (b) ! 0.02
(c) 0.4 (d) 2

3. In the given figure, PA is a tangent from an external point P to a circle with centre O . If +POB = 115c, then
perimeter of +APO is

(a) 25c (b) 20c


(c) 30c (d) 65c

4. In an AP, if d =− 4 , n = 7 and an = 4 , then a is equal to


(a) 6 (b) 7
(c) 20 (d) 28

Install NODIA App to See the Solutions.


Click Here To Install
Page 2 Sample Paper 10 CBSE Mathematics Class 10

5. A bag contains 3 red and 2 blue marbles. If a marble is drawn at random, then the probability of drawing a blue
marble is:
2 1
(a) 5 (b) 4

3 2
(c) 5 (d) 3

6. 225 can be expressed as


(a) 5 # 32 (b) 52 # 3
(c) 52 # 32 (d) 53 # 3

7. In Figure, DE || BC . Find the length of side AD , given that AE = 1.8 cm, BD = 7.2 cm and CE = 5.4 cm .

(a) 2.4 cm (b) 2.2 cm


(c) 3.2 cm (d) 3.4 cm

8. Consider the following distribution :

Marks obtained Number of students


More than or equal to 0 63
More than or equal to 10 58
More than or equal to 20 55
More than or equal to 30 51
More than or equal to 40 48
More than or equal to 50 42

the frequency of the class 30-40 is :


(a) 3 (b) 4
(c) 48 (d) 51

9. If cos 9a = sin a and 9α < 90c, then the value oftan 5α is

(a) 1 (b) 3
3
(c) 1 (d) 0

Install NODIA App to See the Solutions.


Click Here To Install
Page 3 Sample Paper 10 CBSE Mathematics Class 10

10. From the top of a 7 m high building the angle of elevation of the top of a cable tower is 60c and the angle of
depression of its foot is 45c, then the height of the tower is
(a) 14.124 m (b) 17.124 m
(c) 19.124 m (d) 15.124 m

11. A circus artist is climbing a 20 m long rope, which is tightly stretched and tied from the top of a vertical pole to
the ground. If the angle made by the rope with the ground level is 30c, then what is the height of pole?
(a) 20 m (b) 8 m
(c) 10 m (d) 6 m

12. The maximum number of zeroes a cubic polynomial can have, is


(a) 1 (b) 4
(c) 2 (d) 3

13. If triangle ABC is similar to triangle DEF such that 2AB = DE and BC = 8 cm then find EF.
(a) 16 cm (b) 14 cm
(c) 12 cm (d) 15 cm

14. A sphere is melted and half of the melted liquid is used to form 11 identical cubes, whereas the remaining half is
used to form 7 identical smaller spheres. The ratio of the side of the cube to the radius of the new small sphere is
(a) b 3 l (b) b 3 l
4 1/3 8 1/3

(c) (3) 1/3 (d) 2

15. Ratio of volumes of two cones with same radii is


(a) h1 : h2 (b) s1 : s2
(c) r1 : r2 (d) None of these

In the formula x = a + h f
/ f i ui , for finding the mean of grouped frequency distribution, u
/ fi p
16. i is equal to

(a) xi + a (b) h (xi - a)


h

(c) xi - a (d) a - xi
h h

17. If the probability of an event is p , then the probability of its complementary event will be
(a) p-1 (b) p
(c) 1-p (d) 1 - 1
p

18. The distance of the point P (- 3, - 4) from the x -axis (in units) is
(a) 3 (b) - 3
(c) 4 (d) 5

Install NODIA App to See the Solutions.


Click Here To Install
Page 4 Sample Paper 10 CBSE Mathematics Class 10

In the question number 19 and 20, a statement of Assertion (A) is followed by a statement of Reason (R). Choose the
correction option.

19. Assertion : Pair of linear equations : 9x + 3y + 12 = 0, 8x + 6y + 24 = 0 have infinitely many solutions.


Reason : Pair of linear equations a1 x + b1 y + c1 = 0 and a2 x + b2 y + c2 = 0 have infinitely many solutions, if
a1 = b1 = c1
a2 b2 c2
(a) Both assertion (A) and reason (R) are true and reason (R) is the correct explanation of assertion (A).
(b) Both assertion (A) and reason (R) are true but reason (R) is not the correct explanation of assertion (A).
(c) Assertion (A) is true but reason (R) is false.
(d) Assertion (A) is false but reason (R) is true.

20. Assertion : If the circumference of a circle is 176 cm, then its radius is 28 cm.
Reason : Circumference = 2π # radius
(a) Both assertion (A) and reason (R) are true and reason (R) is the correct explanation of assertion (A).
(b) Both assertion (A) and reason (R) are true but reason (R) is not the correct explanation of assertion (A).
(c) Assertion (A) is true but reason (R) is false.
(d) Assertion (A) is false but reason (R) is true.

Section - B
Section B consists of 5 questions of 2 marks each.

21. In the given figure, if ABCD is a trapezium in which AB || CD || EF, then prove that AE
ED = BF
FC

22. In the given figure, from a point P , two tangents PT and PS are drawn to a circle with centre O such that
+SPT = 120c, Prove that OP = 2PS .

Install NODIA App to See the Solutions.


Click Here To Install
Page 5 Sample Paper 10 CBSE Mathematics Class 10

23. If 3 sin θ − cos θ = 0 and 0º < θ < 90º, find the value of θ.

24. A box contains cards numbered 11 to 123. A card is drawn at random from the box. Find the probability that the
number of the drawn card is
(i) A perfect square number
(ii) A multiple of 7.
 O
A letter of English alphabet is chosen at random, find the probability that the letter so chosen is :
(i) a vowel,
(ii) a consonant.

25. In Figure +D = +E and AD = AE , prove that TBAC is an isosceles triangle.


DB EC

 O
ABC is a right triangle right angled at C. Let BC = a, CA = b, AB = c PQR, ST || QR and p be the length of
perpendicular from C to AB . Prove that cp = ab .

Section - C
Section C consists of 6 questions of 3 marks each.

26. Quadratic polynomial 2x2 − 3x + 1 has zeroes as α and β . Now form a quadratic polynomial whose zeroes are 3α
and 3β .

27. Find whether the following pair of linear equations has a unique solutions. If yes, find the solution :
7x - 4y = 49, 5x − 6y = 57 .

Install NODIA App to See the Solutions.


Click Here To Install
Page 6 Sample Paper 10 CBSE Mathematics Class 10

28. In given figure TABC~TDEF. AP bisects +CAB and DQ bisects +FDE.

Prove that :
(1) AP = AB
DQ DE
(2) TCAP~TFDQ.
 O
In the given figure, DE || AC and DF || AE. Prove that BE = BE .
FE EC

29. If cos ^40º + x h = sin 30º, find the value of x .

30. A conical vessel, with base radius 5 cm height 24 cm, is full of water. This water emptied into a cylindrical vessel,
of base radius 10 cm. Find the height to which the water will rise in the cylindrical vessel. Use π = 227
 O
504 cones, each of diameter 3.5 cm and height 3 cm, are melted and recast into a metallic sphere. Find the
diameter of the sphere and hence find its surface area. Use π = 227

31. Three bells toll at intervals of 9, 12, 15 minutes respectively. If they start tolling together, after what time will
they next toll together?

Section - D
Section D consists of 4 questions of 5 marks each.

32. Find for x : 1 + 2 = 6 ; x ! 0, 1, 2


x−2 x−1 x
 O
Find the values of k for which the equation ^3k + 1h x + 2 ^k + 1h x + 1 has equal roots. Also find the roots.
2

Install NODIA App to See the Solutions.


Click Here To Install
Page 7 Sample Paper 10 CBSE Mathematics Class 10

33. In figure O is the centre of a circle of radius 5 cm. T is a point such that OT = 13 cm and OT intersects circle at
E . If AB is a tangent to the circle at E , find the length of AB , where TP and TQ are two tangents to the circle.

34. Find the mode of the following frequency distribution

Class Interval 25-30 30-35 35-40 40-45 45-50 50-55


Frequency 25 34 50 42 38 14

 O

On the sports day of a school, 300 students participated. Their ages are given in the following distribution :

Age (in years) 5-7 7-9 9-11 11-13 13-15 15-17 17-19
Number of students 67 33 41 95 36 13 15
Find the mean and mode of the data.

35. Find the ratio in which the line x − 3y = 0 divides the line segment joining the points ^- 2, - 5h and ^6, 3h . Find
the coordinates of the point of intersection.

Section - E
Case study based questions are compulsory.

36. Volume of a Bird Cage. A company makes rectangular shaped bird cages with height b inches and square bottoms.
The volume of these cages is given by the function V = b 3 − 6b2 + 9b .
(i) Find an expression for the length of each side of the square bottom.
(ii) Use the function to find the volume of a cage with a height of 18 inches.
(iii) Use the remainder theorem to find the volume of a cage with a height of 15 inches.
(iv) Verify the result of (iii) using function ?

Continue on next page....

Install NODIA App to See the Solutions.


Click Here To Install
Page 8 Sample Paper 10 CBSE Mathematics Class 10

37. Dipesh bought 3 notebooks and 2 pens for Rs. 80. His friend Ramesh said that price of each notebook could be
Rs. 25. Then three notebooks would cost Rs.75, the two pens would cost Rs. 5 and each pen could be for Rs. 2.50.
Another friend Amar felt that Rs. 2.50 for one pen was too little. It should be at least Rs. 16. Then the price of
each notebook would also be Rs.16.

Aditya also bought the same types of notebooks and pens as Dipesh. He paid 110 for 4 notebooks and 3 pens.
(i) Whether the estimation of Ramesh and Amar is applicable for Aditya?
(ii) Let the cost of one notebook be x and that of pen be y . Which of the following set describe the given
problem ?
(iii) What is the exact cost of the notebook?
(iv) What is the exact cost of the pen? What is the total cost if they purchase the same type of 15 notebooks
and 12 pens.

Install NODIA App to See the Solutions.


Click Here To Install
Page 9 Sample Paper 10 CBSE Mathematics Class 10

38. Conical Tank : The advantages of cone bottom tanks are found in nearly every industry, especially where getting
every last drop from the tank is important. This type of tank has excellent geometry for draining, especially with
high solids content slurries as these cone tanks provide a better full-drain solution. The conical tank eliminates
many of the problems that flat base tanks have as the base of the tank is sloped towards the centre giving the
greatest possible full-drain system in vertical tank design.

Rajesh has been given the task of designing a conical bottom tank for his client. Height of conical part is equal
to its radius. Length of cylindrical part is the 3 times of its radius. Tank is closed from top. The cross section of
conical tank is given below.

(i) If radius of cylindrical part is taken as 3 meter, what is the volume of above conical tank ?
(ii) What is the area of metal sheet used to make this conical tank ? Assume that tank is covered from top.
(iii) What is the ratio of volume of cylindrical part to the volume of conical part?
(iv) The cost of metal sheet is Rs 2000 per square meter and fabrication cost is 1000 per square meter. What is
the total cost of tank ?
 ******

Install NODIA App to See the Solutions.


Click Here To Install
Page 1 Sample Paper 11 Mathematics STD Class 10

Sample Paper 11
Class- X Exam - 2023-24
Mathematics - Standard

Time Allowed: 3 Hours Maximum Marks : 80


General Instructions :
1. This Question Paper has 5 Sections A-E.
2. Section A has 20 MCQs carrying 1 mark each
3. Section B has 5 questions carrying 02 marks each.
4. Section C has 6 questions carrying 03 marks each.
5. Section D has 4 questions carrying 05 marks each.
6. Section E has 3 case based integrated units of assessment (04 marks each) with sub-parts of the values of 1, 1
and 2 marks each respectively.
7. All Questions are compulsory. However, an internal choice in 2 Qs of 5 marks, 2 Qs of 3 marks and 2 Questions
of 2 marks has been provided. An internal choice has been provided in the 2marks questions of Section E
8. Draw neat figures wherever required. Take π = 227 wherever required if not stated.

Section - A
Section A consists of 20 questions of 1 mark each.

1. The distance between the points (a cos θ + b sin θ, 0), and (0, a sin θ − b cos θ) is
(a) a2 + b2 (b) a2 - b2
(c) a2 + b2 (d) a2 - b2

2. If one zero of the polynomial (3x2 + 8x + k) is the reciprocal of the other, then value of k is
(a) 3 (b) - 3
(c) 1 (d) - 1
3 3

3. If 3x + 4y : x + 2y = 9 : 4 , then 3x + 5y : 3x − y is equal to
(a) 4 : 1 (b) 1 : 4
(c) 7:1 (d) 1 : 7

4. The value of k for which the system of linear equations x + 2y = 3 , 5x + ky + 7 = 0 is inconsistent is


(a) - 14 (b) 2
3 5
(c) 5 (d) 10

5. If α and β are the zeroes of the polynomial 2x2 − 13x + 6 , then α + β is equal to
(a) - 3 (b) 3
(c) 13 (d) - 13
2 2

https://qrbook.page.link/app
Install NODIA App to See the Solutions.
Click Here To Install
Page 2 Sample Paper 11 Mathematics STD Class 10

6. The roots of the quadratic equation x2 − 0.04 = 0 are


(a) ! 0.2 (b) ! 0.02
(c) 0.4 (d) 2

7. If the common difference of an AP is 5, then what is a18 - a13 ?


(a) 5 (b) 20
(c) 25 (d) 30

8. TABC is an equilateral triangle with each side of length 2p . If AD = BC then the value of AD is
(a) 3 (b) 3 p
(c) 2p (d) 4p

9. The base radii of a cone and a cylinder are equal. If their curved surface areas are also equal, then the ratio of the
slant height of the cone to the height of the cylinder is
(a) 2 : 1 (b) 1 : 2
(c) 1:3 (d) 3 : 1

10. In figure, O is the centre of circle. PQ is a chord and PT is tangent at P which makes an angle of 50c with
PQ+POQ is

(a) 130c (b) 90c


(c) 100c (d) 75c

11. A tree casts a shadow 15 m long on the level of ground, when the angle of elevation of the sun is 45c. The height
of a tree is
(a) 10 m (b) 14 m
(c) 8m (d) 15 m

12. The quadratic equation 2x2 − 5 x + 1 = 0 has


(a) two distinct real roots
(b) two equal real roots
(c) no real roots
(d) more than 2 real roots

https://qrbook.page.link/app
Install NODIA App to See the Solutions.
Click Here To Install
Page 3 Sample Paper 11 Mathematics STD Class 10

13. A sector is cut from a circular sheet of radius 100 cm, the angle of the sector being 240c. If another circle of the
area same as the sector is formed, then radius of the new circle is
(a) 79.5 cm (b) 81.5 cm
(c) 83.4 cm (d) 88.5 cm

14. In a frequency distribution, the mid value of a class is 10 and the width of the class is 6. The lower limit of the
class is
(a) 6 (b) 7
(c) 8 (d) 12

15. If a card is selected from a deck of 52 cards, then the probability of its being a red face card is
(a) 3 (b) 3
26 13

(c) 2 (d) 1
13 2

16. If cos ^α + β h = 0 , then sin ^α - β h can be reduced to


(a) cos β (b) cos 2β
(c) sin α (d) sin 2α

17. The point P on x -axis equidistant from the points A (- 1, 0) and B (5, 0) is
(a) (2, 0) (b) (0, 2)
(c) (3, 0) (d) (- 3, 5)

18. The point on the x -axis which is equidistant from the points A (- 2, 3) and B (5, 4) is
(a) (0, 2) (b) (2, 0)
(c) (3, 0) (d) (- 2, 0)

In the question number 19 and 20, a statement of Assertion (A) is followed by a statement of Reason (R). Choose the
correction option.

19. Assertion : When a positive integer a is divided by 3, the values of remainder can be 0, 1 or 2.
Reason : According to Euclid’s Division Lemma a = bq + r , where 0 # r 1 b and r is an integer.
(a) Both assertion (A) and reason (R) are true and reason (R) is the correct explanation of assertion (A).
(b) Both assertion (A) and reason (R) are true but reason (R) is not the correct explanation of assertion (A).
(c) Assertion (A) is true but reason (R) is false.
(d) Assertion (A) is false but reason (R) is true.

20. Assertion : Sum of first 10 terms of the arithmetic progression - 0.5 , - 1.0, - 1.5 , .......... is 31.
Reason : Sum of n terms of an AP is given as Sn = n2 [2a + (n − 1) d ] where a is first term and d common
difference.
(a) Both assertion (A) and reason (R) are true and reason (R) is the correct explanation of assertion (A).
(b) Both assertion (A) and reason (R) are true but reason (R) is not the correct explanation of assertion (A).
(c) Assertion (A) is true but reason (R) is false.
(d) Assertion (A) is false but reason (R) is true.

https://qrbook.page.link/app
Install NODIA App to See the Solutions.
Click Here To Install
Page 4 Sample Paper 11 Mathematics STD Class 10

Section - B
Section B consists of 5 questions of 2 marks each.

21. ABCD is a trapezium in which AB || CD and its diagonals intersect each other at the point O. Show that
AO = CO .
BO DO

22. In given figure, AB is the diameter of a circle with centre O and AT is a tangent. If +AOQ = 58c, find +ATQ .

23. Find the value of cos 2θ , if 2 sin 2θ = 3.

24. Find the mean of the following distribution :

Class 10-25 25-40 40-55 55-70 70-85 85-100


Frequency 2 3 7 6 6 6

 O
Find the mean of the following data :

Class 0- 20 20-40 40-60 60-80 80-100 100-120


Frequency 20 35 52 44 38 31

25. Show that 5 6 is an irrational number.


 O
Write a rational number between 2 and 3.

Section - C
Section C consists of 6 questions of 3 marks each.

26. Which term of the AP 20, 19 1 , 18 1 , 17 3 , ... is the first negative term.
4 2 4

27. If 1 + sin2 θ = 3 sin θ cos θ , prove that tan θ = 1 or ½.

https://qrbook.page.link/app
Install NODIA App to See the Solutions.
Click Here To Install
Page 5 Sample Paper 11 Mathematics STD Class 10

28. A horse is tethered to one corner of a rectangular field of dimensions 70 m # 52 m , by a rope of length 21 m. How
much area of the field can it graze?
 O

In the given figure, a chord AB of the circle with centre O and radius 10 cm, that subtends a right angle at the
centre of the circle. Find the area of the minor segment AQBP . Hence find the area of major segment ALBQA
. (Use π = 3.14 )

29. Find the mode of the following frequency distribution :

Class 15-20 20-25 25-30 30-35 35-40 40-45


Frequency 3 8 9 10 3 2

30. Find the ratio in which the segment joining the points ^1, - 3h and ^4, 5h is divided by x -axis? Also find the
coordinates of this point on x -axis.
 O

The vertices of TABC are A ^6, - 2h, B ^0, - 6h and C ^4, 8h . Find the co-ordinates of mid-points of AB, BC and
AC.

31. Write the smallest number which is divisible by both 306 and 657.

Section - D
Section D consists of 4 questions of 5 marks each.

32. Determine graphically the coordinates of the vertices of triangle, the equations of whose sides are given by
2y − x = 8 , 5y − x = 14 and y − 2x = 1.
 O

Draw the graphs of the equations x − y + 1 = 0 and 3x + 2y − 12 = 0. Determine the co-ordinates of the vertices
of the triangle formed by these lines and the X-axis and shade the triangular region.

Continue on next page....

https://qrbook.page.link/app
Install NODIA App to See the Solutions.
Click Here To Install
Page 6 Sample Paper 11 Mathematics STD Class 10

33. Two tangents PA and PB are drawn from an external point P to a circle with centre O , such that +APB = +x
and +AOB = y . Prove that opposite angles are supplementary.

34. The person standing on the bank of river observes that the angle of elevation of the top of a tree standing on
opposite bank is 60c . When he moves 30 m away from the bank, he finds the angle of elevation to be 30c. Find
the height of tree and width of the river.
 O
As observed from the top of a 100 m high light house from the sea-level, the angles of depression of two ships are
30c and 45c. If one ship is exactly behind the other on the same side of the light house, find the distance between
the two ships [Use 3 = 1.732 ]

35. A hemispherical depression is cut from one face of a cubical block, such that diameter l of hemisphere is equal to
the edge of cube. Find the surface area of the remaining solid.

Section - E
Case study based questions are compulsory.

36. Maximum Profit : A kitchen utensils manufacturer can produce up to 200 utensils per day. The profit made from
the sale of these utensils can be modelled by the function P (x) = − 0.5x + 175x − 330 , where P (x) is the profit in
Rupees, and x is the number of utensils made and sold. Based on this model,
(i) Find the y -intercept and explain what it means in this context.
(ii) Find the x -intercepts and explain what they mean in this context.
(iii) How many utensils should be sold to maximize profit?
 O
What is the maximum profit?

Continue on next page....

https://qrbook.page.link/app
Install NODIA App to See the Solutions.
Click Here To Install
Page 7 Sample Paper 11 Mathematics STD Class 10

37. Tania is very intelligent in maths. She always try to relate the concept of maths in daily life. One day she plans
to cross a river and want to know how far it is to the other side. She takes measurements on her side of the river
and make the drawing as shown below.

(i) Which similarity criterion is used in solving the above problem ?


(ii) Consider the following statement :
S1 : +ACB = +DCE
S2 : +BAC = +CDE
Which of the above statement is/are correct.
(a) S1 and S2 both (b) S1
(c) S2 (d) None
(iii) Consider the following statement :
S3 : AB = CA
DE CD

S 4 : BC = AB
CE DE

S5 : CA = DE
CD AB
Which of the above statements are correct ?
(a) S3 and S5 (b) S 4 and S5
(c) S3 and S 4 (d) All three
(iv) What is the distance x across the river?
 O
What is the approximate length of AD shown in the figure?

Continue on next page....

https://qrbook.page.link/app
Install NODIA App to See the Solutions.
Click Here To Install
Page 8 Sample Paper 11 Mathematics STD Class 10

38. Double-six Dominos : It is a game played with the 28 numbered tiles shown in the diagram.

The 28 dominos are placed in a bag, shuffled, and then one domino is randomly drawn. Give the following answer.
(i) What is the probability the total number of dots on the domino is three or less ?
(ii) What is the probability the total number of dots on the domino is greater than three ?
(iii) What is the probability the total number of dots on the domino does not have a blank half ?
 O
What is the probability the total number of dots on the domino is not a “double” (both sides the same) ?

 ******

https://qrbook.page.link/app
Install NODIA App to See the Solutions.
Click Here To Install
Page 1 Sample Paper 12 CBSE Mathematics Class 10

Sample Paper 12
Class- X Exam - 2023-24
Mathematics - Standard

Time Allowed: 3 Hours Maximum Marks : 80


General Instructions :
1. This Question Paper has 5 Sections A-E.
2. Section A has 20 MCQs carrying 1 mark each
3. Section B has 5 questions carrying 02 marks each.
4. Section C has 6 questions carrying 03 marks each.
5. Section D has 4 questions carrying 05 marks each.
6. Section E has 3 case based integrated units of assessment (04 marks each) with sub-parts.
7. All Questions are compulsory. However, an internal choice in 2 Qs of 5 marks, 2 Qs of 3 marks and 2 Questions
of 2 marks has been provided.
8. Draw neat figures wherever required. Take π = 227 wherever required if not stated.

Section - A
Section A consists of 20 questions of 1 mark each.

1. What do you say about the solution of the pair of linear equations y = 0 and y = − 5 ?
(a) no solution (b) unique solution (c) infinitely solution (d) can’t say anything

2. The HCF and LCM of 378, 180 and 420 of will be


(a) 6 and 3980 (b) 12 and 3780 (c) 6 and 3780 (d) 12 and 3980

3. The quadratic equation x2 − 4x − 3 2 = 0 has


(a) two distinct real roots (b) two equal real roots
(c) no real roots (d) more than 2 real roots

4. The 11th term of an AP - 5 , - 5 , 0, 5 , ....., is


2 2
(a) - 20 (b) 20 (c) - 30 (d) 30

5. If tan 2A = cot ^A + 60ºh , where 2A is an acute angle, the value of A will be


(a) 20° (b) 30° (c) 60° (d) 10°

6. If - 1 is a zero of the polynomial f (x) = x2 − 7x − 8 , then other zero is


(a) 4 (b) 8 (c) 1 (d) –4

7. Given the linear equation 3x + 4y = 9 . Select another linear equation in these two variables such that the
geometrical representation of the pair so formed is intersecting lines.
(a) 3x − 5y = 10 (b) 6x + 8y = 18 (c) 8x + 12y = 18 (d) above all

Install NODIA App to See the Solutions.


Click Here To Install
Page 2 Sample Paper 12 CBSE Mathematics Class 10

8. A fraction becomes 13 when 2 is subtracted from the numerator and it becomes 1


2 when 1 is subtracted from the
denominator. The fraction will be
(a) 157 (b) 158 (c) 156 (d) 9
15

9. Which of the following is not the graph of a quadratic polynomial?

10. If the point P ^x, y h is equidistant from the points Q ^a + b, b − a h and R ^a − b, a + b h then,
(a) 2ay = xy (b) bx = ay (c) ab = xy (d) by = ax

11. The quadratic equation 3x2 + 4 3 x + 4 has


(a) two distinct real roots (b) two equal real roots
(c) no real roots (d) more than 2 real roots

12. In the given figure, two tangents AB and AC are drawn to a circle with centre O such that +BAC = 120c, then
OA is equal to that

(a) 2AB (b) 3AB (c) 4AB (d) 5AB

Install NODIA App to See the Solutions.


Click Here To Install
Page 3 Sample Paper 12 CBSE Mathematics Class 10

ar (TAMN)
13. In figure, MN < BC and AM : MB = 1 : 2 , then = ................... .
ar (TABC)

1 1 8 5
(a) 3 (b) 9 (c) 9 (d) 9

14. 1 − sin θ = ?
1 + sin θ
(a) sin q - cos q (b) sec q - tan q (c) sec q + tan q (d) sin q + cos q

15. Two different dice are tossed together. What is the probability that the number on each die is even ?
(a) 12 (b) 14 (c) 34 (d) 16

16. A circle artist is climbing a 20 m long rope, which is tightly stretched and tied from the top of a vertical pole to
the ground, then the height of pole, if the angle made by the rope with the ground level is 30c, is
(a) 5 m (b) 10 m
(c) 15 m (d) 20 m

17. Ratio of volumes of two cones with same radii is


(a) h1 : h2 (b) s1 : s2 (c) r1 : r2 (d) None of these

18. For the following distribution:

Marks Number of students


Below 10 3
Below 20 12
Below 30 27
Below 40 57
Below 50 75
Below 60 80
The modal class is
(a) 10-20 (b) 20-30 (c) 30-40 (d) 50-60

Install NODIA App to See the Solutions.


Click Here To Install
Page 4 Sample Paper 12 CBSE Mathematics Class 10

In the question number 19 and 20, a statement of Assertion (A) is followed by a statement of Reason (R). Choose the
correction option.

19. Assertion : Sum of first 10 terms of the arithmetic progression - 0.5 , - 1.0, - 1.5 , .......... is 31.
Reason : Sum of n terms of an AP is given as Sn = n2 [2a + (n − 1) d ] where a is first term and d common
difference.
(a) Both assertion (A) and reason (R) are true and reason (R) is the correct explanation of assertion (A).
(b) Both assertion (A) and reason (R) are true but reason (R) is not the correct explanation of assertion (A).
(c) Assertion (A) is true but reason (R) is false.
(d) Assertion (A) is false but reason (R) is true.

20. Assertion : If the outer and inner diameter of a circular path is 10 m and 6 m then area of the path is 16π m2 .
Reason : If R and r be the radius of outer and inner circular path, then area of path is π ^R2 − r2h .
(a) Both assertion (A) and reason (R) are true and reason (R) is the correct explanation of assertion (A).
(b) Both assertion (A) and reason (R) are true but reason (R) is not the correct explanation of assertion (A).
(c) Assertion (A) is true but reason (R) is false.
(d) Assertion (A) is false but reason (R) is true.

Section - B
Section B consists of 5 questions of 2 marks each.

21. Write a rational number between 2 and 3.

22. Find the sum of first 16 terms of the AP 10, 6, 2, .....


 O
What is the sum of five positive integer divisible by 6.

23. If three points (0, 0), (3, 3 ) and (3, λ) form an equilateral triangle, then what is the value of λ ?

24. A chord of a circle of radius 10 cm, subtends a right angle at its centre. What is the length of the chord?
 O
In figure, on a circle of radius 7 cm, tangent PT is drawn from a point P such that PT = 24 cm. If O is the centre
of the circle, then what is the length of PR ?

25. In a family of two children find the probability of having at least one girl.

Install NODIA App to See the Solutions.


Click Here To Install
Page 5 Sample Paper 12 CBSE Mathematics Class 10

Section - C
Section C consists of 6 questions of 3 marks each.

26. The length, breadth and height of a room are 8 m 50 cm, 6 m 25 cm and 4 m 75 cm respectively. Find the length
of the longest rod that can measure the dimensions of the room exactly.

27. If triangle ABC is similar to triangle DEF such that 2AB = DE and BC = 8 cm then find EF.

28. If θ be an acute angle and 5 cosec θ = 7 , then evaluate sin θ + cos2 θ − 1.

29. In given figure, two circles touch each other at the point C . Prove that the common tangent to the circles at C ,
bisects the common tangent at P and Q .

30. In Figure, PQ and AB are two arcs of concentric circles of radii 7 cm and 3.5 cm respectively, with centre O . If
+POQ = 30c, then find the area of shaded region.

 O
A horse is tethered to one corner of a rectangular field of dimensions 70 m # 52 m , by a rope of length 21 m. How
much area of the field can it graze?

31. Two dice are tossed simultaneously. Find the probability of getting
(i) an even number on both dice.
(ii) the sum of two numbers more than 9.

 O

In a family of three children, find the probability of having at least two boys.

Install NODIA App to See the Solutions.


Click Here To Install
Page 6 Sample Paper 12 CBSE Mathematics Class 10

Section - D
Section D consists of 4 questions of 5 marks each.

32. Determine graphically whether the following pair of linear equations :


3x - y = 7
2x + 5y + 1 = 0 has :
(a) unique solution
(b) infinitely many solutions or
(c) no solution.
 O
For Uttarakhand flood victims two sections A and B of class contributed Rs. 1,500. If the contribution of X-A was
Rs. 100 less than that of X-B, find graphically the amounts contributed by both the sections.

33. If the mid-point of the line segment joining A 8 x2 , y +2 1B and B ^x + 1, y − 3h is C ^5, - 2h , find x, y.

34. In TABC, AD is a median and O is any point on AD. BO and CO on producing meet AC and AB at E and F
respectively. Now AD is produced to X such that OD = DX as shown in figure.
Prove that :
(1) EF || BC
(2) AO : AX = AF : AB

 O
In the figure, +BED = +BDE and E is the mid-point of BC . Prove that AF = AD .
CF BE

Install NODIA App to See the Solutions.


Click Here To Install
Page 7 Sample Paper 12 CBSE Mathematics Class 10

35. A milk tanker cylindrical in shape having diameter 2 m and length 4.2 m supplies milk to the two booths in
the ratio of 3 | 2 . One of the milk booths has cuboidal vessel having base area 3.96 sq. m. and the other has a
cylindrical vessel having radius 1 m. Find the level of milk in each of the vessels. Use π = 227

Section - E
Case study based questions are compulsory.

36. Nidhi and Ria are very close friends. Nidhi’s parents own a Maruti Alto. Ria’s parents own a Toyota Liva. Both
the families decide to go for a picnic to Somnath temple in Gujrat by their own cars.

Nidhi’s car travels x km/h while Ria’s car travels 5 km/h more than Nidhi’s car. Nidhi’s car took 4 hrs more than
Ria’s car in covering 400 km.
(i) What will be the distance covered by Ria’s car in two hour? Write the quadratic equation that describe the
speed of Nidhi’s car?
(ii) What is the speed of Nidhi’s car?
(iii) How much time did Ria take to travel 400 km?
(iv) How much time did Nidhi take to travel 400 km?

37. Width of a Lake : The angle of depression to one side of a lake, measured from a balloon 300 meter above the
lake as shown in the accompanying figure, is 45c . The angle of depression to the opposite side of the lake is 30c .
(i) Find the width of the lake.
(ii) Find the ground distance of balloon from sides of lake.

Install NODIA App to See the Solutions.


Click Here To Install
Page 8 Sample Paper 12 CBSE Mathematics Class 10

38. Transport department of a Jaipur wants to buy some Electric buses for the city. For which they wants to analyse
the distance travelled by existing public transport buses in a day.

The following data shows the distance travelled by 60 existing public transport buses in a day.

Daily distance travelled (in km) 200-209 210-219 220-229 230-239 240-249
Number of buses 4 14 26 10 6
Base on the above information, answer the following questions.
(i) Find the median class of daily distance travelled ?
(ii) What is the cumulative frequency of the class preceding the median class ? Find the median of the distance
travelled.
(iii) If the mode of the distance travelled is 223.78 km, find the mean of the distance travelled by the bus .

 ******

Install NODIA App to See the Solutions.


Click Here To Install
Page 1 Sample Paper 13 Mathematics STD Class 10

Sample Paper 13
Class- X Exam - 2023-24
Mathematics - Standard

Time Allowed: 3 Hours Maximum Marks : 80


General Instructions :
1. This Question Paper has 5 Sections A-E.
2. Section A has 20 MCQs carrying 1 mark each
3. Section B has 5 questions carrying 02 marks each.
4. Section C has 6 questions carrying 03 marks each.
5. Section D has 4 questions carrying 05 marks each.
6. Section E has 3 case based integrated units of assessment (04 marks each) with sub-parts of the values of 1, 1
and 2 marks each respectively.
7. All Questions are compulsory. However, an internal choice in 2 Qs of 5 marks, 2 Qs of 3 marks and 2 Questions
of 2 marks has been provided. An internal choice has been provided in the 2 marks questions of Section E
8. Draw neat figures wherever required. Take π = 227 wherever required if not stated.

Section - A
Section A consists of 20 questions of 1 mark each.

1. Consider the following frequency distribution of the heights of 60 students of a class

Height (in cm) 150-155 155-160 160-165 165-170 170-175 175-180


Number of students 15 13 10 8 9 5
The upper limit of the median class in the given data is
(a) 165 (b) 155 (c) 160 (d) 170

2. If one zero of a quadratic polynomial (kx2 + 3x + k) is 2, then the value of k is


(a) 5 (b) - 5 (c) 6 (d) - 6
6 6 5 5

3. The centroid of the triangle whose vertices are (3, - 7), (- 8, 6) and (5, 10) is
(a) (0, 9) (b) (0, 3) (c) (1, 3) (d) (3, 5)

4. The zeroes of the polynomial x2 − 3x − m (m + 3) are


(a) m, m + 3 (b) − m, m + 3 (c) m, − (m + 3) (d) − m, − (m + 3)

5. The value of k for which the system of equations x + y − 4 = 0 and 2x + ky = 3 , has no solution, is
(a) - 2 (b) ! 2 (c) 3 (d) 2

6. If one root of the quadratic equation ax2 + bx + c = 0 is the reciprocal of the other, then
(a) b = c (b) a = b
(c) ac = 1 (d) a = c

https://qrbook.page.link/app
Install NODIA App to See the Solutions.
Click Here To Install
Page 2 Sample Paper 13 Mathematics STD Class 10

7. The pair of equations 3x + y = 81, 81x − y = 3 has


(a) no solution (b) unique solution
(c) infinitely many solutions (d) x = 2 1 , y = 1 7
8 8

8. What is the common difference of an AP in which a18 − a14 = 32 ?


(a) 8 (b) - 8 (c) - 4 (d) 4

9. Two chords AB and CD of a circle intersect at E such that AE = 2.4 cm , BE = 3.2 cm and CE = 1.6 cm . The
length of DE is
(a) 1.6 cm (b) 3.2 cm (c) 4.8 cm (d) 6.4 cm

10. If cos 9a = sin a and 9α < 90c, then the value oftan 5α is

(a) 1 (b) 3 (c) 1 (d) 0


3

11. The first term of AP is p and the common difference is q , then its 10th term is
(a) q + 9p (b) p - 9q (c) p + 9q (d) 2p + 9q

12. The ratio of the length of a rod and its shadow is 1 : 3 then the angle of elevation of the sun is
(a) 90c (b) 45c (c) 30c (d) 75c

13. If a circular grass lawn of 35 m in radius has a path 7 m wide running around it on the outside, then the area of
the path is
(a) 1450 m2 (b) 1576 m2 (c) 1694 m2 (d) 3368 m2

14. Two poles of height 6 m and 11 m stand vertically upright on a plane ground. If the distance between their foot
is 12 m, then distance between their tops is
(a) 12 m (b) 14 m (c) 13 m (d) 11 m

15. If the perimeter of one face of a cube is 20 cm, then its surface area is
(a) 120 cm2 (b) 150 cm2 (c) 125 cm2 (d) 400 cm2

16. A card is drawn from a deck of 52 cards. The event E is that card is not an ace of hearts. The number of outcomes
favourable to E is
(a) 4 (b) 13 (c) 48 (d) 51

17. The co-ordinates of the point which is reflection of point (- 3, 5) in x -axis are
(a) (3, 5) (b) (3, - 5 )
(c) (- 3, - 5) (d) (- 3, 5)

18. C is the mid-point of PQ , if P is (4, x), C is (y, - 1) and Q is (- 2, 4), then x and y respectively are
(a) - 6 and 1 (b) - 6 and 2
(c) 6 and - 1 (d) 6 and - 2

https://qrbook.page.link/app
Install NODIA App to See the Solutions.
Click Here To Install
Page 3 Sample Paper 13 Mathematics STD Class 10

In the question number 19 and 20, a statement of Assertion (A) is followed by a statement of Reason (R). Choose the
correction option.

19. Assertion : The HCF of two numbers is 5 and their product is 150, then their LCM is 30
Reason : For any two positive integers a and b, HCF ^a, b h + LCM ^a, b h = a # b .
(a) Both assertion (A) and reason (R) are true and reason (R) is the correct explanation of assertion (A).
(b) Both assertion (A) and reason (R) are true but reason (R) is not the correct explanation of assertion (A).
(c) Assertion (A) is true but reason (R) is false.
(d) Assertion (A) is false but reason (R) is true.

20. Assertion : 4x2 − 12x + 9 = 0 has repeated roots.


Reason : The quadratic equation ax2 + bx + c = 0 have repeated roots if discriminant D > 0 .
(a) Both assertion (A) and reason (R) are true and reason (R) is the correct explanation of assertion (A).
(b) Both assertion (A) and reason (R) are true but reason (R) is not the correct explanation of assertion (A).
(c) Assertion (A) is true but reason (R) is false.
(d) Assertion (A) is false but reason (R) is true.

Section - B
Section B consists of 5 questions of 2 marks each.

21. In a rectangle ABCD, E is a point on AB such that AE = 23 AB. If AB = 6 km and AD = 3 km, then find DE.

22. In the given figure PQ is chord of length 6 cm of the circle of radius 6 cm. TP and TQ are tangents to the circle
at points P and Q respectively. Find +PTQ .

23. Find the value of sin 30º cos 60º + cos 30º sin 60º is it equal to sin 90º or cos 90º ?

24. Find the mode of the following frequency distribution.

Class 0- 10 10-20 20-30 30-40 40-50 50-60 60-70


Frequency 8 10 10 16 12 6 7

 O
The data regarding marks obtained by 48 students of a class in a class test is given below. Calculate the modal
marks of students.

Marks obtained 0-5 5- 10 10-15 15-20 20-25 25-30 30-35 35-40 40-45 45-50
Number of students 1 0 2 0 0 10 25 7 2 1

https://qrbook.page.link/app
Install NODIA App to See the Solutions.
Click Here To Install
Page 4 Sample Paper 13 Mathematics STD Class 10

25. Show that 571 is a prime number.


 O
If two positive integers p and q are written as p = a b and q = a3 b, where a and b are prime numbers than verify
2 3

LCM (p, q) # HCF (q, q) = pq

Section - C
Section C consists of 6 questions of 3 marks each.

26. Find the middle term of the AP 7, 13, 19, ...., 247.

27. Prove that ^sin θ + cosec θh2 + ^cos θ + sec θh2 = 7 + tan2 θ + cot2 θ

28. The circumference of a circle exceeds the diameter by 16.8 cm. Find the radius of the circle. Use π = 22
7 .
 O
Find the area of shaded region shown in the given figure where a circular arc of radius 6 cm has been drawn with
vertex O of an equilateral triangle OAB of side 12 cm as centre.

29. The marks obtained by 110 students in an examination are given below

Marks 30-35 35-40 40-45 45-50 50-55 55-60 60-65


Number of Students 14 16 28 23 18 8 3

Find the mean marks of the students.

30. If the point C ^- 1, 2h divides internally the line segment joining the points A ^2, 5h and B ^x, y h in the ratio 3: 4,
find the value of x2 + y2 .
 O
Find the ratio in which the point ^- 3, p h divides the line segment joining the points ^- 5, - 4h and ^- 2, 3h .Hence
find the value of p.

31. 144 cartons of Coke cans and 90 cartons of Pepsi cans are to be stacked in a canteen. If each stack is of the same
height and if it equal contain cartons of the same drink, what would be the greatest number of cartons each stack
would have?

https://qrbook.page.link/app
Install NODIA App to See the Solutions.
Click Here To Install
Page 5 Sample Paper 13 Mathematics STD Class 10

Section - D
Section D consists of 4 questions of 5 marks each.

32. Determine graphically whether the following pair of linear equations :


3x - y = 7
2x + 5y + 1 = 0 has :
unique solution
infinitely many solutions or
no solution.
 O
Solve the following pair of linear equations graphically:
x + 3y = 12, 2x − 3y = 12
Also shade the region bounded by the line 2x − 3y = 2 and both the co-ordinate axes.

33. a, b and c are the sides of a right triangle, where c is the hypotenuse. A circle, of radius r , touches the sides of
the triangle. Prove that r = a + b − c .
2

34. A vertical tower stands on horizontal plane and is surmounted by a vertical flag-staff of height 6 m. At a point
on the ground, angle of elevation of the bottom and top of the flag-staff are 30c and 45c respectively. Find the
height of the tower. (Take 3 = 1.73 )
 O
From the top of tower, 100 m high, a man observes two cars on the opposite sides of the tower with the angles of
depression 30c and 45c respectively. Find the distance between the cars. (Use 3 = 1.73 )

35. The internal and external diameters of a hollow hemispherical vessel are 16 cm and 12 cm respectively. If the cost
of painting 1 cm2 of the surface area is Rs. 5.00, find the total cost of painting the vessel all over. (Use π = 3.14 )

Section - E
Case study based questions are compulsory.

36. Riya has a lawn with a flowerbed and grass land. The grass land is in the shape of rectangle while flowerbed is in
the shape of square. The length of the grassland is found to be 3 m more than twice the length of the flowerbed.
Total area of the whole lawn is 1260 m2.

(i) If the length of the flowerbed is x m then what is the total length of the lawn ?
(ii) What is the value of x if the area of total lawn is 1260 m2 ?
(iii) What is the area of grassland ?
 O
What is the ratio of area of flowerbed to area of grassland ?

https://qrbook.page.link/app Install NODIA App to See the Solutions.


Click Here To Install
Page 6 Sample Paper 13 Mathematics STD Class 10

37. Rani wants to make the curtains for her window as shown in the figure. The window is in the shape of a rectangle,
whose width and height are in the ratio 2 : 3 . The area of the window is 9600 square cm.

(i) What is the shape of the window that is uncovered?


(ii) What will be the ratio of two sides of each curtain (other than hypotenuse) ?
(iii) What are the dimensions of the window ?
 O
How much window area is covered by the curtains?

38. Family Structures : For a recent year, 51% of the families in the United States had no children under the age of
18; 20% had one child; 19% had two children; 7% had three children; and 3% had four or more children.

If a family is selected at random, find the following probability.


(i) Find the probability that the family has two or three children.
(ii) Find the probability that the family has more than one child.
(iii) Find the probability that the family has less than three children.
 O
Find the probability that the family has more than three children.

 ******

https://qrbook.page.link/app
Install NODIA App to See the Solutions.
Click Here To Install
Page 1 Sample Paper 14 CBSE Mathematics Class 10

Sample Paper 14
Class- X Exam - 2023-24
Mathematics - Standard

Time Allowed: 3 Hours Maximum Marks : 80


General Instructions :
1. This Question Paper has 5 Sections A-E.
2. Section A has 20 MCQs carrying 1 mark each
3. Section B has 5 questions carrying 02 marks each.
4. Section C has 6 questions carrying 03 marks each.
5. Section D has 4 questions carrying 05 marks each.
6. Section E has 3 case based integrated units of assessment (04 marks each) with sub-parts.
7. All Questions are compulsory. However, an internal choice in 2 Qs of 5 marks, 2 Qs of 3 marks and 2 Questions
of 2 marks has been provided.
8. Draw neat figures wherever required. Take π = 227 wherever required if not stated.

Section - A
Section A consists of 20 questions of 1 mark each.

1. Consider the data:

Class 65-85 85-105 105-125 125-145 145-165 165-185 185-205


Frequency 4 5 13 20 14 7 4
The difference of the upper limit of the median class and the lower limit of the modal class is
(a) 0 (b) 19 (c) 20 (d) 38

2. In the given factor tree what is the composite number x ?

(a) 53 (b) 11130 (c) 5565 (d) 19438

Install NODIA App to See the Solutions.


Click Here To Install
Page 2 Sample Paper 14 CBSE Mathematics Class 10

3. In the given figure, three circles with centres P , Q and R are drawn, such that the circles with centres Q and R
touch each other externally and they touch the circle with centre P , internally. If PQ = 10 cm , PR = 8 cm and
QR = 12 cm , then the diameter of the largest circle is:

(a) 30 cm (b) 20 cm (c) 10 cm (d) None of these

4. If α and β are the zeroes the polynomial 2x2 − 4x + 5, the value of α2 + β2 is


(a) - 7 (b) 1 (c) - 1 (d) - 6

5. In the figure, ABCDE is a pentagon with BE z CD and BC z DE . BC is perpendicular to CD. AB = 5 cm,


AE = 5 cm, BE = 7 cm, BC = x − y and CD = x + y. If the perimeter of ABCDE is 27 cm. The value of x and
y , will be

(a) 3 and 2 (b) 2 and 3 (c) 1 and 6 (d) 6 and 1

6. If tan ^3x + 30ºh = 1 then the value of x. will be


(a) 5° (b) 10° (c) 20° (d) 30°

7. If α and β are the zeroes the polynomial 2x2 − 4x + 5, the value of 12 + 12 is


α β
(a) 254 (b) - 254 (c) 45 (d) - 45

8. In an AP, if a = 3.5 , d = 0 and n = 101, then an will be


(a) 0 (b) 3.5 (c) 103.5 (d) 104.5

9. If am = bl, then what do you say about the solution of the pair of linear equations ax + by = c and lx + my = n ?
(a) no solution (b) unique solution (c) infinitely solution (d) can’t say anything

Install NODIA App to See the Solutions.


Click Here To Install
Page 3 Sample Paper 14 CBSE Mathematics Class 10

10. For what value of p does the pair of linear equations given below has unique solution ?
4x + py + 8 = 0 and 2x + 2y + 2 = 0 .
(a) p = 1 (b) p = 2 (c) p ! 4 (d) p ! 2

11. The quadratic equation 2x2 − 3 2 x + 94 = 0 has


(a) two distinct real roots (b) two equal real roots
(c) no real roots (d) more than 2 real roots

12. The quadratic equation 3x2 + 4 3 x + 4 has


(a) two distinct real roots (b) two equal real roots
(c) no real roots (d) more than 2 real roots

13. The corresponding sides of two similar triangles are in the ratio 3 : 4, then the ratio of the areas of triangles is
...................
(a) 13 (b) 19 (c) 169 (d) 34

14. The points ^3, 0h , ^6, 4h and ^- 1, 3h are the vertices of a


(a) equilateral triangle (b) scalene triangle
(c) isosceles triangle (d) right angled isosceles triangle

15. 1 − tan2 θ = ?
1 + tan2 θ
(a) 1 (b) cos2 q - sin2 q (c) sin2 θ (d) cos2 θ

16. The length of a string between a kite and a point on the ground is 85 m. If the string makes an angle θ with level
ground such that tan θ = 158 , then the height of kite is
(a) 75 m (b) 78.05 m (c) 226 m (d) None of these

17. From a solid circular cylinder with height 10 cm and radius of the base 6 cm, a right circular cone of the same
height and same base is removed, then the volume of remaining solid is
(a) 280 πcm3 (b) 330 πcm3 (c) 240 πcm3 (d) 440 π cm3

18. A letter of English alphabet is chosen at random, what is the probability that the letter so chosen is a vowel ?
(a) 265 21
(b) 26 (c) 132 (d) 137

In the question number 19 and 20, a statement of Assertion (A) is followed by a statement of Reason (R). Choose the
correction option.

19. Assertion : an - an - 1 is not independent of n then the given sequence is an AP.


Reason : Common difference d = an − an − 1 is constant or independent of n .
(a) Both assertion (A) and reason (R) are true and reason (R) is the correct explanation of assertion (A).
(b) Both assertion (A) and reason (R) are true but reason (R) is not the correct explanation of assertion (A).
(c) Assertion (A) is true but reason (R) is false.
(d) Assertion (A) is false but reason (R) is true.

Install NODIA App to See the Solutions.


Click Here To Install
Page 4 Sample Paper 14 CBSE Mathematics Class 10

20. Assertion : If a wire of length 22 cm is bent in the shape of a circle, then area of the circle so formed is 40 cm2 .
Reason : Circumference of the circle = length of the wire.
(a) Both assertion (A) and reason (R) are true and reason (R) is the correct explanation of assertion (A).
(b) Both assertion (A) and reason (R) are true but reason (R) is not the correct explanation of assertion (A).
(c) Assertion (A) is true but reason (R) is false.
(d) Assertion (A) is false but reason (R) is true.

Section - B
Section B consists of 5 questions of 2 marks each.

21. Show that 571 is a prime number.

22. If the sum of first k terms of an AP is 3k2 - k and its common difference is 6. What is the first term?
 O
Which term of the AP 8, 14, 20, 26, ...... will be 72 more than its 41st term.

23. The co-ordinate of the point dividing the line segment joining the points A (1, 3) and B (4, 6) in the ratio 2 : 1 is
......... .

24. In figure, O is the centre of circle. PQ is a chord and PT is tangent at P which makes an angle of 50c with PQ .
Find the angle +POQ .

 O
In the adjoining figure, TP and TQ are the two tangents to a circle with centre O . If +POQ = 110c, then find
the angle +PTQ .

25. Find the probability that a leap year has 53 Sundays

Install NODIA App to See the Solutions.


Click Here To Install
Page 5 Sample Paper 14 CBSE Mathematics Class 10

Section - C
Section C consists of 6 questions of 3 marks each.

26. Prove that 3 is an irrational number.

27. In the figure, PQ is parallel to MN. If KP = 4 and KN = 20.4 cm then find KQ.
PM 13

cos2 (45c + θ) + cos2 (45c − θ)


28. Show that : =1
tan (60c + θ) tan (30c − θ)

29. In the given figure, OP is equal to the diameter of a circle with centre O and PA and PB are tangents. Prove
that ABP is an equilateral triangle.

30. The circumference of a circle exceeds the diameter by 16.8 cm. Find the radius of the circle. Use π = 22
7 .
 O
In the given figure, AOB is a sector of angle 60c of a circle with centre O and radius 17 cm. If AP = OB and
AP = 15 cm, find the area of the shaded region.

Install NODIA App to See the Solutions.


Click Here To Install
Page 6 Sample Paper 14 CBSE Mathematics Class 10

31. A child has a die whose six faces show the letters as shown below:
A A B C C C
The die is thrown once. What is the probability of getting (i) A, (ii) C ?

 O

A game consists of tossing a coin 3 times and noting the outcome each time. If getting the same result in all the
tosses is a success, find the probability of losing the game.

Section - D
Section D consists of 4 questions of 5 marks each.

32. Draw the graphs of the equations x − y + 1 = 0 and 3x + 2y − 12 = 0. Determine the co-ordinates of the vertices
of the triangle formed by these lines and the X-axis and shade the triangular region.

 O

The cost of 2 kg of apples and 1kg of grapes on a day was found to be Rs. 160. After a month, the cost of 4kg of
apples and 2kg of grapes is Rs. 300. Represent the situations algebraically and geometrically.

33. Show that A ^6, 4h, B ^5, - 2h and C ^7, - 2h are the vertices of an isosceles triangle.

34. In TABC, AD is the median to BC and in TPQR, PM is the median to QR. If AB = BC = AD . Prove that
PQ QR PM
TABC~TPQR .

 O

In the right triangle, B is a point on AC such that AB + AD = BC + CD. If AB = x, BC = h and CD = d, then


find x (in term of h and d).

35. A right triangle whose sides are 20 cm and 15 cm is made to revolve about its hypotenuse. Find the volume and
the surface area of the double cone so formed. (Use π = 3.14 )

Install NODIA App to See the Solutions.


Click Here To Install
Page 7 Sample Paper 14 CBSE Mathematics Class 10

Section - E
Case study based questions are compulsory.

36. Optimal Pricing Strategy : The director of the National School of Drama must decide what to charge for a ticket
to the comedy drama. If the price is set too low, the theatre will lose money; and if the price is too high, people
won’t come. From past experience she estimates that the profit P from sales (in hundreds) can be approximated
by P (x) =− x2 + 22x − 40 where x is the cost of a ticket and 0 # x # 25 hundred rupees.

(i) What is the lowest cost of a ticket that would allow the theatre to break even? What is the highest cost that
the theatre can charge to break even?
(ii) If theatre charge Rs 4 hundred for each ticket, what is the profit/loss ?
(iii) If theatre charge Rs 25 hundred for each ticket, what is the profit/loss ?
(iv) What is the maximum profit which can be earned by theatre ?

37. Height of a Pyramid : The angle of elevation to the top of the Egyptian pyramid of Cheops is 30c measured from a
point 50 meter from the base of the pyramid. The angle of elevation from the base of a face of the pyramid is 60c .

(i) Find the height of the Cheops pyramid.


(ii) Find the side of base of pyramid.

Install NODIA App to See the Solutions.


Click Here To Install
Page 8 Sample Paper 14 CBSE Mathematics Class 10

38. An inspector in an enforcement squad of electricity department visit to a locality of 100 families and record their
monthly consumption of electricity, on the basis of family members, electronic items in the house and wastage of
electricity, which is summarise in the following table.

Monthly Consumption (in kwh) Number of families


0-100 2
100-200 5
200-300 x
300-400 12
400-500 17
500-600 20
600-700 y
700-800 9
800-900 7
900-1000 4
Inspector calculated that median of the above data is 525 and after that he lost two data which is given as x and
y in table.
Based on the above information, answer the following questions.
(i) What is the value of lost data x ?
(ii) What is the value of lost data y ?
(iii) What will be the upper limit of the modal class?

 ******

Install NODIA App to See the Solutions.


Click Here To Install
Page 1 Sample Paper 15 Mathematics STD Class 10

Sample Paper 15
Class- X Exam - 2023-24
Mathematics - Standard

Time Allowed: 3 Hours Maximum Marks : 80


General Instructions :
1. This Question Paper has 5 Sections A-E.
2. Section A has 20 MCQs carrying 1 mark each.
3. Section B has 5 questions carrying 02 marks each.
4. Section C has 6 questions carrying 03 marks each.
5. Section D has 4 questions carrying 05 marks each.
6. Section E has 3 case based integrated units of assessment (04 marks each) with sub-parts of the values of 1, 1
and 2 marks each respectively.
7. All Questions are compulsory. However, an internal choice in 2 Qs of 5 marks, 2 Qs of 3 marks and 2 Questions
of 2 marks has been provided. An internal choice has been provided in the 2 marks questions of Section E.
8. Draw neat figures wherever required. Take π = 227 wherever required if not stated.

Section - A
Section A consists of 20 questions of 1 mark each.

1. Ramesh buys a fish from a shop for his aquarium. The shopkeeper takes out one fish at random a tank containing
5 male fish and 9 female fish. Then, the probability that the fish taken out is a male fish, is
(a) 5 (b) 5
13 14

(c) 6 (d) 7
13 13

2. A quadratic polynomial, whose zeroes are - 3 and 4, is


(a) x2 − x + 12 (b) x2 + x + 12

(c) x2 - x - 6 (d) 2x2 + 2x − 24


2 2

3. A number x is selected from the numbers 1, 2, 3 and then a second number y is randomly selected from the
numbers 1, 4, 9 then the probability that the product xy of the two numbers will be less than 9 is
(a) 3 (b) 4
7 9

(c) 5 (d) 7
9 9

4. The value of c for which the pair of equations cx − y = 2 and 6x − 2y = 3 will have is
(a) 3 (b) - 3
(c) - 12 (d) no value

5. If the common difference of an AP is 5, then what is a18 - a13 ?


(a) 5 (b) 20
(c) 25 (d) 30

https://qrbook.page.link/app
Install NODIA App to See the Solutions.
Click Here To Install
Page 2 Sample Paper 15 Mathematics STD Class 10

6. The perimeters of two similar triangles are 25 cm and 15 cm respectively. If one side of the first triangle is 9 cm,
then the corresponding side of second triangle is ................... .
(a) 5.4 cm (b) 5.2 cm
(c) 4.9 cm (d) 5.1 cm

7. In the adjoining figure, PT is a tangent at point C of the circle. O is the circumference of TABC . If +ACP = 118c
, then the measure of +x is

(a) 28c (b) 32c


(c) 42c (d) 38c

8. In the given figure, three circles with centres P , Q and R are drawn, such that the circles with centres Q and R
touch each other externally and they touch the circle with centre P , internally. If PQ = 10 cm , PR = 8 cm and
QR = 12 cm , then the diameter of the largest circle is:

(a) 30 cm (b) 20 cm
(c) 10 cm (d) None of these

9. If TABC is right angled at C , then the value of sec (A + B) is


(a) 0 (b) 1
(c) 2 (d) not defined
3

10. The quadratic equation x2 + x − 5 = 0 has


(a) two distinct real roots (b) two equal real roots
(c) no real roots (d) more than 2 real roots

https://qrbook.page.link/app
Install NODIA App to See the Solutions.
Click Here To Install
Page 3 Sample Paper 15 Mathematics STD Class 10

11. If x sin3 q + y cos3 q = sin q cos q and x sin q = y cos q , than x2 + y2 is equal to
(a) 0 (b) 1/2
(c) 1 (d) 3/2

12. From the top of a 7 m high building the angle of elevation of the top of a cable tower is 60c and the angle of
depression of its foot is 45c, then the height of the tower is
(a) 14.124 m (b) 17.124 m
(c) 19.124 m (d) 15.124 m

13. In a right angled TABC right angled at B , if P and Q are points on the sides AB and BC respectively, then
(a) AQ2 + CP2 = 2 ^AC2 + PQ2h
(b) 2 ^AQ2 + CP2h = AC2 + PQ2
(c) AQ2 + CP2 = AC2 + PQ2
(d) AQ + CP = 1 ^AC + PQ h
2

14. A circle artist is climbing a 20 m long rope, which is tightly stretched and tied from the top of a vertical pole to
the ground, then the height of pole, if the angle made by the rope with the ground level is 30c, is
(a) 5 m (b) 10 m
(c) 15 m (d) 20 m

15. If the perimeter of a semi-circular protractor is 36 cm, then its diameter is


(a) 10 cm (b) 14 cm
(c) 12 cm (d) 16 cm

16. A right circular cylinder of radius r and height h (where, h > 2r ) just encloses a sphere of diameter
(a) r (b) 2r
(c) h (d) 2h

17. The mean and median of the data a , b and c are 50 and 35 respectively, where a < b < c . If c − a = 55 , then
(b - a) is
(a) 8 (b) 7
(c) 3 (d) 5

18. For the following distribution

Marks Number of Students Marks Number of students


Below 10 3 Below 40 57
Below 20 12 Below 50 75
Below 30 28 Below 60 80
The modal class is
(a) 0-20 (b) 20-30
(c) 30-40 (d) 50-60

https://qrbook.page.link/app
Install NODIA App to See the Solutions.
Click Here To Install
Page 4 Sample Paper 15 Mathematics STD Class 10

In the question number 19 and 20, a statement of Assertion (A) is followed by a statement of Reason (R). Choose the
correction option.

19. Assertion : The roots of the quadratic equation x2 + 2x + 2 = 0 are imaginary.


Reason : If discriminant D = b2 − 4ac < 0 then the roots of quadratic equation ax2 + bx + c = 0 are imaginary.
(a) Both assertion (A) and reason (R) are true and reason (R) is the correct explanation of assertion (A).
(b) Both assertion (A) and reason (R) are true but reason (R) is not the correct explanation of assertion (A).
(c) Assertion (A) is true but reason (R) is false.
(d) Assertion (A) is false but reason (R) is true.

20. Assertion : PA and PB are two tangents to a circle with centre O . Such that +AOB = 110c, then +APB = 90c.
Reason : The length of two tangents drawn from an external point are equal.
(a) Both assertion (A) and reason (R) are true and reason (R) is the correct explanation of assertion (A).
(b) Both assertion (A) and reason (R) are true but reason (R) is not the correct explanation of assertion (A).
(c) Assertion (A) is true but reason (R) is false.
(d) Assertion (A) is false but reason (R) is true.

Section - B
Section B consists of 5 questions of 2 marks each.

21. If Sn the sum of first n terms of an AP is given by Sn = 3n2 − 4n , find the nth term.

22. In Figure a quadrilateral ABCD is drawn to circumscribe a circle, with centre O, in such a way that the sides AB,
BC, CD, and DA touch the circle at the points P, Q, R and S respectively. Prove that. AB + CD = BC + DA.

23. Prove that the point ^3, 0h , ^6, 4h and ^- 1, 3h are the vertices of a right angled isosceles triangle.

24. Check whether 4n can end with the digit 0 for any natural number n .

25. The 8th term of an AP is zero. Prove that its 38th term is triple of its 18th term.
 O
If the number x + 3, 2x + 1 and x - 7 are in AP find the value of x .

Continue
 on next page....

https://qrbook.page.link/app
Install NODIA App to See the Solutions.
Click Here To Install
Page 5 Sample Paper 15 Mathematics STD Class 10

Section - C
Section C consists of 6 questions of 3 marks each.

26. Determine the values of m and n so that the following system of linear equation have infinite number of solutions :
^2m − 1h x + 3y − 5 = 0
3x + ^n − 1h y − 2 = 0

27. Solve for x :


2x + 1 + 3x + 9 = 0, x ! 3, − 3
x − 3 2x + 3 ^x − 3h^2x + 3h 2

28. If a circle touches the side BC of a triangle ABC at P and extended sides AB and AC at Q and R , respectively,
prove that AQ = 12 (BC + CA + AB)
 O
In TABD, AB = AC. If the interior circle of TABC touches the sides AB, BC and CA at D, E and F respectively.
Prove that E bisects BC.

29. An aeroplane, when flying at a height of 4000 m from the ground passes vertically above another aeroplane at
an instant when the angles of elevation of the two planes from the same point on the ground are 60º and 45º
respectively. Find the vertical distance between the aeroplanes at that instant. (Use 3 = 1.73)

30. A hemispherical bowl of internal diameter 36 cm contains liquid is filled into 72 cylindrical bottles of diameter 6
cm. Find the height of the each bottle, if 10% liquid is wasted in this transfer.
 O
A hollow cylindrical pipe is made up of copper. It is 21 dm long. The outer and inner diameters of the pipe are 10
cm and 6 cm respectively. Find the volume of copper used in making the pipe.

31. A child has a die whose six faces show the letters as shown below:
A A B C C C
The die is thrown once. What is the probability of getting (i) A, (ii) C ?

Section - D
Section D consists of 4 questions of 5 marks each.

32. If β and 1
β are zeroes of the polynomial ^a2 + a h x2 + 61x + 6a . Find the value of β and α .

 O
Find the zeroes of the quadratic polynomial 7y2 - 113 y - 23 and verify the relationship between the zeroes and the
coefficients.

33. In TABC, AD is the median to BC and in TPQR, PM is the median to QR. If AB = BC = AD . Prove that
PQ QR PM
TABC~TPQR .

https://qrbook.page.link/app Install NODIA App to See the Solutions.


Click Here To Install
Page 6 Sample Paper 15 Mathematics STD Class 10

34. Prove that: sin θ = 2+ sin θ


cot θ + cosec θ cot θ − cosec θ
 O
2
If 3 cot θ − 4 cot θ + 3 = 0 , then find the value of cot2 θ + tan2 θ.

!!! !
35. Find the area of the shaded region in Figure, APD, AQB , BRC and CSD , are semi-circles of diameter 14 cm, 3.5
cm, 7 cm and 3.5 cm respectively. Use π = 227 .

Section - E
Case study based questions are compulsory.

36. Mutual Fund : A mutual fund is a type of financial vehicle made up of a pool of money collected from many
investors to invest in securities like stocks, bonds, money market instruments, and other assets. Mutual funds are
operated by professional money managers, who allocate the fund’s assets and attempt to produce capital gains or
income for the fund’s investors.

Net asset value (NAV) represents a fund’s per share market value. It is the price at which investors buy (“bid
price”) fund shares from a fund company and sell them (“redemption price”) to a fund company.
The following table below shows the net asset value (NAV) per unit of mutual funds of ICICI mutual funds.

Continue on next page....

https://qrbook.page.link/app
Install NODIA App to See the Solutions.
Click Here To Install
Page 7 Sample Paper 15 Mathematics STD Class 10

NAV (Rs) No. of mutual funds


0-5 13
6-10 16
11-15 22
16-20 18
21-25 11
Based on the above information, answer the following questions.
(i) What is the lower limit of model class of NAV?
(ii) What is the mode NAV of mutual funds ?
(iii) What is the mean NAV of mutual funds ?
 O
(iv) What is the median NAV of mutual funds ?

37. Carpooling : It is the sharing of car journeys so that more than one person travels in a car, and prevents the
need for others to have to drive to a location themselves. By having more people using one vehicle, carpooling
reduces each person’s travel costs such as: fuel costs, tolls, and the stress of driving. Carpooling is also a more
environmentally friendly and sustainable way to travel as sharing journeys reduces air pollution, carbon emissions,
traffic congestion on the roads, and the need for parking spaces.

Three friends Amar, Bandhu and Chakradev lives in societies represented by the points A, B and C respectively.
They all work in offices located in a same building represented by the point O . Since they all go to same building
everyday, they decided to do carpooling to save money on petrol. Based on the above information, answer the
following questions.

Continue on next page....

https://qrbook.page.link/app
Install NODIA App to See the Solutions.
Click Here To Install
Page 8 Sample Paper 15 Mathematics STD Class 10

(i) Which society is nearest to the office?


(ii) What is the distance between A and C ?
(iii) Find the least distance between AB , OA and BC ?
(iv) If Bandhu and Chakradev planned to meet at a club situated at the mid-point of the line joining the points
B and C , find the coordinates of this point.

38. Volume of Solid : A cuboidal solid of base x by x + 1 is shown in figure. Height of original solid is x + 2 . A small
cuboidal solid of base x - 2 by x - 2 and height 2 is cut from this solid as shown in figure.

(i) Find the polynomial for the volume of remaining solid.


(ii) Use the remainder theorem to determine the volume of remaining solid at x = 8 inch.
(iii) Use the polynomial to determine the volume at x = 8 inch and verify the result in (ii).
 O
Use the remainder theorem to determine the volume at x = 10 inch.

 ******

https://qrbook.page.link/app
Install NODIA App to See the Solutions.
Click Here To Install
Page 1 Sample Paper 16 Mathematics STD Class 10

Sample Paper 16
Class- X Exam - 2023-24
Mathematics - Standard

Time Allowed: 3 Hours Maximum Marks : 80


General Instructions :
1. This Question Paper has 5 Sections A-E.
2. Section A has 20 MCQs carrying 1 mark each
3. Section B has 5 questions carrying 02 marks each.
4. Section C has 6 questions carrying 03 marks each.
5. Section D has 4 questions carrying 05 marks each.
6. Section E has 3 case based integrated units of assessment (04 marks each) with sub-parts of the values of 1, 1
and 2 marks each respectively.
7. All Questions are compulsory. However, an internal choice in 2 Qs of 5 marks, 2 Qs of 3 marks and 2 Questions
of 2 marks has been provided. An internal choice has been provided in the 2 marks questions of Section E
8. Draw neat figures wherever required. Take π = 227 wherever required if not stated.

Section - A
Section A consists of 20 questions of 1 mark each.

1. The linear factors of the quadratic equation x2 + kx + 1 = 0 are


(a) k $ 2 (b) k # 2
(c) k $- 2 (d) 2 # k # - 2

2. The maximum number of zeroes a cubic polynomial can have, is


(a) 1 (b) 4
(c) 2 (d) 3

3. A fraction becomes 4 when 1 is added to both the numerator and denominator and it becomes 7 when 1 is
subtracted from both the numerator and denominator. The numerator of the given fraction is
(a) 2 (b) 3
(c) 5 (d) 15

4. The sum of exponents of prime factors in the prime-factorisation of 196 is


(a) 3 (b) 4
(c) 5 (d) 2

5. The pair of linear equations 2kx + 5y = 7 , 6x − 5y = 11 has a unique solution, if


(a) k !- 3 (b) k ! 2
3

(c) k!5 (d) k ! 2


9

https://qrbook.page.link/app
Install NODIA App to See the Solutions.
Click Here To Install
Page 2 Sample Paper 16 Mathematics STD Class 10

6. In an AP, if d =− 4 , n = 7 and an = 4 , then a is equal to


(a) 6 (b) 7
(c) 20 (d) 28

7. The 4th term from the end of an AP - 11, - 8 , - 5 , ....., 49 is


(a) 37 (b) 40
(c) 43 (d) 58

8. A chord of a circle of radius 10 cm, subtends a right angle at its centre. The length of the chord (in cm) is
(a) 5 (b) 5 2
2
(c) 10 2 (d) 10 3

9. If TABC is right angled at C , then the value ofcos ^A + B h is


(a) 0 (b) 1

(c) 1 (d) 3
2 2

10. If x − 2y + k = 0 is a median of the triangle whose vertices are at points A (- 1, 3), B (0, 4) and C (- 5, 2), then the
value of k is
(a) 2 (b) 4
(c) 6 (d) 8

11. A circle artist is climbing a 20 m long rope, which is tightly stretched and tied from the top of a vertical pole to
the ground, then the height of pole, if the angle made by the rope with the ground level is 30c, is
(a) 5 m (b) 10 m
(c) 15 m (d) 20 m

12. If the area of a semi-circular field is 15400 sq m, then perimeter of the field is
(a) 160 2 m (b) 260 2 m
(c) 360 2 m (d) 460 2 m

13. In a right angled TABC right angled at B , if P and Q are points on the sides AB and BC respectively, then
(a) AQ2 + CP2 = 2 ^AC2 + PQ2h (b) 2 ^AQ2 + CP2h = AC2 + PQ2
(c) AQ2 + CP2 = AC2 + PQ2 (d) AQ + CP = 1 ^AC + PQ h
2

14. Ratio of lateral surface areas of two cylinders with equal height is
(a) 1 : 2 (b) H : h
(c) R :r (d) None of these

15. For finding the popular size of readymade garments, which central tendency is used?
(a) Mean (b) Median
(c) Mode (d) Both Mean and Mode

https://qrbook.page.link/app
Install NODIA App to See the Solutions.
Click Here To Install
Page 3 Sample Paper 16 Mathematics STD Class 10

16. When a die is thrown, the probability of getting an odd number less than 3 is
(a) 1 (b) 1
6 3

(c) 1 (d) 0
2

17. If the point P (6, 2) divides the line segment joining A (6, 5) and B (4, y) in the ratio 3 : 1 then the value of y is
(a) 4 (b) 3
(c) 2 (d) 1

18. The ratio in which the point (2, y) divides the join of (- 4, 3) and (6, 3), hence the value of y is
(a) 2 : 3, y = 3 (b) 3 : 2, y = 4
(c) 3 : 2, y = 3 (d) 3 : 2, y = 2

In the question number 19 and 20, a statement of Assertion (A) is followed by a statement of Reason (R). Choose the
correction option.

19. Assertion : ^2 - 3 h is one zero of the quadratic polynomial then other zero will be ^2 + 3 h .
Reason : Irrational zeros (roots) always occurs in pairs.
(a) Both assertion (A) and reason (R) are true and reason (R) is the correct explanation of assertion (A).
(b) Both assertion (A) and reason (R) are true but reason (R) is not the correct explanation of assertion (A).
(c) Assertion (A) is true but reason (R) is false.
(d) Assertion (A) is false but reason (R) is true.

20. Assertion : The equation x2 + 3x + 1 = ^x − 2h2 is a quadratic equation.


Reason : Any equation of the form ax2 + bx + c = 0 where a ! 0 , is called a quadratic equation.
(a) Both assertion (A) and reason (R) are true and reason (R) is the correct explanation of assertion (A).
(b) Both assertion (A) and reason (R) are true but reason (R) is not the correct explanation of assertion (A).
(c) Assertion (A) is true but reason (R) is false.
(d) Assertion (A) is false but reason (R) is true.

Section - B
Section B consists of 5 questions of 2 marks each.

21. In Figure +D = +E and AD = AE , prove that TBAC is an isosceles triangle.


DB EC

https://qrbook.page.link/app
Install NODIA App to See the Solutions.
Click Here To Install
Page 4 Sample Paper 16 Mathematics STD Class 10

22. In figure, two tangents RQ and RP are drawn from an external point R to the circle with centre O . If +PRQ = 120c
, then prove that OR = PR + RQ .

23. If 3 sin θ − cos θ = 0 and 0º < θ < 90º, find the value of θ.

24. Find the value of λ , if the mode of the following data is 20 :


15, 20, 25, 18, 13, 15, 25, 15, 18, 17, 20, 25, 20, λ , 18.
 O

The mean and median of 100 observation are 50 and 52 respectively. The value of the largest observation is 100.
It was later found that it is 110. Find the true mean and median.

25. Prove that 3 + 5 is an irrational number.


 O

Show that any positive even integer can be written in the form 6q, 6q + 2 or 6q + 4, where q is an integer.

Section - C
Section C consists of 6 questions of 3 marks each.

26. Show that the sum of all terms of an AP whose first term is a , the second term is b and last term is c , is equal
(a + c) (b + c − 2a)
to
2 (b − a)

27. Prove that ^1 + cot A − cosec Ah^1 + tan A + sec Ah = 2

28. Sides of a right triangular field are 25 m, 24 m and 7 m. At the three corners of the field, a cow, a buffalo and a
horse are tied separately with ropes of 3.5 m each to graze in the field. Find the area of the field that cannot be
grazed by these animals.

Continue on next page....

https://qrbook.page.link/app
Install NODIA App to See the Solutions.
Click Here To Install
Page 5 Sample Paper 16 Mathematics STD Class 10

 O
In the given figure, find the area of the shaded region, enclosed between two concentric circles of radii 7 cm and
14 cm where +AOC = 40c. Use π = 227 .

29. The mean of the following distribution is 48 and sum of all the frequency is 50. Find the missing frequencies x
and y .

Class 20-30 30-40 40-50 50-60 60-70


Frequency 8 6 x 11 y

30. If the co-ordinates of points A and B are ^- 2, - 2h and ^2, - 4h respectively, find the co-ordinates of P such that
AP = 73 AB, where P lies on the line segment AB.
 O
Find the co-ordinates of the points of trisection of the line segment joining the points A ^1, - 2h and B ^- 3, 4h .

31. Three bells toll at intervals of 9, 12, 15 minutes respectively. If they start tolling together, after what time will
they next toll together?

Section - D
Section D consists of 4 questions of 5 marks each.

32. Aftab tells his daughter, ‘7 years ago, I was seven times as old as you were then. Also, 3 years from now, I shall
be three times as old as you will be.’ Represent this situation algebraically and graphically.
 O
Solve the following pair of linear equations graphically:
x - y = 1, 2x + y = 8
Also find the co-ordinates of the points where the lines represented by the above equation intersect y - axis.

33. From a point T outside a circle of centre O , tangents TP and TQ are drawn to the circle. Prove that OT is the
right bisector of line segment PQ .

34. From a point on the ground, the angles of elevation of the bottom and the top of a tower fixed at the top of a 20
m high building are 45c and 60c respectively. Find the height of the tower.
 O
The angle of elevation of the top B of a tower AB from a point X on the ground is 60c. At point Y , 40 m
vertically above X , the angle of elevation of the top is 45c. Find the height of the tower AB and the distance XB .

https://qrbook.page.link/app
Install NODIA App to See the Solutions.
Click Here To Install
Page 6 Sample Paper 16 Mathematics STD Class 10

35. Water is flowing through a cylindrical pipe, of internal diameter 2 cm, into a cylindrical tank of base radius 40 cm,
at the rate of 0.4 m/s. Determine the rise in level of water in the tank in half an hour.

Section - E
Case study based questions are compulsory.

36. John and Priya went for a small picnic. After having their lunch Priya insisted to travel in a motor boat. The speed
of the motor boat was 20 km/hr. Priya being a Mathematics student wanted to know the speed of the current. So
she noted the time for upstream and downstream.

She found that for covering the distance of 15 km the boat took 1 hour more for upstream than downstream.
(i) Let speed of the current be x km/hr. then speed of the motorboat in upstream will be
(ii) What is the relation between speed distance and time?
(iii) Write the correct quadratic equation for the speed of the current ?
 O
(iv) What is the speed of current ?

37. The centroid is the centre point of the object. It is also defined as the point of intersection of all the three medians.
The median is a line that joins the midpoint of a side and the opposite vertex of the triangle. The centroid of the
triangle separates the median in the ratio of 2 : 1. It can be found by taking the average of x- coordinate points
and y-coordinate points of all the vertices of the triangle.
See the figure given below

Continue on next page....

https://qrbook.page.link/app
Install NODIA App to See the Solutions.
Click Here To Install
Page 7 Sample Paper 16 Mathematics STD Class 10

Here D, E and F are mid points of sides BC , AC and AB in same order. G is centroid, the centroid divides
the median in the ratio 2 : 1 with the larger part towards the vertex. Thus AG : GD = 2 : 1
On the basis of above information read the question below.
If G is Centroid of 3 ABC with height h and J is centroid of 3 ADE . Line DE parallel to BC , cuts the
3 ABC at a height h4 from BC . HF = h4 .

(i) What is the length of AH ?


(ii) What is the distance of point A from point G ?
(iii) What is the distance of point A from point J?
 O
(iv) What is the distance GJ ?

38. Abhinav Bindra is retired sport shooter and currently India’s only individual Olympic gold medalist. His gold in
the 10-meter air rifle event at the 2008 Summer Olympics was also India’s first Olympic gold medal since 1980.
He is the first Indian to have held concurrently the world and Olympic titles for the men’s 10-meter air rifle event,
having earned those honors at the 2008 Summer Olympics and the 2006 ISSF World Shooting Championships.
Bindra has also won nine medals at the Commonwealth Games and three gold medals at the Asian Games.

https://qrbook.page.link/app
Install NODIA App to See the Solutions.
Click Here To Install
Page 8 Sample Paper 16 Mathematics STD Class 10

A circular dartboard has a total radius of 8 inch, with circular bands that are 2 inch wide, as shown in figure.
Abhinav is still skilled enough to hit this board 100% of the time so he always score at least two points each time
he throw a dart. Assume the probabilities are related to area, on the next dart that he throw.
(i) What is the probability that he score at least 4 ?
(ii) What is the probability that he score at least 6 ?
(iii) What is the probability that he hit bull’s eye ?

 O
(iv) What is the probability that he score exactly 4 points ?

 ******

https://qrbook.page.link/app
Install NODIA App to See the Solutions.
Click Here To Install
Page 1 Sample Paper 17 CBSE Mathematics Class 10

Sample Paper 17
Class- X Exam - 2023-24
Mathematics - Standard

Time Allowed: 3 Hours Maximum Marks : 80


General Instructions :
1. This Question Paper has 5 Sections A-E.
2. Section A has 20 MCQs carrying 1 mark each
3. Section B has 5 questions carrying 02 marks each.
4. Section C has 6 questions carrying 03 marks each.
5. Section D has 4 questions carrying 05 marks each.
6. Section E has 3 case based integrated units of assessment (04 marks each) with sub-parts.
7. All Questions are compulsory. However, an internal choice in 2 Qs of 5 marks, 2 Qs of 3 marks and 2 Questions
of 2 marks has been provided.
8. Draw neat figures wherever required. Take π = 227 wherever required if not stated.

Section - A
Section A consists of 20 questions of 1 mark each.

1. In the given figure, a circle touches all the four sides of quadrilateral ABCD with AB = 6 cm , BC = 7 cm and
CD = 4 cm , then length of AD is

(a) 3 cm (b) 4 cm (c) 5 cm (d) 6 cm

2. If sum of the zeroes of the quadratic polynomial 3x2 − kx + 6 is 3, then the value of k will be
(a) 1 (b) 4 (c) 6 (d) 9

3. The sum of exponents of prime factors in the prime-factorisation of 1764 is


(a) 3 (b) 4 (c) 5 (d) 6

4. Select the quadratic polynomial whose zeroes are reciprocals of the zeroes of the polynomial f (x) = ax2 + bx + c
, a ! 0, c ! 0.
(a) bx2 + ax + c (b) ax2 + cx + b (c) cx2 + bx + a (d) bx2 + cx + a

Install NODIA App to See the Solutions.


Click Here To Install
Page 2 Sample Paper 17 CBSE Mathematics Class 10

5. In the given figure, the positions of the observer and the object are mentioned, the angle of depression is

(a) 30c (b) 90c (c) 60c (d) 45c

6. For the following distribution.

Class 0-5 5-10 10-15 15-20 20-25


Frequency 10 15 12 20 9
the sum of lower limits of the median class and modal class is
(a) 15 (b) 25 (c) 30 (d) 35

7. If a and b are the zeroes of polynomial x2 + ax + b , the values of a and b are


(a) 1 and 2 (b) 1 and –2 (c) –2 and 1 (d) 2 and 1

8. What do you say about the lines represented by 2x + y = 3 and 4x + 2y = 6 ?


(a) lines are parallel (b) lines are coincident
(c) lines are intersecting (d) can’t say anything

9. Select the value of k for which the pair of Linear equations kx + y = d2 and x + ky = 1 have infinitely many
solutions.
(a) 1 (b) 2 (c) 3 (d) 4

10. The quadratic equation x2 − 4x + 3 2 = 0 has


(a) two distinct real roots (b) two equal real roots
(c) no real roots (d) more than 2 real roots

11. In an AP, if d =− 4 , n = 7 and an = 4 , then a is equal to


(a) 6 (b) 7 (c) 20 (d) 28

12. The perimeters of two similar triangles are 25 cm and 15 cm respectively. If one side of the first triangle is 9 cm,
then the corresponding side of second triangle is ................... .
(a) 4.2 cm (b) 5.4 cm (c) 20 cm (d) 6 cm

13. The distance of a point P (x, y) from the origin is


(a) x2 − xy + y2 (b) x2 + xy + y2 (c) x2 + y2 (d) x2 + 3xy + y2

Install NODIA App to See the Solutions.


Click Here To Install
Page 3 Sample Paper 17 CBSE Mathematics Class 10

14. In a triangle ABC, cos b B + C l will be


2
A
(a) sin 4 (b) cos A (c) sin A2 (d) cos A2

15. sin A − 2 sin3 A = ?


2 cos3 A − cos A
(a) sin A (b) tan A (c) cos A (d) cot A

16. Volume of a spherical shell is given by


(a) 4π (R2 − r2) (b) π (R3 − r3) (c) 4π (R3 − r3) (d) 4
3 π (R3 − r3)

17. The sum of first 16 terms of the AP 10, 6, 2, ..... is


(a) - 320 (b) 320 (c) - 352 (d) - 400

18. A bag contains 6 red and 5 blue balls. What is the probability that the ball drawn is not red?
(a) 115 (b) 116 (c) 112 (d) 117

In the question number 19 and 20, a statement of Assertion (A) is followed by a statement of Reason (R). Choose the
correction option.

19. Assertion : The roots of the quadratic equation x2 + 2x + 2 = 0 are imaginary.


Reason : If discriminant D = b2 − 4ac < 0 then the roots of quadratic equation ax2 + bx + c = 0 are imaginary.
(a) Both assertion (A) and reason (R) are true and reason (R) is the correct explanation of assertion (A).
(b) Both assertion (A) and reason (R) are true but reason (R) is not the correct explanation of assertion (A).
(c) Assertion (A) is true but reason (R) is false.
(d) Assertion (A) is false but reason (R) is true.

20. Assertion : In a circle of radius 6 cm, the angle of a sector 60c. Then the area of the sector is 18 76 cm2 .
Reason : Area of the circle with radius r is πr2 .
(a) Both assertion (A) and reason (R) are true and reason (R) is the correct explanation of assertion (A).
(b) Both assertion (A) and reason (R) are true but reason (R) is not the correct explanation of assertion (A).
(c) Assertion (A) is true but reason (R) is false.
(d) Assertion (A) is false but reason (R) is true.

Section - B
Section B consists of 5 questions of 2 marks each.

21. Check whether 4n can end with the digit 0 for any natural number n .

22. If α and β are zeroes of x2 − ^k − 6h x + 2 ^2k − 1h , find the value of k if α + β = 12 αβ .


 O
Find the quadratic polynomial, the sum and product of whose zeroes are - 3 and 2 respectively. Hence find the
zeroes.

23. If the distance between the points A (4, p) and B (1, 0) is 5 units then what are the values of p ?

Install NODIA App to See the Solutions.


Click Here To Install
Page 4 Sample Paper 17 CBSE Mathematics Class 10

24. In the figure, PQRS is a trapezium in which PQ || RS. On PQ and RS, there are points E and F respectively
such that EF intersects SQ at G . Prove that EQ # GS = GQ # FS.

 O
In the given figure, +A = +B and AD = BE. Show that DE || AB.

25. Harpreet tosses two different coins simultaneously. What is the probability that she gets :
(i) at least one head ?
(ii) one head and one tail ?

Section - C
Section C consists of 6 questions of 3 marks each.

26. Three bells toll at intervals of 9, 12, 15 minutes respectively. If they start tolling together, after what time will
they next toll together?

27. In the given figure, if +A = 90º, +B = 90º, OB = 4.5 cm OA = 6 cm and AP = 4 cm then find QB.

Install NODIA App to See the Solutions.


Click Here To Install
Page 5 Sample Paper 17 CBSE Mathematics Class 10

28. If sin A = 3 , find the value of 2 cot2 A - 1.


2

29. If a circle touches the side BC of a triangle ABC at P and extended sides AB and AC at Q and R , respectively,
prove that AQ = 12 (BC + CA + AB)

30. If the total surface area of a solid hemisphere is 462 cm2, find its volume. Use π = 22
7

 O
A wooden article was made by scooping out a hemisphere from each end of a solid cylinder, as shown in Figure.
If the height of the cylinder is 10 cm and its base is of radius 3.5 cm, find the total surface area of the article.

31. Find the median of the following data :

Height (in cm) Less than 120 Less than 140 Less than 160 Less than 180 Less than 200
Number of students 12 26 34 40 50

 O
Find the mean of the following distribution :

Height (in cm) Less than 75 Less than 100 Less than 125 Less than 150 Less than 175 Less than 200
No. of students 5 11 14 18 21 28
Height (in cm) Less than 225 Less than 250 Less than 275 Less than 300
No. of students 33 37 45 50

Section - D
Section D consists of 4 questions of 5 marks each.

32. If the roots of the quadratic equation ^x − a h^x − b h + ^x − b h^x − c h + ^x − c h^x − a h = 0 are equal. Then show that
a = b = c.
 O
A fast train takes 3 hours less than a slow train for a journey of 60 km. If the speed of the slow train is 10 km/h
less than that of the fast train, find the speed of each train.

33. Find the ratio in which the point P ^ 34 , 125 h divides the line segment joining the point A ^ 12 , 32 h and ^2, - 5h .

Install NODIA App to See the Solutions.


Click Here To Install
Page 6 Sample Paper 17 CBSE Mathematics Class 10

34. From a point T outside a circle of centre O , tangents TP and TQ are drawn to the circle. Prove that OT is the
right bisector of line segment PQ .
 O
In Figure, PQ and RS are two parallel tangents to a circle with centre O and another tangent AB with point of
contact C intersecting PQ at A and RS at B . Prove that +AOB = 90c.

35. Water is flowing at the rate of 15 km/hr through a cylindrical pipe of diameter 14 cm into a cuboidal pond which
is 50 m long and 44 m wide. In what time the level of water in pond rise by 21 cm ?

Section - E
Case study based questions are compulsory.

36. TOWER OF PISA : To prove that objects of different weights fall at the same rate, Galileo dropped two objects
with different weights from the Leaning Tower of Pisa in Italy. The objects hit the ground at the same time. When
an object is dropped from a tall building, it falls about 16 feet in the first second, 48 feet in the second, and 80
feet in the third second, regardless of its weight.
(i) How many feet would an object fall in the sixth second?
(ii) How many feet would an object fall in the six second?
(iii) How many feet would an object fall in the eight second?

37. Speed Limit Enforcement : Rajendra works in traffic police and manage traffic on highway. His van is having radar
detection equipment. He takes up a hidden position 50 meter from the highway. Using a sighting device he finds
the angle between his position and a road sign in the distance is 60c .

Continue on next page....

Install NODIA App to See the Solutions.


Click Here To Install
Page 7 Sample Paper 17 CBSE Mathematics Class 10

He then uses a stop watch to determine how long it takes a vehicle to pass her location and reach the road sign.
In quick succession—an 18-wheeler, a truck, and a car pass her position, with the time each takes to travel this
distance noted. Find the speed of each vehicle in miles per hour if
(i) the 18-wheeler takes 8 sec,
(ii) the truck takes 6 sec,
(iii) the car takes 4 sec.
38. Abhinav Bindra is retired sport shooter and currently India’s only individual Olympic gold medalist. His gold in
the 10-meter air rifle event at the 2008 Summer Olympics was also India’s first Olympic gold medal since 1980.
He is the first Indian to have held concurrently the world and Olympic titles for the men’s 10-meter air rifle event,
having earned those honors at the 2008 Summer Olympics and the 2006 ISSF World Shooting Championships.
Bindra has also won nine medals at the Commonwealth Games and three gold medals at the Asian Games.

A circular dartboard has a total radius of 8 inch, with circular bands that are 2 inch wide, as shown in figure.
Abhinav is still skilled enough to hit this board 100% of the time so he always score at least two points each time
he throw a dart. Assume the probabilities are related to area, on the next dart that he throw.
(i) What is the probability that he score at least 4 ?
(ii) What is the probability that he score at least 6 ?
(iii) What is the probability that he hit bull’s eye ?
(iv) What is the probability that he score exactly 4 points ?

 ******

Install NODIA App to See the Solutions.


Click Here To Install
Page 1 Sample Paper 18 Mathematics STD Class 10

Sample Paper 18
Class- X Exam - 2023-24
Mathematics - Standard

Time Allowed: 3 Hours Maximum Marks : 80


General Instructions :
1. This Question Paper has 5 Sections A-E.
2. Section A has 20 MCQs carrying 1 mark each
3. Section B has 5 questions carrying 02 marks each.
4. Section C has 6 questions carrying 03 marks each.
5. Section D has 4 questions carrying 05 marks each.
6. Section E has 3 case based integrated units of assessment (04 marks each) with sub-parts of the values of 1, 1
and 2 marks each respectively.
7. All Questions are compulsory. However, an internal choice in 2 Qs of 5 marks, 2 Qs of 3 marks and 2 Questions
of 2 marks has been provided. An internal choice has been provided in the 2 marks questions of Section E
8. Draw neat figures wherever required. Take π = 227 wherever required if not stated.

Section - A
Section A consists of 20 questions of 1 mark each.

1. If the difference of mode and median of a data is 24, then the difference of median and mean is
(a) 12 (b) 24 (c) 08 (d) 36

2. If α and β are zeroes and the quadratic polynomial f ^x h = x2 − x − 4 , then the value of 1 + 1 − αβ is
α β
(a) 15 (b) - 15
4 4
(c) 4 (d) 15

3. The values of x and y in the given figure are

(a) 7, 13 (b) 13, 7 (c) 9, 12 (d) 12, 9

4. The father’s age is six times his son’s age. Four years hence, the age of the father will be four times his son’s age.
The present ages (in year) of the son and the father are, respectively.
(a) 4 and 24 (b) 5 and 30
(c) 6 and 36 (d) 3 and 24

https://qrbook.page.link/app
Install NODIA App to See the Solutions.
Click Here To Install
Page 2 Sample Paper 18 Mathematics STD Class 10

5. If the sum of the zeroes of the quadratic polynomial kx2 + 2x + 3k is equal to their product, then k equals
(a) 1 (b) - 1
3 3

(c) 2 (d) - 2
3 3

6. In an AP, if a = 3.5 , d = 0 and n = 101, then an will be


(a) 0 (b) 3.5
(c) 103.5 (d) 104.5

7. The pair of equations x + 2y + 5 = 0 and − 3x − 6y + 1 = 0 has


(a) a unique solution (b) exactly two solutions
(c) infinitely many solutions (d) no solution

8. It is given that, TABC + TEDF such that AB = 5 cm, AC = 7 cm, DF = 15 cm and DE = 12 cm then the sum
of the remaining sides of the triangles is
(a) 23.05 cm (b) 16.8 cm
(c) 6.25 cm (d) 24 cm

9. QP is a tangent to a circle with centre O at a point P on the circle. If TOPQ is isosceles, then +OQR equals.
(a) 30c (b) 45c
(c) 60c (d) 90c

10. If sin α = 1 and cos β = 1 , then the value of ^α + β h is


2 2
(a) 0c (b) 30c
(c) 60c (d) 90c

11. The famous mathematician associated with finding the sum of the first 100 natural numbers is
(a) Pythagoras (b) Newton
(c) Gauss (d) Euclid

12. In the given figure, the positions of the observer and the object are mentioned, the angle of depression is

(a) 30c (b) 90c


(c) 60c (d) 45c

https://qrbook.page.link/app
Install NODIA App to See the Solutions.
Click Here To Install
Page 3 Sample Paper 18 Mathematics STD Class 10

13. Volume of a spherical shell is given by


(a) 4π (R2 − r2) (b) π (R3 − r3)
(c) 4π (R3 − r3) (d) 4
3 π (R3 − r3)

14. The probability of getting a bad egg in a lot of 400 is 0.035. The number of bad eggs in the lot is
(a) 7 (b) 14
(c) 21 (d) 28

15. The area of the circle that can be inscribed in a square of side 6 cm is
(a) 36π cm2 (b) 18π cm2
(c) 12π cm2 (d) 9π cm2

16. The distance between the points (a cos θ + b sin θ, 0), and (0, a sin θ − b cos θ) is
(a) a2 + b2 (b) a2 - b2
(c) a2 + b2 (d) a2 - b2

17. If the centre of a circle is (3, 5) and end points of a diameter are (4, 7) and (2, y ), then the value of y is
(a) 3 (b) - 3
(c) 7 (d) 4

18. The point P on x -axis equidistant from the points A (- 1, 0) and B (5, 0) is
(a) (2, 0) (b) (0, 2)
(c) (3, 0) (d) (- 3, 5)

In the question number 19 and 20, a statement of Assertion (A) is followed by a statement of Reason (R). Choose the
correction option.

19. Assertion : If one zero of poly-nominal p _x i = ^k2 + 4h x2 + 13x + 4k is reciprocal of other, then k = 2 .
Reason : If ^x − αh is a factor of p ^x h , then p ^αh = 0 i.e. α is a zero of p _x i .
(a) Both assertion (A) and reason (R) are true and reason (R) is the correct explanation of assertion (A).
(b) Both assertion (A) and reason (R) are true but reason (R) is not the correct explanation of assertion (A).
(c) Assertion (A) is true but reason (R) is false.
(d) Assertion (A) is false but reason (R) is true.

20. Assertion : The values of x are - a2 , a for a quadratic equation 2x2 + ax − a2 = 0 .


Reason : For quadratic equation ax2 + bx + c = 0
2
x = − b ! b − 4ac
2a
(a) Both assertion (A) and reason (R) are true and reason (R) is the correct explanation of assertion (A).
(b) Both assertion (A) and reason (R) are true but reason (R) is not the correct explanation of assertion (A).
(c) Assertion (A) is true but reason (R) is false.
(d) Assertion (A) is false but reason (R) is true.

https://qrbook.page.link/app
Install NODIA App to See the Solutions.
Click Here To Install
Page 4 Sample Paper 18 Mathematics STD Class 10

21. In the given figure, G is the mid-point of the side PQ of TPQR and GH || QR. Prove that H is the mid-point of
the side PR or the triangle PQR.

22. In figure, O is the centre of a circle. PT are tangents to the circle from an external point P . If +TPQ = 70c,
find +TRQ .

23. Evaluate : cos 45º + 1


sec 30º sec 60º

24. Find the arithmetic mean of the following frequency distribution :

xi 3 4 5 7 10
fi 3 4 8 5 10

 O
Given below is the distribution of weekly pocket money received by students of a class. Calculate the pocket money
that is received by most of the students.

Pocket Money (in Rs.) 0-20 20-40 40-60 60-80 80-100 100-120 120-140
Number of students. 2 2 3 12 18 5 2

25. a and b are two positive integers such that the least prime factor of a is 3 and the least prime factor of b is 5.
Then calculate the least prime factor of (a + b).
 O
What are the values of x and y in the given figure ?

https://qrbook.page.link/app
Install NODIA App to See the Solutions.
Click Here To Install
Page 5 Sample Paper 18 Mathematics STD Class 10

Section - C
Section C consists of 6 questions of 3 marks each.

26. If in an AP, the sum of first m terms is n and the sum of its first n terms is m , then prove that the sum of its
first (m + n) terms is − (m + n).

27. Prove that sin A − cos A − 1 = 1


sin A + cos A − 1 sec A − tan A

28. Find the area of minor segment of a circle of radius 14 cm, when its centre angle is 60c. Also find the area of
corresponding major segment. Use π = 227 .
 O
In the given figure, TPQR is an equilateral triangle of side 8 cm and D, E, F are centres of circular arcs, each of
radius 4 cm. Find the area of shaded region. (Use π = 3.14 ) and 3 = 1.732

29. The table below shows the daily expenditure on food of 25 households in a locality. Find the mean daily expenditure
on food.

Daily expenditure (in <) 100-150 150-200 200-250 250-300 300-350


Number of households 4 5 12 2 2

30. If the distance of P ^x, y h from A ^6, 2h and B ^- 2, 6h are equal, prove that y = 2x.
 O
If ^a, b h is the mid-point of the segment joining the points A ^10, - 6h and B ^k, 4h and a − 2b = 18, find the value
of k and the distanceAB .

https://qrbook.page.link/app
Install NODIA App to See the Solutions.
Click Here To Install
Page 6 Sample Paper 18 Mathematics STD Class 10

31. Find HCF and LCM of 16 and 36 by prime factorization and check your answer.

Section - D
Section D consists of 4 questions of 5 marks each.

32. For Uttarakhand flood victims two sections A and B of class contributed Rs. 1,500. If the contribution of X-A was
Rs. 100 less than that of X-B, find graphically the amounts contributed by both the sections.
 O
Draw the graph of the following equations:
2x - y = 1, x + 2y = 13
Find the solution of the equations from the graph and shade the triangular region formed by the lines and the y
-axis.

33. Prove that the parallelogram circumscribing a circle is a rhombus.

34. From the top of a 7 m high building the angle of elevation of the top of a tower is 60c and the angle of depression
of its foot is 45c. Determine the height of the tower.
 O
A vertical tower stands on a horizontal plane and is surmounted by a flagstaff of height 5 m. From a point on the
ground the angles of elevation of top and bottom of the flagstaff are 60c and 30c respectively. Find the height of
the tower and the distance of the point from the tower. (take 3 = 1.732 )

35. Water is flowing at the rate of 15 km/hr through a cylindrical pipe of diameter 14 cm into a cuboidal pond which
is 50 m long and 44 m wide. In what time the level of water in pond rise by 21 cm ?

Section - E
Case study based questions are compulsory.
36. Nidhi and Ria are very close friends. Nidhi’s parents own a Maruti Alto. Ria’s parents own a Toyota Liva. Both
the families decide to go for a picnic to Somnath temple in Gujrat by their own cars.

Nidhi’s car travels x km/h while Ria’s car travels 5 km/h more than Nidhi’s car. Nidhi’s car took 4 hrs more than
Ria’s car in covering 400 km.
(i) What will be the distance covered by Ria’s car in two hour?
(ii) Write the quadratic equation that describe the speed of Nidhi’s car?
(iii) What is the speed of Nidhi’s car?
 O
(iv) How much time did Ria take to travel 400 km?

https://qrbook.page.link/app
Install NODIA App to See the Solutions.
Click Here To Install
Page 7 Sample Paper 18 Mathematics STD Class 10

37. Two poles, 30 feet and 50 feet tall, are 40 feet apart and perpendicular to the ground. The poles are supported
by wires attached from the top of each pole to the bottom of the other, as in the figure. A coupling is placed at
C where the two wires cross.

(i) What is the horizontal distance from C to the taller pole?


(ii) How high above the ground is the coupling ?
(iii) How far down the wire from the smaller pole is the coupling ?

38. Blood Group : Blood type or blood group is a medical term. It describes the type of blood a person has. It is
a classification of blood based on the presence or absence of inherited antigenic substances on the surface of red
blood cells (RBCs). Blood types predict whether a serious reaction will occur in a blood transfusion.

In a sample of 50 people, 21 had type O blood, 22 had type A blood, 5 had type B blood, and 2 had type AB
blood. Set up a frequency distribution and find the following probabilities.
(i) What is the probability that a person has type O blood ?
(ii) What is the probability that a person has type A or type B blood ?
(iii) What is the probability that a person has neither type A nor type O blood ?
 O
(iv) What is the probability that a person does not have type AB blood ?

 ******

https://qrbook.page.link/app
Install NODIA App to See the Solutions.
Click Here To Install
Page 1 Sample Paper 19 Mathematics STD Class 10

Sample Paper 19
Class- X Exam - 2023-24
Mathematics - Standard

Time Allowed: 3 Hours Maximum Marks : 80


General Instructions :
1. This Question Paper has 5 Sections A-E.
2. Section A has 20 MCQs carrying 1 mark each
3. Section B has 5 questions carrying 02 marks each.
4. Section C has 6 questions carrying 03 marks each.
5. Section D has 4 questions carrying 05 marks each.
6. Section E has 3 case based integrated units of assessment (04 marks each) with sub-parts of the values of 1, 1
and 2 marks each respectively.
7. All Questions are compulsory. However, an internal choice in 2 Qs of 5 marks, 2 Qs of 3 marks and 2 Questions
of 2 marks has been provided. An internal choice has been provided in the 2 marks questions of Section E
8. Draw neat figures wherever required. Take π = 227 wherever required if not stated.

Section - A
Section A consists of 20 questions of 1 mark each.

1. The quadratic equation x2 − 4x − 3 2 = 0 has


(a) two distinct real roots (b) two equal real roots
(c) no real roots (d) more than 2 real roots

2. If the square of difference of the zeroes of the quadratic polynomial x2 + px + 45 is equal to 144, then the value
of p is
(a) ! 9 (b) ! 12
(c) ! 15 (d) ! 18

3. In the adjoining figure, TP and TQ are the two tangents to a circle with centre O . If +POQ = 110c, then +PTQ
is

(a) 60c (b) 70c


(c) 80c (d) 90c

https://qrbook.page.link/app
Install NODIA App to See the Solutions.
Click Here To Install
Page 2 Sample Paper 19 Mathematics STD Class 10

4. The pair of equations x = a and y = b graphically represents lines which are


(a) parallel (b) intersecting at (b, a)
(c) coincident (d) intersecting at (a, b)

5. A set of numbers consists of three 4’s, five 5’s, six 6’s, eight 8’s and seven 10’s. The mode of this set of numbers is
(a) 6 (b) 7
(c) 8 (d) 10

6. If x2 + y2 = 25 , xy = 12 , then x is
(a) (3, 4) (b) (3, - 3)
(c) (3, 4, - 3, - 4) (d) (3, - 3)

7. TABC is an equilateral triangle with each side of length 2p . If AD = BC then the value of AD is

(a) 3 (b) 3p
(c) 2p (d) 4p

8. TABC and TBDE are two equilateral triangle such that D is the mid-point of BC . Ratio of the areas of triangles
ABC and BDE is ................. .
(a) 1 : 4 (b) 4 : 1
(c) 1:3 (d) 3 : 1

9. From an external point P , tangents PA and PB are drawn to a circle with centre O . If CD is the tangent to the
circle at a point E and PA = 14 cm . The perimeter of TPCD is
(a) 14 cm (b) 21 cm
(c) 28 cm (d) 35 cm

10. (cos 4 A - sin 4 A) is equal to


(a) 1 - 2 cos2 A (b) 2 sin2 A - 1
(c) sin2 A - cos2 A (d) 2 cos2 A - 1

11. An observer, 1.5 m tall is 20.5 away from a tower 22 m high, then the angle of elevation of the top of the tower
from the eye of observer is
(a) 30c (b) 45c
(c) 60c (d) 90c

12. A tree casts a shadow 15 m long on the level of ground, when the angle of elevation of the sun is 45c. The height
of a tree is
(a) 10 m (b) 14 m
(c) 8m (d) 15 m

https://qrbook.page.link/appInstall NODIA App to See the Solutions.


Click Here To Install
Page 3 Sample Paper 19 Mathematics STD Class 10

13. From a solid circular cylinder with height 10 cm and radius of the base 6 cm, a right circular cone of the same
height and same base is removed, then the volume of remaining solid is
(a) 280 πcm3 (b) 330 πcm3
(c) 240 πcm3 (d) 440 π cm3

14. If median is 137 and mean is 137.05, then the value of mode is
(a) 156.90 (b) 136.90
(c) 186.90 (d) 206.90

15. If a number x is chosen at random from the numbers - 2, - 1, 0, 1, 2 . Then, the probability that x2 1 2 is
(a) 2 (b) 4
5 5

(c) 1 (d) 3
5 5

16. If the circumference of a circle increases from 4 π to 8 π , then its area is


(a) halved (b) doubled
(c) tripled (d) quadrupled

17. Which of the following relationship is the correct?


(a) P (E ) + P (E ) = 1 (b) P (E ) − P (E ) = 1
(c) P (E ) = 1 + P (E ) (d) None of these

18. A chord of a circle of radius 10 cm, subtends a right angle at its centre. The length of the chord (in cm) is
(a) 5 (b) 5 2
2
(c) 10 2 (d) 10 3

In the question number 19 and 20, a statement of Assertion (A) is followed by a statement of Reason (R). Choose the
correction option.

19. Assertion : an - an - 1 is not independent of n then the given sequence is an AP.


Reason : Common difference d = an − an − 1 is constant or independent of n .
(a) Both assertion (A) and reason (R) are true and reason (R) is the correct explanation of assertion (A).
(b) Both assertion (A) and reason (R) are true but reason (R) is not the correct explanation of assertion (A).
(c) Assertion (A) is true but reason (R) is false.
(d) Assertion (A) is false but reason (R) is true.

20. Assertion : The value of sin θ = 43 is not possible.


Reason : Hypotenuse is the largest side in any right angled triangle.
(a) Both assertion (A) and reason (R) are true and reason (R) is the correct explanation of assertion (A).
(b) Both assertion (A) and reason (R) are true but reason (R) is not the correct explanation of assertion (A).
(c) Assertion (A) is true but reason (R) is false.
(d) Assertion (A) is false but reason (R) is true.

https://qrbook.page.link/app
Install NODIA App to See the Solutions.
Click Here To Install
Page 4 Sample Paper 19 Mathematics STD Class 10

Section - B
Section B consists of 5 questions of 2 marks each.

21. How many two digits numbers are divisible by 3?

22. From an external point P , tangents PA and PB are drawn to a circle with centre O. If +PAB = 50º, then find
+AOB.

23. Find the ratio in which the point ^- 3, k h divides the line segment joining the points ^- 5, - 4h and ^- 2, 3h .Also
find the value of k .

24. Write a rational number between 2 and 3.

 O
Explain why (7 # 13 # 11) + 11 and (7 # 6 # 5 # 4 # 3 # 2 # 1) + 3 are composite numbers.

25. Find the 7th term from the end of AP 7, 10, 13, .... 184.
 O
The fourth term of an AP is 11. The sum of the fifth and seventh terms of the AP is 34. Find the common
difference.

Section - C
Section C consists of 6 questions of 3 marks each.

26. A fraction becomes 13 when 2 is subtracted from the numerator and it becomes 1
2 when 1 is subtracted from the
denominator- Find the fraction.

27. Solve for x : 1 − 1 = 11 x !- 4, - 7 .


x+4 x+7 30

28. Prove that the rectangle circumscribing a circle is a square.


 O
If O is centre of a circle, PQ is a chord and the tangent PR at P makes an angle of 50c with PQ , find +POQ .

https://qrbook.page.link/app
Install NODIA App to See the Solutions.
Click Here To Install
Page 5 Sample Paper 19 Mathematics STD Class 10

29. An electric pole is 10 m high. A steel wire tied to top of the pole is affixed at a point on the ground to keep the
pole up right. If the wire makes an angle of 45º with the horizontal through the foot of the pole, find the length
of the wire. [Use 2 = 1.414 ]

30. From a solid right circular cylinder of height 14 cm and base radius 6 cm, a right circular cone of same height and
same base removed. Find the volume of the remaining solid.
 O
A metallic cylinder has radius 3 cm and height 5 cm. To reduce its weights, a conical hole is drilled in the cylinder.
The conical hole has a radius of 32 cm and its depth 89 cm. Calculate the ratio of the volume of metal left in the
cylinder to the volume of metal taken out in conical shape.

31. Two dice are tossed simultaneously. Find the probability of getting
(i) an even number on both dice.
(ii) the sum of two numbers more than 9.

Section - D
Section D consists of 4 questions of 5 marks each.

32. Find the zeroes of the quadratic polynomial 7y2 - 113 y - 23 and verify the relationship between the zeroes and the
coefficients.
 O
If α and β are the zeroes the polynomial 2x2 − 4x + 5, find the values of
(i) α2 + β2 (ii) 1 + 1
α β

(iii) ^α − βh (iv) 12 + 12
2
α β
(v) α2 + β2

33. In TABC, AD is a median and O is any point on AD. BO and CO on producing meet AC and AB at E and F
respectively. Now AD is produced to X such that OD = DX as shown in figure.
Prove that :
(1) EF || BC
(2) AO : AX = AF : AB

https://qrbook.page.link/app
Install NODIA App to See the Solutions.
Click Here To Install
Page 6 Sample Paper 19 Mathematics STD Class 10

34. If sin A = 3 calculate sec A.


4
 O
Evaluate : 4 ^sin 30º + cos 60ºh − 3 ^cos 45 − sin 90ºh
4 4 2 2

35. In Figure, a square OABC is inscribed in a quadrant OPBQ . If OA = 15 cm , find the area of the shaded region.
(Use π = 3.14 ).

Section - E
Case study based questions are compulsory.

36. Heart Rate : The heart rate is one of the ‘vital signs,’ or the important indicators of health in the human body. It
measures the number of times per minute that the heart contracts or beats. The speed of the heartbeat varies as a
result of physical activity, threats to safety, and emotional responses. The resting heart rate refers to the heart rate
when a person is relaxed. While a normal heart rate does not guarantee that a person is free of health problems, it
is a useful benchmark for identifying a range of health issues. After the age of 10 years, the heart rate of a person
should be between 60 and 100 beats per minute while they are resting.

Thirty women were examined by doctors of AIIMS and the number of heart beats per minute were recorded and
summarised as follows.

Number of heart beats per minute Number of women ^ f ih


65-68 2
68-71 4
71-74 3
74-77 8
77-80 7
80-83 4
83-86 2

Continue on next page....

https://qrbook.page.link/app
Install NODIA App to See the Solutions.
Click Here To Install
Page 7 Sample Paper 19 Mathematics STD Class 10

Based on the above information, answer the following questions.


(i) What is the mean heart beats per minute for these women ?
(ii) What is the upper limit of median value of heart beats per minute for these women ?
(iii) What is the lower limit of mode value of heart beats per minute for these women ?
 O
How many women are having heart beat in range 68-77?

37. Morning assembly is an integral part of the school’s schedule. Almost all the schools conduct morning assemblies
which include prayers, information of latest happenings, inspiring thoughts, speech, national anthem, etc. A good
school is always particular about their morning assembly schedule. Morning assembly is important for a child’s
development. It is essential to understand that morning assembly is not just about standing in long queues and
singing prayers or national anthem, but it’s something beyond just prayers. All the activities carried out in
morning assembly by the school staff and students have a great influence in every point of life. The positive effects
of attending school assemblies can be felt throughout life.

Have you noticed that in school assembly you always stand in row and column and this make a coordinate system.
Suppose a school have 100 students and they all assemble in prayer in 10 rows as given below.

Here A, B, C and D are four friend Amar, Bharat, Colin and Dravid.
(i) What is the distance between A and B ?
(ii) What is the distance between C and D ?
(iii) What is the distance between A and C ?
 O
What is the distance between D and B ?

https://qrbook.page.link/appInstall NODIA App to See the Solutions.


Click Here To Install
Page 8 Sample Paper 19 Mathematics STD Class 10

38. Volume of a Bird Cage. A company makes rectangular shaped bird cages with height b inches and square bottoms.
The volume of these cages is given by the function V = b 3 − 6b2 + 9b .
(i) Find an expression for the length of each side of the square bottom.
(ii) Use the function to find the volume of a cage with a height of 18 inches.
(iii) Use the remainder theorem to find the volume of a cage with a height of 15 inches.

 ******

https://qrbook.page.link/app
Install NODIA App to See the Solutions.
Click Here To Install
Page 1 Sample Paper 20 Mathematics STD Class 10

Sample Paper 20
Class- X Exam - 2023-24
Mathematics - Standard

Time Allowed: 3 Hours Maximum Marks : 80


General Instructions :
1. This Question Paper has 5 Sections A-E.
2. Section A has 20 MCQs carrying 1 mark each
3. Section B has 5 questions carrying 02 marks each.
4. Section C has 6 questions carrying 03 marks each.
5. Section D has 4 questions carrying 05 marks each.
6. Section E has 3 case based integrated units of assessment (04 marks each) with sub-parts of the values of 1, 1
and 2 marks each respectively.
7. All Questions are compulsory. However, an internal choice in 2 Qs of 5 marks, 2 Qs of 3 marks and 2 Questions
of 2 marks has been provided. An internal choice has been provided in the 2 marks questions of Section E
8. Draw neat figures wherever required. Take π = 227 wherever required if not stated.

Section - A
Section A consists of 20 questions of 1 mark each.

1. If one of the zeroes of a quadratic polynomial of the form x2 + ax + b is the negative of the other, then it
(a) has no linear term and the constant term is negative.
(b) has no linear term and the constant term is positive.
(c) can have a linear term but the constant term is negative.
(d) can have a linear term but the constant term is positive.

2. A girl calculates that the probability of her winning the first prize in a lottery is 0.08. If 6000 tickets are sold, then
how many tickets has she bought?
(a) 40 (b) 240
(c) 480 (d) 750

3. If a pair of linear equations is consistent, then the lines will be


(a) parallel (b) always coincident
(c) intersecting or coincident (d) always intersecting

4. The sum and product of the zeroes of a quadratic polynomial are 3 and - 10 respectively. The quadratic polynomial
is
(a) x2 − 3x + 10 (b) x2 + 3x − 10
(c) x2 - 3x - 10 (d) x2 + 3x + 10

5. Value(s) of k for which the quadratic equation 2x2 − kx + k = 0 has equal roots is/are
(a) 0 (b) 4
(c) 8 (d) 0, 8

https://qrbook.page.link/app
Install NODIA App to See the Solutions.
Click Here To Install
Page 2 Sample Paper 20 Mathematics STD Class 10

6. The total number of factors of prime number is


(a) 1 (b) 0
(c) 2 (d) 3

7. The 11th term of an AP - 5 , - 5 , 0, 5 , ....., is


2 2
(a) - 20 (b) 20
(c) - 30 (d) 30

8. If the first term of an AP is - 5 and the common difference is 2, then the sum of the first 6 terms is
(a) 0 (b) 5
(c) 6 (d) 15

9. From an external point Q , the length of tangent to a circle is 12 cm and the distance of Q from the centre of circle
is 13 cm. The radius of circle (in cm) is
(a) 10 (b) 5
(c) 12 (d) 7

10. The value of the polynomial x8 − x5 + x2 − x + 1 is


(a) positive for all the real numbers
(b) negative for all the real numbers
(c) 0
(d) depends on value of x

11. If 4 tan θ = 3 , then c 4 sin q − cos q m is equal to


4 sin q + cos q
(a) 2 (b) 1
3 3

(c) 1 (d) 3
2 4

12. The sum of the areas of two circle, which touch each other externally, is 153 π . If the sum of their radii is 15, then
the ratio of the larger to the smaller radius is
(a) 4 :1 (b) 2 : 1
(c) 3 :1 (d) None of these

13. If the mean of the numbers 27 + x , 31 + x, 89 + x 107 + x, 156 + x is 82, then the mean of 130 + x, 126 + x,
68 + x, 50 + x, and 1 + x is
(a) 75 (b) 157
(c) 82 (d) 80

14. If the point P (k, 0) divides the line segment joining the points A (2, - 2) and B (- 7, 4) in the ratio 1 : 2, then
the value of k is
(a) 1 (b) 2
(c) -2 (d) - 1

https://qrbook.page.link/app
Install NODIA App to See the Solutions.
Click Here To Install
Page 3 Sample Paper 20 Mathematics STD Class 10

15. Aruna has only < 1 and < 2 coins with her. If the total number of coins that she has is 50 and the amount of money
with her is < 75, then the number of < 1 and < 2 coins are, respectively
(a) 35 and 15 (b) 35 and 20
(c) 15 and 35 (d) 25 and 25

16. Ratio of volumes of two cylinders with equal height is


(a) H : h (b) R : r
2 2
(c) R :r (d) None of these

17. The point which divides the line segment joining the points (8, - 9) and (2, 3) in the ratio 1 : 2 internally lies in the
(a) I quadrant (b) II quadrant
(c) III quadrant (d) IV quadrant

18. The co-ordinates of the point which is reflection of point (- 3, 5) in x -axis are
(a) (3, 5) (b) (3, - 5 )
(c) (- 3, - 5) (d) (- 3, 5)

In the question number 19 and 20, a statement of Assertion (A) is followed by a statement of Reason (R). Choose the
correction option.

19. Assertion : In the 3ABC , AB = 24 cm , BC = 10 cm and AC = 26 cm , then 3ABC is a right angle triangle.
Reason : If in two triangles, their corresponding angles are equal, then the triangles are similar.
(a) Both assertion (A) and reason (R) are true and reason (R) is the correct explanation of assertion (A).
(b) Both assertion (A) and reason (R) are true but reason (R) is not the correct explanation of assertion (A).
(c) Assertion (A) is true but reason (R) is false.
(d) Assertion (A) is false but reason (R) is true.

20. Assertion : In a circle of radius 6 cm, the angle of a sector 60c. Then the area of the sector is 18 76 cm2 .
Reason : Area of the circle with radius r is πr2 .
(a) Both assertion (A) and reason (R) are true and reason (R) is the correct explanation of assertion (A).
(b) Both assertion (A) and reason (R) are true but reason (R) is not the correct explanation of assertion (A).
(c) Assertion (A) is true but reason (R) is false.
(d) Assertion (A) is false but reason (R) is true.

Section - B
Section B consists of 5 questions of 2 marks each.

21. In the figure of TABC, the points D and E are on the sides CA, CB respectively such that DE || AB,
AD = 2x, DC = x + 3, BE = 2x − 1 and CE = x. Then, find x.

Continue on next page....

https://qrbook.page.link/app
Install NODIA App to See the Solutions.
Click Here To Install
Page 4 Sample Paper 20 Mathematics STD Class 10

 O
In the figure of TABC, DE || AB. If AD = 2x, DC = x + 3, BE = 2x − 1 and CE = x, then find the value of x.

22. Prove that tangents drawn at the ends of a chord of a circle make equal angles with the chord.

cos2 (45c + θ) + cos2 (45c − θ)


23. Show that : =1
tan (60c + θ) tan (30c − θ)

24. Find the mean of the following distribution :

Class interval 0-6 6-12 12-18 18-24 24-30


Frequency 5 4 1 6 4

 O
Find the sum of the lower limit of the median class and the upper limit of the modal class :

Classes 10-20 20-30 30-40 40-50 50-60 60-70


Frequency 1 3 5 9 7 3

https://qrbook.page.link/app Install NODIA App to See the Solutions.


Click Here To Install
Page 5 Sample Paper 20 Mathematics STD Class 10

25. Find the smallest natural number by which 1200 should be multiplied so that the square root of the product is a
rational number.
 O
Complete the following factor tree and find the composite number x .

Section - C
Section C consists of 6 questions of 3 marks each.

26. The 17 th term of an AP is 5 more than twice its 8 th term. If 11 th term of AP is 43, then find its nth term.

27. Prove that : 2 (sin6 θ + cos6 θ) − 3 (sin 4 θ + cos 4 θ) + 1 = 0

28. In a circle of radius 21 cm, an arc subtends an angle of 60c at the centre. Find the area of sector formed by the arc.
 O
A road which is 7 m wide surrounds a circular park whose circumference is 88 m. Find the area of the road.

29. The weekly expenditure of 500 families is tabulated below :

Weekly Expenditure(Rs.) Number of families


0-1000 150
1000-2000 200
2000-3000 75
3000-4000 60
4000-5000 15
Find the median expenditure.

30. The co-ordinates of the vertices of TABC are A ^7, 2h, B ^9, 10h and C ^1, 4h . If E and F are the mid-points of AB
and AC respectively, prove that EF = 12 BC .
 O
Find the ratio in which the line 2x + 3y − 5 = 0 divides the line segment joining the points ^8, - 9h and ^2, 1h . Also
find the co-ordinates of the point of division.

https://qrbook.page.link/appInstall NODIA App to See the Solutions.


Click Here To Install
Page 6 Sample Paper 20 Mathematics STD Class 10

31. The length, breadth and height of a room are 8 m 50 cm, 6 m 25 cm and 4 m 75 cm respectively. Find the length
of the longest rod that can measure the dimensions of the room exactly.

Section - D
Section D consists of 4 questions of 5 marks each.

32. Solve graphically the pair of linear equations :


3x − 4y + 3 = 0 and 3x + 4y − 21 = 0
Find the co-ordinates of the vertices of the triangular region formed by these lines and x -axis. Also, calculate
the area of this triangle.
 O
Solve the following pair of equations graphically:
2x + 3y = 12, x − y − 1 = 0 .
Shade the region between the two lines represented by the above equations and the X -axis.

33. In Figure, PQ is a chord of length 8 cm of a circle of radius 5 cm and centre O . The tangents at P and Q intersect
at point T . Find the length of TP .

34. Two poles of equal heights are standing opposite to each other on either side of the road which is 80 m wide. From
a point P between them on the road, the angle of elevation of the top of a pole is 60c and the angle of depression
from the top of the other pole of point P is 30c. Find the heights of the poles and the distance of the point P
from the poles.
 O
Two post are k metre apart and the height of one is double that of the other. If from the mid-point of the line
segment joining their feet, an observer finds the angles of elevation of their tops to be complementary, then find
the height of the shorted post.

35. A toy is in the form of a cylinder of diameter 2 2 m and height 3.5 m surmounted by a cone whose vertical angle
is 90c. Find total surface area of the toy.

Section - E
Case study based questions are compulsory.

36. Optimal Pricing Strategy : The director of the National School of Drama must decide what to charge for a ticket
to the comedy drama. If the price is set too low, the theatre will lose money; and if the price is too high, people
won’t come. From past experience she estimates that the profit P from sales (in hundreds) can be approximated
by P (x) =− x2 + 22x − 40 where x is the cost of a ticket and 0 # x # 25 hundred rupees.

Continue on next page....

https://qrbook.page.link/app
Install NODIA App to See the Solutions.
Click Here To Install
Page 7 Sample Paper 20 Mathematics STD Class 10

(i) What is the lowest and highest cost of a ticket that would allow the theatre to break even?
(ii) If theatre charge Rs 4 hundred for each ticket, what is the profit/loss ?
 O
If theatre charge Rs 25 hundred for each ticket, what is the profit/loss ?
(iii) What is the maximum profit which can be earned by theatre ?

37. Radio towers are used for transmitting a range of communication services including radio and television. The
tower will either act as an antenna itself or support one or more antennas on its structure, including microwave
dishes. They are among the tallest human-made structures. There are 2 main types: guyed and self-supporting
structures.
On a similar concept, a radio station tower was built in two sections A and B . Tower is supported by wires from
a point O . Distance between the base of the tower and point O is 36 m. From point O , the angle of elevation of
the top of section B is 30c and the angle of elevation of the top of section A is 45c.

(i) What is the height of the section B ?


(ii) What is the height of the section A ?
(iii) What is the length of the wire structure from the point O to the top of section A ?
 O

What is the length of the wire structure from the point O to the top of section B ?

https://qrbook.page.link/app
Install NODIA App to See the Solutions.
Click Here To Install
Page 8 Sample Paper 20 Mathematics STD Class 10

38. Hospital Stays : Hospital records indicated that maternity patients stayed in the hospital for the number of days
shown in the distribution.

Number of days stayed Frequency


3 13
4 22
5 45
6 14
7 6
100

Find these probabilities.


(i) A patient stayed exactly 5 days.
(ii) A patient stayed less than 6 days.
(iii) A patient stayed at most 4 days.
 O
(iv) A patient stayed at least 5 days.

 ******

https://qrbook.page.link/app
Install NODIA App to See the Solutions.
Click Here To Install
Page 1 Sample Paper 21 Mathematics STD Class 10

Sample Paper 21
Class- X Exam - 2023-24
Mathematics - Standard

Time Allowed: 3 Hours Maximum Marks : 80


General Instructions :
1. This Question Paper has 5 Sections A-E.
2. Section A has 20 MCQs carrying 1 mark each
3. Section B has 5 questions carrying 02 marks each.
4. Section C has 6 questions carrying 03 marks each.
5. Section D has 4 questions carrying 05 marks each.
6. Section E has 3 case based integrated units of assessment (04 marks each) with sub-parts of the values of 1, 1
and 2 marks each respectively.
7. All Questions are compulsory. However, an internal choice in 2 Qs of 5 marks, 2 Qs of 3 marks and 2 Questions
of 2 marks has been provided. An internal choice has been provided in the 2 marks questions of Section E
8. Draw neat figures wherever required. Take π = 227 wherever required if not stated.

Section - A
Section A consists of 20 questions of 1 mark each.

1. If sin q − cos q = 0 , then the value of ^sin 4 q + cos 4 qh is


(a) 1 (b) 3
4

(c) 1 (d) 1
2 4

2. Lowest value of x2 + 4x + 2 is
(a) 0 (b) - 2
(c) 2 (d) 4

3. The equation 2x2 + 2 ^p + 1h x + p = 0, where p is real, always has roots that are
(a) Equal (b) Equal in magnitude but opposite in sign
(c) Irrational (d) Real

4. Two concentric circles are of radii 10 cm and 8 cm, then the length of the chord of the larger circle which touches
the smaller circle is
(a) 6 cm (b) 12 cm
(c) 18 cm (d) 9 cm

5. The sum of first 16 terms of the AP 10, 6, 2, ..... is


(a) - 320 (b) 320
(c) - 352 (d) - 400

https://qrbook.page.link/app
Install NODIA App to See the Solutions.
Click Here To Install
Page 2 Sample Paper 21 Mathematics STD Class 10

6. Which of the following statement is false?


(a) All isosceles triangles are similar. (b) All quadrilateral are similar.
(c) All circles are similar. (d) None of the above

7. In the given figure, DE z BC . The value of EC is

(a) 1.5 cm (b) 3 cm


(c) 2 cm (d) 1 cm

8. A race track is in the form of a ring whose inner and outer circumference are 437 m and 503 m respectively. The
area of the track is
(a) 66 sq. cm. (b) 4935 sq. cm.
(c) 9870 sq. cm (d) None of these

9. If a regular hexagon is inscribed in a circle of radius r, then its perimeter is


(a) 3r (b) 6r
(c) 9r (d) 12r

10. If a letter is chosen at random from the letter of English alphabet, then the probability that it is a letter of the
word DELHI is
(a) 1 (b) 1
5 26

(c) 5 (d) 21
26 26

11. If x = p sec θ and y = q tan θ , then


(a) x2 − y2 = p2 q2 (b) x2 q2 − y2 p2 = pq
(c) x2 q2 − y2 p2 = 21 2 (d) x2 q2 − y2 p2 = p2 q2
pq

12. The pair of equations y = 0 and y =− 7 has


(a) one solution (b) two solutions
(c) infinitely many solutions (d) no solution

13. The length of a string between a kite and a point on the ground is 85 m. If the string makes an angle θ with level
ground such that tan θ = 158 , then the height of kite is
(a) 75 m (b) 78.05 m
(c) 226 m (d) None of these

https://qrbook.page.link/app
Install NODIA App to See the Solutions.
Click Here To Install
Page 3 Sample Paper 21 Mathematics STD Class 10

14. If the angle of depression of an object from a 75 m high tower is 30c, then the distance of the object from the
tower is
(a) 25 3 m (b) 50 3 m
(c) 75 3 m (d) 150 m

15. Ratio of volumes of two cones with same radii is


(a) h1 : h2 (b) s1 : s2
(c) r1 : r2 (d) None of these

16. The median of a set of 9 distinct observations is 20.5. If each of the largest 4 observation of the set is increased by
2, then the median of the new set
(a) Is increased by 2
(b) Is decreased by 2
(c) Is two times the original median
(d) Remains the same as that of the original set

17. Mode of the following grouped frequency distribution is

Class Frequency
3-6 2
6-9 5
9-12 10
12-15 23
15-18 21
18-21 12
21-24 03
(a) 13.6 (b) 15.6
(c) 14.6 (d) 16.6

18. A fair die is thrown once. The probability of getting a composite number less than 5 is
(a) 1 (b) 1
3 6

(c) 2 (d) 0
3

In the question number 19 and 20, a statement of Assertion (A) is followed by a statement of Reason (R). Choose the
correction option.

19. Assertion : The equation 8x2 + 3kx + 2 = 0 has equal roots then the value of k is ! 83 .
Reason : The equation ax2 + bx + c = 0 has equal roots if D = b2 − 4ac = 0
(a) Both assertion (A) and reason (R) are true and reason (R) is the correct explanation of assertion (A).
(b) Both assertion (A) and reason (R) are true but reason (R) is not the correct explanation of assertion (A).
(c) Assertion (A) is true but reason (R) is false.
(d) Assertion (A) is false but reason (R) is true.

https://qrbook.page.link/appInstall NODIA App to See the Solutions.


Click Here To Install
Page 4 Sample Paper 21 Mathematics STD Class 10

20. Assertion : If in a circle, the radius of the circle is 3 cm and distance of a point from the centre of a circle is 5 cm,
then length of the tangent will be 4 cm.
Reason : (hypotenuse) 2 = (base) 2 + (height) 2
(a) Both assertion (A) and reason (R) are true and reason (R) is the correct explanation of assertion (A).
(b) Both assertion (A) and reason (R) are true but reason (R) is not the correct explanation of assertion (A).
(c) Assertion (A) is true but reason (R) is false.
(d) Assertion (A) is false but reason (R) is true.

Section - B
Section B consists of 5 questions of 2 marks each.

21. If the sum of first m terms of an AP is the same as the sum of its first n terms, show that the sum of its first
(m + n) terms is zero.

22. A circle is inscribed in a TABC touching AB , BC and AC at P , Q and R respectively. If AB = 10 cm


AR = 7 cm and CR = 5 cm , then find the length of BC

23. Find a relation between x and y such that the point P ^x, y h is equidistant from the points A ^- 5, 3h and B ^7, 2h .

24. Explain whether 3 # 12 # 101 + 4 is a prime number or a composite number.

 O

Show that 5 6 is an irrational number.

25. Is 184 a term of the sequence 3, 7, 11, .......?

 O

The ninth term of an AP is - 32 and the sum of its eleventh and thirteenth term is - 94 . Find the common
difference of the AP

Section - C
Section C consists of 6 questions of 3 marks each.

26. Solve graphically :


2x − 3y + 13 = 0 ; 3x − 2y + 12 = 0

27. Solve the following equation: 1 - 1 = 3 , x ! 0 , 2


x x-2

https://qrbook.page.link/app
Install NODIA App to See the Solutions.
Click Here To Install
Page 5 Sample Paper 21 Mathematics STD Class 10

28. If tangents PA and PB drawn from an external point P to a circle with centre O are inclined to each other at
an angle of 80c, then find +POA .
 O
In the given figure, OP is equal to the diameter of a circle with centre O and PA and PB are tangents. Prove
that ABP is an equilateral triangle.

29. The angle of elevation of the top of a building from the foot of a tower is 30c and the angle of elevation of the
top of a tower from the foot of the building is 60c. If the tower is 50 m high, then find the height of the building.

30. From a solid cylinder whose height is 15 cm and the diameter is 16 cm, a conical cavity of the same height and
same diameter is hollowed out, Find the total surface area of remaining solid. (Given your answer in terms of π ).
 O
A solid right-circular cone of height 60 cm and radius 30 cm is dropped in a right-circular cylinder full of water of
height 180 cm and radius 60 cm. Find the volume of water left in the cylinder in cubic metre. Use π = 227 .

31. An integer is chosen between 70 and 100. Find the probability that it is
(i) a prime number (ii) divisible by 7

Section - D
Section D consists of 4 questions of 5 marks each.

32. Polynomial x 4 + 7x3 + 7x2 + px + q is exactly divisible by x2 + 7x + 12 , then find the value of p and q .

 O
If α and β are the zeroes of polynomial p ^x h = 3x2 + 2x + 1, find the polynomial whose zeroes are 1 − α and
1+α
1−β
.
1+β

33. In the given figure, DEFG is a square and +BAC = 90c. Show that FG2 = BG # FC .

https://qrbook.page.link/app
Install NODIA App to See the Solutions.
Click Here To Install
Page 6 Sample Paper 21 Mathematics STD Class 10

34. If sin θ + cos θ = 3 , then prove that tan θ + cot θ = 1.


 O
Evaluate :
tan2 30º sin 30º + cos 60º sin2 90º tan2 60º − 2 tan 45º cos2 0º sin 90º

35. Four equal circles are described at the four corners of a square so that each touches two of the others. The shaded
area enclosed between the circle is 247 cm2. Find the radius of each circle.

Section - E
Case study based questions are compulsory.

36. Air Quality Iindex : AQI is an index for reporting air quality on a daily basis. The purpose of the AQI is to help
people know how the local air quality impacts their health. The Environmental Protection Agency (EPA) calculates
the AQI for five major air pollutants :
1. Ground-level ozone
2. Particle pollution/particulate matter (PM2.5/pm 10)
3. Carbon Monoxide
4. Sulfur dioxide
5. Nitrogen dioxide
The higher the AQI value, the greater the level of air pollution and the greater the health concerns.

Following frequency distribution shows the Air Quality Index of different localities of Delhi on 27th December 2020
reported by Times of India Newspaper on 28th December 2020.

AIQ Number of weeks f


270-280 4
280-290 10
290-300 14
300-310 20
310-320 24
320-330 8
Total 80

Continue on next page....

https://qrbook.page.link/app Install NODIA App to See the Solutions.


Click Here To Install
Page 7 Sample Paper 21 Mathematics STD Class 10

Based on the above information, answer the following questions.


(i) Estimate the mean AQI.
(ii) In which class does the median of distribution lie ?
(iii) In which class does the mode of distribution lie ?

 O
What is the median AQI?

37. Resident Welfare Association (RWA) of a Gulmohar Society in Delhi have installed three electric poles A, B and
C in a society’s common park. Despite these three poles, some parts of the park are still in dark.
So, RWA decides to have one more electric pole D in the park.

The park can be modelled as a coordinate systems given below.

On the basis of the above information, answer any four of the following questions:
(i) What is the position of the pole C ?
(ii) What is the distance of the pole B from the corner O of the park ?
(iii) Find the position of the fourth pole D so that four points A, B C and D form a parallelogram .
 O
What is the distance between poles A and C ?

https://qrbook.page.link/app
Install NODIA App to See the Solutions.
Click Here To Install
Page 8 Sample Paper 21 Mathematics STD Class 10

38. Box : For the box to satisfy certain requirements, its length must be three unit greater than the width, and its
height must be two unit less than the width.

(i) If width is taken as x , find the polynomial that represent volume of box.
(ii) Find the polynomial that represent the area of paper sheet used to make box.
(iii) If it must have a volume of 18 unit, what must be its length and height ?
 O
If box is made of a paper sheet which cost is Rs 100 per square unit, what is the cost of paper?

 ******

https://qrbook.page.link/app
Install NODIA App to See the Solutions.
Click Here To Install
Page 1 Sample Paper 22 CBSE Mathematics Class 10

Sample Paper 22
Class- X Exam - 2023-24
Mathematics - Standard

Time Allowed: 3 Hours Maximum Marks : 80


General Instructions :
1. This Question Paper has 5 Sections A-E.
2. Section A has 20 MCQs carrying 1 mark each
3. Section B has 5 questions carrying 02 marks each.
4. Section C has 6 questions carrying 03 marks each.
5. Section D has 4 questions carrying 05 marks each.
6. Section E has 3 case based integrated units of assessment (04 marks each) with sub-parts.
7. All Questions are compulsory. However, an internal choice in 2 Qs of 5 marks, 2 Qs of 3 marks and 2 Questions
of 2 marks has been provided.
8. Draw neat figures wherever required. Take π = 227 wherever required if not stated.

Section - A
Section A consists of 20 questions of 1 mark each.

1. The HCF and the LCM of 12, 21, 15 respectively are


(a) 3, 140 (b) 12, 420
(c) 3, 420 (d) 420, 3

2. If the mean of a , b , c is M and ab + bc + ca = 0 , the mean of a2 , b2 and c2 is KM2 , then K is equal to


(a) 3 (b) 9
(c) 6 (d) 4

3. The 2 digit number which becomes 56 th of itself when its digits are reversed. The difference in the digits of the
number being 1, then the two digits number is
(a) 45 (b) 54
(c) 36 (d) None of these

4. Each root of x2 − bx + c = 0 is decreased by 2. The resulting equation is x2 − 2x + 1 = 0 , then


(a) b = 6, c = 9 (b) b = 3, c = 5
(c) b = 2, c =− 1 (d) b = − 4, c = 3

5. If the nth term of an AP is given by an = 5n − 3 , then the sum of first 10 terms if


(a) 225 (b) 245
(c) 255 (d) 270

Install NODIA App to See the Solutions.


Click Here To Install
Page 2 Sample Paper 22 CBSE Mathematics Class 10

6. In the given figure, x is

(a) ab (b) ac
a+b b+c

(c) bc (d) ac
b+c a+c

7. If the perimeter of one face of a cube is 20 cm, then its surface area is
(a) 120 cm2 (b) 150 cm2
(c) 125 cm2 (d) 400 cm2

8. From an external point Q , the length of tangent to a circle is 12 cm and the distance of Q from the centre of circle
is 13 cm. The radius of circle (in cm) is
(a) 10 (b) 5
(c) 12 (d) 7

9. The length of a string between a kite and a point on the ground is 85 m. If the string makes an angle θ with level
ground such that tan θ = 158 , then the height of kite is
(a) 75 m (b) 78.05 m
(c) 226 m (d) None of these

10. A tree casts a shadow 15 m long on the level of ground, when the angle of elevation of the sun is 45c. Find the
height of a tree.
(a) 15 m (b) 10 m
(c) 7.5 m (d) 12 m

11. If the sum of the circumferences of two circles with radii R1 and R2 is equal to the circumference of a circle of
radius R , then
(a) R1 + R2 = R (b) R1 + R2 > R
(c) R1 + R2 > R (d) R1 + R2 < R

Install NODIA App to See the Solutions.


Click Here To Install
Page 3 Sample Paper 22 CBSE Mathematics Class 10

12. In the given figure, if +A = 90º, +B = 90º, OB = 4.5 cm OA = 6 cm and AP = 4 cm then find QB.

(a) 3 cm (b) 6 cm
(c) 4.5 cm (d) 3.5 cm

13. Twelve solid spheres of the same size are made by melting a solid metallic cylinder of base diameter 2 cm and
height 16 cm. The diameter of each sphere is
(a) 4 cm (b) 3 cm
(c) 2 cm (d) 6 cm

14. Consider the following frequency distribution

Class 0-5 6-11 12-17 18-23 24-29


Frequency 13 10 15 8 11
The upper limit of the median class is
(a) 17 (b) 17.5
(c) 18 (d) 18.5

15. The probability that a number selected at random from the numbers 1, 2, 3, ......, 15 is a multiple of 4 is
(a) 4 (b) 2
15 15

(c) 1 (d) 1
15 5

16. One ticket is drawn at random from a bag containing tickets numbered 1 to 40. The probability that the selected
ticket has a number which is a multiple of 5 is
(a) 1 (b) 3
5 5

(c) 4 (d) 1
5 3

17. The zeroes of the quadratic polynomial x2 + kx + k where k ! 0 ,


(a) cannot both be positive
(b) cannot both be negative
(c) are always unequal
(d) are always equal

Install NODIA App to See the Solutions.


Click Here To Install
Page 4 Sample Paper 22 CBSE Mathematics Class 10

18. If the point P (6, 2) divides the line segment joining A (6, 5) and B (4, y) in the ratio 3 : 1 then the value of y is
(a) 4 (b) 3
(c) 2 (d) 1

In the question number 19 and 20, a statement of Assertion (A) is followed by a statement of Reason (R). Choose the
correction option.

19. Assertion : x3 + x has only one real zero.


Reason : A polynomial of n th degree must have n real zeroes.
(a) Both assertion (A) and reason (R) are true and reason (R) is the correct explanation of assertion (A).
(b) Both assertion (A) and reason (R) are true but reason (R) is not the correct explanation of assertion (A).
(c) Assertion (A) is true but reason (R) is false.
(d) Assertion (A) is false but reason (R) is true.

20. Assertion : The value of sec2 10c - cot2 80c is 1.


Reason : The value of sin 30c = 12 .
(a) Both assertion (A) and reason (R) are true and reason (R) is the correct explanation of assertion (A).
(b) Both assertion (A) and reason (R) are true but reason (R) is not the correct explanation of assertion (A).
(c) Assertion (A) is true but reason (R) is false.
(d) Assertion (A) is false but reason (R) is true.

Section - B
Section B consists of 5 questions of 2 marks each.

21. In the given figure, TABC ~TPQR. Find the value of y + z.

22. In the given figure, BOA is a diameter of a circle and the tangent at a point P meets BA when produced at T. If
+PBO = 30º , what is the measure of +PTA ?

Install NODIA App to See the Solutions.


Click Here To Install
Page 5 Sample Paper 22 CBSE Mathematics Class 10

23. If tan 2A = cot (A − 18c), where 2A is an acute angle, find the value of A.

24. A bag contains 5 red, 8 green and 7 white balls. One ball is drawn at random from the bag, find the probability
of getting :
(i) not a white ball,
(ii) neither a green nor a red ball.

 O
Two coins are tossed together. Find the probability of getting both heads or both tails.

25. In TABC, AD = BC, such that AD2 = BD # CD. Prove that TABC is right angled at A.

 O
In the figure of TABC, the points D and E are on the sides CA, CB respectively such that DE || AB,
AD = 2x, DC = x + 3, BE = 2x − 1 and CE = x. Then, find x.

OR

In the figure of TABC, DE || AB. If AD = 2x, DC = x + 3, BE = 2x − 1 and CE = x, then find the value of x.

Install NODIA App to See the Solutions.


Click Here To Install
Page 6 Sample Paper 22 CBSE Mathematics Class 10

Section - C
Section C consists of 6 questions of 3 marks each.

26. Find a quadratic polynomial whose zeroes are reciprocals of the zeroes of the polynomial f (x) = ax2 + bx + c ,
a ! 0, c ! 0.

27. Solve the following pair of linear equations :


8x + 5y = 9
3x + 2y = 4

28. Two right triangles ABC and DBC are drawn on the same hypotenuse BC and on the same side of BC . If AC
and BD intersect at P , prove that AP # PC = BP # DP .

 O
In the given figure, two triangles ABC and DBC lie on the same side of BC such that PQ || BA and PR || BD.
Prove that QR || AD.

29. The rod of TV disc antenna is fixed at right angles to wall AB and a rod CD is supporting the disc as shown in
Figure. If AC = 1.5 m long and CD = 3 m , find (i) tan θ (ii) sec θ + cosec θ .

30. A vessel is in the form of a hemispherical bowl surmounted by a hollow cylinder of same diameter. The diameter
of the hemispherical bowl is 14 cm and the total height of the vessel is 13 cm. Find the total surface area of the
vessel. Use π = 227
 O
A sphere of diameter 12 cm, is dropped in a right circular cylindrical vessel, partly filled with water. If the sphere
is completely submerged in water, the water level into the cylindrical vessel rises by 3 5 cm. Find the diameter of
9
the cylindrical vessel.

Install NODIA App to See the Solutions.


Click Here To Install
Page 7 Sample Paper 22 CBSE Mathematics Class 10

31. The 34 th part of a conical vessel of internal radius 5 cm and height 24 cm is full of water. The water emptied into
a cylindrical vessel with internal radius 10 cm. Find the height of water in cylindrical vessel.

Section - D
Section D consists of 4 questions of 5 marks each.

Solve for x : b 2x l + b 2x l − 24 = 0, x ! 5
2
32.
x−5 x−5
 O
Solve for x : x + 3 − 1 − x = 17 ; x ! 0, 2
x−2 x 4

33. If the angle between two tangents drawn from an external point P to a circle of radius a and centre O, is 60º,
then find the length of OP.

34. The median of the following data is 525. Find the values of x and y , if total frequency is 100 :

Class 0-100 100-200 200-300 300-400 400-500 500-600 600-700 700-800 800-900 900-1000
Frequency 2 5 x 12 17 20 x 9 7 4

 O
On annual day of a school, 400 students participated in the function. Frequency distribution showing their ages
is as shown in the following table :

Ages (in years) 05-07 07-09 09-11 11-13 13-15 15-17 17-19
Number of students 70 120 32 100 45 28 5
Find mean and median of the above data.

35. To conduct Sports Day activities, in your rectangular school ground ABCD , lines have been drawn with chalk
powder at a distance of 1 m each. 100 flower pots have been placed at a distance of 1 m from each other along AD
, as shown in Figure. Niharika runs ¼th the distance AD on the 2nd line and posts a green flag. Preet runs 15 th
distance AD on the eighth line and posts a red flag.
(i) What is the distance between the two flags?
(ii) If Rashmi has to post a blue flag exactly half way between the line segment joining the two flags, where
should she post the blue flag?

Install NODIA App to See the Solutions.


Click Here To Install
Page 8 Sample Paper 22 CBSE Mathematics Class 10

Section - E
Case study based questions are compulsory.

36. Box : For the box to satisfy certain requirements, its length must be three unit greater than the width, and its
height must be two unit less than the width.

(i) If width is taken as x , find the polynomial that represent volume of box.
(ii) Find the polynomial that represent the area of paper sheet used to make box.
(iii) If it must have a volume of 18 unit, what must be its length and height ?
 O
(iv) If box is made of a paper sheet which cost is Rs 100 per square unit, what is the cost of paper?

37. MASK : Masks are an additional step to help prevent people from getting and spreading COVID-19. They provide
a barrier that keeps respiratory droplets from spreading. Wear a mask and take every day preventive actions in
public settings.

Due to ongoing Corona virus outbreak, Wellness Medical store has started selling masks of decent quality. The
store is selling two types of masks currently type A and type B .

Continue on next page....

Install NODIA App to See the Solutions.


Click Here To Install
Page 9 Sample Paper 22 CBSE Mathematics Class 10

The cost of type A mask is Rs. 15 and of type B mask is Rs. 20. In the month of April, 2020, the store sold 100
masks for total sales of Rs. 1650.
(i) How many masks of each type were sold in the month of April? If the store had sold 50 masks of each type,
what would be its sales in the month of April?
(ii) Due to great demand and short supply, the store has increased the price of each type by Rs. 5 from May 1,
2020. In the month of May, 2020, the store sold 310 masks for total sales of Rs. 6875. How many masks of
each type were sold in the month of May?
(iii) What percent of masks of each type sale was increased in the month of May, compared with the sale of
month April?
 O
(v) What extra profit did store earn by increasing price in May month.

38. In a toys manufacturing company, wooden parts are assembled and painted to prepare a toy. For the wood
processing activity center, the wood is taken out of storage to be sawed, after which it undergoes rough polishing,
then is cut, drilled and has holes punched in it. It is then fine polished using sandpaper. For the retail packaging
and delivery activity center, the polished wood sub-parts are assembled together, then decorated using paint.

One specific toy is in the shape of a cone mounted on a cylinder. The total height of the toy is 110 mm and
the height of its conical part is 77 mm. The diameters of the base of the conical part is 72 mm and that of the
cylindrical part is 40 mm.
(i) If its cylindrical part is to be painted red, what is the surface area need to be painted ?
(ii) If its conical part is to be painted blue, what is the surface area need to be painted ?
(iii) How much of the wood have been used in making the toy ?
 O
(iv) If the cost of painting the toy is 2 paise for 8π mm2 , then what is the cost of painting of a box of 100 toys?

 ******

Install NODIA App to See the Solutions.


Click Here To Install
Page 1 Sample Paper 23 Mathematics STD Class 10

Sample Paper 23
Class- X Exam - 2023-24
Mathematics - Standard

Time Allowed: 3 Hours Maximum Marks : 80


General Instructions :
1. This Question Paper has 5 Sections A-E.
2. Section A has 20 MCQs carrying 1 mark each
3. Section B has 5 questions carrying 02 marks each.
4. Section C has 6 questions carrying 03 marks each.
5. Section D has 4 questions carrying 05 marks each.
6. Section E has 3 case based integrated units of assessment (04 marks each) with sub-parts of the values of 1, 1
and 2 marks each respectively.
7. All Questions are compulsory. However, an internal choice in 2 Qs of 5 marks, 2 Qs of 3 marks and 2 Questions
of 2 marks has been provided. An internal choice has been provided in the 2 marks questions of Section E
8. Draw neat figures wherever required. Take π = 227 wherever required if not stated.

Section - A
Section A consists of 20 questions of 1 mark each.

1. In the adjoining figure, the length of BC is

(a) 2 3 cm (b) 3 3 cm
(c) 4 3 cm (d) 3 cm

2. If the sum of the zeroes of the polynomial f ^x h = 2x3 − 3kx2 + 4x − 5 is 6, then the value of k is
(a) 2 (b) - 2
(c) 4 (d) - 4

3. For what value of k , do the equations 3x − y + 8 = 0 and 6x − ky = − 16 represent coincident lines ?

(a) 1 (b) - 1
2 2
(c) 2 (d) - 2

https://qrbook.page.link/app
Install NODIA App to See the Solutions.
Click Here To Install
Page 2 Sample Paper 23 Mathematics STD Class 10

4. If the height and length of the shadow of a man are equal, then the angle of elevation of the sun is,
(a) 45c (b) 60c
(c) 90c (d) 120c

5. The condition for one root of the quadratic equation ax2 + bx + c = 0 to be twice the other, is
(a) b2 = 4ac (b) 2b2 = 9ac
(c) c 2 = 4a + b 2 (d) c2 = 9a − b2

6. The quadratic equation x2 + 4x − 3 2 = 0 has


(a) two distinct real roots (b) two equal real roots
(c) no real roots (d) more than 2 real roots

7. If in a lottery, there are 5 prizes and 20 blanks, then the probability of getting a prize is
(a) 2 (b) 4
5 5

(c) 1 (d) 1
5

8. In an AP, if a = 1, an = 20 and Sn = 399 , then n is equal to


(a) 19 (b) 21
(c) 38 (d) 42

9. In Figure, DE || BC . Find the length of side AD , given that AE = 1.8 cm, BD = 7.2 cm and CE = 5.4 cm .

(a) 2.4 cm (b) 2.2 cm


(c) 3.2 cm (d) 3.4 cm

10. In the given figure, PA is a tangent from an external point P to a circle with centre O . If +POB = 115c, then
perimeter of +APO is

Continue on next page....

https://qrbook.page.link/app
Install NODIA App to See the Solutions.
Click Here To Install
Page 3 Sample Paper 23 Mathematics STD Class 10

(a) 25c (b) 20c


(c) 30c (d) 65c

11. QP is a tangent to a circle with centre O at a point P on the circle. If TOPQ is isosceles, then +OQR equals.
(a) 30c (b) 45c
(c) 60c (d) 90c

12. If b tan θ = a , the value of a sin θ − b cos θ is


a sin θ + b cos θ
(a) a−b (b) a2 + b2
a2 + b2 a +b

(c) a2 + b2 2 2
(d) a2 − b2
a2 − b2 a +b

13. The top of two poles of height 20 m and 14 m are connected by a wire. If the wire makes an angle of 30c with the
horizontal, then the length of the wire is
(a) 12 m (b) 10 m
(c) 8m (d) 6 m

14. In the given figure, OACB is a quadrant of a circle of radius 7 cm. The perimeter of the quadrant is

(a) 11 cm (b) 18 cm
(c) 25 cm (d) 36 cm

15. If the coordinates of the point of intersection of less than ogive and more than ogive is (13.5,20), then the value
of median is
(a) 13.5 (b) 20
(c) 33.5 (d) 7.5

16. Two circles of radii 20 cm and 37 cm intersect in A and B . If O1 and O2 are their centres and AB = 24 cm, then
the distance O1 O2 is equal to
(a) 44 cm (b) 51 cm
(c) 40.5 cm (d) 45 cm

https://qrbook.page.link/app
Install NODIA App to See the Solutions.
Click Here To Install
Page 4 Sample Paper 23 Mathematics STD Class 10

17. While computing the mean of grouped data, we assume that the frequencies are
(a) evenly distributed over all the classes
(b) centred at the class marks of the classes
(c) centred at the upper limits of the classes
(d) centred at the lower limits of the classes

18. The probability that a two digit number selected at random will be a multiple of 3 and not a multiple of 5 is
(a) 2 (b) 4
15 15

(c) 1 (d) 4
15 90

In the question number 19 and 20, a statement of Assertion (A) is followed by a statement of Reason (R). Choose the
correction option.

19. Assertion : ABC and DEF are two similar triangles such that BC = 4 cm , EF = 5 cm and area of TABC = 64 cm2
, then area of TDEF = 100 cm2 .
Reason : The areas of two similar triangles are in the ratio of the squares of the corresponding altitudes.
(a) Both assertion (A) and reason (R) are true and reason (R) is the correct explanation of assertion (A).
(b) Both assertion (A) and reason (R) are true but reason (R) is not the correct explanation of assertion (A).
(c) Assertion (A) is true but reason (R) is false.
(d) Assertion (A) is false but reason (R) is true.

20. Assertion : Total surface area of the cylinder having radius of the base 14 cm and height 30 cm is 3872 cm2 .
Reason : If r be the radius and h be the height of the cylinder, then total surface area = (2πrh + 2πr2).
(a) Both assertion (A) and reason (R) are true and reason (R) is the correct explanation of assertion (A).
(b) Both assertion (A) and reason (R) are true but reason (R) is not the correct explanation of assertion (A).
(c) Assertion (A) is true but reason (R) is false.
(d) Assertion (A) is false but reason (R) is true.

Section - B
Section B consists of 5 questions of 2 marks each.

21. How many terms of AP 3, 5, 7, 9, ..... must be taken to get the sum 120?

22. Prove that in two concentric circles, the chord of the larger circle, which touches the smaller circle is bisected at
the point of contact.

23. The x -coordinate of a point P is twice its y-coordinate. If P is equidistant from Q ^2, - 5h and R ^- 3, 6h , find
the co-ordinates of P.

Continue on next page....

https://qrbook.page.link/app
Install NODIA App to See the Solutions.
Click Here To Install
Page 5 Sample Paper 23 Mathematics STD Class 10

24. Complete the following factor tree and find the composite number x .

 O
Show that 571 is a prime number.

25. Find, 100 is a term of the AP 25, 28, 31, ...... or not.
 O
th
The seventeenth term of an AP exceeds its 10 term by 7. Find the common difference.

Section - C
Section C consists of 6 questions of 3 marks each.

26. Solve graphically : 2x + 3y = 2 , x − 2y = 8

27. Solve for x : 1 + 2 = 1 , x ! 0, 2 , 2 .


x 2x − 3 x − 2 3

28. An isosceles triangle ABC , with AB = AC , circumscribes a circle, touching BC at P , AC at Q and AB at R .


Prove that the contact point P bisects BC .
 O
From a point P , which is at a distant of 13 cm from the centre O of a circle of radius 5 cm, the pair of tangents
PQ and PR are drawn to the circle, then the area of the quadrilateral PQOR (in cm2).

29. The top of two poles of height 16 m and 10 m are connected by a length l meter. If wire makes an angle of 30c
with the horizontal, then find l .

30. A solid is in the shape of a cone mounted on a hemisphere of same base radius. If the curved surface areas of the
hemispherical part and the conical part are equal, then find the ratio of the radius and the height of the conical
part.
Continue on next page....

https://qrbook.page.link/appInstall NODIA App to See the Solutions.


Click Here To Install
Page 6 Sample Paper 23 Mathematics STD Class 10

 O
A tent is in the shape of cylinder surmounted by a conical top of same diameter. If the height and diameter of
cylindrical part are 2.1 m and 3 m respectively and the slant height of conical part is 2.8 m, find the cost of canvas
needed to make the tent if the canvas is available at the rate of Rs.500 per square meter. Use π = 227 .

31. Find the probability that 5 Sundays occur in the month of November of a randomly selected year.

Section - D
Section D consists of 4 questions of 5 marks each.

32. If α and β are the zeroes of the polynomial p ^x h = 2x2 + 5x + k satisfying the relation, α2 + β2 + αβ = 21
4 , then
find the value of k .
 O
If β and 1
β are zeroes of the polynomial ^a + a h x + 61x + 6a . Find the value of β and α .
2 2

33. In Figure, if TABC + TDEF and their sides of lengths (in cm) are marked along them, then find the lengths of
sides of each triangle.

34. If sec θ = x + 1 , x ! 0 find ^sec θ + tan θh .


4x
 O
Evaluate :
sin2 30º cos2 45º + 4 tan2 30º + 1 sin 90º − 2 cos2 90º + 1
2 24

https://qrbook.page.link/app Install NODIA App to See the Solutions.


Click Here To Install
Page 7 Sample Paper 23 Mathematics STD Class 10

35. Find the area of the shaded region in Figure, if ABCD is a rectangle with sides 8 cm and 6 cm and O is the centre
of circle. (Take π = 3.14 )

Section - E
Case study based questions are compulsory.

36. 100 Metres Race : The 100 metres is a sprint race in track and field competitions. The shortest common outdoor
running distance, it is one of the most popular and prestigious events in the sport of athletics. It has been
contested at the summer Olympics since 1896 for men and since 1928 for women. The World Championships 100
metres has been contested since 1983. The reigning 100 m Olympic or world champion is often named “the fastest
man or woman in the world”.

A stopwatch was used to find the time that it took a group of students to run 100 m.

Time (in sec) 0-20 20-40 40-60 60-80 80-100


No. of students 8 10 13 6 3
Based on the above information, answer the following questions.
(i) Estimate the mean time taken by a student to finish the race.
(ii) What will be the upper limit of the modal class ?
(iii) What is the sum of lower limits of median class and modal class ?

 O
How many students finished the race within 1 minute?

Continue on next page....

https://qrbook.page.link/app
Install NODIA App to See the Solutions.
Click Here To Install
Page 8 Sample Paper 23 Mathematics STD Class 10

37. Satellite Images : Satellite images are images of Earth collected by imaging satellites operated by governments
and businesses around the world. Satellite imaging companies sell images by licensing them to governments and
businesses such as Apple Maps and Google Maps. It should not be confused for astronomy images collected by
space telescope.

Barun lives in Jaipur in Vaishali. Satellite image of his colony is shown in given figure. In this view, his house is
pointed out by a flag, which is situated at the point of intersection of x and y - axes. If he goes 2 cm east and 3
cm north from the house, then he reaches to a grocery store, If he goes 4 cm west and 6 cm south from the house,
then he reaches to his office. If he goes 6 cm east and 8 cm south from the house, then he reaches to a food court.
If he goes 6 cm west and 8 cm north from the house, he reaches to a his kid’s school.
Based on the above information, answer the following questions.
(i) Find the distance between grocery store and food court.
(ii) Find the distance of the school from the house.
(iii) If the grocery store and office lie on a line, what is the ratio of distance of house from grocery store to that
from office ?
 O
Find the ratio of distances of house from school to food court.

38. Swimming Pool : The volume of water in a rectangular, in-ground, swimming pool is given by V (x) = x 3 + 11x2 + 24x
where V (x) is the volume in cubic feet when the water is x ft high.
(i) Find the dimension of base of pool.
(ii) Use the remainder theorem to find the volume when x = 3 ft.
(iii) If the volume is 100 ft 3 of water, what is the height x ?
 O
(iv) If the maximum capacity of the pool is 520 ft 3 what is the maximum depth?

 ******

https://qrbook.page.link/app
Install NODIA App to See the Solutions.
Click Here To Install
Page 1 Sample Paper 24 Mathematics STD Class 10

Sample Paper 24
Class- X Exam - 2023-24
Mathematics - Standard

Time Allowed: 3 Hours Maximum Marks : 80


General Instructions :
1. This Question Paper has 5 Sections A-E.
2. Section A has 20 MCQs carrying 1 mark each.
3. Section B has 5 questions carrying 02 marks each.
4. Section C has 6 questions carrying 03 marks each.
5. Section D has 4 questions carrying 05 marks each.
6. Section E has 3 case based integrated units of assessment (04 marks each) with sub-parts of the values of 1, 1
and 2 marks each respectively.
7. All Questions are compulsory. However, an internal choice in 2 Qs of 5 marks, 2 Qs of 3 marks and 2 Questions
of 2 marks has been provided. An internal choice has been provided in the 2 marks questions of Section E.
8. Draw neat figures wherever required. Take π = 227 wherever required if not stated.

Section - A
Section A consists of 20 questions of 1 mark each.

1. AB and CD are two common tangents to circles which touch each other at a point C . If D lies on AB such that
CD = 4 cm then AB is
(a) 12 cm (b) 8 cm
(c) 4 cm (d) 6 cm

2. If one of the zeroes of the quadratic polynomial ^k − 1h x2 + kx + 1 is - 3 , then the value of k is


(a) 4 (b) - 4
3 3

(c) 2 (d) - 2
3 3

3. If the lines given by 3x + 2ky = 2 and 2x + 5y + 1 = 0 are parallel, then the value of k is
(a) - 5 (b) 2
4 5

(c) 15 (d) 3
4 2

4. The following data gives the distribution of total household expenditure (in <) of manual workers in a city.

Expenditure (in <) Frequency


1000-1500 24
1500-2000 40
2000-2500 33
2500-3000 28
3000-3500 30

https://qrbook.page.link/app
Install NODIA App to See the Solutions.
Click Here To Install
Page 2 Sample Paper 24 Mathematics STD Class 10

Expenditure (in <) Frequency


3500-4000 22
4000-4500 16
4500-5000 07
Then, find the average expenditure which is done by the maximum number of manual workers.
(a) 1747.26 (b) 1847.26
(c) 1947.26 (d) 2047.26

5. The quadratic equation 2x2 − 3 2 x + 94 = 0 has


(a) two distinct real roots (b) two equal real roots
(c) no real roots (d) more than 2 real roots

6. The quadratic equation 3x2 + 4 3 x + 4 has


(a) two distinct real roots (b) two equal real roots
(c) no real roots (d) more than 2 real roots

7. A letter is chosen at random from the letters of the word ASSASSINATION, then the probability that the letter
chosen is a vowel is in the form of 6 , then x is equal to
2x + 1
(a) 5 (b) 6
(c) 7 (d) 8

8. In TABC, DE || BC, find the value of x .

(a) 3 (b) 2 (c) 4 (d) 1

9. In the given figure, two tangents AB and AC are drawn to a circle with centre O such that +BAC = 120c, then
OA is equal to that

(a) 2AB (b) 3AB (c) 4AB (d) 5AB

https://qrbook.page.link/app
Install NODIA App to See the Solutions.
Click Here To Install
Page 3 Sample Paper 24 Mathematics STD Class 10

10. Given that sin α = 3 and cos β = 0 , then the value of β − α is


2
(a) 0c (b) 90c
(c) 60c (d) 30c

11. If sec 5A = cosec ^A + 30ch , where 5A is an acute angle, then the value of A is
(a) 15c (b) 5c
(c) 20c (d) 10c

12. Two poles of height 6 m and 11 m stand vertically upright on a plane ground. If the distance between their foot
is 12 m, then distance between their tops is
(a) 12 m (b) 14 m
(c) 13 m (d) 11 m

13. A 6 m high tree cast a 4 m long shadow. At the same time, a flag pole cast a shadow 50 m long. How long is the
flag pole?
(a) 75 m (b) 100 m
(c) 150 m (d) 50 m

14. The ratio of the length of a rod and its shadow is 1 : 3 then the angle of elevation of the sun is
(a) 90c (b) 45c
(c) 30c (d) 75c

15. If two solid hemispheres of same base radius r are joined together along their bases, then curved surface area of
this new solid is
(a) 4πr2 (b) 6πr2
(c) 3πr2 (d) 8πr2

16. If the mean of the observation x, x + 3, x + 5, x + 7 and x + 10 is 9, the mean of the last three observation is

(a) 10 1 (b) 10 2
3 3

(c) 11 1 (d) 11 2
3 3

17. If the radius of a circle is diminished by 10%, then its area is diminished by
(a) 10% (b) 19%
(c) 36% (d) 20%

18. Two dice are thrown together. The probability that sum of the two numbers will be a multiple of 4, is:

(a) 1 (b) 1
2 3

(c) 1 (d) 1
8 4

https://qrbook.page.link/app
Install NODIA App to See the Solutions.
Click Here To Install
Page 4 Sample Paper 24 Mathematics STD Class 10

In the question number 19 and 20, a statement of Assertion (A) is followed by a statement of Reason (R). Choose the
correction option.

19. Assertion : If n th term of an AP is 7 - 4n , then its common differences is - 4 .


Reason : Common difference of an AP is given by d = an + 1 − an .
(a) Both assertion (A) and reason (R) are true and reason (R) is the correct explanation of assertion (A).
(b) Both assertion (A) and reason (R) are true but reason (R) is not the correct explanation of assertion (A).
(c) Assertion (A) is true but reason (R) is false.
(d) Assertion (A) is false but reason (R) is true.

20. Assertion : The two tangents are drawn to a circle from an external point, then they subtend equal angles at the
centre.
Reason : A parallelogram circumscribing a circle is a rhombus.
(a) Both assertion (A) and reason (R) are true and reason (R) is the correct explanation of assertion (A).
(b) Both assertion (A) and reason (R) are true but reason (R) is not the correct explanation of assertion (A).
(c) Assertion (A) is true but reason (R) is false.
(d) Assertion (A) is false but reason (R) is true.

Section - B
Section B consists of 5 questions of 2 marks each.

21. Which term of the AP 3, 15, 27, 39, ... will be 120 more than its 21st term?

22. Prove that the lengths of two tangents drawn from an external point to a circle are equal.

23. If the point P ^x, y h is equidistant from the points Q ^a + b, b − a h and R ^a − b, a + b h , then prove that bx = ay .

24. What are the values of x and y in the given figure ?

25. In a certain AP 32th term is twice the 12th term. Prove that 70th term is twice the 31st term.
 O
Find the middle term of the AP 213, 205, 197, .... 37.

https://qrbook.page.link/app
Install NODIA App to See the Solutions.
Click Here To Install
Page 5 Sample Paper 24 Mathematics STD Class 10

Section - C
Section C consists of 6 questions of 3 marks each.

26. Represent the following pair of linear equations graphically and hence comment on the condition of consistency
of this pair.
x − 5y = 6 and 2x − 10y = 12 .

27. Solve for x : x + 1 + x − 2 = 4 − 2x + 3 ; x ! 1, − 2, 2


x−1 x+2 x−2

28. Prove that the tangent at any point of a circle is perpendicular to the radius through the point of contact.
 O
In the figure, PQ is a tangent to a circle with centre O . If +OAB = 30º , find +ABP and +AOB.

29. The angles of depression of the top and bottom of a 50 m high building from the top of a tower are 45º and 60º
respectively. Find the height of the tower and the horizontal distance between the tower and the building. (Use
3 = 1.73)

30. A solid is in the shape of a hemisphere surmounted by a cone. If the radius of hemisphere and base radius of cone
is 7 cm and height of cone is 3.5 cm, find the volume of the solid. ^Take π = 227 h
 O
The rain water from 22m # 20 m roof drains into cylindrical vessel of diameter 2 m and height 3.5 m. If the rain
water collected from the roof fills 45th of cylindrical vessel then find the rainfall in cm.

31. In a family of three children, find the probability of having at least two boys.

Section - D
Section D consists of 4 questions of 5 marks each.

32. If α and β are the zeroes of polynomial p ^x h = 3x2 + 2x + 1, find the polynomial whose zeroes are 1 − α and
1+α
1−β
.
1+β
 O
2
If α and β are zeroes of the polynomial p (x) = 6x − 5x + k such that α − β = 16 , Find the value of k.

https://qrbook.page.link/app
Install NODIA App to See the Solutions.
Click Here To Install
Page 6 Sample Paper 24 Mathematics STD Class 10

33. In the figure, +BED = +BDE and E is the mid-point of BC . Prove that AF = AD .
CF BE

34. Prove that: tan θ + cot θ = 1 + sec θ cosec θ


1 − cot θ 1 − tan θ
 O
If 15 tan θ + 4 sec θ = 23, then find the value of ^sec θ + cosec θh2 − sin2 θ .
2 2

35. Fig. depicts a racing track whose left and right ends are semi-circular. The distance between the two inner parallel
line segments is 60 m and they are each 106 m long. If the track is 10 m wide everywhere, find the area of the track.

Section - E
Case study based questions are compulsory.

36. Sulfur dioxide (SO 2 ) is a colorless gas at ambient temperature and pressure. It is soluble in water and forms
sulfurous acid which is slowly oxidized to sulfuric acid by dissolved oxygen.

Continue on next page....

https://qrbook.page.link/app Install NODIA App to See the Solutions.


Click Here To Install
Page 7 Sample Paper 24 Mathematics STD Class 10

Sulfur dioxide is a major air pollutant and has significant impacts upon human health. In addition, the concentration
of sulfur dioxide in the atmosphere can influence the habitat suitability for plant communities, as well as animal
life. Sulfur dioxide emissions are a precursor to acid rain and atmospheric particulates.
To find out the concentration of SO 2 in the air (in parts per million, i.e. ppm), the data was collected for 30
localities in a delhi and is presented below:

Concentration of SO 2 (in ppm) Frequency


0.00-0.04 4
0.04-0.08 9
0.08-0.12 9
0.12-0.16 2
0.16-0.20 4
0.20-0.24 2
Based on the above information, answer the following questions.
(i) What is the mean concentration of SO 2 in the air?
(ii) What is the lower limit of median value of concentration of SO 2 in the air?
(iii) Find the median value concentration of SO 2 in the air.
(iv) How many localities are having SO 2 in range 0.04-0.16 ppm ?

37. Ajay, Bhigu and Colin are fast friend since childhood. They always want to sit in a row in the classroom . But
teacher doesn’t allow them and rotate the seats row-wise everyday. Bhigu is very good in maths and he does
distance calculation everyday. He consider the centre of class as origin and marks their position on a paper in a
co-ordinate system. One day Bhigu make the following diagram of their seating position.

(i) What is the distance of point A from origin ?


(ii) What is the distance between A and B ?
(iii) What is the distance between B and C ?
(iv) A point D lies on the line segment between points A and B such that AD : DB = 4 : 3 . What are the the
coordinates of point D ?

https://qrbook.page.link/app
Install NODIA App to See the Solutions.
Click Here To Install
Page 8 Sample Paper 24 Mathematics STD Class 10

38. Cost of Production : The cost to produce bottled spring water is given by C (x) = 16x − 63 where x is the
number of thousands of bottles. The total income (revenue) from the sale of these bottles is given by the function
R (x) =− x2 + 326x − 7463 .
(i) Since Profit = Revenue − Cost , find the profit function.
(ii) How many bottles sold will produce the maximum profit?
(iii) What is the maximum profit?
(iv) Find the profit when 245 thousand bottles are sold. Use remainder theorem

 ******

https://qrbook.page.link/app
Install NODIA App to See the Solutions.
Click Here To Install
Page 1 Sample Paper 25 CBSE Mathematics Class 10

Sample Paper 25
Class- X Exam - 2023-24
Mathematics - Standard

Time Allowed: 3 Hours Maximum Marks : 80


General Instructions :
1. This Question Paper has 5 Sections A-E.
2. Section A has 20 MCQs carrying 1 mark each
3. Section B has 5 questions carrying 02 marks each.
4. Section C has 6 questions carrying 03 marks each.
5. Section D has 4 questions carrying 05 marks each.
6. Section E has 3 case based integrated units of assessment (04 marks each) with sub-parts.
7. All Questions are compulsory. However, an internal choice in 2 Qs of 5 marks, 2 Qs of 3 marks and 2 Questions
of 2 marks has been provided.
8. Draw neat figures wherever required. Take π = 227 wherever required if not stated.

Section - A
Section A consists of 20 questions of 1 mark each.

1. If the angle of depression of an object from a 75 m high tower is 30c, then what is the distance of the object from
the tower?
(a) 75 2 m (b) 75 3 m
(c) 75 m (d) 75 2.5 m

2. If one zero of a quadratic polynomial (kx2 + 3x + k) is 2, then the value of k is


(a) 5 (b) - 5
6 6

(c) 6 (d) - 6
5 5

3. Two concentric circles are of radii 10 cm and 8 cm, then the length of the chord of the larger circle which touches
the smaller circle is
(a) 6 cm (b) 12 cm
(c) 18 cm (d) 9 cm

4. If ½ is a root of the equation x2 + kx − 54 = 0 , then the value of k is


(a) 2 (b) - 2
(c) 1 (d) 1
4 2

5. Consider the data:


Class 65-85 85-105 105-125 125-145 145-165 165-185 185-205
Frequency 4 5 13 20 14 7 4
The difference of the upper limit of the median class and the lower limit of the modal class is
(a) 0 (b) 19
(c) 20 (d) 38

Install NODIA App to See the Solutions.


Click Here To Install
Page 2 Sample Paper 25 CBSE Mathematics Class 10

6. An AP starts with a positive fraction and every alternate term is an integer. If the sum of the first 11 terms is 33,
then the fourth term is
(a) 2 (b) 3
(c) 5 (d) 6

7. If the sum of the zeroes of the quadratic polynomial kx2 + 2x + 3k is equal to their product, then k equals
(a) 1 (b) - 1
3 3

(c) 2 (d) - 2
3 3

8. TABC and TBDE are two equilateral triangle such that D is the mid-point of BC . Ratio of the areas of triangles
ABC and BDE is ................. .
(a) 1 : 4 (b) 4 : 1
(c) 1:3 (d) 3 : 1

9. If sin θ = a , then cos θ is equal to


b
(a) b (b) b
b2 - a2 a

(c) b2 - a2 (d) a
b b - a2
2

10. An observer, 1.5 m tall is 20.5 away from a tower 22 m high, then the angle of elevation of the top of the tower
from the eye of observer is
(a) 30c (b) 45c
(c) 60c (d) 90c

11. HCF of two numbers is 27 and their LCM is 162. If one of the numbers is 54, then the other number is
(a) 36 (b) 35
(c) 9 (d) 81

12. Volumes of two spheres are in the ratio 64 : 27. The the ratio of their surface areas is
(a) 3 : 4 (b) 4 : 3
(c) 9 : 16 (d) 16 : 9

13. Volume of a spherical shell is given by


(a) 4π (R2 − r2) (b) π (R3 − r3)
(c) 4π (R3 − r3) (d) 4
3 π (R3 − r3)

14. While computing mean of grouped data, we assume that the frequencies are
(a) evenly distributed over all the classes
(b) centred at the class marks of the classes
(c) centred at the upper limits of the classes
(d) centred at the lower limits of the classes

Install NODIA App to See the Solutions.


Click Here To Install
Page 3 Sample Paper 25 CBSE Mathematics Class 10

15. If an event cannot occur, then its probability is


(a) 1 (b) 3
4

(c) 1 (d) 0
2

16. In Figure, in TABC , DE z BC such that AD = 2.4 cm, AB = 3.2 cm and AC = 8 cm, then what is the length
of AE ?

(a) 4 cm (b) 6 cm
(c) 8 cm (d) 3 cm

17. The probability of getting a number greater then 3 in throwing a die is


1 1
(a) 3 (b) 4

3 2
(c) 4 (d) 3

18. The distance of the point (- 12, 5) from the origin is


(a) 12 (b) 5
(c) 13 (d) 169

In the question number 19 and 20, a statement of Assertion (A) is followed by a statement of Reason (R). Choose the
correction option.

19. Assertion : x + y − 4 = 0 and 2x + ky − 3 = 0 has no solution if k = 2 .


Reason : a1 x + b1 y + c1 = 0 and a2 x + b2 y + c2 = 0 are consistent if a1 ! k1 .
a2 k2
(a) Both assertion (A) and reason (R) are true and reason (R) is the correct explanation of assertion (A).
(b) Both assertion (A) and reason (R) are true but reason (R) is not the correct explanation of assertion (A).
(c) Assertion (A) is true but reason (R) is false.
(d) Assertion (A) is false but reason (R) is true.

20. Assertion : If the outer and inner diameter of a circular path is 10 m and 6 m then area of the path is 16π m2 .
Reason : If R and r be the radius of outer and inner circular path, then area of path is π ^R2 − r2h .
(a) Both assertion (A) and reason (R) are true and reason (R) is the correct explanation of assertion (A).
(b) Both assertion (A) and reason (R) are true but reason (R) is not the correct explanation of assertion (A).
(c) Assertion (A) is true but reason (R) is false.
(d) Assertion (A) is false but reason (R) is true.

Install NODIA App to See the Solutions.


Click Here To Install
Page 4 Sample Paper 25 CBSE Mathematics Class 10

Section - B
Section B consists of 5 questions of 2 marks each.

21. In the given figure, +A = +B and AD = BE. Show that DE || AB.

22. In the given figure, if AB = AC , prove that BE = CE.

23. Find the value of cos 2θ , if 2 sin 2θ = 3.

24. A lot consists of 144 ball pens of which 20 are defective and others are good. Nuri will buy a pen if it is good, but
will not buy if it is defective. The shopkeeper draws one pen at random and gives it to her. What is the probability
that :
(i) she will buy it ?
(ii) she will not buy it ?

 O

A bag contains 3 red, 4 green and 5 white candles, one candle is drawn at random from the bag, find the
probability that candle is not red.

25. ABCD is a trapezium in which AB || CD and its diagonals intersect each other at the point O. Show that
AO = CO .
BO DO

Continue on next page....

Install NODIA App to See the Solutions.


Click Here To Install
Page 5 Sample Paper 25 CBSE Mathematics Class 10

 O
In the given figures, find the measure of +X.

Section - C
Section C consists of 6 questions of 3 marks each.

26. If the sum and product of the zeroes of the polynomial ax2 − 5x + c are equal to 10 each, find the value of 'a' and
'c' .
27. Given the linear equation 2x + 3y − 8 = 0 , write another linear equation in two variables such that the geometrical
representation of the pair so formed is :
(a) intersecting lines
(b) parallel lines
(c) coincident lines.

28. If in TABC, AD is median and AE = BC, then prove that AB2 + AC2 = 2AD2 + 1 BC2 .
2
 O

In the given figure, find the value of x in terms of a, b and c.

Install NODIA App to See the Solutions.


Click Here To Install
Page 6 Sample Paper 25 CBSE Mathematics Class 10

29. If in a triangle ABC right angled at B, AB = 6 units and BC = 8 units, then find the value of sin A cos C + cos A sin C

30. A metallic solid sphere of radius 10.5 cm melted and recasted into smaller solid cones each of radius 3.5 cm and
height 3 cm. How may cones will be made ?
 O
A cylindrical tub, whose diameter is 12 cm and height 15 cm is full of ice-cream. The whole ice-cream is to be
divided into 10 children in equal ice-cream cones, with conical base surmounted by hemispherical top. If the height
of conical portion is twice the diameter of base, find the diameter of conical part of ice-cream cones.

31. Write the smallest number which is divisible by both 306 and 657.

Section - D
Section D consists of 4 questions of 5 marks each.

32. Find x in terms of a, b and c :


a + b = 2c , x ! a, b, c
x−a x−b x−c
 O
Write all the values of p for which the quadratic equation x2 + px + 16 = 0 has equal roots. Find the roots of the
equation so obtained.

33. In figure, a circle is inscribed in a TABC having sides BC = 8 cm , AB = 10 cm and AC = 12 cm . Find the length
BL, CM and AN .

34. If the median of the following frequency distribution is 32.5. Find the values of f1 and f2 .

Class 0- 10 10-20 20-30 30-40 40-50 50-60 60-70 Total


Frequency f1 5 9 12 f2 3 2 40

 O
Literacy rates of 40 cities are given in the following table. It is given that mean literacy rate is 63.5, then find the
missing frequencies x and y .

Literacy rate (in %) 35-40 40-45 45-50 50-55 55-60 60-65 65-70 70-75 75-80 80-85 85-90
Number of cities 1 2 3 x y 6 8 4 2 3 2

Install NODIA App to See the Solutions.


Click Here To Install
Page 7 Sample Paper 25 CBSE Mathematics Class 10

35. In a classroom, 4 friends are seated at the points A, B , C , and D as shown in Figure. Champa and Chameli walk
into the class and after observing for a few minutes Champa asks Chameli, Don’t you think ABCD is a square?
Chameli disagrees. Using distance formula, find which of them is correct.

Section - E
Case study based questions are compulsory.

36. Swimming Pool : The volume of water in a rectangular, in-ground, swimming pool is given by V (x) = x 3 + 11x2 + 24x
where V (x) is the volume in cubic feet when the water is x ft high.
(i) Find the dimension of base of pool.
(ii) Use the remainder theorem to find the volume when x = 3 ft.
(iii) If the volume is 100 ft 3 of water, what is the height x ?
 O
(iv) If the maximum capacity of the pool is 520 ft what is the maximum depth?
3

Install NODIA App to See the Solutions.


Click Here To Install
Page 8 Sample Paper 25 CBSE Mathematics Class 10

37. Architect : An architect is a skilled professional who plans and designs buildings and generally plays a key role
in their construction. Architects are highly trained in the art and science of building design. Since they bear
responsibility for the safety of their buildings’ occupants, architects must be professionally licensed.

Varsha is a licensed architect and design very innovative house. She has made a house layout for her client which
is given below. In the layout, the design and measurements has been made such that area of two bedrooms and
kitchen together is 95 sq. m.

(i) Which pair of linear equations does describe this situation ?


(ii) What is the length of the outer boundary of the layout.
(iii) What is the area of bedroom 1 ? What is the area of living room in the layout ?
(iv) What is the cost of laying tiles in Kitchen at the rate of Rs. 50 per sq. m ?

Install NODIA App to See the Solutions.


Click Here To Install
Page 9 Sample Paper 25 CBSE Mathematics Class 10

38. Atal Tunnel : Atal Tunnel (also known as Rohtang Tunnel) is a highway tunnel built under the Rohtang Pass
in the eastern Pir Panjal range of the Himalayas on the Leh-Manali Highway in Himachal Pradesh. At a length
of 9.02 km, it is the longest tunnel above 10,000 feet (3,048 m) in the world and is named after former Prime
Minister of India, Atal Bihari Vajpayee. The tunnel reduces the travel time and overall distance between Manali
and Keylong on the way to Leh. Moreover, the tunnel bypasses most of the sites that were prone to road blockades,
avalanches, and traffic snarls.

Earth is excavated to make a railway tunnel. The tunnel is a cylinder of radius 7 m and length 450 m. A level
surface is laid inside the tunnel to carry the railway lines. Figure given below shows the circular cross - section
of the tunnel. The level surface is represented by AB , the centre of the circle is O and +AOB = 90c. The space
below AB is filled with rubble (debris from the demolition buildings).

(i) How much volume of earth is removed to make the tunnel ?


(ii) If the cost of excavation of 1 cubic meter is Rs 250, what is the total cost of excavation?
(iii) A coating is to be done on the surface of inner curved part of tunnel. What is the area of tunnel to be being
coated ? Costing of coating is Rs 30 per m2 . What is the total cost of coating ?
 O
(iv) How much volume of debris is required to fill the ground surface of tunnel ?

 ******

Install NODIA App to See the Solutions.


Click Here To Install
Page 1 Sample Paper 26 CBSE Mathematics Class 10

Sample Paper 26
Class- X Exam - 2023-24
Mathematics - Standard

Time Allowed: 3 Hours Maximum Marks : 80


General Instructions :
1. This Question Paper has 5 Sections A-E.
2. Section A has 20 MCQs carrying 1 mark each
3. Section B has 5 questions carrying 02 marks each.
4. Section C has 6 questions carrying 03 marks each.
5. Section D has 4 questions carrying 05 marks each.
6. Section E has 3 case based integrated units of assessment (04 marks each) with sub-parts.
7. All Questions are compulsory. However, an internal choice in 2 Qs of 5 marks, 2 Qs of 3 marks and 2 Questions
of 2 marks has been provided.
8. Draw neat figures wherever required. Take π = 227 wherever required if not stated.

Section - A
Section A consists of 20 questions of 1 mark each.

1. If one zero of the quadratic polynomial x2 + 3x + k is 2, then the value of k is


(a) 10 (b) - 10
(c) -7 (d) - 2

2. HCF of 144 and 198 is


(a) 9 (b) 18
(c) 6 (d) 12

3. x and y are 2 different digits. If the sum of the two digit numbers formed by using both the digits is a perfect
square, then value of x + y is
(a) 10 (b) 11
(c) 12 (d) 13

4. The times, in seconds, taken by 150 athletes to run a 110 m hurdle race are tabulated below

Class Frequency
13.8-14 2
14-14.2 4
14.2-14.4 5
14.4-14.6 71
14.6-14.8 48
14.8-15 20
The number of athletes who completed the race in less than 14.6 second is :
(a) 11 (b) 71
(c) 82 (d) 130

Install NODIA App to See the Solutions.


Click Here To Install
Page 2 Sample Paper 26 CBSE Mathematics Class 10

5. The real roots of the equation x2/3 + x1/3 − 2 = 0 are


(a) 1, 8 (b) - 1, - 8
(c) - 1, 8 (d) 1, - 8

6. If the sum of the first 2n terms of 2, 5, 8, .......... is equal to the sum of the first n terms of 57, 59, 61, ..........,
then n is equal to
(a) 10 (b) 12
(c) 11 (d) 13

7. The top of two poles of height 20 m and 14 m are connected by a wire. If the wire makes an angle of 30c with the
horizontal, then find the length of the wire.
(a) 12 m (b) 20 m
(c) 8m (d) 6 m

8. The perimeters of two similar triangles are 25 cm and 15 cm respectively. If one side of the first triangle is 9 cm,
then the corresponding side of second triangle is ................... .
(a) 5.4 cm (b) 5.2 cm
(c) 4.9 cm (d) 5.1 cm

9. Two circles of radii 20 cm and 37 cm intersect in A and B . If O1 and O2 are their centres and AB = 24 cm, then
the distance O1 O2 is equal to
(a) 44 cm (b) 51 cm
(c) 40.5 cm (d) 45 cm

10. If cos ^α + β h = 0 , then sin ^α - β h can be reduced to


(a) cos β (b) cos 2β
(c) sin α (d) sin 2α

11. The zeroes of the polynomial x2 − 3x − m (m + 3) are


(a) m, m + 3 (b) − m, m + 3
(c) m, − (m + 3) (d) − m, − (m + 3)

12. A 6 m high tree cast a 4 m long shadow. At the same time, a flag pole cast a shadow 50 m long. How long is the
flag pole?
(a) 75 m (b) 100 m
(c) 150 m (d) 50 m

13. It is proposed to build a single circular park equal in area to the sum of areas of two circular parks of diameters
16 m and 12 m in a locality. The radius of the new park would be
(a) 10 m (b) 15 m
(c) 20 m (d) 24 m

14. If the radius of the sphere is increased by 100%, the volume of the corresponding sphere is increased by
(a) 200% (b) 500%
(c) 700% (d) 800%

Install NODIA App to See the Solutions.


Click Here To Install
Page 3 Sample Paper 26 CBSE Mathematics Class 10

15. Which of the following cannot be the probability of an event?


(a) 1 (b) 0.1
3

(c) 3% (d) 17
16

16. In TABC, DE || BC, find the value of x .

(a) 3 (b) 2
(c) 4 (d) 1

17. Out of one digit prime numbers, one number is selected at random. The probability of selecting an even number is
1 1
(a) 3 (b) 4

3 2
(c) 4 (d) 3

18. Distance of point P (3, 4) from x -axis is


(a) 3 units (b) 4 units
(c) 5 units (d) 1 units

In the question number 19 and 20, a statement of Assertion (A) is followed by a statement of Reason (R). Choose the
correction option.

19. Assertion : If the height of a cone is 24 cm and diameter of the base is 14 cm, then the slant height of the cone
is 15 cm.
Reason : If r be the radius and h be the slant height of the cone, then slant height = h2 + r2 .
(a) Both assertion (A) and reason (R) are true and reason (R) is the correct explanation of assertion (A).
(b) Both assertion (A) and reason (R) are true but reason (R) is not the correct explanation of assertion (A).
(c) Assertion (A) is true but reason (R) is false.
(d) Assertion (A) is false but reason (R) is true.

20. Assertion : If the number of runs scored by 11 players of a cricket team of India are 5, 19, 42, 11, 50, 30, 21, 0,
52, 36, 27 then median is 30.
Reason : Median = b n + 1 l value, if n is odd.
th

2
(a) Both assertion (A) and reason (R) are true and reason (R) is the correct explanation of assertion (A).
(b) Both assertion (A) and reason (R) are true but reason (R) is not the correct explanation of assertion (A).
(c) Assertion (A) is true but reason (R) is false.
(d) Assertion (A) is false but reason (R) is true.

Install NODIA App to See the Solutions.


Click Here To Install
Page 4 Sample Paper 26 CBSE Mathematics Class 10

Section - B
Section B consists of 5 questions of 2 marks each.

21. In the given figure, OA # OB = OC # OD, show that +A = +C and +B = +D.

22. In figure, a circle touches all the four sides of a quadrilateral ABCD . If AB = 6 cm, BC = 9 cm and CD = 8 cm,
then find the length of AD .

23. Find the value of sin 30º cos 60º + cos 30º sin 60º is it equal to sin 90º or cos 90º ?

24. Two different dice are thrown together. Find the probability that the product of the number appeared is less than
18.
 O
Harpreet tosses two different coins simultaneously. What is the probability that she gets :
(i) at least one head ?
(ii) one head and one tail ?

25. In a rectangle ABCD, E is a point on AB such that AE = 23 AB. If AB = 6 km and AD = 3 km, then find DE.

 O
In the given figure, if AB || DC, find the value of x.

Continue on next page....

Install NODIA App to See the Solutions.


Click Here To Install
Page 5 Sample Paper 26 CBSE Mathematics Class 10

Section - C
Section C consists of 6 questions of 3 marks each.

26. If one the zero of a polynomial 3x2 − 8x + 2k + 1 is seven times the other, find the value of k .

27. Solve for x and y :


ax + by = a + b
2
3x + 5y = 4

28. In the given figure, P and Q are the points on the sides AB and AC respectively of TABC, such that AP = 3.5
cm, PB = 7 cm, AQ = 3 cm and QC = 6 cm. If PQ = 4.5 cm, find BC.

 O
In the given figure, AB = AC. E is a point on CB produced. If AD is perpendicular to BC and EF perpendicular
to AC , prove that TABD is similar to TCEF.

Install NODIA App to See the Solutions.


Click Here To Install
Page 6 Sample Paper 26 CBSE Mathematics Class 10

2
29. Evaluate : 5 cos 60º + 4 cos2 30º − tan2 45º
sin 30º + cos2 60º
2

30. A solid sphere of diameter 6 cm is dropped in a right circular cylindrical vessel partly filled with water. The
diameter of the cylindrical vessel is 12 cm. If the sphere is completely submerged into water, by how much will the
level of water rise in the cylindrical vessel ?
 O
A hemispherical tank, of diameter 3 m, is full of water. It is being emptied by a pipe at the rate of 3 74 litre per
second. How much time will it take to make the tank half empty ? Use π = 227

31. 144 cartons of Coke cans and 90 cartons of Pepsi cans are to be stacked in a canteen. If each stack is of the same
height and if it equal contain cartons of the same drink, what would be the greatest number of cartons each stack
would have?

Section - D
Section D consists of 4 questions of 5 marks each.

32. Solve for x : x − 1 + 2x + 1 = 2 where x ]- 1 , 1


2x + 1 x−1 2
 O
Find the positive values of k for which quadratic equations x2 + kx + 64 = 0 and x2 − 8x + k = 0 both will have
the real roots.

33. PB is a tangent to the circle with centre O to B.AB is a chord of length 24 cm at a distance of 5 cm from the
centre. It the tangent is length 20 cm, find the length of PO .

34. The arithmetic mean of the following frequency distribution is 53. Find the value of k .

Class 0-20 20-40 40-60 60-80 80-100


Frequency 12 15 32 k 13

Continue on next page....

Install NODIA App to See the Solutions.


Click Here To Install
Page 7 Sample Paper 26 CBSE Mathematics Class 10

 O
The following distribution gives the weights of 60 students of a class. Find the mean and mode weights of the
students.

Weight (in kg) 40-44 44-48 48-52 52-56 56-60 60-64 64-68 68-72
Number of students 4 6 10 14 10 8 6 2

35. Point A lies on the line segment XY joining X ^6, - 6h and Y ^- 4, - 1h in such a way that XA
= 25 . If point A also
lies on the line 3x + k ^y + 1h = 0 , find the value of k .
XY

Section - E
Case study based questions are compulsory.

36. Overflow Pan : A metalworker makes an overflow pan by cutting equal squares with sides of length x from the
corners of a 30 cm by 20 cm piece of aluminium, as shown in the figure. The sides are then folded up and the
corners sealed.
(i) Find a polynomial function V (x) that gives the volume of the pan.
(ii) Find the volume of the pan if the height is 6 cm. Use remainder theorem.

37. Mr. RK Agrawal is owner of a famous amusement park in Delhi. The ticket charge for the park is Rs 150 for
children and Rs 400 for adult.

Generally he does not go to park and it is managed by team of staff. One day Mr Agrawal decided to random
check the park and went there. When he checked the cash counter, he found that 480 tickets were sold and Rs
134500 was collected.
(i) Let the number of children visited be x and the number of adults visited be y . Which of the following is
the correct system of equations that model the problem ?
(ii) How many children visited the park ? How many adults visited the park?
(iii) How much amount collected if 300 children and 350 adults visited the park?
(iv) One day total visited children and adults together is 750 and the total amount collected is Rs 212500. What
are the number of children and adults visited the park ?

Install NODIA App to See the Solutions.


Click Here To Install
Page 8 Sample Paper 26 CBSE Mathematics Class 10

38. A bakery is an establishment that produces and sells flour-based food baked in an oven such as bread, cookies,
cakes, pastries, and pies. Some retail bakeries are also categorized as cafés, serving coffee and tea to customers who
wish to consume the baked goods on the premises.

Tania runs a bakery shop and her bakery is very famous for tasty biscuits. The amount of mixture required to
make one biscuit is 18 cu cm. Before it is cooked, the mixture is rolled into a sphere. After the biscuit is cooked,
the biscuit becomes a cylinder of radius 3 cm and height 0.7 cm. The increase in volume is due to air being trapped
in the biscuit. Biscuits are packed in a cylindrical card box of height 14 cm. The arrangement of biscuits is shown
below.

(i) What is the volume of the biscuits after it is cooked ? What is the volume of air trapped, while cooking the
biscuit ?
(ii) How many biscuits will be there in a box ?
(iii) How much space is vacant in box after biscuits are packed ?
 O

(iv) If weight of 7 biscuits is 50 grams, what will be the weight of box of biscuits?

 ******

Install NODIA App to See the Solutions.


Click Here To Install
Page 1 Sample Paper 27 CBSE Mathematics Class 10

Sample Paper 27
Class- X Exam - 2023-24
Mathematics - Standard

Time Allowed: 3 Hours Maximum Marks : 80


General Instructions :
1. This Question Paper has 5 Sections A-E.
2. Section A has 20 MCQs carrying 1 mark each
3. Section B has 5 questions carrying 02 marks each.
4. Section C has 6 questions carrying 03 marks each.
5. Section D has 4 questions carrying 05 marks each.
6. Section E has 3 case based integrated units of assessment (04 marks each) with sub-parts.
7. All Questions are compulsory. However, an internal choice in 2 Qs of 5 marks, 2 Qs of 3 marks and 2 Questions
of 2 marks has been provided.
8. Draw neat figures wherever required. Take π = 227 wherever required if not stated.

Section - A
Section A consists of 20 questions of 1 mark each.

1. A pole casts a shadow of length 2 3 m on the ground, when the Sun’s elevation is 60º. Find the height of the pole.
(a) 4 m (b) 6 m
(c) 2m (d) 3 m

2. The quadratic polynomial, the sum of whose zeroes is - 5 and their product is 6, is
(a) x2 + 5x + 6 (b) x2 − 5x + 6
(c) x2 - 5x - 6 (d) − x2 + 5x + 6

3. A single letter is selected at random from the word PROBABILITY. The probability that the selected letter is a
vowel is
(a) 2 (b) 3
11 11

(c) 4 (d) 0
11

4. The value of k for which the system of linear equations x + 2y = 3 , 5x + ky + 7 = 0 is inconsistent is

(a) - 14 (b) 2
3 5
(c) 5 (d) 10

5. The quadratic equation 2x2 − 5 x + 1 = 0 has


(a) two distinct real roots (b) two equal real roots
(c) no real roots (d) more than 2 real roots

Install NODIA App to See the Solutions.


Click Here To Install
Page 2 Sample Paper 27 CBSE Mathematics Class 10

6. The first four terms of an AP whose first term is - 2 and the common difference is - 2 are
(a) - 2, 0, 2, 4 (b) - 2, 4, - 8, 16
(c) - 2, - 4, - 6, - 8 (d) - 2, - 4, - 8, - 16

7. 2 3 is
(a) an integer (b) a rational number
(c) an irrational number (d) a whole number

8. In the given figure, if +A = 90º, +B = 90º, OB = 4.5 cm OA = 6 cm and AP = 4 cm then find QB.

(a) 3 cm (b) 6 cm
(c) 4.5 cm (d) 3.5 cm

9. In Figure, in TABC , DE z BC such that AD = 2.4 cm, AB = 3.2 cm and AC = 8 cm, then what is the length
of AE ?

(a) 4 cm (b) 6 cm
(c) 8 cm (d) 3 cm

10. For the following distribution.

Class 0-5 5-10 10-15 15-20 20-25


Frequency 10 15 12 20 9
the sum of lower limits of the median class and modal class is
(a) 15 (b) 25
(c) 30 (d) 35

Install NODIA App to See the Solutions.


Click Here To Install
Page 3 Sample Paper 27 CBSE Mathematics Class 10

11. If TABC is right angled at C , then the value of cos ^A + B h is


(a) 0 (b) 1

(c) 1 (d) 3
2 2

12. The radius of a circle whose circumference is equal to the sum of the circumferences of the two circles of diameters
36 cm and 20 cm is
(a) 56 cm (b) 42 cm
(c) 28 cm (d) 16 cm

13. The base radii of a cone and a cylinder are equal. If their curved surface areas are also equal, then the ratio of the
slant height of the cone to the height of the cylinder is
(a) 2 : 1 (b) 1 : 2
(c) 1:3 (d) 3 : 1

14. The diameter of a sphere is 6 cm. It is melted and drawn into a wire of diameter 2 mm. The length of the wire is
(a) 12 m (b) 18 m
(c) 36 m (d) 66 m

15. The mean weight of 9 students is 25 kg. If one more student is joined in the group the mean is unaltered, then the
weight of the 10th student is
(a) 25 kg (b) 24 kg
(c) 26 kg (d) 23 kg

16. The probability expressed as a percentage of a particular occurrence can never be


(a) less than 100 (b) less than 0
(c) greater than 1 (d) anything but a whole number

17. In the adjoining figure, TP and TQ are the two tangents to a circle with centre O . If +POQ = 110c, then +PTQ
is

(a) 60c (b) 70c


(c) 80c (d) 90c

18. If A ^ m3 , 5h is the mid-point of the line segment joining the points Q (- 6, 7) and R (- 2, 3), then the value of m is
(a) - 12 (b) - 4
(c) 12 (d) - 6

Install NODIA App to See the Solutions.


Click Here To Install
Page 4 Sample Paper 27 CBSE Mathematics Class 10

In the question number 19 and 20, a statement of Assertion (A) is followed by a statement of Reason (R). Choose the
correction option.

19. Assertion : If both zeros of the quadratic polynomial x2 − 2kx + 2 are equal in magnitude but opposite in sign then
value of k is ½.
Reason : Sum of zeros of a quadratic polynomial ax2 + bx + c is -ab
(a) Both assertion (A) and reason (R) are true and reason (R) is the correct explanation of assertion (A).
(b) Both assertion (A) and reason (R) are true but reason (R) is not the correct explanation of assertion (A).
(c) Assertion (A) is true but reason (R) is false.
(d) Assertion (A) is false but reason (R) is true.

20. Assertion : In the figure, if BC = 20 m , then height AB is 11.56 m.

perpendicular
Reason : tan θ = AB = where θ is the angle +ACB .
BC base
(a) Both assertion (A) and reason (R) are true and reason (R) is the correct explanation of assertion (A).
(b) Both assertion (A) and reason (R) are true but reason (R) is not the correct explanation of assertion (A).
(c) Assertion (A) is true but reason (R) is false.
(d) Assertion (A) is false but reason (R) is true.

Section - B
Section B consists of 5 questions of 2 marks each.

21. In the given figure, CB || QR and CA || PR. If AQ = 12 cm, AR = 20 cm, PB = CQ = 15 cm, calculate PC and
BR.

Install NODIA App to See the Solutions.


Click Here To Install
Page 5 Sample Paper 27 CBSE Mathematics Class 10

22. In given figure, AB is the diameter of a circle with centre O and AT is a tangent. If +AOQ = 58c, find +ATQ .

23. Evaluate : cos 45º + 1


sec 30º sec 60º

24. A box contains 12 balls of which some are red in colour. If 6 more red balls are put in the box and a ball is drawn
at random the probability of drawing a red ball doubles than what it was before. Find the number of red balls in
the bag.
 O
Two different dice are tossed together. Find the probability :
(i) that the number on each die is even.
(ii) that the sum of numbers appearing on the two dice is 5.

25. In the given figure, G is the mid-point of the side PQ of TPQR and GH || QR. Prove that H is the mid-point of
the side PR or the triangle PQR.

 O
In TABC, if X and Y are points on AB and AC respectively such that AX
XB = 34 , AY = 5 and YC = 9, then state
whether XY and BC parallel or not.

Section - C
Section C consists of 6 questions of 3 marks each.

26. If α and β are the zeroes of the polynomial f (x) = x2 − 4x − 5 then find the value of α2 + β2

27. Solve graphically :


2x − 3y + 13 = 0 ; 3x − 2y + 12 = 0

Install NODIA App to See the Solutions.


Click Here To Install
Page 6 Sample Paper 27 CBSE Mathematics Class 10

28. In the given figure, DB = BC, DE = AB and AC = BC. Prove that BE = AC .


DE BC

 O
A 6 m high tree cast a 4 m long shadow. At the same time, a flag pole cast a shadow 50 m long. How long is the
flag pole?

29. In the given +PQR, right-angled at Q, QR = 9 cm and PR − PQ = 1 cm. Determine the value of sin R + cos R.

30. A well of diameter 4 m is dug 14 m deep. The earth taken out is spread evenly all around the well to form a 40 m
high embankment. Find the width of the embankment.
 O
The 34 th part of a conical vessel of internal radius 5 cm and height 24 cm is full of water. The water emptied into
a cylindrical vessel with internal radius 10 cm. Find the height of water in cylindrical vessel.

31. Find HCF and LCM of 16 and 36 by prime factorization and check your answer.

Section - D
Section D consists of 4 questions of 5 marks each.

32. Solve, for x : 3 x2 + 10x + 7 3 = 0


 O
2
If x =− 2 is a root of the equation 3x + 7x + p = 0 , find the value of k so that the roots of the equation
x2 + k ^4x + k − 1h + p = 0 are equal.

Continue on next page....

Install NODIA App to See the Solutions.


Click Here To Install
Page 7 Sample Paper 27 CBSE Mathematics Class 10

33. In figure, a circle with centre O is inscribed in a quadrilateral ABCD such that, it touches the sides BC , AB,
AD and CD at points P, Q, R and S respectively. If AB = 29 cm, AD = 23 cm, +B = 90c and DS = 5 cm, then
find the radius of the circle (in cm).

34. The table below show the salaries of 280 persons:

Salary (In thousand <) No. of Persons


5-10 49
10-15 133
15-20 63
20-25 15
25-30 6
30-35 7
35-40 4
40-45 2
45-50 1
Calculate the median salary of the data.
 O
The median of the following data is 525. Find the values of x and y if the total frequency is 100.

Class Interval Frequency


0-100 2
100-200 5
200-300 x
300-400 12
400-500 17
500-600 20
600-700 y
700-800 9
800-900 7
900-1000 4

35. Find the ratio in which the y -axis divides the line segment joining the points ^- 1, - 4h and ^5, - 6h . Also find the
coordinates of the point of intersection.

Install NODIA App to See the Solutions.


Click Here To Install
Page 8 Sample Paper 27 CBSE Mathematics Class 10

Section - E
Case study based questions are compulsory.

36. Cost of Production : The cost to produce bottled spring water is given by C (x) = 16x − 63 where x is the
number of thousands of bottles. The total income (revenue) from the sale of these bottles is given by the function
R (x) =− x2 + 326x − 7463 .
(i) Since Profit = Revenue − Cost , find the profit function.
(ii) How many bottles sold will produce the maximum profit?
(iii) What is the maximum profit?
(iv) Find the profit when 245 thousand bottles are sold. Use remainder theorem

37. Jodhpur is the second-largest city in the Indian state of Rajasthan and officially the second metropolitan city
of the state. Jodhpur was historically the capital of the Kingdom of Marwar, which is now part of Rajasthan.
Jodhpur is a popular tourist destination, featuring many palaces, forts, and temples, set in the stark landscape
of the Thar Desert. It is popularly known as the “Blue City” among people of Rajasthan and all over India. The
old city circles the Mehrangarh Fort and is bounded by a wall with several gates. The city has expanded greatly
outside the wall, though, over the past several decades. Jodhpur is also known for the rare breed of horses known
as Marwari or Malani, which are only found here.

Last year we visited Jodhpur in a group of 25 friends. When we went mehrangarh fort we found following fare for
ride :

Ride Normal Hours Fare Peak Hours Fare


Horse Rs 50 3 Times
Elephant Rs 100 2 Times
Some people choose to ride on horse and rest choose to ride on elephant.
(i) First day we rode in normal hours and we paid Rs 1950 for ride. Let x be the number of horses hired and
y be the number elephants hired. Which of the following is the correct system of equation that model the
problem ?
(ii) How many horses were hired ? How many elephant were hired ?
(iii) Next day we rode in peak hours, then how much total fare was paid by our group?
(iv) What was the increase in total fare because of peak hours ride ?

Install NODIA App to See the Solutions.


Click Here To Install
Page 9 Sample Paper 27 CBSE Mathematics Class 10

38. Boiler : The boiler is essentially a closed vessel inside which water is stored. Fuel is burnt in a furnace and hot
gasses are produced. These hot gasses come in contact with water vessel where the heat of these hot gases transfer
to the water and consequently steam is produced in the boiler. Then this steam is piped to the turbine of thermal
power plant.

Rajesh has been given the task of designing a boiler for NTPC. Boiler consist of a cylindrical part in middle and
two hemispherical part at its both end. The cross section of boiler is given below. Length of cylindrical part is the
3 times of radius of hemispherical part.

(i) Which of the following is correct expression for the surface area of cylindrical part of boiler?
(ii) Which of the following is correct expression for the total surface area of boiler?
(iii) Which of the following is correct expression for the volume of boiler? What is the ratio of volume to the
surface area?
(iv) If r = 3 m , what is the volume of boiler?

 ******

Install NODIA App to See the Solutions.


Click Here To Install
Page 1 Sample Paper 28 CBSE Mathematics Class 10

Sample Paper 28
Class- X Exam - 2023-24
Mathematics - Standard

Time Allowed: 3 Hours Maximum Marks : 80


General Instructions :
1. This Question Paper has 5 Sections A-E.
2. Section A has 20 MCQs carrying 1 mark each
3. Section B has 5 questions carrying 02 marks each.
4. Section C has 6 questions carrying 03 marks each.
5. Section D has 4 questions carrying 05 marks each.
6. Section E has 3 case based integrated units of assessment (04 marks each) with sub-parts.
7. All Questions are compulsory. However, an internal choice in 2 Qs of 5 marks, 2 Qs of 3 marks and 2 Questions
of 2 marks has been provided.
8. Draw neat figures wherever required. Take π = 227 wherever required if not stated.

Section - A
Section A consists of 20 questions of 1 mark each.

1. In a circle of radius 14 cm, an arc subtends an angle of 45c at the centre, then the area of the sector is
(a) 71 cm2 (b) 76 cm2
(c) 77 cm2 (d) 154 cm2

2. The graph of y = p (x), where p (x) is a polynomial in variable x , is as follows.

The number of zeroes of p (x) is .......... .


(a) 2 (b) 3
(c) 4 (d) 5

3. The zeroes of the quadratic polynomial 3 x2 − 8x + 4 3 are


1
(a) 2 3 and 3 (b) 2 3 and 3
1 2
(c) 3
and 3 (d) 3
and 2 3

Install NODIA App to See the Solutions.


Click Here To Install
Page 2 Sample Paper 28 CBSE Mathematics Class 10

4. Select the quadratic polynomial p ^x h with 3 and - 25 as sum and product of its zeroes, respectively.
(a) x2 - 3x - 25 (b) x2 - 3x - 2
(c) 5x2 - 15x - 2 (d) 15x2 − 5x + 25

5. If ad ! bc, then what do you say about the solution of the pair of linear equations ax + by = p and cx + dy = q ?
(a) no solution (b) unique solution
(c) infinitely solution (d) can’t say anything

6. What are the values of x and y for the following pair of linear equations ?
3x + 2y − 7 = 0
4x + y − 6 = 0
(a) 1 and 2 (b) 2 and 2
(c) 1 and 1 (d) –1 and –1

7. The quadratic equation 2x2 − 3 2 x + 94 = 0 has


(a) two distinct real roots (b) two equal real roots
(c) no real roots (d) more than 2 real roots

8. From an external point P , tangents PA and PB are drawn to a circle with centre O . If CD is the tangent to the
circle at a point E and PA = 14 cm . The perimeter of TPCD is
(a) 14 cm (b) 21 cm
(c) 28 cm (d) 35 cm

9. What is the common difference of an AP in which a18 − a14 = 32 ?


(a) 8 (b) - 8
(c) -4 (d) 4

10. The nth term of the AP a , 3a , 5a , ... is


(a) na (b) (2n - 1) a
(c) (2n + 1) a (d) 2na

11. If tan A = cot B , then the value of (A + B) is


(a) 90° (b) 120°
(c) 60° (d) 180°

12. If cos A = 2 , the value of 4 + 4 tan2 A will be


5
(a) 1 (b) 2
(c) 4 (d) 25

13. If the angle of depression of an object from a 75 m high tower is 30c, then the distance of the object from the
tower is
(a) 25 3 m (b) 50 3 m
(c) 75 3 m (d) 150 m

Install NODIA App to See the Solutions.


Click Here To Install
Page 3 Sample Paper 28 CBSE Mathematics Class 10

14. TABC is an equilateral triangle of side 2a , then length of one of its altitude is ................... .
(a) a 3 (b) a2 3
(c) a3 2 (d) a 2

15. If the radius of the sphere is increased by 100%, the volume of the corresponding sphere is increased by
(a) 200% (b) 500%
(c) 700% (d) 800%

16. In the formula x = a +


/ f i d i , for finding the mean of grouped data d ’s are deviation from a of
/ fi i

(a) lower limits of the classes (b) upper limits of the classes
(c) mid-points of the classes (d) frequencies of the class marks

17. a and b are two positive integers such that the least prime factor of a is 3 and the least prime factor of b is 5.
Then the least prime factor of (a + b) will be
(a) 1 (b) 2
(c) 3 (d) 4

18. A pair of dice is thrown once. What is the probability of getting a doublet?
1
(a) 2 (b) 13
1 1
(c) 6 (d) 5

In the question number 19 and 20, a statement of Assertion (A) is followed by a statement of Reason (R). Choose the
correction option.

19. Assertion : 4x2 − 12x + 9 = 0 has repeated roots.


Reason : The quadratic equation ax2 + bx + c = 0 have repeated roots if discriminant D > 0 .
(a) Both assertion (A) and reason (R) are true and reason (R) is the correct explanation of assertion (A).
(b) Both assertion (A) and reason (R) are true but reason (R) is not the correct explanation of assertion (A).
(c) Assertion (A) is true but reason (R) is false.
(d) Assertion (A) is false but reason (R) is true.

20. Assertion : The value of y is 6, for which the distance between the points P ^2, - 3h and Q ^10, y h is 10.
Reason : Distance between two given points A ^x1, y1h and B ^x2, y2h is given,
AB = ^x2 − x1h2 + ^y2 − y1h2
(a) Both assertion (A) and reason (R) are true and reason (R) is the correct explanation of assertion (A).
(b) Both assertion (A) and reason (R) are true but reason (R) is not the correct explanation of assertion (A).
(c) Assertion (A) is true but reason (R) is false.
(d) Assertion (A) is false but reason (R) is true.

Install NODIA App to See the Solutions.


Click Here To Install
Page 4 Sample Paper 28 CBSE Mathematics Class 10

Section - B
Section B consists of 5 questions of 2 marks each.

21. Explain why (7 # 13 # 11) + 11 and (7 # 6 # 5 # 4 # 3 # 2 # 1) + 3 are composite numbers.

22. Find the zeroes of the quadratic polynomial 3 x2 − 8x + 4 3 .

 O
Find a quadratic polynomial, the sum and product of whose zeroes are 6 and 9 respectively. Hence find the zeroes.

23. What is the distance of point P (3, 4) from x -axis?

24. If triangle ABC is similar to triangle DEF such that 2AB = DE and BC = 8 cm then find EF.
 O
In the figure, PQ is parallel to MN. If KP = 4 and KN = 20.4 cm then find KQ.
PM 13

25. There are 30 cards of the same size in a bag in which the numbers 1 to 30 are written. One card is taken out of
the bag at random. Find the probability that the number on the selected card is not divisible by 3.

Section - C
Section C consists of 6 questions of 3 marks each.

26. Given that 5 is irrational, prove that 2 5 - 3 is an irrational number.

27. In Figure, DE || BC . Find the length of side AD , given that AE = 1.8 cm, BD = 7.2 cm and CE = 5.4 cm .

Install NODIA App to See the Solutions.


Click Here To Install
Page 5 Sample Paper 28 CBSE Mathematics Class 10

cos2 (45c + θ) + cos2 (45c − θ)


28. Show that : =1
tan (60c + θ) tan (30c − θ)

29. If tangents PA and PB drawn from an external point P to a circle with centre O are inclined to each other at
an angle of 80c, then find +POA .

30. A right circular cone of radius 3 cm, has a curved surface area of 47.1 cm2. Find the volume of the cone. (Use
π = 3.14 )
 O
The sum of the radius of base and height of a solid right circular cylinder is 37 cm. If the total surface area of the
solid cylinder is 1628 sq. cm, find the volume of the cylinder. π = 227 .

31. Compute the mode for the following frequency distribution:

Size of items (in cm) 0- 4 4- 8 8- 12 12-16 16-20 20-24 24-28


Frequency 5 7 9 17 12 10 6

 O
The mean of the following frequency distribution is 18. The frequency f in the class interval 19-21 is missing.
Determine f .

Class interval 11-13 13-15 15-17 17-19 19-21 21-23 23-25


Frequency 3 6 9 13 f 5 4

Section - D
Section D consists of 4 questions of 5 marks each.

32. Solve for x : 1 + 2 = 4 x !- 1, - 2, - 4


x+1 x+2 x+4
 O
A two digit number is such that product of its digits is 14. If 45 is added to the number, the digits interchange
their places. Find the number.

33. Find the ratio in which the point ^- 3, k h divides the line segment joining the points ^- 5, - 4h and ^- 2, 3h .Also
find the value of k .

34. In Figure, PQ is a chord of length 8 cm of a circle of radius 5 cm and centre O . The tangents at P and Q intersect
at point T . Find the length of TP .

Continue on next page....

Install NODIA App to See the Solutions.


Click Here To Install
Page 6 Sample Paper 28 CBSE Mathematics Class 10

 O
A right triangle ABC , right angled at A is circumscribing a circle. If AB = 6 cm and BC = 10 cm, find the
radius r of the circle.

35. Sides of a right triangular field are 25 m, 24 m and 7 m. At the three corners of the field, a cow, a buffalo and a
horse are tied separately with ropes of 3.5 m each to graze in the field. Find the area of the field that cannot be
grazed by these animals.

Section - E
Case study based questions are compulsory.

36. Salary : In investigating different job opportunities, you find that firm A will start you at Rs 25,000 per year and
guarantee you a raise of Rs 1,200 each year whereas firm B will start you at Rs 28,000 per year but will guarantee
you a raise of only Rs 800 each year.
(i) Over a period of 15 years, how much would you receive from firm A?
(ii) Over a period of 15 years, how much would you receive from firm B?
(iii) What would be your annual salary at firm A for the tenth year?
(iv) What would be your annual salary at firm B for the tenth year?

37. From his hotel room window on the fourth floor, Ranjan notices some window washers high above him on the
hotel across the street.

Continue on next page....

Install NODIA App to See the Solutions.


Click Here To Install
Page 7 Sample Paper 28 CBSE Mathematics Class 10

Curious as to their height above ground, he quickly estimates the buildings are 60 m apart, the angle of elevation
to the workers is about 60c , and the angle of depression to the base of the hotel is about 30c .
(i) How high above ground is the window of Ranjan’s hotel room?
(ii) How high above ground are the workers?

38. Political survey questions are questions asked to gather the opinions and attitudes of potential voters. Political
survey questions help you identify supporters and understand what the public needs. Using such questions, a
political candidate or an organization can formulate policies to gain support from these people.

A survey of 100 voters was taken to gather information on critical issues and the demographic information
collected is shown in the table. One out of the 100 voters is to be drawn at random to be interviewed on the India
Today News on prime time.

Women Men Totals


Republican 17 20 37
Democrat 22 17 39
Independent 8 7 15
Green Party 6 3 5
Totals 53 47 100

(i) What is the probability the person is a woman or a Republican ?


(ii) What is the probability the person is a Democrat ?
(iii) What is the probability the person is a Independent men ?
(iv) What is the probability the person is a Independent men or green party men ?

 ******

Install NODIA App to See the Solutions.


Click Here To Install
Page 1 Sample Paper 29 CBSE Mathematics Class 10

Sample Paper 29
Class- X Exam - 2023-24
Mathematics - Standard

Time Allowed: 3 Hours Maximum Marks : 80


General Instructions :
1. This Question Paper has 5 Sections A-E.
2. Section A has 20 MCQs carrying 1 mark each
3. Section B has 5 questions carrying 02 marks each.
4. Section C has 6 questions carrying 03 marks each.
5. Section D has 4 questions carrying 05 marks each.
6. Section E has 3 case based integrated units of assessment (04 marks each) with sub-parts.
7. All Questions are compulsory. However, an internal choice in 2 Qs of 5 marks, 2 Qs of 3 marks and 2 Questions
of 2 marks has been provided.
8. Draw neat figures wherever required. Take π = 227 wherever required if not stated.

Section - A
Section A consists of 20 questions of 1 mark each.

1. Two lines are given to be parallel. The equation of one of the lines is 4x + 3y = 14 , then the equation of the second
line will be
(a) 12x + 9x = 42 (b) 12x + 9y = 5
(c) 12x + 8y = 15 (d) 12x + 8y = 42

2. If the height and length of the shadow of a man are equal, then the angle of elevation of the sun is,
(a) 45c (b) 60c
(c) 90c (d) 120c

3. A die is thrown once. What is the probability of getting a prime number?


1
(a) 3 (b) 23
1 1
(c) 2 (d) 4

4. Thus (a) is correct option. If one root of the equation (k − 1) x2 − 10x + 3 = 0 is the reciprocal of the other then
the value of k is .........
(a) 2 (b) 3
(c) 4 (d) 5

5. If p and q are the zeroes of polynomial f ^x h = 2x2 − 7x + 3 , the value of p2 + q2 will be


39
(a) 5 (b) 395
37 4
(c) 4 (d) 37

Install NODIA App to See the Solutions.


Click Here To Install
Page 2 Sample Paper 29 CBSE Mathematics Class 10

6. Select the least number that is divisible by all numbers between 1 and 10 (both inclusive).
(a) 2520 (b) 5040
(c) 1010 (d) 2020

7. If m and n are the zeroes of the polynomial 3x2 + 11x − 4 , then value of m
n + mn will be
(a) 12
145 (b) - 145
12

(c) - 145
12 (d) 145
12

8. The quadratic equation x2 + x − 5 = 0 has


(a) two distinct real roots (b) two equal real roots
(c) no real roots (d) more than 2 real roots

9. The 4th term from the end of an AP - 11, - 8 , - 5 , ....., 49 is


(a) 37 (b) 40
(c) 43 (d) 58

1 - p 1 - 2p
10. The common difference of the AP 1 , , , ... is
p p p
(a) 1 (b) 1
p

(c) -1 (d) - 1
p

11. If the point C (k, 4) divides the line segment joining two points A (2, 6) and B (5, 1) in ratio 2 : 3, the value of k
is ......... .
5
(a) 16 (b) 165
9 5
(c) 5 (d) 9

12. If x = 3 sin θ + 4 cos θ and y = 3 cos θ − 4 sin θ then x2 + y2 is


(a) 25 (b) 45
(c) 7 (d) 49

13. TABC and TBDE are two equilateral triangle such that D is the mid-point of BC . Ratio of the areas of triangles
ABC and BDE is ................. .
(a) 1 : 1 (b) 3 : 1
(c) 2:1 (d) 4 : 1

14. In the figure given below, ABCD is a rectangle. The values of x and y will be

Continue on next page....

Install NODIA App to See the Solutions.


Click Here To Install
Page 3 Sample Paper 29 CBSE Mathematics Class 10

(a) 3 and 19 (b) 19 and 3


(c) 4 and 18 (d) 18 and 4

15. If k + 1 = sec2 θ ^1 + sin θh^1 − sin θh, then the value of k. will be
(a) 0 (b) 1
(c) 2 (d) 15

16. If the sum of the areas of two circles with radii R1 and R2 is equal to the area of a circle of radius R , then
(a) R1 + R2 = R (b) R12 + R22 = R2
(c) R1 + R2 < R (d) R12 + R22 < R2

17. The base radii of a cone and a cylinder are equal. If their curved surface areas are also equal, then the ratio of the
slant height of the cone to the height of the cylinder is
(a) 2 : 1 (b) 1 : 2
(c) 1:3 (d) 3 : 1

18. While computing mean of grouped data, we assume that the frequencies are
(a) evenly distributed over all the classes
(b) centred at the class marks of the classes
(c) centred at the upper limits of the classes
(d) centred at the lower limits of the classes

In the question number 19 and 20, a statement of Assertion (A) is followed by a statement of Reason (R). Choose the
correction option.

19. Assertion : The equation x2 + 3x + 1 = ^x − 2h2 is a quadratic equation.


Reason : Any equation of the form ax2 + bx + c = 0 where a ! 0 , is called a quadratic equation.
(a) Both assertion (A) and reason (R) are true and reason (R) is the correct explanation of assertion (A).
(b) Both assertion (A) and reason (R) are true but reason (R) is not the correct explanation of assertion (A).
(c) Assertion (A) is true but reason (R) is false.
(d) Assertion (A) is false but reason (R) is true.

20. Assertion : If in a circle, the radius of the circle is 3 cm and distance of a point from the centre of a circle is 5 cm,
then length of the tangent will be 4 cm.
Reason : (hypotenuse) 2 = (base) 2 + (height) 2
(a) Both assertion (A) and reason (R) are true and reason (R) is the correct explanation of assertion (A).
(b) Both assertion (A) and reason (R) are true but reason (R) is not the correct explanation of assertion (A).
(c) Assertion (A) is true but reason (R) is false.
(d) Assertion (A) is false but reason (R) is true.

Install NODIA App to See the Solutions.


Click Here To Install
Page 4 Sample Paper 29 CBSE Mathematics Class 10

Section - B
Section B consists of 5 questions of 2 marks each.

21. Explain whether 3 # 12 # 101 + 4 is a prime number or a composite number.

22. Form a quadratic polynomial p ^x h with 3 and - 2 as sum and product of its zeroes, respectively.
5
 O

If α and β are the zeroes of the polynomial f (x) = 5x2 − 7x + 1 then find the value of ` αβ + αβ j

23. If A ^ m3 , 5h is the mid-point of the line segment joining the points Q (- 6, 7) and R (- 2, 3), then what is the value
of m ?

24. In Figure, in TABC , DE z BC such that AD = 2.4 cm, AB = 3.2 cm and AC = 8 cm, then what is the length
of AE ?

 O
In Figure +D = +E and AD = AE , prove that TBAC is an isosceles triangle.
DB EC

25. A bag contains cards bearing numbers from 11 to 30. A card is taken out from the bag at random. Find the
probability that the selected card has multiple of 5 on it.

Section - C
Section C consists of 6 questions of 3 marks each.

26. Given that 2 is irrational, prove that (5 + 3 2 ) is an irrational number.

Install NODIA App to See the Solutions.


Click Here To Install
Page 5 Sample Paper 29 CBSE Mathematics Class 10

27. What is the value of x in given figure?

28. Evaluate : cos 45º + 1


sec 30º sec 60º

29. An isosceles triangle ABC , with AB = AC , circumscribes a circle, touching BC at P , AC at Q and AB at R .


Prove that the contact point P bisects BC .

30. A glass is in the shape of a cylinder of radius 7 cm and height 10 cm. Find the volume of juice in litre required to
fill 6 such glasses. Use π = 227
 O
The volume of a right circular cylinder with its height equal to the radius is 25 71 cm3 . Find the height of the
cylinder. ^Use π = 227 h

31. Find the mode of the following frequency distribution :

Class 15-20 20-25 25-30 30-35 35-40 40-45


Frequency 3 8 9 10 3 2

 O
The marks obtained by 110 students in an examination are given below

Marks 30-35 35-40 40-45 45-50 50-55 55-60 60-65


Number of Students 14 16 28 23 18 8 3

Find the mean marks of the students.

Section - D
Section D consists of 4 questions of 5 marks each.

32. Write all the values of p for which the quadratic equation x2 + px + 16 = 0 has equal roots. Find the roots of the
equation so obtained.
 O
In a flight of 600 km, an aircraft was slowed down due to bad weather. The average speed of the trip was reduced
by 200 km/hr and the time of flight increased by 30 minutes. Find the duration of flight.

Install NODIA App to See the Solutions.


Click Here To Install
Page 6 Sample Paper 29 CBSE Mathematics Class 10

33. If the point P ^x, y h is equidistant from the points Q ^a + b, b − a h and R ^a − b, a + b h , then prove that bx = ay .

34. a, b and c are the sides of a right triangle, where c is the hypotenuse. A circle, of radius r , touches the sides of
the triangle. Prove that r = a + b − c .
2
 O
In figure O is the centre of a circle of radius 5 cm. T is a point such that OT = 13 cm and OT intersects circle at
E . If AB is a tangent to the circle at E , find the length of AB , where TP and TQ are two tangents to the circle.

35. Four equal circles are described at the four corners of a square so that each touches two of the others. The shaded
area enclosed between the circle is 247 cm2. Find the radius of each circle.

Section - E
Case study based questions are compulsory.

36. Arc of a Baby Swing : When Mackenzie’s baby swing is started, the first swing (one way) is a 30 inch arc. As the
swing slows down, each successive arc is 1.5 inch less than the previous one.
(i) Find the length of the tenth swing.
(ii) How far Mackenzie has travelled during the 10 swings ?

Install NODIA App to See the Solutions.


Click Here To Install
Page 7 Sample Paper 29 CBSE Mathematics Class 10

37. Statue of Unity : It is a colossal statue of Indian statesman and independence activist Sardar Vallabh bhai Patel,
who was the first Deputy Prime Minister and Home minister of independent India.

Patel was highly respected for his leadership in uniting the 562 princely states of India to form the single Union
of India. It is located in the state of Gujarat and it is the world’s tallest statue.
(i) For a person standing 240 m from the center of the base of the statue, the angle of elevation to the top of
the statue is 45c . How tall is the statue?
(ii) A cop in helicopter near the top of the statue, notices a car wreck some distance from the statue. If the angle
of depression from the cop’s eyes to the wreck is 60c , how far away is the accident from the centre of base
of the statue?

38. Eight Ball : This is a game played on a pool table with 15 balls numbered 1 through 15 and a cue ball that is solid
white. Of the 15 numbered balls, 8 are a solid (nonwhite) color and numbered 1 through 8, and seven are striped
balls numbered 9 through 15.

The fifteen numbered pool balls (no cueball) are placed in a large bowl and mixed, then one is drawn out.
(i) What is the probability of drawing the eight ball ?
(ii) What is the probability of drawing a number greater than fifteen ?
(iii) What is the probability of drawing an even number ?
(iv) What is the probability of drawing a multiple of three ?

 ******

Install NODIA App to See the Solutions.


Click Here To Install
Page 1 Sample Paper 30 CBSE Mathematics Class 10

Sample Paper 30
Class- X Exam - 2023-24
Mathematics - Standard

Time Allowed: 3 Hours Maximum Marks : 80


General Instructions :
1. This Question Paper has 5 Sections A-E.
2. Section A has 20 MCQs carrying 1 mark each
3. Section B has 5 questions carrying 02 marks each.
4. Section C has 6 questions carrying 03 marks each.
5. Section D has 4 questions carrying 05 marks each.
6. Section E has 3 case based integrated units of assessment (04 marks each) with sub-parts.
7. All Questions are compulsory. However, an internal choice in 2 Qs of 5 marks, 2 Qs of 3 marks and 2 Questions
of 2 marks has been provided.
8. Draw neat figures wherever required. Take π = 227 wherever required if not stated.

Section - A
Section A consists of 20 questions of 1 mark each.

1. If the first term of an AP is - 5 and the common difference is 2, then the sum of the first 6 terms is
(a) 0 (b) 5
(c) 6 (d) 15

2. What are the values of x and y for the following system of linear equations ?
2x - y = 2
x + 3y = 15
(a) 4 and 5 (b) 3 and 4
(c) 5 and 4 (d) 4 and 4

In the formula x = a + h f
/ f i ui , for finding the mean of grouped frequency distribution, u
/ fi p
3. i is equal to

(a) xi + a (b) h (xi - a)


h

(c) xi - a (d) a - xi
h h

Install NODIA App to See the Solutions.


Click Here To Install
Page 2 Sample Paper 30 CBSE Mathematics Class 10

4. In given figure, the graph of a polynomial p (x) is shown. The number of zeroes of p (x) will be

(a) 1 (b) 2
(c) 3 (d) 4

5. If α and β are the zeroes the polynomial 2x2 − 4x + 5, the value of ^α − βh2 is
(a) 2 (b) 1
(c) -1 (d) - 6

6. If α and β are the zeroes of a polynomial x2 − 4 3 x + 3, then the value of α + β − αβ will be


(a) 3 ^2 - 3 h (b) 3 ^2 + 3 h
(c) 3 ^4 + 3h (d) 3 ^4 - 3h

7. If 2x + y = 23 and 4x − y = 19 , the value of (5y - 2x) and ^ yx - 2h will be


(a) - 75 and 31 (b) 31 and - 75
2 2
(c) 37 and 7 (d) 7 and 37

8. 1+ cot2 α =?
1 + cosec α
(a) cos α (b) tan α
(c) cosec α (d) sin α

9. In TABC , AB = 6 3 cm , AC = 12 cm and BC = 6 cm , then +B = .................. .


(a) 30c (b) 60c
(c) 45c (d) 90c

10. The quadratic equation 5x2 − 3x + 1 = 0 has


(a) two distinct real roots (b) two equal real roots
(c) no real roots (d) more than 2 real roots

Install NODIA App to See the Solutions.


Click Here To Install
Page 3 Sample Paper 30 CBSE Mathematics Class 10

11. The decimal representation of 21 will


16 # 15
(a) terminate after 2 decimal place (b) terminate after 3 decimal place
(c) terminate after 4 decimal places (d) terminate after 5 decimal places

12. The co-ordinate of the point dividing the line segment joining the points A (1, 3) and B (4, 6) in the ratio 2 : 1 is
......... .
(a) (5, 3) (b) (3, 5)
(c) (4, 6) (d) (6, 4)

13. If sin θ + sin2 θ = 1 then cos2 θ + cos 4 θ = ?


(a) 1 (b) 2
(c) 2 2 (d) 2 3

14. A bag contains cards with numbers written on it from 1–80. A card is pulled out at random. What is the
probability that the card shows a perfect square?
1
(a) 2 (b) 15
1 3
(c) 10 (d) 10

15. The ratio of the length of a rod and its shadow is 1 : 3 then the angle of elevation of the sun is
(a) 90c (b) 45c
(c) 30c (d) 75c

16. If the circumference of a circle and the perimeter of a square are equal, then
(a) Area of the circle = Area of the square
(b) Area of the circle > Area of the square
(c) Area of the circle < Area of the square
(d) Nothing definite can be said about the relation between the areas of the circle and square

17. The first term of AP is p and the common difference is q , then its 10th term is
(a) q + 9p (b) p - 9q
(c) p + 9q (d) 2p + 9q

18. Ratio of lateral surface areas of two cylinders with equal height is
(a) 1 : 2 (b) H : h
(c) R :r (d) None of these

In the question number 19 and 20, a statement of Assertion (A) is followed by a statement of Reason (R). Choose the
correction option.

19. Assertion : The equation 8x2 + 3kx + 2 = 0 has equal roots then the value of k is ! 83 .
Reason : The equation ax2 + bx + c = 0 has equal roots if D = b2 − 4ac = 0
(a) Both assertion (A) and reason (R) are true and reason (R) is the correct explanation of assertion (A).
(b) Both assertion (A) and reason (R) are true but reason (R) is not the correct explanation of assertion (A).
(c) Assertion (A) is true but reason (R) is false.
(d) Assertion (A) is false but reason (R) is true.

Install NODIA App to See the Solutions.


Click Here To Install
Page 4 Sample Paper 30 CBSE Mathematics Class 10

20. Assertion : PA and PB are two tangents to a circle with centre O . Such that +AOB = 110c, then +APB = 90c.
Reason : The length of two tangents drawn from an external point are equal.
(a) Both assertion (A) and reason (R) are true and reason (R) is the correct explanation of assertion (A).
(b) Both assertion (A) and reason (R) are true but reason (R) is not the correct explanation of assertion (A).
(c) Assertion (A) is true but reason (R) is false.
(d) Assertion (A) is false but reason (R) is true.

Section - B
Section B consists of 5 questions of 2 marks each.

21. Complete the following factor tree and find the composite number x .

22. If one zero of the polynomial 2x2 + 3x + λ is 12 , find the value of λ and the other zero.
 O
Find the value of k such that the polynomial x − ^k + 6h x + 2 ^2k + 1h has sum of its zeros equal to half of their
2

product.

23. If the centre of a circle is (3, 5) and end points of a diameter are (4, 7) and (2, y ), what is the value of y ?

24. In the given figure, TABC ~TPQR. Find the value of y + z.

 O
In an equilateral triangle of side 24 cm, find the length of the altitude.

25. A letter of English alphabet is chosen at random, find the probability that the letter so chosen is :
(i) a vowel,
(ii) a consonant.

Install NODIA App to See the Solutions.


Click Here To Install
Page 5 Sample Paper 30 CBSE Mathematics Class 10

Section - C
Section C consists of 6 questions of 3 marks each.

26. 144 cartons of Coke cans and 90 cartons of Pepsi cans are to be stacked in a canteen. If each stack is of the same
height and if it equal contain cartons of the same drink, what would be the greatest number of cartons each stack
would have?

27. In TABC, DE || BC, find the value of x .

28. If tan 2A = cot (A − 18c), where 2A is an acute angle, find the value of A.

29. Prove that the tangent at any point of a circle is perpendicular to the radius through the point of contact.

30. Isha is 10 years old girl. On the result day, Isha and her father Suresh were very happy as she got first position
in the class. While coming back to their home, Isha asked for a treat from her father as a reward for her success.
They went to a juice shop and asked for two glasses of juice.
Aisha, a juice seller, was serving juice to her customers in two types of glasses.
Both the glasses had inner radius 3 cm. The height of both the glasses was 10 cm.
First Type : A glass with hemispherical raised bottom.

Second Type : A glass with conical raised bottom of height 1.5 cm.

Isha insisted to have the juice in first type of glass and her father decided to have the juice in second type of glass.
Out of the two, Isha or her father Suresh, who got more quantity of juice to drink and by how much?
 O
A sphere of maximum volume is cut out from a solid hemisphere of radius 6 cm. Find the volume of the cut out
sphere.

Install NODIA App to See the Solutions.


Click Here To Install
Page 6 Sample Paper 30 CBSE Mathematics Class 10

31. The weekly expenditure of 500 families is tabulated below :

Weekly Expenditure(Rs.) Number of families


0-1000 150
1000-2000 200
2000-3000 75
3000-4000 60
4000-5000 15
Find the median expenditure.
 O
Following frequency distribution shows the expenditure on milk of 30 households in a locality :

Daily expenditure on milk (Rs.) 0-30 30-60 60-90 90-120 120-150


Number of households 5 6 9 6 4
Find the mode for the above data.

Section - D
Section D consists of 4 questions of 5 marks each.

32. Find the value of p for which the quadratic equation ^p + 1h x2 − 6 ^p + 1h x + 3 ^p + 9h = 0 , p !- 1 has equal roots.
Hence find the roots of the equation.
 O
If the price of a book is reduced by < 5, a person can by 4 more books for < 600. Find the original price of the
book.

33. If A ^5, 2h , B ^2, - 2h and C ^- 2, t h are the vertices of a right angled triangle with +B = 90º , then find the value
of t.

34. In Figure the radius of incircle of TABC of area 84 cm2 and the lengths of the segments AP and BP into which
side AB is divided by the point of contact are 6 cm and 8 cm Find the lengths of the sides AC and BC.

Continue on next page....

Install NODIA App to See the Solutions.


Click Here To Install
Page 7 Sample Paper 30 CBSE Mathematics Class 10

 O
In the given figure, O is the centre of the circle. Determine +APC , if DA and DC are tangents and +ADC = 50c.

35. Fig. depicts a racing track whose left and right ends are semi-circular. The distance between the two inner parallel
line segments is 60 m and they are each 106 m long. If the track is 10 m wide everywhere, find the area of the track.

Section - E
Case study based questions are compulsory.

36. It takes 5 toothpicks to build the top trapezoid shown at below. You need 9 toothpicks to build 2 adjoined
trapezoids and 13 toothpicks for 3 trapezoids.
(i) If 1000 toothpicks are available, how many trapezoids will be in the last complete row?
(ii) How many complete rows will there be?
(iii) How many toothpicks will you use to construct these rows?
(iv) Use the numbers in this problem to carefully describe the difference between a sequence and a series.

37. From the observation deck of a seaside building 200 m high, Jignesh sees two fishing boats in the distance. The
angle of depression to the nearer boat is 60c while for the boat farther away the angle is 45c .
(i) How far out to sea is the nearer boat?
(ii) How far apart are the two boats?

Continue on next page....

Install NODIA App to See the Solutions.


Click Here To Install
Page 8 Sample Paper 30 CBSE Mathematics Class 10

38. Family Structures : For a recent year, 51% of the families in the United States had no children under the age of
18; 20% had one child; 19% had two children; 7% had three children; and 3% had four or more children.

If a family is selected at random, find the following probability.


(i) Find the probability that the family has two or three children.
(ii) Find the probability that the family has more than one child.
(iii) Find the probability that the family has less than three children.
(iv) Based on the answers to parts (i), (ii) and (iii) which is most likely to occur? Explain why.

 ******

Install NODIA App to See the Solutions.


Click Here To Install
SOLUTION
SECTION A
1.(c) a3b2
Explanation: Given that, p = ab2 = a × b × b
And q= a3b = a × a × a × b
LCM of p and q= LCM (ab2, a3b)
= a × b × b × a × a
= a3b2 ...
[Since, LCM is the product of the greatest power of each prime factor
involved in the numbers]
Caution
 Students usually make mistakes while calculating prime factors. They should always start
with the lowest prime number.

2. (c) 3
Explanation:
The number of zeroes of a polynomial is equal to the number of points at
which graph of the polynomial cut the x-axis.
Here, graph of P(x) cuts x-axis at three points, namely, A, B and C. So,
number of zeroes of P(x) is three.
3. (d) 10
Explanation:
The system of equations will be inconsistent, if
a1 b1c1
= b ¹
a2 c2
2

Here, a1 = 1, b1 = 2, c1 = – 3
a2 = 5 b2 = k, c2 = 7
1 2 3
So, = ↑
5 k 7
i.e., k = 10
4. (b) –m, m + 3
Explanation: Given, equation is:
x2 – 3x – m(m + 3) = 0

.30/1/1 1 P.T.O.
Þ x2 – [(m + 3) – m]x – m(m + 3) = 0
Þ x2 – (m + 3) x + mx – m (m + 3) = 0
Þ x[x – (m + 3)] + m [x – (m + 3)] = 0
Þ (x + m) [x – (m + 3)] = 0
Þ x = –m, (m + 3)
5.(c) 7
Explanation: Let d be the common difference of the two A.P.’s and a1 be
the first term of the first A.P. and A1 be the first term of the second A.P.
Now, a1 = –1, A1 = –8
We know that, an = a + (n – 1)d
\ a4 = a1 + (4 – 1)d
= a + 3d = –1 + 3d
A4 = A1 + (4 – 1)d
= –8 + 3d
Now, the difference between their 4th terms will be
|a4 – A4| = (–1 + 3d) – (–8 + 3d)
= –1 + 3d + 8 – 3d
= 7
Hence, the required difference is 7.
6.(b)12 units
Explanation: Let A(0, 4), O(0, 0) and B(3, 0) be the vertices of DAOB.
Using distance formula, we get
OA = ( 0 − 0 )2 + ( 4 − 0 )2 = 16 = 4 units

OB = ( 3 − 0 )2 + ( 0 − 0 )2 = 9 = 3 units

AB = ( 3 − 0 )2 + ( 0 − 4 )2 = 9 + 16 = 25

= 5 units

\ Perimeter of DAOB = OA + OB + AB
= 4 + 3 + 5
= 12 units
Thus, the required perimeter of the triangle is 12 units.

.30/1/1 2
7.
(a) 2
Explanation: Since A and B lie on the circle having centre O.
Therefore, OA = OB
( 4 − 2 )2 + ( 3 − 3 )2 = ( x − 2 )2 + ( 5 − 3 )2

2 = ( x − 2) + 4
2

(x – 2)2 + 4 = 4
(x – 2)2 = 0
x–2=0
x=2
8. (c) 8 cm
Explanation: Given ABCD is a trapezium in which BC || AD and AD =
4 cm.
AO DO 1
Also, the diagonals AC and BD intersect at O such that = =
OC OB 2
In DAOD and DCOB,
ÐOAD = ÐOCB [alternate angles]
ÐODA = ÐOBC [alternate angles]
ÐAOD = ÐBOC [vertically opposite angles]
We know that AAA similarity criterion states that in two triangles, if
corresponding angles are equal, then their corresponding sides are in the
same ratio and hence the two triangles are similar.
\ DAOD ~ DCOB
We know that two triangles are similar if their corresponding sides are
proportional.
AO DO AD
Þ = =
CO BO BC
1 AD
Þ =
2 BC
1 4
Þ =
2 BC
\ BC = 8 cm

.30/1/1 3 P.T.O.
9. (d) 8 cm
Explanation:
C B
5 cm E
3cm D
X
O
5 cm

cm
8
A

According to the question,


Radius of circle, AO = OC = 5 cm
AE = 8 cm
AE = AO + OE
⇒ OE = AE – AO
⇒ OE = (8 – 5) = 3 cm
Now, ∠OAX = ∠BAX = 90°
[Tangent at any point of a circle is perpendicular to the radius through
the point of contact]
Also, CD || XY meets AB at E, so we have
90° + ∠AEC = 180°
⇒ ∠AEC = 90°
⇒ ∠OEC = 90°
By Pythagoras theorem, in DOEC,
OC2 = OE2 + EC2
⇒ (5)2 = (3)2 + (EC)2
⇒ EC2 = 25 – 9 = 16
⇒ EC = 4
Also, CE = ED
[since, perpendicular from centre to the chord bisects the chord]
⇒ CD = 2 × EC = 8 cm
Concept Applied
 Perpendicular from centre to the chord bisects the chord.

10.
(c) 132 cm
Explanation: Time covered by minute hand from 10:10 am to 10:25 am
= 15 minutes

.30/1/1 4
 Angle covered by minute hand in 1 minute
= 6°
 Angle covered by minute hand in 15 minutes
= 6° × 15
= 90°
So, distance covered by tip of minute hand
θ
= × 2πr
360
90 22
= × 2 × × 84
360 7
1
= × 2 × 22 ×12
4
= 2 × 22 × 3
= 132 cm
1
11. (b)
2
cot y° AC / BC CD CD 1
= = = =
cot x ° AC / CD BC 2CD 2
Explanation: In DADC,
AC
cot xº = ...(i)
CD
and, in DABC,
AC AC
cot yº = = ...(ii)
BC 2CD
[ D is mid-point of BC]

AC
cot y° 2CD 1
So, = AC =
cot x ° 2
CD

12. (d) 3
Explanation: Given, 3 tan A = 4
4
Þ tan A =
3

.30/1/1 5 P.T.O.
3 sin A + 2 cos A
Now,
3 sin A − 2 cos A
Divide numerator and denominator by cos A
3 sin A 2 cos A 4
+ 3× + 2
cos A cos A 3 tan A + 2 3 4 + 2 6
= = = 3
= 3 sin A 2 cos A = 3 tan A − 2 = 4 4 − 2 2
− 3× − 2
cos A cos A 3
13. Let AB be the girl and BM be her shadow. Similarly, let PQ be the
lamppost and QM be its shadow. Also, let θ be the angle of elevation of the
Sun.
P

A
3.2 m
1.6 m
q M
Q 3.2 m B 4.8 m

\ AB = 1.6 m, BM = 4.8 m, QB = 3.2 m


and ∠M = q
So, QM = QB + BM
= 3.2 + 4.8 = 8 m
Now in DABM,
AB 1.6 1
tan q = = = ...(i)
BM 4.8 3
Also, in DPQM,
PQ PQ
tan q = =
QM 8
1 PQ
= ⇒  [From (i)]
3 8
8
⇒ PQ = = 2.67 m
3
Hence, the height of the lamppost is 2.67 m.
14. (c) 16 : 9
Explanation: Let the radii of the two circles be r and R, the circumferences
of the circles be c and C and the areas of the two circles be a and A.

.30/1/1 6
c 4
Now, =
C 3
2≠r 4
⇒ =
2≠R 3
r 4
⇒ =
R 3
Now, the ratio between their areas is given by
a ≠r 2
=
A ≠R 2
2
r
=  
R
2
4
=  3 
 
a 16
=
A 9

π 
15. (d) 2r  + 1 
6 
Explanation: Let, AOB be a sector of radius r and central angle 60°,

r x
60°

A B
Perimeter of sector
AOB= OA + OB + arc AB
where, OA= OB = r
θ
Length of arc AB = × 2πr
360
60
= ×2×p×r
360
≠r
=
3

.30/1/1 7 P.T.O.
≠r
⇒ Perimeter of sector AOB = r + r +
3
 π
= r 1 + 1 + 
 3
 π π 
= r  2 + 3  = 2r  6 + 1 
 

 π π 
r  2 + == 2r  + 1 
 3 6 

16. (b) 14
Explanation: It is given that,
Total no. of eggs = 400
Probability of getting a bad egg = 0.035
No. of bad eggs
\ Probability of getting a bad egg = = 0.035
Total no. of eggs
No. of bad eggs
⇒ = 0.035
400
⇒ No. of bad eggs = 0.035 × 400 = 14.
Hence, the number of bad eggs in the lot is 14.
9
17. (d)
22
Explanation: Total no. of cards in a pack = 52
After removing red-colored – jack, queen, king, and aces,
Cards removed = 2 + 2 + 2 + 2 = 8
\ Remaining number of cards = 52 – 8 = 44
Number of red cards left = 26 – 8 = 18
18 9
\ Required probability = =
44 22
18. (b) 25
Explanation:
Class Frequency Cumulative
Frequency
0–5 10 10
5–10 15 25
10–15 12 37

.30/1/1 8
15–20 20 57
20–25 9 66
Here, N = 66
N
\ = 33, which lies in the interval 10 – 15.
2
So, the lower limit of the median class is 10.
The highest frequency is 20, which lies in the interval 15 – 20.
Therefore, the lower limit of modal class is 15.
So, the required sum is 10 + 15 = 25.
19. (c) Assertion (A) is true but reason (R) is false.
Explanation : When cubes are joined end to end, it forms a cuboid.
where, l = 2 × 7 = 14 cm
b = 7 cm
and h = 7 cm
Total surface area of cuboid = 2(lb + bh + hl)
= 2 (14 × 7 + 7 × 7 + 7 × 14)
= 490 cm2
Hence, assertion is true but reason is false.
And pythagoras theorem h2 = l2 + b2
20. (a) B
 oth assertion (A) and reason (R) are true and reason (R) is the correct
explanation of assertion (A).
1 1 1
Explanation: Given sequence is, , , .
bc ab ac
Multiplying each term by abc, we get
1
× abc = a
bc
1
× abc = b
ac
1
× abc = c
ab
Resulting sequence is a, b, c.
Hence, both assertion and reason are true and reason is the correct
explanation of the assertion

.30/1/1 9 P.T.O.
SECTION B
Given numbers are 306 and 657.
21.
The smallest number divisible by 306 and 657
= LCM(306, 657)
Prime factors of 306 = 2 × 3 × 3 × 17
Prime factors of 657 = 3 × 3 × 73
\ LCM of (306, 657) = 2 × 3 × 3 × 17 × 73
= 22338
Hence, the smallest number divisible by 306 and 657 is 22,338.
22. In ΔABC, DE || AC
BD BE
= ...(i) (Using BPT)
AD EC
A

B
F E
C

In ΔABE, DF || AE
BD BE
= ....(ii) (Using BPT)
AD FE
From (i) and (ii)
BD BE BF
= =
AD EC FE
BF BE
Thus, =
FE EC
23. A

r
2r
P O

Let ∠APO =q
OA 1
sin q = =
OP 2
⇒ q = 30°

.30/1/1 10
⇒ ∠APB = 2q = 60°
Also, ∠PAB = ∠PBA = 60° (QPA = PB)
⇒ DAPB is equilateral.
Given: OP = 2r and OA = r
Now, AP ⊥ OA [Q Tangent ⊥ Radius]
\ DOAP is a right triangle.
Let ∠OPA = q
Then, in DOAP,
OA
sin q =
OP
r 1
= = = sin 30°
2r 2
⇒ q = 30°
Since, tangents are equally inclined to the line, joining the centre of the
circle to their common external point.

\ ∠OPA = ∠OPB = 1 ∠APB


2
or, ∠APB = 2∠OPA
= 2q
= 2 × 30° = 60°
Also, in DAPB,
AP = PB [Q Tangents drawn from an external point are equal]
⇒ ∠ABP= ∠PAB = x° (say) [Equal angles opposite to equal sides]
180° − ∠ APB
So, x° =
2
180° − 60°
= = 60°
2
⇒ ∠APB = ∠PAB = ∠ABP = 60°
\ DAPB is an equilateral triangle.
1
24. (b)
2
cot y° AC / BC CD CD 1
= = = =
cot x ° AC / CD BC 2CD 2

Explanation: In DADC,
AC
cot xº = ...(i)
CD

.30/1/1 11 P.T.O.

and, in DABC,
AC AC
cot yº=
= ...(ii)
BC 2CD
[ D is mid-point of BC]
AC
cot y° 2CD
So, = AC
cot x °
CD
1
=
2

OR
K + 1= sec2q(1 + sinq)(1 – sinq)
= sec2q(1 – sin2q)[ a2 – b2 = (a + b)(a – b)]
= sec2qcos2q  [ 1 – sin2q = cos2q]
1
= 2
2
.cos θ
cos θ
K+1=1
K=0
25.

60°

M 3.5 N

From the figure,


OM = ON [radii]
⇒ ∠OMN = ∠ONM = 60º
⇒ ∠MON = 60º [given]
⇒ ∆MON is an equilateral triangle.
θ
Arc length of minor sector = × 2πr
360º
60º 22 7
= × 2× ×
360º 7 2

.30/1/1 12
22
=
6
11
= cm
3
OR
Let, the measure of ∠A, ∠B, ∠C and ∠D be q1, q2, q3 and q4 respectively
Required area = Area of sector with centre A + Area of sector with centre
B + Area of sector with centre C + Area of sector with centre D
θ1 θ θ
× π × 72 + 2 × π × 72 + 3 × π × 72
= 360 360 360
θ
+ 4 × π × 72
360
( θ1 + θ2 + θ3 + θ4 )
= × π × 72
360
( 360 ) 22
= × ×7 ×7
360 7  (By angle sum property of a triangle)
= 154 cm 2

SECTION C
26. Prime factorisations are:
24= 2 × 2 × 2 × 3
32= 2 × 2 × 2 × 2 × 2
LCM (24, 32)= 2 × 2 × 2 × 2 × 2 × 3
= 96
According to the question, full packs and same number of pencils and
crayons are bought. i.e., 96.
96
Thus, 96 crayons or = 3 packs of crayons
32
96
96 pencils of = 4 packs of pencils.
24
27. Let α, β be the zeroes of f(x) = ax2 + bx + c.
b c
Thus, α+β= – and αβ =
a a

.30/1/1 13 P.T.O.
1 1
Now, and are the zeroes of required polynomial.
α β
1 1 α+β –b / a b
So, + = = = –
α β αβ c/a c
1 1 1 1 a
And, . = αβ = c / a = c
α β
\ Required polynomial is:
x2 – (sum of zeroes)x + product of zeroes.
2  −b  a
⇒ x −  x +
 c  c
⇒ cx2 + bx + a
So, the required polynomial is cx2 + bx + a.
28. Let the actual speed of the train be x km/hr and let the actual time taken
be y hours.
Distance covered is xy km
If the speed is increased by 6 km/hr, then time of journey is reduced
by 4 hours i.e., when speed is (x + 6) km/hr, time of journey is (y − 4)
hours.
\ Distance covered = (x + 6) (y − 4)
Þ xy = (x + 6)(y − 4)
Þ −4x + 6y − 24 = 0
Þ −2x + 3y − 12 = 0 ...(i)
Similarly xy = (x − 6)(y + 6)
Þ 6x − 6y − 36 = 0
Þ x−y−6=0 ...(ii)
Solving (i) and (ii) we get x = 30 and y = 24
Putting the values of x and y in equation (i), we obtain
Distance = (30 × 24) km = 720 km.
Hence, the length of the journey is 720 km.
OR
p
Let the fraction be .
q
According to the question,
p +1 p 1
= 1 and =
q -1 q +1 2

.30/1/1 14
⇒ p – q = –2 and 2p – q = 1
Subtracting the two equations, we get
(p – q) – (2p – q) = –2 – 1
⇒ –p = –3
⇒ p = 3
Putting p = 3 in the equation p – q = –2, we get
3 – q = –2
⇒ q = 5
\ p = 3 and q = 5
3
Thus, the required fraction is
.
5
29. Let ABCD be the rhombus circumscribing the circle with centre O,
such that AB, BC, CD and DA touch the circle at points P, Q, R and S
respectively.
D R C

S
O
Q

A P B

We know that the tangents drawn to a circle from an exterior point
are equal in length
\ AP = AS ...(i)
BP = BQ ...(ii)
CR = CQ ...(iii)
DR = DS ...(iv)
Adding (i), (ii), (iii) and (iv) we get
AP + BP + CR + DR = AS + BQ + CQ + DS
(AP + BP) + (CR + DR) = (AS + DS) + (BQ + CQ)
\ AB + CD = AD + BC ...(v)
Since AB = DC and AD = BC (opposite sides of parallelogram ABCD)
putting in (v) we get,
2AB = 2AD
or AB = AD.
\ AB= BC = DC = AD
Since a parallelogram with equal adjacent sides is a rhombus, so ABCD
is a rhombus

.30/1/1 15 P.T.O.
OR
X P Y
1
2

O
C
4
3
X’ Q B Y’

Join OC
In DOPA and DOCA
OP = OC(radii of same circle)
PA = CA
(length of two tangents from an external point)
AO = AO (Common)
Therefore, DOPA @ DOCA
(By SSS congruency criterion)
Hence, ∠1 = ∠2 (CPCT)
Similarly ∠3 = ∠4
∠PAB + ∠QBA = 180°
(co interior angles are supplementary as XY || X’Y’)
2∠2 + 2∠4 = 180°
∠2 + ∠4 = 90° ...(i)
∠2 + ∠4 + ∠AOB = 180° (Angle sum property)
Using (i), we get, ∠AOB = 90°
sin3 θ / cos3 θ cos3 θ / sin3 θ
30. LHS : +
1 + sin 2 θ / cos2 θ 1 + cos2 θ / sin 2 θ
sin3 θ / cos3 θ cos3 θ / sin3 θ
= +
(cos2 θ + sin 2 θ) / cos2 θ (sin 2 θ + cos2 θ) / sin 2 θ
sin3 θ cos3 θ
= +
cos θ sin θ
sin 4 θ + cos4 θ
=
cos θ sin θ
(sin 2 θ + cos2 θ)2 − 2 sin 2 θ cos2 θ
=
cos θ sin θ
1 − 2 sin 2 θ cos2 θ
=
cos θ sin θ

.30/1/1 16
1 2 sin 2 θ cos2 θ
= −
cos θ sin θ cos θ sin θ
= secq cosecq – 2sinq cosq
= RHS
31.
Class Number of Cumulative
persons Frequency
0 - 10 3 3
10 - 20 7 10
20 - 30 12 22
30 - 40 18 40
40 - 50 14 54
50 - 60 17 71

N 71
N = 71, =
2 2

= 35.5
Median class: 30 – 40
l = 30, h = 10, f = 18, cf = 22
N 
  cf 
 
Median = l +  2  ×h
 f 
 
 
 35.5 – 22 
 
= 30 +  18  × 10
 
= 30 + 7.5 = 37.5
The median age of the person visiting the museum is 37.5 years.

SECTION D
Let the time taken by larger pipe alone to fill the tank = x hours
32.
Therefore, the time taken by the smaller pipe
= x + 10 hours
Water filled by larger pipe running for 4 hours
4
= litres
x
Water filled by smaller pipe running for 9 hours

.30/1/1 17 P.T.O.
9
= litres
x + 10
4 9
We know that,
+ = 1
x x + 10 2
Which on simplification gives:
x2 − 16x − 80 = 0
x2 − 20x + 4x − 80 = 0
x(x – 20) + 4(x – 20) = 0
(x + 4)(x – 20) = 0
x = –4, 20
x cannot be negative.
Thus, x = 20
x + 10 = 30
Larger pipe would alone fill the tank in 20 hours and smaller pipe would
fill the tank alone in 30 hours.
OR
Let the original speed of the train be x km/h.
360 360
\ − = 48
x x +5
Þ x2 + 5x − 2250 = 0
Þ (x + 50)(x − 45)= 0 \ x = 45
Hence original speed of the train = 45 km/h
Explanation: Let the original speed of the train be x km/h.
\ Increased speed = (x + 5) km/h.
So, time taken to cover 360 km at original speed,
360
t1 = hr
x
And, time taken to cover 360 km at increased speed,
360
t2 = hr
x +5
Now, according to the question,
48  48 
t1 – t2 = hr  Q48 min. = hr 
60  60 

.30/1/1 18
360 360 48
Þ − =
x x +5 60
x +5− x  4
Þ 360   =
 x( x + 5)  5
Þ 4x(x + 5) = 360 × 5 × 5
Þ x(x + 5) = 2250
Þ x + 5x – 2250 = 0
­2

Þ x2 + 50x – 45x – 2250 = 0


Þ x(x + 50) – 45(x + 50) = 0
Þ (x – 45) (x + 50) = 0
Þ x – 45 = 0
[x + 50 ¹ 0, as speed cannot be negative]
Þ x = 45
Hence, the original speed of the train is 45 km/h.
33. Given: l || m and line segments AB, CD and EF are concurrent at P.
AE AC CE
To prove: = =
BF BD FD
Proof: In ∆APC and ∆BPD
ÐAPC = ÐBPD [Vertically opposite angles]
ÐPAC = ÐPBD [Alternate angles]
\ ∆APC ~ ∆BPD[By AA similarity criterion]
AP AC PC
\ = = ..(i)
BP BD PD
In ∆APE and ∆BPF,
ÐAPE = ÐBPF [Vertically opposite angles]
ÐPAE = ÐPBF [Alternate angles]
\ ∆APE ~ ∆BPF [By AA similarity criterion]
AP
\ = AE = PE ...(ii)
BP BF PF
In ∆PEC and ∆PFD,
ÐEPC = ÐFPD [Vertically opposite angles]
ÐPCE = ÐPDF [Alternate angles]
\ ∆PEC ~ ∆PFD [By AA similarity criterion]

.30/1/1 19 P.T.O.
PE EC PC
\ = =
PF FD PD  ...(iii)

From eqns. (i), (ii) and (iii), we get

AP AC PC AE PE EC
= = = = =
BP BD PD BF PF FD

AE AC CE
\ = =
BF BD FD

Hence, proved.
34. First Solid Second Solid

(A) SA for first new solid (S1):


6 × 7 × 7 + 2p × 3.52 – p ×3.52
= 294 + 77 – 38.5
= 332.5 cm2
SA for second new solid (S2):
6 × 7 × 7 + 2p × 3.52 – p × 3.52
= 294 + 77 – 38.5
= 332.5 cm2
So S1 : S2 = 1:1
(B) Volume for first new solid (V1)
2
=7×7×7– p × 3.53
3
539
= 343 −
6
1519
= cm3
6
Volume for second new solid (V2)

.30/1/1 20
2
=7×7×7+ p × 3.53
3
539 2597
= 343 + = cm3
6 6

OR
Let h be height of the cylinder, and r the common radius of the cylinder
and hemisphere.


Then, the total surface area = CSA of cylinder + CSA of hemisphre
= 2 prh + 2pr2 = 2pr(h + r)
22
= × 390(145 + 30) cm2
7
22
= × 30 × 175 cm2
7
= 33000 cm­2 = 3.3 m2
Detailed Answer :
Given: The height of the cylinder is 1.45 m and its radius is 30 cm.
Let h be height of the cylinder, and r the common radius of the cylinder
and hemisphere. Then,
The total surface area of the bird-bath = CSA of cylinder + CSA of
hemisphere + Area of the circle in bottom
= 2prh + 2pr2
22 22
= (2 × × 30 × 145) + (2 × × 302)
7 7  [Q 1.45 m = 145 cm]

.30/1/1 21 P.T.O.
191400 39600
= +
7 7
231000
=
7
= 33000 cm2
= 3.3 m2
Hence, the total surface area of the birdbath is 3.3 m2.
35.
Cumulative
Class Frequency
frequency
0-5 12 12
5-10 a 12 + a
10-15 12 24 + a
15-20 15 39 + a
20-25 b 39 + a + b
25-30 6 45 + a + b
30-35 6 51 + a + b
35-40 4 55 + a + b
Total 70
Then, 55 + a + b = 70
a+b= 15 ...(i)
Given, median = 16
Then, median class is15 – 20
N
− cf
l + 2 ×h
Median = l +
\ f
Here, l = 15
h=5
cf = 24 + a
f = 15
N = 70
35 − 24 − a
\ 16 = 15 + ×5
15

.30/1/1 22
11 − a
Þ 1= ×5
15
Þ 3 = 11 – a
Þ a=8
Put the value ‘a’ in equation (i), we get
b = 15 – 8 = 7
Hence, the values of ‘a’ and ‘b’ are 8 and 7.

SECTION E
36. (A) Since each row is increasing by 10 seats,
so it is an AP with first term a = 30, and
common difference d = 10.
So number of seats in 10th row
a10 = a + 9d
= 30 + 9 × 10 = 120
n
(B) Sn = [2a + (n – 1)d]
2
n
1500 = [2 × 30 + (n -1)10]
2
3000 = 50n + 10n2
n2 + 5n – 300 = 0
n2 + 20n – 15n – 300 = 0
(n + 20)(n – 15) = 0
Rejecting the negative value, n = 15
OR
No. of seats already put up to the 10th row
= S10
10
S10= [2  30  (10  1)10]
2
= 5(60 + 90) = 750
So, the number of seats still required to be put are 1500 – 750 = 750
(C) If no. of rows = 17
then the middle row is the 9th row
a8 = a + 8d

.30/1/1 23 P.T.O.
= 30 + 80
= 110 seats
37. (A) B(1,2), F(-2,9)
BF2 = (–2 – 1)2 + (9 – 2)2
= (–3)2 + (7)2
= 9 + 49
= 58
So, BF = 58 units

(B) R
13
12
Q
S 11 P
10
T 9 O
F E
8
U G 7 D N
6
5
V H 4 C M
3
A B
W 2 L
1
0
X K
0 –9–8–7 –6 –5 –4 –3 –2 –1 –11 2 3 4 5 6 7 8
I –2J
–3
–4

W(–6, 2), X(–4, 0), O(5, 9), P(3, 11)


Clearly WXOP is a rectangle
Point of intersection of diagonals of a rectangle is the mid point of the
 -6 + 5 2 + 9 
diagonals. So the required point is mid point of WO or XP =  , 
 2 2 

 -1 11 
=  2 , 2 
 
(C) A(–2, 2), G(–4, 7)
Let the point on y-axis be Z(0, y)
AZ2 = GZ2
(0 + 2)2 + (y – 2)2 = (0 + 4)2 + (y – 7)2
(2)2 + y2 + 4 – 4y = (4)2 + y2 + 49 – 14y
8 – 4y = 65 – 14y
10y = 57
So , y = 5.7
i.e. the required point is (0, 5.7)

.30/1/1 24
OR
13
R Q
12
S 11 P
10
T 9 O
F E
8
U G 7 D N
6
5
V H 4 C M
3
A B
W 2 L
1
0
X K
0 –9–8–7 –6 –5 –4 –3 –2 –1 –11 2 3 4 5 6 7 8
I –2J
–3
–4

A(–2, 2), F(–2, 9), G(–4, 7), H(–4, 4)


Clearly GH = 7 – 4 = 3units
AF = 9 – 2 = 7 units
So, height of the trapezium AFGH = 2 units
1
So, area of AFGH = (AF + GH) x height
2
1
= (7 + 3) × 2
2
= 10 sq. units
38.

PC
(A) sin 60º =
PA
3 18
⇒ =
2 PA
⇒ PA = 12 3m
PC
(B) sin 30º =
PB
1 18
⇒ =
2 PB
⇒ PB = 36 m

.30/1/1 25 P.T.O.
PC
(C) tan 60º =
AC
18
⇒ 3=
AC

⇒ AC = 6 3 m
tan 30º = 6 3
1 18
⇒ =
3 CB

⇒ CB = 18 3 m
Width AB = AC + CB

= 6 3 +18 3 = 24 3 m
OR

RB = PC = 18 m and PR = CB = 18 3 m
QR
tan 30º =
PR
1 QR
⇒ =
3 18 3
⇒ QR = 18 m
QB = QR + RB = 18 + 18
= 36 m.
Hence, height BQ is 36 m.

.30/1/1 26
Maths 11 Sample paperS
FOR CLASS X
PREPARED BY: M. S. KUMAR SWAMY, TGT(MATHS)
KENDRIYA VIDYALAYA GACHIBOWLI, GPRA CAMPUS, HYD-32
SAMPLE PAPER TEST 01 FOR BOARD EXAM 2024

SUBJECT: MATHEMATICS MAX. MARKS : 80


CLASS : X DURATION : 3 HRS
General Instruction:
1. This Question Paper has 5 Sections A-E.
2. Section A has 20 MCQs carrying 1 mark each.
3. Section B has 5 questions carrying 02 marks each.
4. Section C has 6 questions carrying 03 marks each.
5. Section D has 4 questions carrying 05 marks each.
6. Section E has 3 case based integrated units of assessment (04 marks each) with sub-parts of the
values of 1, 1 and 2 marks each respectively.
7. All Questions are compulsory. However, an internal choice in 2 Qs of 5 marks, 2 Qs of 3 marks and
2 Questions of 2 marks has been provided. An internal choice has been provided in the 2marks
questions of Section E
8. Draw neat figures wherever required. Take π =22/7 wherever required if not stated.
SECTION – A
Questions 1 to 20 carry 1 mark each.

1. In a formula racing competition, the time taken by two racing cars A and B to complete 1 round of
the track is 30 minutes and p minutes respectively. If the cars meet again at the starting point for the
first time after 90 minutes and the HCF (30, p) = 15, then the value of p is
(a) 45 minutes (b) 60 minutes (c) 75 minutes (d) 180 minutes

2. The solution of the following pair of equation is:


x – 3y = 2, 3x – y = 14
(a) x = 5, y = 1 (b) x = 2, y = 3 (c) x = 1, y = 2 (d) x = 1, y = 4

3. If two positive integers a and b are written as a = x3y2 and b = xy3, where x and y are prime
numbers, then the HCF (a, b) is:
(a) xy (b) xy2 (c) x3y3 (d) x2y2

4. The ratio in which x-axis divides the join of (2, -3) and (5, 6) is:
(a) 1: 2 (b) 3 : 4 (c) 1: 3 (d) 1: 5

5. The 11th and 13th terms of an AP are 35 and 41 respectively, its common difference is
(a) 38 (b) 32 (c) 6 (d) 3

6. A medicine-capsule is in the shape of a cylinder of radius 0.25 cm with two hemispheres stuck to
each of its ends. The length of the entire capsule is 2 cm. What is the total surface area of the
capsule? (Take π as 3.14)

(a) 0.785 cm2 (b) 0.98125 cm2 (c) 2.7475 cm2 (d) 3.14 cm2

Prepared by: M. S. KumarSwamy, TGT(Maths) Page - 1-


7. A 1.6 m tall girl stands at distance of 3.2 m from a lamp post and casts shadow of 4.8 m on the
ground, then the height of the lamp post is
(a) 8 m (b) 4 m (c) 6 m (d) 8/3 m

8. A tangent is drawn from a point at a distance of 17 cm of circle (O, r) of radius 8 cm. The length of
tangent is
(a) 5 cm (b) 9 cm (c) 15 cm (d) 23 cm

9. The runs scored by a batsman in 35 different matches are given below:


Runs Scored 0-15 15-30 30-45 45-60 60-75 75-90
Frequency 5 7 4 8 8 3
The lower limit of the median class is
(a) 15 (b) 30 (c) 45 (d) 60

10. If in two triangles, DEF and PQR, ∠ =∠ and ∠ =∠ , then which of the following is not true?
EF DF EF DE DE DF EF DE
(a)  (b)  (c)  (d) 
PR PQ RP PQ QR PQ RP QR

11. In the given figure, if AB = 14 cm, then the value of tan B is:

4 14 5 13
(a) (b) (c) (d)
3 3 3 3

12. Two cubes each with 6 cm edge are joined end to end. The surface area of the resulting cuboid is
(a) 180 ² (b) 360 ² (c) 300 ² (d) 260 ²

13. A cone, a hemisphere and cylinder are of the same base and of the same height. The ratio of their
volumes is
(a) 1 : 2 : 3 (b) 2 : 1 : 3 (c) 3 : 1 : 2 (d) 3 : 2 : 1

14. The probability of getting a bad egg in a lot of 400 is 0.035. The number of bad eggs in the lot is
(a) 7 (b) 14 (c) 21 (d) 28

15. If 3 sin θ – cos θ = 0 and 0° < θ < 90°, find the value of θ.
(a) 30° (b) 45° (c) 60° (d) 90°

16. Find the value of k for which the equation x2 + k(2x + k − 1)+ 2 = 0 has real and equal roots.
(a) 2 (b) 3 (c) 4 (d) 5

17. In the below figure, the pair of tangents AP and AQ drawn from an external point A to a circle with
centre O are perpendicular to each other and length of each tangent is 5 cm. Then radius of the
circle is
(a) 10 cm (b) 7.5 cm (c) 5 cm (d) 2.5 cm

Prepared by: M. S. KumarSwamy, TGT(Maths) Page - 2-


18. The radii of two cylinders are in the ratio 5 : 7 and their heights are in the ratio 3 : 5. The ratio of
their curved surface area is
(a) 3 : 7 (b) 7 : 3 (c) 5 : 7 (d) 3 : 5

Direction : In the question number 19 & 20 , A statement of Assertion (A) is followed by a


statement of Reason(R) . Choose the correct option

19. Assertion (A): If x = 2 sin2θ and y = 2 cos2θ + 1 then the value of x + y = 3.


Reason (R): For any value of θ, sin2θ + cos2θ = 1

20. Assertion (A): The length of the minute hand of a clock is 7 cm. Then the area swept by the
minute hand in 5 minute is 77/6 cm2.

Reason (R): The length of an arc of a sector of angle q and radius r is given by l   2 r
3600

SECTION-B
Questions 21 to 25 carry 2M each

21. Find the point on y-axis which is equidistant from the points (5, – 2) and (–3, 2).

22. X is a point on the side BC of ∆ABC. XM and XN are drawn parallel to AB and AC respectively
meeting AB in N and AC in M. MN produced meets CB produced at T. Prove that TX2 = TB × TC.

23. The probability of selecting a blue marble at random from a jar that contains only blue, black and
green marbles is 1/5. The probability of selecting a black marble at random from the same jar is 1/4.
If the jar contains 11 green marbles, find the total number of marbles in the jar.

24. In figure PA and PB are tangents to the circle drawn from an external point P. CD is the third
tangent touching the circle at Q. If PA = 15 cm, find the perimeter of ΔPCD.

OR
Two concentric circles are of radii 8 cm and 5 cm. Find the length of the chord of the larger circle
which touches the smaller circle.

Prepared by: M. S. KumarSwamy, TGT(Maths) Page - 3-


25. For what value of k, the following system of equations have infinite solutions:
2x – 3y = 7, (k + 2)x – (2k + 1)y = 3 (2k – 1)?
OR
Sumit is 3 times as old as his son. Five years later, he shall be two and a half time as old as his son.
How old is Sumit at present?

SECTION-C
Questions 26 to 31 carry 3 marks each

26. Find the coordinates of the points which divide the line segment joining A (–2, 2) and B (2, 8) into
four equal parts.

27. If PQ is a tangent drawn from an external point P to a circle with centre O and QOR is a diameter
where length of QOR is 8 cm such that ∠POR = 120°, then find OP and PQ.

p2 1
28. If sec θ + tan θ = p, prove that sin θ = .
p2 1
OR
If sin θ + cos θ = √3 , then prove that tan θ + cot θ = 1.

29. Daily wages of 110 workers, obtained in a survey, are tabulated below:
Daily Wages (in Rs. ) 100-120 120-140 140-160 160-180 180-200 200-220 220-240
Number of Workers 10 15 20 22 18 12 13
Compute the mean daily wages and modal daily wages of these workers.

30. Solve the following linear equations:


152x – 378y = –74 and –378x + 152y = –604

31. The sum of the 5th and the 9th terms of an AP is 30. If its 25th term is three times its 8th term, find
the AP.
OR
If the ratio of the sum of first n terms of two AP’s is (7n + 1) : (4n + 27), find the ratio of their mth
terms.

SECTION-D
Questions 32 to 35 carry 5M each

32. From a solid cylinder whose height is 2.4 cm and diameter 1.4 cm, a conical cavity of the same
height and same diameter is hollowed out. Find the total surface area of the remaining solid to the
nearest cm2.
OR
Rasheed got a playing top (lattu) as his birthday present, which surprisingly had no colour on it. He
wanted to colour it with his crayons. The top is shaped like a cone surmounted by a hemisphere.
The entire top is 5 cm in height and the diameter of the top is 3.5 cm. Find the area he has to
colour.

33. A motor boat whose speed is 15 km/hr in still water goes 30 km downstream and comes back in 4
hours 30 minutes. Find the speed of the stream.

34. State and prove Basic Proportional Theorem.

35. The lower window of a house is at a height of 2 m above the ground and its upper window is 4 m
vertically above the lower window. At certain instant, the angles of elevation of a balloon from

Prepared by: M. S. KumarSwamy, TGT(Maths) Page - 4-


these windows are observed to be 60° and 30°, respectively. Find the height of the balloon above
the ground.
OR
From the top of a 60 m high building, the angles of depression of the top and the bottom of a tower
are 45° and 60° respectively. Find the height of the tower. [Take √3 = 1.73]

SECTION-E (Case Study Based Questions)


Questions 36 to 38 carry 4M each

36. Shivani took a pack of 52 cards. She kept aside all the black face cards and shuffled the remaining
cards well.

Based on the above information answer the following questions.


(i) Write the number of total possible outcomes.
(ii) She draws a card from the well-shuffled pack of remaining cards. What is the probability that
the card is a face card?
(iii) Write the probability of drawing a black card.
OR
(iii) What is the probability of getting neither a black card nor an ace card?

37. In the month of April to June 2022, the exports of passenger cars from India increased by 26% in
the corresponding quarter of 2021–22, as per a report. A car manufacturing company planned to
produce 1800 cars in 4th year and 2600 cars in 8th year. Assuming that the production increases
uniformly by a fixed number every year.

Based on the above information answer the following questions.


(i) Find the production in the 1st year. (1)
(ii) Find the production in the 12th year. (1)
(iii) Find the total production in first 10 years. (2)
OR
(iii) In how many years will the total production reach 31200 cars? (2)

Prepared by: M. S. KumarSwamy, TGT(Maths) Page - 5-


38. Aditya plantations have two rectangular fields of the same width but different lengths. They are
required to plant 168 trees in the smaller field and 462 trees in the larger field. In both fields, the
trees will be planted in the same number of rows but in different number of columns.

(i) What is the maximum number of rows in which the trees can be planted in each of the fields? (2)
(ii) If the trees are planted in the number of rows obtained in part (i), how many columns will each
field have?
(iii) If total cost of planted trees in one column is Rs. 500, then find the cost to plant the trees in
smaller field.
OR
If the total cost of planted trees in one column is Rs. 500, the find the cost to plant the trees in larger
field.

Prepared by: M. S. KumarSwamy, TGT(Maths) Page - 6-


PM SHRI KENDRIYA VIDYALAYA GACHIBOWLI, GPRA CAMPUS, HYD-32
SAMPLE PAPER TEST 02 FOR BOARD EXAM 2024

SUBJECT: MATHEMATICS MAX. MARKS : 80


CLASS : X DURATION : 3 HRS
General Instruction:
1. This Question Paper has 5 Sections A-E.
2. Section A has 20 MCQs carrying 1 mark each.
3. Section B has 5 questions carrying 02 marks each.
4. Section C has 6 questions carrying 03 marks each.
5. Section D has 4 questions carrying 05 marks each.
6. Section E has 3 case based integrated units of assessment (04 marks each) with sub-parts of the
values of 1, 1 and 2 marks each respectively.
7. All Questions are compulsory. However, an internal choice in 2 Qs of 5 marks, 2 Qs of 3 marks and
2 Questions of 2 marks has been provided. An internal choice has been provided in the 2marks
questions of Section E
8. Draw neat figures wherever required. Take π =22/7 wherever required if not stated.
SECTION – A
Questions 1 to 20 carry 1 mark each.

1. If two positive integers p and q can be expressed as p = ab2 and q = a3b; a, b being prime numbers,
then LCM (p, q) is
(a) ab (b) a2b2 (c) a3b2 (d) a3b3
2. The perimeter of a triangle with vertices (0, 4), (0, 0) and (3, 0) is
(a) 5 units (b) 12 units (c) 11 units (d) (7 + √5) units
3. The zeroes of the polynomial x2 – 3x – m(m + 3) are
(a) m, m + 3 (b) –m, m + 3 (c) m, – (m + 3) (d) –m, – (m + 3)
4. The area of a quadrant of a circle, whose circumference is 22 cm, is
11 77 77 77
(a) cm2 (b) cm2 (c) cm2 (d) cm2
8 8 2 4
5. The pair of linear equations 2x + 3y = 5 and 4x + 6y = 10 is
(a) inconsistent (b) consistent (c) dependent consistent (d) none of these

6. If the circumference of a circle and the perimeter of a square are equal, then
(a) Area of the circle = Area of the square
(b) Area of the circle > Area of the square
(c) Area of the circle < Area of the square
(d) Nothing definite can be said about the relation between the areas of the circle and square.

7. The sum of the lower limit of median class and the upper limit of the modal class of the following
data is:
Marks 0 – 10 10 – 20 20 – 30 30 – 40 40 – 50 50 – 60
No. of students 8 10 12 22 30 18
(a) 70 (b) 80 (c) 90 (d) 100
8. A card is selected at random from a well shuffled deck of 52 cards. The probability of its being a
face card is
(a) 3/26 (b) 3/13 (c) 2/13 (d) 1/2

Prepared by: M. S. KumarSwamy, TGT(Maths) Page - 1-


7
9. In ABC right angled at B, sin A = , then the value of cos C is ………….
25
7 24 7 24
(a) (b) (c) (d)
25 25 24 7
10. The radius of the largest right circular cone that can be cut out from a cube of edge 4.2 cm is
(a) 2.1 cm (b) 4.2 cm (c) 3.1 cm (d) 2.2 cm

11. Volume and surface area of a solid hemisphere are numerically equal. What is the diameter of
hemisphere?
(a) 9 units (b) 6 units (c) 4.5 units (d) 18 units

12. In the ∆ABC, DE ∥ BC and AD = 3x − 2, AE = 5x − 4, BD = 7x − 5, CE = 5x − 3, then find the


value of x
(a) 1 (b) 7/10 (c) both (a) & (b) (d) none of these

13. Two circles touch each other externally at C and AB is common tangent of circles, then ∠ACB is
(a) 70° (b) 60° (c) 100° (d) 90°

5sin   3cos 
14. If 5 tan θ = 4, then the value of is
5sin   2cos 
(a) 1/6 (b) 1/7 (c) 1/4 (d) 1/5

15. Given that sin α = 1/2 and cos β = 1/2, then the value of (β – α) is
(a) 0° (b) 30° (c) 60° (d) 90°

16. Two identical solid hemispheres of equal base radius are stuck along their bases. The total surface
area of the combination is
(a) πr2 (b) 2πr2 (c) 3πr2 (d) 4πr2

17. Nature of roots of quadratic equation 2x2 – 4x + 3 = 0 is


(a) real (b) equal (c) not real (d) none of them

18. If ∆ABC ~ ∆EDF and ∆ABC is not similar to ∆DEF, then which of the following is not true?
(a) BC. EF = AC. FD (b) AB. EF = AC. DE
(c) BC. DE = AB. EF (d) BC. DE = AB. FD

Direction : In the question number 19 & 20 , A statement of Assertion (A) is followed by a


statement of Reason(R) . Choose the correct option

19. Assertion (A): The value of y is 3, if the distance between the points P(2, -3) and Q(10, y) is 10.
Reason (R): Distance between two points is given by ( x2  x1 )2  ( y2  y1 ) 2
(a) Both Assertion (A) and Reason (R) are true and Reason (R) is the correct explanation of
Assertion (A)
(b) Both assertion (A) and reason (R) are true and reason (R) is not the correct explanation of
Assertion (A)
(c) Assertion (A) is true but reason(R) is false.
(d) Assertion (A) is false but reason(R) is true.

20. Assertion (A): 6n never ends with the digit zero, where n is natural number.
Reason (R): Any number ends with digit zero, if its prime factor is of the form 2m × 5n, where m, n
are natural numbers.

Prepared by: M. S. KumarSwamy, TGT(Maths) Page - 2-


(a) Both Assertion (A) and Reason (R) are true and Reason (R) is the correct explanation of
Assertion (A).
(b) Both Assertion (A) and Reason (R) are true but Reason (R) is not the correct explanation of
Assertion (A).
(c) Assertion (A) is true but Reason (R) is false.
(d) Assertion (A) is false but Reason (R) is true.

SECTION-B
Questions 21 to 25 carry 2M each

21. For what values of k will the following pair of linear equations have infinitely many solutions? kx
+ 3y – (k – 3) = 0 and 12x + ky – k = 0

22. In the given figure, AP = 3 cm, AR = 4.5 cm, AQ = 6 cm, AB = 5 cm, AC = 10 cm. Find the length
of AD

23. Two concentric circles are of radii 5 cm and 3 cm. Find the length of the chord of the larger circle
which touches the smaller circle.

1
24. If sin (A + B) = √3/2 and sin (A – B) = , 0 ≤ A + B ≤ 90° and A > B, then find A and B.
2
OR
(1  sin  )(1  sin  )
If tan θ =3/4, evaluate
(1  cos  )(1  cos  )

25. Find the area of the sector of a circle with radius 4 cm and of angle 30°. Also, find the area of the
corresponding major sector. (Use π = 3.14)

SECTION-C
Questions 26 to 31 carry 3 marks each

26. Four bells toll at an interval of 8, 12, 15 and 18 seconds respectively. All the four begin to toll
together. Find the number of times they toll together in one hour excluding the one at the start.

27. Find the zeroes of the quadratic polynomial 6x2– 3 – 7x and verify the relationship between the
zeroes and the coefficients of the polynomial.
OR
Find the quadratic polynomial sum and product of whose zeros are –1 and –20 respectively. Also
find the zeroes of the polynomial so obtained.

28. The area of a rectangle gets reduced by 9 square units, if its length is reduced by 5 units and breadth
is increased by 3 units. If we increase the length by 3 units and the breadth by 2 units, the area
increases by 67 square units. Find the dimensions of the rectangle.

Prepared by: M. S. KumarSwamy, TGT(Maths) Page - 3-


29. In the below figure, XY and X′Y′ are two parallel tangents to a circle with centre O and another
tangent AB with point of contact C intersecting XY at A and X′Y′ at B. Prove that ∠AOB = 90°.

OR
In the below figure, two equal circles, with centres O and O', touch each other at X. OO' produced
meets the circle with centre O' at A. AC is tangent to the circle with centre O, at the point C. O'D is
DO '
perpendicular to AC. Find the value of .
CO

sin   cos   1
30. Prove that  sec   tan 
sin   cos   1

31. Two dice are thrown at the same time. What is the probability that the sum of the two numbers
appearing on the top of the dice is (i) 5? (ii) 10? (iii) at least 9?

SECTION-D
Questions 32 to 35 carry 5M each

32. A motor boat whose speed is 18 km/h in still water takes 1 hour more to go 24 km upstream than to
return downstream to the same spot. Find the speed of the stream.
OR
An express train takes 1 hour less than a passenger train to travel 132 km between Mysore and
Bangalore (without taking into consideration the time they stop at intermediate stations). If the
average speed of the express train is 11km/h more than that of the passenger train, find the average
speed of the two trains.

33. State and prove Basic Proportional Theorem.

34. If the median of the following distribution is 58 and sum of all the frequencies is 140. What is the
value of x and y?
Class 15 – 25 25 – 35 35 – 45 45 – 55 55 – 65 65 – 75 75 – 85 85 – 95
Frequency 8 10 x 25 40 y 15 7

Prepared by: M. S. KumarSwamy, TGT(Maths) Page - 4-


35. A toy is in the form of a hemisphere surmounted by a right circular cone of the same base radius as
that of the hemisphere. If the radius of the base of the cone is 21 cm and its volume is 2/3 of the
volume of the hemisphere, calculate the height of the cone and the surface area of the toy.
OR
A vessel full of water is in the form of an inverted cone of height 8 cm and the radius of its top,
which is open, is 5 cm. 100 spherical lead balls are dropped into the vessel. One fourth of the water
flows out of the vessel. Find the radius of a spherical ball.

SECTION-E (Case Study Based Questions)


Questions 36 to 38 carry 4M each

36. The top of a table is shown in the figure given below:

On the basis of above information answer the following questions.


(i) Find the distance between points A and B.
(ii) Write the co-ordinates of the mid point of line segment joining points M and Q.
(iii) If G is taken as the origin, and x, y axis put along GF and GB, then find the point denoted by
coordinates (4, 2) and (8, 4).
OR
Find the coordinates of H, G and also find the distance between them.

Prepared by: M. S. KumarSwamy, TGT(Maths) Page - 5-


37. Ananya saves Rs. 24 during the first month Rs. 30 in the second month and Rs. 36 in the third
month. She continues to save in this manner.

On the basis of above information answer the following questions.


(i) Whether the monthly savings of Ananya form an AP or not? If yes then write the first term and
common difference.
(ii) What is the amount that she will save in 15th month?
(iii) In which month, will she save Rs. 66?
OR
What is the common difference of an AP whose nth term is 8 – 5n?

38. A person/observer on the sea coast observes two ships in the sea, both the ships are in same straight
path one behind the other.
If the observer is on his building of height 20 meters (including observer) and he observes the angle
of depression of two ships as 45° and 60° respectively.

On the basis of above information answer the following questions.


(i) If a person observes a ship whose angle of depression is 60° then how much distance is the ship
away from the building?
(ii) If a person observes another ship whose angle of depression is 45° then how much distance that
ship is away from the building?
(iii) If a person observes the ship whose angle of depression changes from 60° to 30° then how far
be ship from the building if the observer is at 20 m of height (including him)?
OR
At a time when a person observes two ships whose angle of depressions are 60° and 45° the
distance between the ships is (in meter).

Prepared by: M. S. KumarSwamy, TGT(Maths) Page - 6-


PM SHRI KENDRIYA VIDYALAYA GACHIBOWLI, GPRA CAMPUS, HYD-32
SAMPLE PAPER TEST 03 FOR BOARD EXAM 2024

SUBJECT: MATHEMATICS MAX. MARKS : 80


CLASS : X DURATION : 3 HRS
General Instruction:
1. This Question Paper has 5 Sections A-E.
2. Section A has 20 MCQs carrying 1 mark each.
3. Section B has 5 questions carrying 02 marks each.
4. Section C has 6 questions carrying 03 marks each.
5. Section D has 4 questions carrying 05 marks each.
6. Section E has 3 case based integrated units of assessment (04 marks each) with sub-parts of the
values of 1, 1 and 2 marks each respectively.
7. All Questions are compulsory. However, an internal choice in 2 Qs of 5 marks, 2 Qs of 3 marks and
2 Questions of 2 marks has been provided. An internal choice has been provided in the 2marks
questions of Section E
8. Draw neat figures wherever required. Take π =22/7 wherever required if not stated.
SECTION – A
Questions 1 to 20 carry 1 mark each.

1. A card is selected from a deck of 52 cards. The probability of being a red face card is
(a) 3/26 (b) 3/13 (c) 2/13 (d) 1/2

2. If two tangents inclined at an angle of 60ᵒ are drawn to a circle of radius 3cm, then the length of
each tangent is equal to
3 3
(a) cm (b) 3 cm (c) 6 cm (d) 3 3
2

3. If the mean of a frequency distribution is 8.1 and fi = 20, fixi = 132 + 5k, then k =
(a) 3 (b) 4 (c) 5 (d) 6

4. If the radii of two circles are in the ratio of 4 : 3, then their areas are in the ratio of :
(a) 4 : 3 (b) 8 : 3 (c) 16 : 9 (d) 9 : 16

5. If one zero of the quadratic polynomial x2 + 3x + k is 2, then the value of k is


(a) 10 (b) –10 (c) 5 (d) –5

6. If two positive integers a and b are written as a = x3y2 and b = xy3; x, y are prime numbers, then
HCF (a, b) is
(a) xy (b) xy2 (c) x3y3 (d) x2y2

7. When 2120 is expressed as the product of its prime factors we get


(a) 2 × 5³ × 53 (b) 2³ × 5 × 53 (c) 5 × 7² × 31 (d) 5² × 7 × 33

8. In the ∆ABC, D and E are points on side AB and AC respectively such that DE || BC.
If AE = 2 cm, AD = 3 cm and BD = 4.5 cm, then CE equals
(a) 1 cm (b) 2 cm (c) 3 cm (d) 4 cm

9. If the distance between the points (2, –2) and (–1, x) is 5, one of the values of x is
(a) –2 (b) 2 (c) –1 (d) 1

Prepared by: M. S. KumarSwamy, TGT(Maths) Page - 1-


10. The value of k for which the pair of equation kx – y = 2 and 6x – 2y = 3 has unique solution
(a) k = 3 (b) k ≠ 3 (c) k ≠ 0 (d) k = 0

11. The median class of the following data is:


Marks 0 – 10 10 – 20 20 – 30 30 – 40 40 – 50 50 – 60
No. of students 8 10 12 22 30 18
(a) 20 – 30 (b) 30 – 40 (c) 40 – 50 (d) 50 – 60
12. The ratio of outer and inner perimeters of circular path is 23:22. If the path is 5 m wide, the
diameter of the inner circle is
(a) 55 m (b) 110 m (c) 220 m (d) 230 m

13. In ΔABC, right angled at B, AB = 5 cm and sin C = 1/2. Determine the length of side AC.
(a) 10 cm (b) 15 cm (c) 20 cm (d) none of these
14. If x2 + k (4x + k -1) + 2 = 0 has equal roots, then k = ………
2 2 3 1 3 1
(a)  ,1 (b) , 1 (c) , (d) ,
3 3 2 3 2 3
15. If x = a cos θ and y = b sin θ, then the value of b2x2 + a2y2 is
(a) a2 + b2 (b) a2/b2 (c) a2b2 (d) None of these

16. ABCD is a trapezium with AD ∥ BC and AD = 4cm. If the diagonals AC and BD intersect each
other at O such that AO/OC = DO/OB =1/2, then BC =
(a) 6cm (b) 7cm (c) 8cm (d) 9cm
17. The value of (sin 45° + cos 45°) is
(a) 1√2 (b) √2 (c) √3/2 (d) 1

18. Volumes of two spheres are in the ratio 64:27. The ratio of their surface areas is
(a) 3:4 (b) 4:3 (c) 9:16 (d) 16:9

Direction : In the question number 19 & 20 , A statement of Assertion (A) is followed by a


statement of Reason(R) . Choose the correct option

19. Statement A (Assertion): If product of two numbers is 5780 and their HCF is 17, then their LCM
is 340
Statement R( Reason) : HCF is always a factor of LCM
(a) Both Assertion (A) and Reason (R) are true and Reason (R) is the correct explanation of
Assertion (A).
(b) Both Assertion (A) and Reason (R) are true but Reason (R) is not the correct explanation of
Assertion (A).
(c) Assertion (A) is true but Reason (R) is false.
(d) Assertion (A) is false but Reason (R) is true.

20. Assertion (A): The point (0, 4) lies on y-axis.


Reason (R): The y co-ordinate of the point on x-axis is zero.
(a) Both Assertion (A) and Reason (R) are true and Reason (R) is the correct explanation of
Assertion (A)
(b) Both assertion (A) and reason (R) are true and reason (R) is not the correct explanation of
Assertion (A)
(c) Assertion (A) is true but reason(R) is false.
(d) Assertion (A) is false but reason(R) is true.

Prepared by: M. S. KumarSwamy, TGT(Maths) Page - 2-


SECTION-B
Questions 21 to 25 carry 2M each

21. If the system of equations 2x + 3y = 7 and (a + b)x + (2a – b)y = 21 has infinitely many solutions,
then find a and b.

tan 2  cot 2 
22. Simplify: 
1  tan 2  1  cot 2 
OR
If 7 sin2A + 3 cos2A = 4, then find tan A
22
23. If the perimeter of a protractor is 72 cm, calculate its area. (Use π = )
7
OR
Two circular pieces of equal radii and maximum area, touching each other are cut out from a
Rectangular card board of dimensions 14 cm × 7 cm. Find the area of the remaining card board.
[Use π = 22/7 ]

24. In the given figure below, AD/AE=AC/BD and ∠1=∠2. Show that Δ BAE~ ΔCAD .

25. Find the length of the tangent from an external point P at a distance of 20 cm from the centre of a
circle of radius 12 cm.

SECTION-C
Questions 26 to 31 carry 3 marks each

26. Two numbers are in the ratio of 1 : 3. If 5 is added to both the numbers, the ratio becomes 1 : 2.
Find the numbers.

OR
A train covered a certain distance at a uniform speed. If the train would have been 6 km/h faster, it
would have taken 4 hours less than the scheduled time. And, if the train were slower by 6 km/hr; it
would have taken 6 hours more than the scheduled time. Find the length of the journey.

27. Given that √3 is irrational, prove that 2 + 5√3 is irrational.

28. Two dice are thrown at the same time. What is the probability that the sum of the two numbers
appearing on the top of the dice is (i) 7? (ii) 14? (iii) equal to 12?

a a sin   b cos  a 2  b 2
29. If tan   , prove that 
b a sin   b cos  a 2  b 2

30. If the zeroes of the polynomial x2 + px + q are double in value to the zeroes of 2x2 – 5x – 3, then
find the values of p and q

Prepared by: M. S. KumarSwamy, TGT(Maths) Page - 3-


31. In the given figure, AB is a chord of length 8 cm of a circle of radius 5 cm. The tangents to the
circle at A and B intersect at P. Find the length of AP.

OR
Prove that opposite sides of a quadrilateral circumscribing a circle subtend supplementary angles at
the centre of the circle.

SECTION-D
Questions 32 to 35 carry 5M each

32. A motorboat whose speed in still water is 9 km/h, goes 15km downstream and comes back to the
same spot, in a total time of 3 hours 45 minutes. Find the speed of the stream.
OR
A takes 6 days less than the time taken by B to finish a piece of work. If both A and B together can
finish it in 4 days, find the time taken by B to finish the work.
33. Prove that if a line is drawn parallel to one side of a triangle intersecting the other two sides in
distinct points, then the other two sides are divided in the same ratio.
Using the above theorem prove that a line through the point of intersection of the diagonals and
parallel to the base of the trapezium divides the non parallel sides in the same ratio.

34. Ramesh made a bird-bath for his garden in the shape of a cylinder with a
hemispherical depression at one end. The height of the cylinder is 1.45 m
and its radius is 30 cm. Find the total surface area of the bird-bath.

OR

A tent is in shape of a cylinder surmounted by a conical top. If the height


and diameter of the cylindrical part are 2.1m and 4m respectively and the
slant height of the top is 2.8m. Find the area of canvas used for making
the tent. Also find the cost of canvas of the tent at the rate of 500 per m2.

35. A life insurance agent found the following data for distribution of ages of 100 policy holders.
Calculate the median age, if policies are given only to persons having age 18 years onwards but
less than 60 years.

Age (in years) Number of policy holders


Below 20 2
20 – 25 4
25 – 30 18
30 – 35 21
35 – 40 33
40 – 45 11
45 – 50 3
50 – 55 6
55 – 60 2

Prepared by: M. S. KumarSwamy, TGT(Maths) Page - 4-


SECTION-E (Case Study Based Questions)
Questions 36 to 38 carry 4M each

36. Aditya is a fitness freak and great athlete. He always wants to make his nation proud by winning
medals and prizes in the athletic activities.

An upcoming activity for athletes was going to be organised by Railways. Aditya wants to
participate in 200 m race. He can currently run that distance in 51 seconds. But he wants to increase
his speed, so to do it in 31 seconds. With each day of practice, it takes him 2 seconds less.
(i) He wants to makes his best time as 31 sec. In how many days will be able to achieve his target?
(ii) What will be the difference between the time taken on 5th day and 7th day.
OR
(ii) Which term of the arithmetic progression 3, 15, 27, 39 .... will be 120 more than its 21st term?

37. A 1.2 m tall girl spots a balloon moving with the wind in a horizontal line at a height of 88.2 m
from the ground. The angle of elevation of the balloon from the eyes of the girl at any instant is
60°. After 30 seconds, the angle of elevation reduces to 30° (see the below figure).

Based on the above information, answer the following questions. (Take √3 =1.732)
(i) Find the distance travelled by the balloon during the interval. (2)
(ii) Find the speed of the balloon. (2)
OR
(ii) If the elevation of the sun at a given time is 30°, then find the length of the shadow cast by a
tower of 150 feet height at that time. (2)
38. Resident Welfare Association (RWA) of a Gulmohar Society in Delhi have installed three electric
poles A, B and C in a society’s common park. Despite these three poles, some parts of the park are

Prepared by: M. S. KumarSwamy, TGT(Maths) Page - 5-


still in dark. So, RWA decides to have one more electric pole D in the park. The park can be
modelled as a coordinate systems given below.

On the basis of the above information, answer any four of the following questions:
(i) What is the position of the pole C? (1)
(ii) What is the distance of the pole B from the corner O of the park? (1)
(iii) Find the position of the fourth pole D so that four points A, B, C and D form a parallelogram.
(2)
OR
(iii) What is the distance between poles A and C? (2)

Prepared by: M. S. KumarSwamy, TGT(Maths) Page - 6-


PM SHRI KENDRIYA VIDYALAYA GACHIBOWLI, GPRA CAMPUS, HYD-32
SAMPLE PAPER TEST 04 FOR BOARD EXAM 2024

SUBJECT: MATHEMATICS MAX. MARKS : 80


CLASS : X DURATION : 3 HRS
General Instruction:
1. This Question Paper has 5 Sections A-E.
2. Section A has 20 MCQs carrying 1 mark each.
3. Section B has 5 questions carrying 02 marks each.
4. Section C has 6 questions carrying 03 marks each.
5. Section D has 4 questions carrying 05 marks each.
6. Section E has 3 case based integrated units of assessment (04 marks each) with sub-parts of the
values of 1, 1 and 2 marks each respectively.
7. All Questions are compulsory. However, an internal choice in 2 Qs of 5 marks, 2 Qs of 3 marks and
2 Questions of 2 marks has been provided. An internal choice has been provided in the 2marks
questions of Section E
8. Draw neat figures wherever required. Take π =22/7 wherever required if not stated.
SECTION – A
Questions 1 to 20 carry 1 mark each.

1. The point on the x-axis which is equidistant from (– 4, 0) and (10, 0) is:
(a) (7, 0) (b) (5, 0) (c) (0, 0) (d) (3, 0)

2. If a cylinder is covered by two hemispheres shaped lid of equal shape, then the total curved surface
area of the new object will be
(a) 4πrh + 2πr2 (b) 4πrh – 2πr2 (c) 2πrh + 4πr2 (d) 2πrh + 4πr

3. If the LCM of a and 18 is 36 and the HCF of a and 18 is 2, then a =


(a) 1 (b) 2 (c) 3 (d) 4

4. The sum of exponents of prime factors in the prime-factorisation of 196 is:


(a) 3 (b) 4 (c) 5 (d) 6

5. The values of k for which the quadratic equation 2x2 – kx + k = 0 has equal roots is
(a) 0 only (b) 8 only (c) 0,8 (d) 4

6. A number x is chosen at random from the numbers -3, -2, -1, 0, 1, 2, 3 the probability that |x| < 2 is
(a) 1/7 (b) 2/7 (c) 3/7 (d) 5/7
7. If x = 2sin2θ and y = 2cos2θ + 1 then x + y is:
(a) 3 (b) 2 (c) 1 (d) 1/2
1
8. If is a root of the equation x2 + kx – 5/4 = 0, then the value of k is
2
(a) 2 (b) – 2 (c) ¼ (d) ½

9. The pair of equations x + 2y + 5 = 0 and –3x – 6y + 1 = 0 have


(a) a unique solution (b) exactly two solutions
(c) infinitely many solutions (d) no solution

10. The point which lies on the perpendicular bisector of the line segment joining point A (–2, –5) and
B (2, 5) is:
(a) (0, 0) (b) (0, –1) (c) (–1, 0) (d) (1, 0)

Prepared by: M. S. KumarSwamy, TGT(Maths) Page - 1-


11. A card is selected at random from a well shuffled deck of 52 playing cards. The probability of its
being a face card is
(a) 3/13 (b) 4/13 (c) 6/13 (d) 9/13

12. The ratio in which the line segment joining the points P(-3, 10) and Q(6, –8) is divided by O(-1, 6)
is:
(a) 1:3 (b) 3:4 (c) 2:7 (d) 2:5

13. A box contains cards numbered 6 to 50. A card is drawn at random from the box. The probability
that the drawn card has a number which is a perfect square is :
(a) 1/45 (b) 2/15 (c) 4/45 (d) 1/9

14. In a circle of diameter 42cm, if an arc subtends an angle of 60º at the centre, then the length of the
arc is:
(a) 22/7 cm (b) 11cm (c) 22 cm (d) 44 cm

15. If the lines 3x + 2ky – 2 = 0 and 2x + 5y + 1 = 0 are parallel, then what is the value of k?
(a) 4/15 (b) 15/4 (c) ⅘ (d) 5/4

16. For the following distribution:


Marks Below 10 Below 20 Below 30 Below 40 Below 50 Below 60
No. of students 3 12 27 57 75 80
the modal class is
(a) 10-20 (b) 20-30 (c) 30-40 (d) 50-60

17. The distance of the point P (2, 3) from the x-axis is


(a) 2 (b) 3 (c) 1 (d) 5

18. A circus artist is climbing a 30 m long rope, which is tightly stretched and tied from the top of a
vertical pole to the ground. Find the distance of the pole to the peg in the ground, if the angle made
by the rope with the ground level is 30⁰.
(a) 20√3 m (b)15√3 m (c)10√3 m (d) 20 m

Direction : In the question number 19 & 20 , A statement of Assertion (A) is followed by a


statement of Reason(R) . Choose the correct option

19. Assertion (A): The largest number that divide 70 and125 which leaves remainder 5 and 8 is 13
Reason (R): HCF (65,117) =13
(a) Both A and R are true and R is the correct explanation of A
(b) Both A and R are true but R is not the correct explanation of A
(c) A is true and R is false
(d) A is false and R is true

20. Assertion (A): In ∆ABC, DE || BC such that AD = (7x - 4) cm, AE = (5x - 2) cm, DB = (3x + 4)
cm and EC = 3x cm than x equal to 5.
Reason (R): If a line is drawn parallel to one side of a triangle to intersect the other two sides in
distant point, than the other two sides are divided in the same ratio.
(a) Both Assertion (A) and Reason (R) are true and Reason (R) is the correct explanation of
Assertion (A).
(b) Both Assertion (A) and Reason (R) are true but Reason (R) is not the correct explanation of
Assertion (A).
(c) Assertion (A) is true but Reason (R) is false.
(d) Assertion (A) is false but Reason (R) is true.

Prepared by: M. S. KumarSwamy, TGT(Maths) Page - 2-


SECTION-B
Questions 21 to 25 carry 2M each

21. Find the value of m for which the pair of linear equations:
2x + 3y – 7 = 0 and (m – 1) x + (m + 1) y = (3m – 1) has infinitely many solutions

22. Find the zeroes of the quadratic polynomials p(t) = 5t2 + 12t + 7 and verify the relationship between
the zeroes and the coefficients.

23. Two dice are thrown at the same time. Find the probability of getting (i) same number on both dice
(ii) different numbers on both dice.
OR
Cards marked with number 3, 4, 5, …, 50 are placed in a box and mixed thoroughly. A card is
drawn at random from the box. Find the probability that the selected card bears (i) a perfect square
number (ii) a single digit number

24. A quadrilateral ABCD is drawn to circumscribe a circle. Prove that AB + CD = AD + BC.

25. Find the points on the x–axis which are at a distance of 2√5 from the point (7, –4). How many such
points are there?
OR
3
If A and B are (-2, -2) and (2, -4) respectively, find the coordinates of P such that AP = AB and P
7
lies on the line segment AB.

SECTION-C
Questions 26 to 31 carry 3 marks each

26. On a morning walk, three persons step off together and their steps measure 40 cm, 42 cm and 45
cm, respectively. Find the minimum distance each should walk so that each can cover the same
distance in complete steps.
OR
Show that 5 + 2√7 is an irrational number, where √7 is given to be an irrational number.

27. From a point on a ground, the angle of elevation of bottom and top of a transmission tower fixed on
the top of a 20 m high building are 45° and 60° respectively. Find the height of the tower.

28. In the below figure, if ∠1 =∠2 and ΔNSQ = ΔMTR, then prove that ΔPTS ~ ΔPRQ.

p2 1
29. If cosecθ + cotθ = p, then prove that cos  
p2 1

Prepared by: M. S. KumarSwamy, TGT(Maths) Page - 3-


30. If 2x + y = 23 and 4x – y = 19, find the values of 5y – 2x and y/x – 2.

31. In the given figure, OP is equal to diameter of the circle. Prove that ABP is an equilateral triangle.

OR
A circle is inscribed in a ΔABC having sides 8 cm, 10 cm and 12 cm as shown in the following
figure. Find AD, BE and CF.

SECTION-D
Questions 32 to 35 carry 5M each
1
32. Two pipes running together can fill a cistern in 3 hours. If one pipe takes 3 hours more than the
13
other to fill it, find the time in which each pipe would fill the cistern.
OR
If Zeba was younger by 5 years than what she really is, then the square of her age (in years) would
have been 11 more than five times her actual age. What is her age now? [NCERT Exemplar]
33. State and prove Basic Proportional Theorem.
34. A survey regarding the heights (in cm) of 50 girls of class Xth of a school was conducted and the
following data was obtained. Find the mean, median and mode of the given data.
Heights (in cm) 120 – 130 130 – 140 140 – 150 150 – 160 160 – 170
No. of Girls 2 8 12 20 8
35. A chord of a circle of radius 15 cm subtends an angle of 60° at the centre. Find the areas of the
corresponding minor and major segments of the circle. (Use π = 3.14 and √3 = 1.73)
OR
PQRS is a diameter of a circle of radius 6 cm. The lengths PQ, QR and RS are equal. Semi-circles
are drawn on PQ and QS as diameters as shown in below figure. Find the perimeter and area of the
shaded region

Prepared by: M. S. KumarSwamy, TGT(Maths) Page - 4-


SECTION-E (Case Study Based Questions)
Questions 36 to 38 carry 4M each
36. In a toys manufacturing company, wooden parts are assembled and painted to prepare a toy. One
specific toy is in the shape of a cone mounted on a cylinder. For the wood processing activity
center, the wood is taken out of storage to be sawed, after which it undergoes rough polishing, then
is cut, drilled and has holes punched in it. It is then fine polished using sandpaper. For the retail
packaging and delivery activity center, the polished wood sub-parts are assembled together, then
decorated using paint. The total height of the toy is 26 cm and the height of its conical part is 6 cm.
The diameters of the base of the conical part is 5 cm and that of the cylindrical part is 3 cm. On the
basis of the above information, answer the following questions:

(a) If its cylindrical part is to be painted yellow, find the surface area need to be painted. [1]
(b) If its conical part is to be painted green, find the surface area need to be painted. [2]
OR
(b) Find the volume of the wood used in making this toy. [2]
(c) If the cost of painting the toy is 3 paise per sq cm, then find the cost of painting the toy. (Use π =
3.14) [1]
37. Radio towers are used for transmitting a range of communication services including radio and
television. The tower will either act as an antenna itself or support one or more antennas on its
structure, including microwave dishes. They are among the tallest human-made structures. There
are 2 main types: guyed and self-supporting structures. On a similar concept, a radio station tower
was built in two sections A and B.
Tower is supported by wires from a point O. Distance between the base of the tower and point O is
36 m. From point O, the angle of elevation of the top of section B is 30° and the angle of elevation
of the top of section A is 45°.

Prepared by: M. S. KumarSwamy, TGT(Maths) Page - 5-


(i) What is the height of the section B? (1)
(ii) What is the height of the section A? (1)
(iii) What is the length of the wire structure from the point O to the top of section A? (2)
OR
(iii) What is the length of the wire structure from the point O to the top of section B? (2)

38. Mohan is an auto driver. His autorickshaw was too old and he had to spend a lot of money on repair
and maintenance every now and then. One day he got to know about the EV scheme of the
Government of India where he can not only get a good exchange bonus but also avail heavy
discounts on the purchase of an electric vehicle. So, he took a loan of 71,18,000 from a reputed
bank and purchased a new autorickshaw.
Mohan repays his total loan of 118000 rupees by paying every month starting with the first
instalment of 1000 rupees.

(i) If he increases the instalment by 100 rupees every month, then what amount will be paid by him
in the 30th instalment? [1]
(ii) If he increases the instalment by 100 rupees every month, then what amount of loan does he still
have to pay after 30th instalment? [2]
OR
(ii)If he increases the instalment by 200 rupees every month, then what amount would he pay in
40th instalment? [2]
(iii) If he increases the instalment by 100 rupees every month, then what amount will be paid by
him in the 100th instalment [1]

Prepared by: M. S. KumarSwamy, TGT(Maths) Page - 6-


PM SHRI KENDRIYA VIDYALAYA GACHIBOWLI, GPRA CAMPUS, HYD-32
SAMPLE PAPER TEST 05 FOR BOARD EXAM 2024
SUBJECT: MATHEMATICS MAX. MARKS : 80
CLASS : X DURATION : 3 HRS
General Instruction:
1. This Question Paper has 5 Sections A-E.
2. Section A has 20 MCQs carrying 1 mark each.
3. Section B has 5 questions carrying 02 marks each.
4. Section C has 6 questions carrying 03 marks each.
5. Section D has 4 questions carrying 05 marks each.
6. Section E has 3 case based integrated units of assessment (04 marks each) with sub-parts of the
values of 1, 1 and 2 marks each respectively.
7. All Questions are compulsory. However, an internal choice in 2 Qs of 5 marks, 2 Qs of 3 marks
and 2 Questions of 2 marks has been provided. An internal choice has been provided in the 2marks
questions of Section E
8. Draw neat figures wherever required. Take π =22/7 wherever required if not stated.
SECTION – A
Questions 1 to 20 carry 1 mark each.
1. Three cubes each of side 15 cm are joined end to end. The total surface area of the cuboid is:
(a) 3150 cm2 (b) 1575 cm2 (c) 1012.5 cm2 (d) 576.4 cm2
2. The midpoint of a line segment joining two points A(2, 4) and B(-2, -4) is
(a) (-2, 4) (b) (2, -4) (c) (0, 0) (d) (-2, -4)
3. If the distance between the points A(2, -2) and B(-1, x) is equal to 5, then the value of x is:
(a) 2 (b) -2 (c) 1 (d) -1
4. If cos A = 4/5, then the value of tan A is
(a) 3/5 (b) 3/4 (c) 4/3 (d) 5/3
5. If cos θ + cos2 θ = 1, the value of sin2 θ + sin4 θ is :
(a) –1 (b) 0 (c) 1 (d) 2
6. The HCF and the LCM of 12, 21, 15 respectively are
(a) 3, 140 (b) 12, 420 (c) 3, 420 (d) 420, 3
7. If the sum of LCM and HCF of two numbers is 1260 and their LCM is 900 more than their HCF,
then the product of two numbers is
(a) 205400 (b) 203400 (c) 194400 (d) 198400
8. If the zeroes of the quadratic polynomial x2 + (a + 1)x + b are 2 and -3, then
(a) a = -7, b = -1 (b) a = 5, b = -1 (c) a = 2, b = –6 (d) a = 0, b = –6
9. In the given figure, from an external point P, two tangents PQ and PR are drawn to a circle of
radius 4 cm with centre O. If ∠QPR = 90°, then length of PQ is

(a) 3 cm (b) 4 cm (c) 2 cm (d) 2.2 cm

Prepared by: M. S. KumarSwamy, TGT(Maths) Page - 1 -


10. In the given figure, quadrilateral ABCD is circumscribed, touching the circle at P, Q, R and S
such that ∠DAB = 90°, If CR = 23 cm and CB = 39 cm and the radius of the circle is 14 cm, then
the measure of AB is

(a) 37 cm (b) 16cm (c) 30 cm (d) 39 cm

11. If the circumference of a circle increases from 2π to 4π then its area ......the original area :
(a) Half (b) Double (c) Three times (d) Four times

12. In the figure given below, AD = 4 cm, BD = 3 cm and CB = 12 cm, then cot θ equals :

(a) 3/4 (b) 5/12 (c) 4/3 (d) 12/5

13. The perimeters of two similar triangles are 26 cm and 39 cm. The ratio of their areas will be :
(a) 2 : 3 (b) 6 : 9 (c) 4 : 6 (d) 4 : 9

14. If ∆ABC ~ ∆EDF and ∆ABC is not similar to ∆DEF, then which of the following is not true?
(a) BC.EF = AC.FD (b) AB.EF = AC.DE (c) BC.DE = AB.EF (d) BC.DE = AB.FD

15. The radii of 2 cylinders are in the ratio 2 : 3 and their heights are in the ratio 5 : 3. Then, the ratio
of their volumes is:
(a) 19 : 20 (b) 20 : 27 (c) 18:25 (d) 17:23

16. Consider the following frequency distribution


Class 0–5 6 – 11 12 – 17 18 – 23 24 – 29
Frequency 13 10 15 8 11
The upper limit of the median class is
(a) 7 (b) 17.5 (c) 18 (d) 18.5

17. Consider the following distribution:


Marks obtained Number of students
More than or equal to 0 63
More than or equal to 10 58
More than or equal to 20 55
More than or equal to 30 51
More than or equal to 40 48
More than or equal to 50 42
the frequency of the class 30-40 is
(a) 4 (b) 48 (c) 51 (d) 3

Prepared by: M. S. KumarSwamy, TGT(Maths) Page - 2 -


18. Two dice are thrown simultaneously. The probability that the product of the numbers appearing
on the dice is 7 is
(a) 7/36 (b) 2/36 (c) 0 (d) 1/36

Direction : In the question number 19 & 20 , A statement of Assertion (A) is followed by a


statement of Reason(R) . Choose the correct option
19. Assertion (A): The mid-point of the line segment joining the points A (3, 4) and B (k, 6) is P (x,
y) and x + y – 10 = 0, the value of k is 7
 x  x y  y2 
Reason (R): Midpoint of line segment is  1 2 , 1 
 2 2 
(a) Both A and R are true and R is the correct explanation of A
(b) Both A and R are true but R is not the correct explanation of A
(c) A is true and R is false
(d) A is false and R is true
20. Assertion (A): For any two positive integers a and b, HCF(a, b) x LCM(a, b) = a x b
Reason (R): The HCF of two numbers is 5 and their product is 150. Then their LCM is 40.
(a) Both Assertion (A) and Reason (R) are true and Reason (R) is the correct explanation of
Assertion (A)
(b) Both assertion (A) and reason (R) are true and reason (R) is not the correct explanation of
Assertion (A)
(c) Assertion (A) is true but reason(R) is false.
(d) Assertion (A) is false but reason(R) is true.

SECTION-B
Questions 21 to 25 carry 2M each

21. A quadrilateral ABCD is drawn to circumscribe a circle. Prove that AB + CD = AD + BC.

QR QT
22. In the figure,  and ∠1 = ∠2, Show that ∆PQS ∼ ∆TQR.
QS PR

OR
ABCD is a trapezium in which AB || CD and its diagonals intersect each other at the point O.
Using a similarity criterion of two triangles, show that =
1 1
23. If sin(A – B) = , cos(A + B) = , 00< A + B ≤900 , A > B. Find A and B.
2 2
24. Find the value of p if the pair of equations 2x + 3y – 5 = 0 and px – 6y – 8 = 0 has a unique
solution.

Prepared by: M. S. KumarSwamy, TGT(Maths) Page - 3 -


25. The short and long hands of a clock are 4 cm and 6 cm long respectively. Find the sum of
distances travelled by their tips in 2 days
OR
A car has two wipers which do not overlap. Each wiper has a blade of length 21 cm sweeping
through an angle of 120°. Find the total area cleaned at each sweep of the blades
SECTION-C
Questions 26 to 31 carry 3 marks each
26. 4 Bells toll together at 9.00 am. They toll after 7, 8, 11 and 12 seconds respectively. How many
times will they toll together again in the next 3 hours?
OR
Given that √3 is irrational, prove that (2 + 5√3) is an irrational number.
27. Find the ratio in which the line 2x + y – 4 = 0 divides the line segment joining the points A (2, –
2) and B (3, 7)
28. In the given figure, PA and PB are the tangent segments to a circle with centre O. Show that the
points A, O, B and P are concyclic.

OR
In the given figure, ABC is a triangle in which ∠B = 90°, BC = 48 cm and AB = 14 cm. A circle
is inscribed in the triangle, whose centre is O. Find radius r of in-circle.

29. From a pack of 52 playing cards, jacks, queens, kings and aces of red colour are removed. From
the remaining a card is drawn at random. Find the probability that the card drawn is (i) a black
queen (ii) a red card (iii) a face card.
30. If a, b are the zeroes of the polynomial 2x2 – 5x + 7, then find a polynomial whose zeroes are 2a
+ 3b, 3a + 2b

cos A 1  sin A
31. Prove that   2sec A
1  sin A cos A

SECTION-D
Questions 32 to 35 carry 5M each
32. The mean of the following frequency distribution is 62.8 and the sum of all the frequencies is 50.
Compute the missing frequencies f1 and f2.
Class 0-20 20-40 40-60 60-80 80-100 100-120
Frequency 5 f1 10 f2 7 8

Prepared by: M. S. KumarSwamy, TGT(Maths) Page - 4 -


33. A train, travelling at a uniform speed for 360 km, would have taken 48 minutes less to travel the
same distance if its speed were 5 km/h more. Find the original speed of the train.
OR
Two water taps together can fill a tank in 6 hours. The tap of larger diameter takes 9 hours less
than the smaller one to fill the tank separately. Find the time in which each tap can separately fill
the tank.
34. 200 logs are stacked in the following manner: 20 logs in the bottom row, 19 in the next row, 18 in
the row next to it and so on (see below figure). In how may rows are the 200 logs placed and how
many logs are in the top row?

OR
The sum of the third and the seventh terms of an AP is 6 and their product is 8. Find the sum of
first sixteen terms of the AP.
35. Prove that if a line is a drawn parallel to one side of a triangle intersecting the other two sides in
distinct points, then the other two sides are divided in the same ratio. Using the above theorem.
Prove that = if LM || CB and LN || CD as shown in the figure.

SECTION-E (Case Study Based Questions)


Questions 36 to 38 carry 4M each
36. Mayank a student of class 7th loves watching and playing with birds of different kinds. One day
he had an idea in his mind to make a bird-bath on his garden. His brother who is studying in class
10th helped him to choose the material and shape of the birdbath. They made it in the shape of a
cylinder with a hemispherical depression at one end as shown in the Figure below. They opted for
the height of the hollow cylinder as 1.45 m and its radius is 30 cm. The cost of material used for
making bird bath is Rs. 40 per square meter.

(i) Find the curved surface area of the hemisphere. (Take π = 3.14)
(ii) Find the total surface area of the bird-bath. (Take π = 22/7)
(iii) What is total cost for making the bird bath?
OR
(iii) Mayank and his brother thought of increasing the radius of hemisphere to 35 cm with same
material so that birds get more space, then what is the new height of cylinder?

Prepared by: M. S. KumarSwamy, TGT(Maths) Page - 5 -


37. Tower Bridge is a Grade I listed combined bascule and suspension bridge in London, built
between 1886 and 1894, designed by Horace Jones and engineered by John Wolfe Barry. The
bridge is 800 feet (240 m) in length and consists of two bridge towers connected at the upper
level by two horizontal walkways, and a central pair of bascules that can open to allow shipping.
In this bridge, two towers of equal heights are standing opposite each other on either side of the
road, which is 80 m wide. During summer holidays, Neeta visited the tower bridge. She stood at
some point on the road between these towers. From that point between the towers on the road, the
angles of elevation of the top of the towers was 60° and 30° respectively.

(i) Find the distances of the point from the base of the towers where Neeta was standing while
measuring the height. [2]
(ii) Neeta used some applications of trigonometry she learned in her class to find the height of the
towers without actually measuring them. What would be the height of the towers she would have
calculated? [2]
OR
(ii) Find the distance between Neeta and top of tower AB? Also, Find the distance between Neeta
and top tower CD? [2]

38. On the roadway, Points A and B, which stand in for Chandigarh and Kurukshetra, respectively,
are located nearly 90 kilometres apart. At the same time, a car departs from Kurukshetra and one
from Chandigarh. These cars will collide in 9 hours if they are travelling in the same direction,
and in 9/7 hours if they are travelling in the other direction. Let X and Y be two cars that are
travelling at x and y kilometres per hour from places A and B, respectively. On the basis of the
above information, answer the following questions:

(a) When both cars move in the same direction, then find the situation which can be represented
algebraically. [2]
OR
(a) When both cars move in the opposite direction, then find the situation which can be
represented algebraically. [2]
(b) Find the speed of car x. [1]
(c) Find the speed of car y. [1]

Prepared by: M. S. KumarSwamy, TGT(Maths) Page - 6 -


PM SHRI KENDRIYA VIDYALAYA GACHIBOWLI, GPRA CAMPUS, HYD-32
SAMPLE PAPER TEST 06 FOR BOARD EXAM 2024

SUBJECT: MATHEMATICS MAX. MARKS : 80


CLASS : X DURATION : 3 HRS
General Instruction:
1. This Question Paper has 5 Sections A-E.
2. Section A has 20 MCQs carrying 1 mark each.
3. Section B has 5 questions carrying 02 marks each.
4. Section C has 6 questions carrying 03 marks each.
5. Section D has 4 questions carrying 05 marks each.
6. Section E has 3 case based integrated units of assessment (04 marks each) with sub-parts of the
values of 1, 1 and 2 marks each respectively.
7. All Questions are compulsory. However, an internal choice in 2 Qs of 5 marks, 2 Qs of 3 marks
and 2 Questions of 2 marks has been provided. An internal choice has been provided in the
2marks questions of Section E
8. Draw neat figures wherever required. Take π =22/7 wherever required if not stated.
SECTION – A
Questions 1 to 20 carry 1 mark each.
1. The solution of the following pair of equation is:
x – 3y = 2, 3x – y = 14
(a) x = 5, y = 1 (b) x = 2, y = 3 (c) x = 1, y = 2 (d) x = 1, y = 4

2. What is the positive real root of 64x2 – 1 = 0?


(a) 1/8 (b) 1/4 (c) 1/2 (d) 1/6

3. In ∆ABC and ∆DEF, ∠B = ∠E, ∠F = ∠C and AB = 3DE. Then, the two triangles are
(a) congruent but not similar (b) similar but not congruent
(c) neither congruent nor similar (d) congruent as well as similar

4. The LCM of smallest two-digit composite number and smallest composite number is:
(a) 12 (b) 4 (c) 20 (d) 44

2sin A  3cos A
5. If cosec A = 13/12, then the value of
4sin A  9cos A
(a) 4 (b) 5 (c) 6 (d) 3

OA OC
6. In the figure, if  , then
OD OB

Which pair of angles are equal?


(a) ∠A = ∠C, ∠B = ∠D (b) ∠A = ∠B, ∠C = ∠D
(c) ∠C = ∠B, ∠A = ∠D (d) None of these

Prepared by: M. S. KumarSwamy, TGT(Maths) Page - 1 -


7. The value of ‘a’, if HCF (a, 18) = 2 and LCM (a, 18) = 36, is: (1)
(a) 2 (b) 5 (c) 7 (d) 4

8. If r = 3 is a root of quadratic equation kr2 – kr – 3 = 0, then the value of k is:


(a) 1/2 (b) 3 (c) 1/3 (d) 1/4

9. The ratio in which x-axis divides the join of (2, -3) and (5, 6) is:
(a) 1: 2 (b) 3 : 4 (c) 1: 3 (d) 1: 5

10. If tan θ = 1, then the value of sec θ + cosec θ is:


(a) 3√2 (b) 4√2 (c) 2√2 (d) √2

11. If the area of circle is numerically equal to twice its circumference, then the diameter of the
circle is
(a) 4 units (b) 6 units (c) 8 units (d) 12 units

12. In the given figure, if TP and TQ are tangents to a circle with centre O, so that ∠POQ = 110°,
then ∠PTQ is

(a) 110° (b) 90° (c) 80° (d) 70°

13. If the angle of elevation of the top of a tower from a point of observation at a distance of 100 m
from its base is 45°, then the height of the tower is:
(a) 160 m (b) 100 m (c) 200 m (d) 150 m

14. If the perimeter of a circle is equal to that of a square, then the ratio of the area of circle to the
area of the square is
(a) 14: 11 (b) 12: 13 (c) 11:14 (d) 13:12

15. For the following distribution:


Class 0-5 6-11 12-17 18-23 24-29
Frequency 13 10 15 8 11
the upper limit of the median class is
(a) 18.5 (b) 20.5 (c) 25.5 (d) 17.5

16. If the mean of the following distribution is 2.6, then the value of y is
Variable (x) 1 2 3 4 5
Frequency 4 5 y 1 2
(a) 3 (b) 8 (c) 13 (d) 24
17. Two different dice are thrown together. The probability of getting the sum of the two numbers
less than 7 is:
(a) 5/12 (b) 7/12 (c) 12/5 (d) 3/11

18. The radii of 2 cylinders are in the ratio 2 : 3 and their heights are in the ratio 5 : 3. Then, the
ratio of their volumes is:
(a) 19 : 20 (b) 20 : 27 (c) 18:25 (d) 17:23

Prepared by: M. S. KumarSwamy, TGT(Maths) Page - 2 -


DIRECTION: In the question number 19 and 20, a statement of Assertion (A) is followed by a
statement of Reason (R).
Choose the correct option:
(a) Both Assertion (A) and Reason (R) are true and Reason (R) is the correct explanation of
Assertion (A).
(b) Both Assertion (A) and Reason (R) are true but Reason (R) is not the correct explanation of
Assertion (A).
(c) Assertion (A) is true but Reason (R) is false.
(d) Assertion (A) is false but Reason (R) is true.
19. Assertion (A): The value of y is 3, if the distance between the points P(2, -3) and Q(10, y) is 10.
Reason (R): Distance between two points is given by ( x2  x1 ) 2  ( y2  y1 )2

20. Assertion: The HCF of two numbers is 9 and their LCM is 2016. If the one number is 54, then
the other number is 336.
Reason: Relation between numbers and their HCF and LCM is product of two numbers a, b =
HCF (a, b) × LCM (a, b).

SECTION – B
Questions 21 to 25 carry 2 marks each.

21. Two concentric circles are of radii 5 cm and 3 cm. Find the length of the chord of the larger
circle which touches the smaller circle.

22. Evaluate: 3 cos2 60° sec2 30° – 2 sin2 30° tan2 60°.

23. The length of the minute hand of a clock is 14 cm. Find the area swept by the minute hand in 5
minutes.
OR
A horse is tied to a peg at one corner of a square shaped grass field of side 15 m by means of a 5
m long rope. Find the area of that part of the field in which the horse can graze (use π = 3.14)

24. For what value of k for which the following pair of linear equations have infinitely many
solutions: 2x + 3y = 7, (k – 1)x + (k + 2)y = 3k is

25. In the below left figure, two chords AB and CD intersect each other at the point P. Prove that
(i) ΔAPC ~ ΔDPB (ii) AP. PB = CP. DP

OR
If in the given above right sided figure, AB || DE and BD || EF, then prove that DC2 = CF x AC

SECTION – C
Questions 26 to 31 carry 3 marks each.
26. A part of monthly hostel charges in a college is fixed and the remaining depends on the number
of days one has taken food in the mess. When a student ‘A’ takes food for 22 days, he has to

Prepared by: M. S. KumarSwamy, TGT(Maths) Page - 3 -


pay Rs. 1380 as hostel charges; whereas a student ‘B’, who takes food for 28 days, pays Rs.
1680 as hostel charges. Find the fixed charges and the cost of food per day.
OR
Meena went to a bank to withdraw Rs 2,000. She asked the cashier to give her Rs. 50 and Rs.
100 notes only. Meena got 25 notes in all. How many notes of Rs. 50 and Rs. 100 she received?

27. Prove that √5 is an irrational number.

28. What number should be added to the polynomial x2 – 5x + 4 so that 3 is the zero of the
polynomial?

cos2  sin 2 
29. Prove that:   1  sin  cos 
1  tan  1  cot 
OR
If cos θ + sin θ = √2 cos θ, show that cos θ – sin θ = √2 sin θ.

30. All the black face cards are removed from a pack of 52 playing cards. The reaming cards are
well shuffled and then a card is drawn at random. Find the probability of getting (i) face card (ii)
red card (iii) black card.

31. In the given figure, OP is equal to diameter of the circle. Prove that ABP is an equilateral
triangle.

SECTION – D
Questions 32 to 35 carry 5 marks each.
32. Some students planned a picnic. The total budget for food was Rs. 2,000. But 5 students failed
to attend the picnic and thus the cost of food for each member increased by Rs. 20. How many
students attended the picnic and how much did each student pay for the food?
OR
If Zeba was younger by 5 years than what she really is, then the square of her age (in years)
would have been 11 more than five times her actual age. What is her age now?

33. A tent is in the shape of a cylinder surmounted by a conical top. If the height and diameter of
the cylindrical part are 2.1 m and 4 m respectively, and the slant height of the top is 2.8 m, find
the area of the canvas used for making the tent. Also, find the cost of the canvas of the tent at
the rate of Rs. 500 per m2.
OR
A rectangular metal block has length 15 cm, breadth 10 cm and height 5 cm. From this block, a
circular hole of diameter 7 cm is drilled out. Find: (i) the volume of the remaining solid (ii) the
surface area of the remaining solid.
34. Prove that “If a line is drawn parallel to one side of a triangle to intersect the other two sides in
distinct points, the other two sides are divided in the same ratio.”
In the figure, find EC if AD/DB = AE/EC using the above theorem.

Prepared by: M. S. KumarSwamy, TGT(Maths) Page - 4 -


35. The distribution below gives the makes of 100 students of a class, if the median marks are 24,
find the frequencies f1 and f2
Marks 0-5 5-10 10-15 15-20 20-25 25-30 30-35 35-40
No. of students 4 6 10 f1 25 f2 18 5

SECTION – E(Case Study Based Questions)


Questions 36 to 38 carry 4 marks each.

36. Case Study – 1


In a GPS, The lines that run east-west are known as lines of latitude, and the lines running
north-south are known as lines of longitude. The latitude and the longitude of a place are its
coordinates and the distance formula is used to find the distance between two places. The
distance between two parallel lines is approximately 150 km. A family from Uttar Pradesh
planned a round trip from Lucknow (L) to Puri (P) via Bhuj (B) and Nashik (N) as shown in the
given figure below.

Based on the above information answer the following questions using the coordinate geometry.
(i) Find the distance between Lucknow (L) to Bhuj(B).
(ii) If Kota (K), internally divide the line segment joining Lucknow (L) to Bhuj (B) into 3 : 2
then find the coordinate of Kota (K).
(iii) Name the type of triangle formed by the places Lucknow (L), Nashik (N) and Puri (P)
OR

Prepared by: M. S. KumarSwamy, TGT(Maths) Page - 5 -


Find a place (point) on the longitude (y-axis) which is equidistant from the points Lucknow (L)
and Puri (P).

37. Case Study – 2


Saving money is a good habit and it should be inculcated in children from the beginning. A
father brought a piggy bank for his son Aditya. He puts one five-rupee coin of his savings in the
piggy bank on the first day. He increases his savings by one five-rupee coin daily.

(i) If the piggy bank can hold 190 coins of five rupees in all, find the number of days he can
contribute to put the five-rupee coins into it. (2)
(ii) Find the total money he saved. (2)
OR
If 6 times the 6th term of an A.P., is equal to 9 times the 9th term, find its 15th term. (2)

38. Case Study – 3


Ram is watching the top and bottom of a lighthouse from the top of the building. The angles of
elevation and depression of the top and bottom of a lighthouse from the top of a 60 m high
building are 30° and 60° respectively.

Find (i) the difference between the heights of the lighthouse and the building. (2)
(ii) the distance between the lighthouse and the building. (2)
OR
The ratio of the height of a light house and the length of its shadow on the ground is √3 : 1 What
is the angle of elevation? (2)

Prepared by: M. S. KumarSwamy, TGT(Maths) Page - 6 -


PM SHRI KENDRIYA VIDYALAYA GACHIBOWLI,GPRA CAMPUS, HYD-32
SAMPLE PAPER TEST 07 FOR BOARD EXAM 2024

SUBJECT: MATHEMATICS MAX. MARKS : 80


CLASS : X DURATION : 3 HRS
General Instruction:
1. This Question Paper has 5 Sections A-E.
2. Section A has 20 MCQs carrying 1 mark each.
3. Section B has 5 questions carrying 02 marks each.
4. Section C has 6 questions carrying 03 marks each.
5. Section D has 4 questions carrying 05 marks each.
6. Section E has 3 case based integrated units of assessment (04 marks each) with sub-parts of the
values of 1, 1 and 2 marks each respectively.
7. All Questions are compulsory. However, an internal choice in 2 Qs of 5 marks, 2 Qs of 3 marks
and 2 Questions of 2 marks has been provided. An internal choice has been provided in the 2marks
questions of Section E
8. Draw neat figures wherever required. Take π =22/7 wherever required if not stated.
SECTION – A
Questions 1 to 20 carry 1 mark each.
1. The pair of equations x + 2y + 5 = 0 and –3x – 6y + 1 = 0 have
(a) a unique solution (b) exactly two solutions
(c) Infinitely many solutions (d) no solution

2. If p and q are the zeroes of the quadratic polynomial f(x) = 2x2 – 7x + 3, find the value of p + q –
pq is
(a) 1 (b) 2 (c) 3 (d) None of these

1
3. If sin 2A = tan² 45° where A is an acute angle, then the value of A is
2
(a) 60° (b) 45° (c) 30° (d) 15°

4. A girl walks 200m towards East and then 150m towards North. The distance of the girl from the
starting point is
(a) 350m (b) 250m (c) 300 m (d) 325 m

5. In ΔABC right angled at B, if cotC = √3 , then then cosAsinC + sinAcosC =


(a) –1 (b) 0 (c) 1 (d) √3 / 2

6. If is an acute angle and tan + cot = 2, then the value of sin3 + cos3 is
1 2
(a) 1 (b) (c) (d) 2
2 2
7. If the distance between the points (4,p) and (1,0) is 5, then value of p is
(a) 4 only (b) ±4 (c) -4 only (d) 0
8. If p and q are positive integers such that p = a3b2 and q = a2b, where ‘a’ and ‘b’ are prime
numbers, then the LCM ( p, q) is …..
(a) ab (b) a2b2 (c) a3b2 (d) a3b3

9. 108 can be expressed as a product of its primes as ……………..


(a) 23 x 32 (b) 23 x 33 (c) 22 x 32 (d) 22 x 33
Prepared by: M. S. KumarSwamy, TGT(Maths) Page - 1 -
10. If angle between two radii of a circle is 130⁰, the angle between the tangents at the ends of the
radii is :
(a) 90⁰ (b) 50⁰ (c) 70⁰ (d) 40⁰

11. The relationship between mean, median and mode for a moderately skewed distribution is
(a) mode = median – 2 mean (b) mode = 3 median – 2 mean
(c) mode = 2 median – 3 mean (d) mode = median – mean

12. For the following distribution:


Marks Below Below Below Below Below Below
10 20 30 40 50 60
No. of Students 3 12 27 57 75 80
the modal class is
(a) 10 – 20 (b) 20 – 30 (c) 30 – 40 (d) 50 – 60

13. The area of a quadrant of a circle, whose circumference is 22 cm, is


11 2 77 77 77
(a) cm (b) cm2 (c) cm2 (d) cm2
8 8 2 4

14. If the quadratic equation x2 + 4x + k = 0 has real and equal roots, then
(a) k < 4 (b) k > 4 (c) k = 4 (d) k ≥ 4

15. Volumes of two spheres are in the ratio 64 : 27. The ratio of their surface areas is
(a) 3 : 4 (b) 4 : 3 (c) 9 : 16 (d) 16 : 9

16. The area of the square that can be inscribed in a circle of radius 8 cm is
(a) 256 cm² (b) 128 cm² (c) 64√2 cm² (d) 64 cm²

17. Two dice are thrown at the same time and the product of numbers appearing on them is noted.
The probability that the product is a prime number is
(a) 1/3 (b) 1/6 (c) 1/5 (d) 5/6

18. In ABC, DE || AB, If CD = 3 cm, EC = 4 cm, BE = 6 cm, then DA is equal to


(a) 7.5 cm (b) 3 cm (c) 4.5 cm (d) 6 cm

DIRECTION: In the question number 19 and 20, a statement of Assertion (A) is followed by a
statement of Reason (R).
Choose the correct option

19. Assertion (A): The point (0, 4) lies on y -axis.


Reason (R): The x co-ordinate on the point on y -axis is zero.
(a) Both Assertion (A) and Reason (R) are true and Reason (R) is the correct explanation of
Assertion (A)
(b) Both assertion (A) and reason (R) are true and reason (R) is not the correct explanation of
Assertion (A)
(c) Assertion (A) is true but reason(R) is false.
(d) Assertion (A) is false but reason(R) is true.

20. Assertion (A): If HCF ( 90, 144) = 18, then LCM (90, 144) = 720
Reason (R): HCF (a, b) x LCM (a, b) = a x b
(a) Both Assertion (A) and Reason (R) are true and Reason (R) is the correct explanation of
Assertion (A).

Prepared by: M. S. KumarSwamy, TGT(Maths) Page - 2 -


(b) Both Assertion (A) and Reason (R) are true but Reason (R) is not the correct explanation of
Assertion (A).
(c) Assertion (A) is true but Reason (R) is false.
(d) Assertion (A) is false but Reason (R) is true.

SECTION – B
Questions 21 to 25 carry 2 marks each.

21. The length of the minute hand of a clock is 14 cm. Find the area swept by the minute hand in 5
minutes.
OR
In a circle of radius 21 cm, an arc subtends an angle of 60° at the centre. Find (i) the length of the
arc (ii) area of the sector formed by the arc

22. For what value of k will the following system of linear equations have no solution?
3x + y = 1; (2k – 1) x + (k – 1) y = 2k + 1

23. From a point P, two tangents PA and PB are drawn to a circle C(0, r). If OP = 2r, then find ∠APB.
Prove that triangle APB is an equilateral triangle.

1
24. If tan (A + B) = 3 and tan (A – B) = ; 0° < A+B ≤ 90°; A > B, find A and B.
3
OR
If xsin θ + ycos θ = sinθ cosθ and xsinθ = ysinθ then find x2 + y2.
3 3

25. ABCD is a trapezium in which AB || CD and its diagonals intersect each other at the point O.
OA OB
Using a similarity criterion of two triangles, show that 
OC OD

SECTION – C
Questions 26 to 31 carry 3 marks each.
26. The sum of the digits of a two-digit number is 9. Also 9 times this number is twice the number
obtained by reversing the order of the digits. Find the number.
OR
Yash scored 40 marks in a test, getting 3 marks for each right answer and losing 1 mark for each
wrong answer. Had 4 marks been awarded for each correct answer and 2 marks been deducted for
each incorrect answer, then Yash would have scored 50 marks. How many questions were there in
the test?

27. Prove that (sinA + cosecA)2 + (cosA + secA)2 = 7 + tan2A + cot2A

28. Prove that √5 is an irrational number.

Prepared by: M. S. KumarSwamy, TGT(Maths) Page - 3 -


29. Find the zeroes of the quadratic polynomial x2 – 2x – 8 and verify the relationship between the
zeroes and the coefficients of the polynomial.

30. Prove that the lengths of the tangents drawn from an external point to a circle are equal.
OR
In the figure XY and X'Y' are two parallel tangents to a circle with centre O and another tangent
AB with point of contact C interesting XY at A and X'Y' at B, what is the measure of ∠AOB.

31. Two dice are thrown at the same time. What is the probability that the sum of the two numbers
appearing on the top of the dice is
(i) 8? (ii) 7? (iii) less than or equal to 12?

SECTION – D
Questions 32 to 35 carry 5 marks each.

32. State and Prove Basic Proportionality Theorem.

33. A person on tour has Rs.360 for his expenses. If he extends his tour for 4 days, he has to cut down
his daily expenses by Rs.3. Find the original duration of the tour.
OR
Rs.6500 were divided equally among a certain number of persons. Had there been 15 more
persons, each would have got Rs.30 less. Find the original number of persons.

34. Ramesh made a bird-bath for his garden in the shape of a cylinder with a hemispherical
depression at one end. The height of the cylinder is 1.45 m and its radius is 30 cm. Find the total
surface area of the bird-bath.

OR

A tent is in shape of a cylinder surmounted by a conical top. If the height and diameter of the
cylindrical part are 2.1m and 4m respectively and the slant height of the top is 2.8m. Find the area
of canvas used for making the tent. Also find the cost of canvas of the tent at the rate of 500 per
m2 .

Prepared by: M. S. KumarSwamy, TGT(Maths) Page - 4 -


35. The following frequency distribution gives the monthly consumption of 68 consumers of a
locality. Find median, mean and mode of the data and compare them.
Monthly consumption of Number of consumers
electricity (in units)
65-85 4
85-105 5
105-125 13
125-145 20
145-165 14
165-185 8
185-205 4

SECTION – E(Case Study Based Questions)


Questions 36 to 38 carry 4 marks each.

36. Case Study-2


In order to conduct sports day activities in your school, lines have been drawn with chalk powder
at a distance of 1 m each in a rectangular shaped ground ABCD. 100 flower pots have been
placed at the distance of 1 m from each other along AD, as shown in the following figure.
1 1
Niharika runs ( )th distance AD on the 2nd line and posts a green Flag. Preet runs ( ) th
4 5
distance AD on the eighth line and posts are red flags. Taking A as the origin AB along x-axis
and AD along y-axis, answer the following questions:
(i) Find the coordinates of the green flag. (1)
(ii) Find the distance between the two flags. (1)
(iii) If Rashmi has to post a blue flag exactly halfway between the line segment joining the two
flags, where should she post her flag? (2)
OR
(iii) If Joy has to post a flag at one fourth distance from the green flag, in the line segment joining
the green and red flags, then where should he post his flag? (2)

37. Case Study – 3


Lakshaman Jhula is located 5 kilometers north-east of the city of Rishikesh in the Indian state of
Uttarakhand. The bridge connects the villages of Tapovan to Jonk. Tapovan is in Tehri Garhwal
district, on the west bank of the river, while Jonk is in Pauri Garhwal district, on the east bank.
Lakshman Jhula is a pedestrian bridge also used by motorbikes. It is a landmark of Rishikesh. A
Prepared by: M. S. KumarSwamy, TGT(Maths) Page - 5 -
group of Class X students visited Rishikesh in Uttarakhand on a trip. They observed from a point
(P) on a river bridge that the angles of depression of opposite banks of the river are 60° and 30°
respectively. The height of the bridge is about 18 meters from the river.

Based on the above information answer the following questions.


(i) Find the distance PA. (1)
(ii) Find the distance PB (1)
(iii) Find the width AB of the river. (2)
OR
(iii) Find the height BQ if the angle of the elevation from P to Q be 30°. (2)

38. Case Study-1


Mohan takes a loan from a bank for his car. Mohan replays his total loan of Rs.118000 by paying
every month starting with the first instalment of Rs.1000. If he increases the instalment by Rs.100
every month.

(i) What is the first term and common difference of given question. (1)
(ii) The amount paid buy him in 30th instalment. (1)
(iii) The amount paid by him in the 30 instalments is (2)
(OR)
(iii) What amount does he still have to pay after 30th instalment? (2)

Prepared by: M. S. KumarSwamy, TGT(Maths) Page - 6 -


PM SHRI KENDRIYA VIDYALAYA GACHIBOWLI,GPRA CAMPUS,HYD-32
SAMPLE PAPER TEST 08 FOR BOARD EXAM 2024

SUBJECT: MATHEMATICS MAX. MARKS : 80


CLASS : X DURATION : 3 HRS
General Instruction:
1. This Question Paper has 5 Sections A-E.
2. Section A has 20 MCQs carrying 1 mark each.
3. Section B has 5 questions carrying 02 marks each.
4. Section C has 6 questions carrying 03 marks each.
5. Section D has 4 questions carrying 05 marks each.
6. Section E has 3 case based integrated units of assessment (04 marks each) with sub-parts of the
values of 1, 1 and 2 marks each respectively.
7. All Questions are compulsory. However, an internal choice in 2 Qs of 5 marks, 2 Qs of 3 marks
and 2 Questions of 2 marks has been provided. An internal choice has been provided in the
2marks questions of Section E
8. Draw neat figures wherever required. Take π =22/7 wherever required if not stated.
SECTION – A
Questions 1 to 20 carry 1 mark each.
1. The pair of linear equations 2x + 3y = 5 and 4x + 6y = 10 is
(a) inconsistent (b) consistent (c) dependent consistent (d) none of these

2. Points A(3, 1), B(5, 1), C(a, b) and D(4, 3) are vertices of a parallelogram ABCD. The values of
a and b are respectively
(a) a = 6, b = 3 (b) a = 2, b = 1 (c) a = 4, b = 2 (d) None of these

3. If ΔABC ~ ΔEDF and ΔABC is not similar to ΔDEF, then which of the following is not true?
(a) BC · EF = AC · FD (b) AB · EF = AC · DE
(c) BC · DE = AB · EF (d) BC · DE = AB · FD

4. If sec A = 15/7 and A + B = 90°, find the value of cosec B.


(a) 8/7 (b) 12/7 (c) 7/15 (d) 15/7

5. The LCM of two numbers is 14 times their HCF. The sum of LCM and HCF is 600. If one
number is 280, then the other number is
(a) 20 (b) 28 (c) 60 (d) 80

6. When 2120 is expressed as the product of its prime factors we get


(a) 2 × 5³ × 53 (b) 2³ × 5 × 53 (c) 5 × 7² × 31 (d) 5² × 7 × 33

7. If p and q are the zeroes of the quadratic polynomial f(x) = 2x2 – 7x + 3, find the value of p + q
– pq is
(a) 1 (b) 2 (c) 3 (d) None of these

8. ABCD is a trapezium with AD ∥ BC and AD = 4cm. If the diagonals AC and BD intersect each
other at O such that AO/OC = DO/OB =1/2, then BC =
(a) 6cm (b) 7cm (c) 8cm (d) 9cm

9. If the angle between two radii of a circle is 140°, then the angle between the tangents at the ends
of the radii is
(a) 90° (b) 50° (c) 70° (d) 40°

Prepared by: M. S. KumarSwamy, TGT(Maths) Page - 1 -


10. The number of revolutions made by a circular wheel of radius 0.7 m in rolling a distance of 176
m is
(a) 22 (b) 24 (c) 75 (d) 40

11. In ΔABC, right angled at B, AB = 5 cm and sin C = 1/2. Determine the length of side AC.
(a) 10 cm (b) 15 cm (c) 20 cm (d) none of these

12. In the ∆ABC, D and E are points on side AB and AC respectively such that DE || BC. If AE = 2
cm, AD = 3 cm and BD = 4.5 cm, then CE equals

(a) 1 cm (b) 2 cm (c) 3 cm (d) 4 cm

13. The median class of the following data is:


Marks 0 – 10 10 – 20 20 – 30 30 – 40 40 – 50 50 – 60
No. of students 8 10 12 22 30 18
(a) 20 – 30 (b) 30 – 40 (c) 40 – 50 (d) 50 – 60

14. Two dice are thrown simultaneously. What is the probability of getting doublet?
(a) 1/36 (b) 1/6 (c) 5/6 (d) 11/36

4sin   cos 
15. If 4 tan  = 3, then the value of is
4sin   cos 
(a) 1/2 (b) 1/3 (c) 1/4 (d) 1/5

16. The area of the square that can be inscribed in a circle of radius 8 cm is
(a) 256 cm² (b) 128 cm² (c) 64√2 cm² (d) 64 cm²

17. The ratio of the total surface area to the lateral surface area of a cylinder with base radius 80 cm
and height 20 cm is
(a) 1 : 2 (b) 2 : 1 (c) 3 : 1 (d) 5 : 1

18. The mean and mode of a frequency distribution are 28 and 16 respectively. The median is
(a) 22 (b) 23.5 (c) 24 (d) 24.5

DIRECTION: In the question number 19 and 20, a statement of Assertion (A) is followed by a
statement of Reason (R).
Choose the correct option
(a) Both assertion (A) and reason (R) are true and reason (R) is the correct explanation of
assertion (A)
(b) Both assertion (A) and reason (R) are true and reason (R) is not the correct explanation of
assertion (A)
(c) Assertion (A) is true but reason (R) is false.
(d) Assertion (A) is false but reason (R) is true.

19. Assertion (A): The value of y is –6, for which the distance between the points P(2, –3) and
Q(10, y) is 10.

Prepared by: M. S. KumarSwamy, TGT(Maths) Page - 2 -


Reason (R): Distance between two given points A (x1, y1) and B (x2, y2) is given by AB =
( x2  x1 ) 2  ( y2  y1 )2

20. Assertion (A): The number 6n never end with digit 0 for any natural number n..
Reason (R): The number 9n never end with digit 0 for any natural number n.

SECTION – B
Questions 21 to 25 carry 2 marks each.

21. In the given figure, find the value of x in terms of a, b and c.

22. XY and MN are the tangents drawn at the end points of the diameter DE of the circle with
centre O. Prove that XY || MN.

23. A rope by which a cow is tethered is increased from 16mto 23m. How much additional ground
does it have now to graze?
OR
In the below figure, OACB is a quadrant of a circle with centre O and radius 3.5 cm. If OD = 2
cm, find the area of the (i) quadrant OACB, (ii) shaded region.

24. In figure, ABCD is a rectangle. Find the values of x and y.

25. Find A and B, if sin (A + 2B) = √3/2 and cos (A + B) = 1/2.


OR
If (1 + cos A) (1 – cos A) = 3/4 , find the value of tan A.

SECTION – C
Questions 13 to 22 carry 3 marks each.
26. A part of monthly hostel charges in a college is fixed and the remaining depends on the number
of days one has taken food in the mess. When a student ‘A’ takes food for 22 days, he has to
pay Rs. 1380 as hostel charges; whereas a student ‘B’, who takes food for 28 days, pays Rs.
1680 as hostel charges. Find the fixed charges and the cost of food per day.
OR
Prepared by: M. S. KumarSwamy, TGT(Maths) Page - 3 -
The ratio of income of two persons is 9 : 7 and the ratio of their expenditure is 4 : 3, if each of
them manage to save Rs. 2000/month. Find their monthly incomes.

sin   cos   1
27. Prove that:  sec   tan 
sin   cos   1

28. Prove that √5 is and irrational number.

29. Find the zeroes of the quadratic polynomial 6x2 – 7x – 3 and verify the relationship between the
zeroes and the coefficients of the polynomial.
30. Prove that the intercept of a tangent between two parallel tangents to a circle subtends a right
angle at the center
OR

Prove that opposite sides of a quadrilateral circumscribing a circle subtend supplementary


angles at the centre of the circle.

31. One card is drawn at random from a well-shuffled deck of 52 playing cards. Find the probability
that the card drawn is (i) either a red card or a king, (ii) neither a red card nor a queen.

SECTION – D
Questions 32 to 35 carry 5 marks each.

32. A vessel is in the form of an inverted cone. Its height is 8 cm and the radius of its top which is
open, is 5 cm. It is filled with water upto the brim. When lead shots, each of which is a sphere of
radius 0.5 cm are dropped into the vessel, one-fourth of water flows out. Find the number of
lead shots dropped into the vessel.
OR

A copper wire of diameter 8 mm is evenly wrapped on a cylinder of length 24 cm and diameter


49 cm to cover the whole surface. Find (i) the length of the wire (ii) the volume of the wire.

33. Prove that if a line is drawn parallel to one side of a triangle intersecting the other two sides in
distinct points, then the other two sides are divided in the same ratio.
Using the above theorem prove that a line through the point of intersection of the diagonals and
parallel to the base of the trapezium divides the non parallel sides in the same ratio.

3
34. Two water taps together can fill a tank in 9 hours. The tap of larger diameter takes 10 hours
8
less than the smaller one to fill the tank separately. Find the time in which each tap can
separately fill the tank.
OR

A rectangular park is to be designed whose breadth is 3 m less than its length. Its area is to be 4
square metres more than the area of a park that has already been made in the shape of an
isosceles triangle with its base as the breadth of the rectangular park and of altitude 12 m. Find
its length and breadth.

35. If the median of the distribution given below is 28.5, find the values of x and y.
Class 0-10 10-20 20-30 30-40 40-50 50-60 Total
Frequency 5 x 20 15 y 5 60

Prepared by: M. S. KumarSwamy, TGT(Maths) Page - 4 -


SECTION – E(Case Study Based Questions)
Questions 35 to 37 carry 4 marks each.
36. Anita’s mother start a new shoe shop. To display the shoes, she put 3 pairs of shoes in 1st row, 5
pairs in 2nd row, 7 pairs in 3rd row and so on.

On the basis of above information, answer the following questions.


(i) If she puts a total of 120 pairs of shoes, then find the number of rows required. (2)
(ii) What is the difference of pairs of shoes in 17th row and 10th row. (2)

37. A 1.2 m tall girl spots a balloon moving with the wind in a horizontal line at a height of 88.2 m
from the ground. The angle of elevation of the balloon from the eyes of the girl at any instant is
60°. After 30 seconds, the angle of elevation reduces to 30° (see the below figure).

Based on the above information, answer the following questions. (Take √3 =1.732)
(i) Find the distance travelled by the balloon during the interval. (2)
(ii) Find the speed of the balloon. (2)

38. In the sport of cricket the Captain sets the field according to a plan. He instructs the players to
take a position at a particular place. There are two reasons to set a cricket field—to take wickets
and to stop runs being scored.

The following graph shows the position of players during a cricket match.

(i) Find the coordinate of the point on y-axis which are equidistant from the points representing
the players at Cover P(2, –5) and Mid-wicket Q(–2, 9) [2]
(ii) Find the ratio in which x-axis divides the line segment joining the points Extra Cover S(3, –
3) and Fine Leg (–2, 7). [2]

Prepared by: M. S. KumarSwamy, TGT(Maths) Page - 5 -


Prepared by: M. S. KumarSwamy, TGT(Maths) Page - 6 -
PM SHRI KENDRIYA VIDYALAYA GACHIBOWLI,GPRA CAMPUS,HYD-32
SAMPLE PAPER TEST 09 FOR BOARD EXAM 2024

SUBJECT: MATHEMATICS MAX. MARKS : 80


CLASS : X DURATION : 3 HRS
General Instruction:
1. This Question Paper has 5 Sections A-E.
2. Section A has 20 MCQs carrying 1 mark each.
3. Section B has 5 questions carrying 02 marks each.
4. Section C has 6 questions carrying 03 marks each.
5. Section D has 4 questions carrying 05 marks each.
6. Section E has 3 case based integrated units of assessment (04 marks each) with sub-parts of the
values of 1, 1 and 2 marks each respectively.
7. All Questions are compulsory. However, an internal choice in 2 Qs of 5 marks, 2 Qs of 3 marks
and 2 Questions of 2 marks has been provided. An internal choice has been provided in the
2marks questions of Section E
8. Draw neat figures wherever required. Take π =22/7 wherever required if not stated.
SECTION – A
Questions 1 to 20 carry 1 mark each.
1. If LCM(x, 18) = 36 and HCF(x, 18) = 2, then x is:
(a) 2 (b) 3 (c) 4 (d) 5 1

2. In ΔABC right angled at B, if tanA = √3 , then then cosA cosC – sinAsinC =


(a) –1 (b) 0 (c) 1 (d) √3 / 2

3. If 2sin2 β – cos2 β = 2, then β is:


(a) 0° (b) 90° (c) 45° (d) 30° 1

4. The ratio of LCM and HCF of the least composite and the least prime numbers is:
(a) 1: 2 (b) 2: 1 (c) 1: 1 (d) 1: 3 1

5. The value of k for which the lines 5x + 7y = 3 and 15x + 21y = k coincide is:
(a) 9 (b) 5 (c) 7 (d) 18
1 1
6. Write a quadratic polynomial whose sum of zeroes is and product of zeroes is .
4 4
(a) 4x2 + x + 1 (b) x2 + 4x – 1
(c) 2x2 + 3x – 1 (d) x2 – 2x + 1

7. The vertices of a parallelogram in order are A(1, 2), B(4, y), C(x, 6) and D(3, 5). Then (x, y) is:
(a) (6, 3) (b) (3, 6) (c) (5, 6) (d) (1, 4)

8. A horse is tied to a pole with 28 m long rope. The perimeter of the field where the horse can
graze is (Take π = 22/7)
(a) 60 cm (b) 85 cm (c) 124 cm (d) 176 cm

9. Two dice are thrown at the same time. The probability of getting not doublet is
(a) 1/3 (b) 1/6 (c) 1/5 (d) 5/6
10. ΔABC~ΔPQR. If AM and PN are altitudes of ΔABC and ΔPQR respectively and AB2: PQ2 = 4:
9, then AM: PN =
(a) 16: 81 (b) 4: 9 (c) 3: 2 (d) 2: 3
Prepared by: M. S. KumarSwamy, TGT(Maths) Page - 1 -
11. In the given figure, PA and PB are tangents to the circle with centre O. If ∠APB = 60°, then
∠OAB is

(a) 30° (b) 60° (c) 90° (d) 15°

12. If the difference of Mode and Median of a data is 24, then the difference of median and mean is
(a) 8 (b) 12 (c) 24 (d) 36

13. For the following distribution:


Class 0-5 5-10 10-15 15-20 20-25
Frequency 10 15 12 20 9
the sum of lower limits of the median class and modal class is
(a) 15 (b) 25 (c) 30 (d) 35

5sin   3cos 
14. If 5 tan θ = 4, then the value of is
5sin   2cos 
(a) 1/6 (b) 1/7 (c) 1/4 (d) 1/5

15. The ratio of the volumes of two spheres is 8 : 27. The ratio between their surface areas is
(a) 2 : 3 (b) 4 : 27 (c) 8 : 9 (d) 4 : 9
16. The area of the circle that can be inscribed in a square of 6cm is
(a) 36π cm2 (b) 18π cm2 (c) 12 π cm2 (d) 9π cm2

17. In the figure, if DE || BC, AD = 3 cm, BD = 4 cm and BC = 14 cm, then DE equals :


(a) 7 cm (b) 6 cm (c) 4 cm (d) 3 cm

18. ABCD is a trapezium with AD ∥ BC and AD = 4cm. If the diagonals AC and BD intersect each
other at O such that AO/OC = DO/OB =1/2, then BC =
(a) 6cm (b) 7cm (c) 8cm (d) 9cm

DIRECTION: In the question number 19 and 20, a statement of Assertion (A) is followed by a
statement of Reason (R).
Choose the correct option

19. Assertion (A): The number 6n, n being a natural number, ends with the digit 5.
Reason (R): The number 9n cannot end with digit 0 for any natural number n.
(a) Both assertion (A) and reason (R) are true and reason (R) is the correct explanation of
assertion (A)

Prepared by: M. S. KumarSwamy, TGT(Maths) Page - 2 -


(b) Both assertion (A) and reason (R) are true and reason (R) is not the correct explanation of
assertion (A)
(c) Assertion (A) is true but reason (R) is false.
(d) Assertion (A) is false but reason (R) is true.

20. Assertion (A): The point (–1, 6) divides the line segment joining the points (–3, 10) and (6, –8)
in the ratio 2 : 7 internally.
Reason (R): Given three points, i.e. A, B, C form an equilateral triangle, then AB = BC = AC.
(a) Both assertion (A) and reason (R) are true and reason (R) is the correct explanation of
assertion (A)
(b) Both assertion (A) and reason (R) are true and reason (R) is not the correct explanation of
assertion (A)
(c) Assertion (A) is true but reason (R) is false.
(d) Assertion (A) is false but reason (R) is true.

SECTION – B
Questions 21 to 25 carry 2 marks each.

21. If sin(A + B) = 1 and cos(A – B) = √3/2, 0°< A + B ≤ 90° and A > B, then find the measures of
angles A and B.
OR
cos   sin  1  3
Find an acute angle θ when 
cos   sin  1  3

22. In the given figure below, AD/AE=AC/BD and ∠1=∠2. Show that Δ BAE~ ΔCAD .

23. If 217x + 131y = 913, 131x + 217y = 827, then find the value of x and y

24. A circle is inscribed in a ΔABC having AB= 10cm, BC = 12cm and CA = 8cm and touching
these sides at D, E, F respectively. Find the lengths of AD, BE and CF

25. The length of the minute hand of a clock is 6cm. Find the area swept by it when it moves from
5:25 pm to 6:00 pm.
OR
In the given figure, the shape of the top of a table is that of a sector of a circle with centre O and
∠AOB = 90°. If AO = OB = 42 cm, then find the perimeter of the top of the table is [Take π =
22/7]

Prepared by: M. S. KumarSwamy, TGT(Maths) Page - 3 -


SECTION – C
Questions 26 to 31 carry 3 marks each.
26. A train covered a certain distance at a uniform speed. If the train would have been 6 km/h faster,
it would have taken 4 hours less than the scheduled time. And, if the train were slower by 6
km/hr; it would have taken 6 hours more than the scheduled time. Find the length of the
journey.
OR
Anuj had some chocolates, and he divided them into two lots A and B. He sold the first lot at the
rate of ₹2 for 3 chocolates and the second lot at the rate of ₹1 per chocolate, and got a total of
₹400. If he had sold the first lot at the rate of ₹1 per chocolate, and the second lot at the rate of
₹4 for 5 chocolates, his total collection would have been ₹460. Find the total number of
chocolates he had.

sin   cos   1
27. Prove that:  sec   tan 
sin   cos   1

28. Given that √5 is irrational, prove that 2 + 3√5 is irrational.

29. Find the zeroes of the polynomial x2 + x – 2, and verify the relation between the coefficients
and the zeroes of the polynomial.

30. Two coins are tossed simultaneously. What is the probability of getting
(i) At least one head? (ii) At most one tail? (iii) A head and a tail?

31. Prove that a parallelogram circumscribing a circle is a rhombus

OR

In the figure XY and X'Y' are two parallel tangents to a circle with centre O and another tangent
AB with point of contact C interesting XY at A and X'Y' at B, what is the measure of ∠AOB.

Prepared by: M. S. KumarSwamy, TGT(Maths) Page - 4 -


SECTION – D
Questions 32 to 35 carry 5 marks each.

32. Prove that if a line is drawn parallel to one side of a triangle intersecting the other two sides in
distinct points, then the other two sides are divided in the same ratio.
In the figure, find EC if AD/DB = AE/EC using the above theorem.

33. A cubical block of side 10 cm is surmounted by a hemisphere. What is the largest diameter that
the hemisphere can have? Find the cost of painting the total surface area of the solid so formed,
at the rate of Rs. 5 per 100 sq. cm. [Use π = 3.14]
OR
Due to heavy floods in a state, thousands were rendered homeless. 50 schools collectively
decided to provide place and the canvas for 1500 tents and share the whole expenditure equally.
The lower part of each tent is cylindrical with base radius 2.8 m and height 3.5 m and the upper
part is conical with the same base radius, but of height 2.1 m. If the canvas used to make the
tents costs ₹120 per m2, find the amount shared by each school to set up the tents.

34. The median of the following data is 868. Find the values of x and y, if the total frequency is 100
Class Frequency
800 – 820 7
820 – 840 14
840 – 860 x
860 – 880 25
880 – 900 y
900 – 920 10
920 – 940 5

1
35. Two pipes running together can fill a cistern in 3 hours. If one pipe takes 3 hours more than
13
the other to fill it, find the time in which each pipe would fill the cistern.
OR
In a flight of 600km, an aircraft was slowed down due to bad weather. Its average speed for the
trip was reduced by 200 km/hr from its usual speed and the time of the flight increased by 30
min. Find the scheduled duration of the flight.

SECTION – E(Case Study Based Questions)


Questions 36 to 38 carry 4 marks each.

36. Case Study – 1


Anita purchased a new building for her business. Being in the prime location, she decided to
make some more money by putting up an advertisement sign for a rental ad income on the roof
of the building.

Prepared by: M. S. KumarSwamy, TGT(Maths) Page - 5 -


From a point P on the ground level, the angle of elevation of the roof of the building is 30° and
the angle of elevation of the top of the sign board is 45°. The point P is at a distance of 24 m
from the base of the building.

On the basis of the above information, answer the following questions:


(i) Find the height of the building (without the sign board). (2)
OR
Find the height of the building (with the sign board) (2)
(ii) Find the height of the sign board. (1)
(iii) Find the distance of the point P from the top of the sign board. (1)

37. Case Study-2


The school auditorium was to be constructed to accommodate at least 1500 people. The chairs
are to be placed in concentric circular arrangement in such a way that each succeeding circular
row has 10 seats more than the previous one.

Prepared by: M. S. KumarSwamy, TGT(Maths) Page - 6 -


(i) If the first circular row has 30 seats, how many seats will be there in the 10th row? (1)
(ii) For 1500 seats in the auditorium, how many rows need to be there? (2)
OR
If 1500 seats are to be arranged in the auditorium, how many seats are still left to be put after
10th row? (2)
(iii) If there were 17 rows in the auditorium, how many seats will be there in the middle row?(1)

38. Case Study-3


The diagrams show the plans for a sun room. It will be built onto the wall of a house. The four
walls of the sunroom are square clear glass panels. The roof is made using
• Four clear glass panels, trapezium in shape, all the same size
• One tinted glass panel, half a regular octagon in shape

(i) Find the mid-point of the segment joining the points J (6, 17) and I (9, 16). (1)
(ii) Find the distance between the points A and S. (1)
(iii) Find the co-ordinates of the point which divides the line segment joining the points A and B
in the ratio 1:3 internally. (2)
OR
(iii) If a point (x,y) is equidistant from the Q(9,8) and S(17,8),then find the relation between x
and y. (2)

Prepared by: M. S. KumarSwamy, TGT(Maths) Page - 7 -


PM SHRI KENDRIYA VIDYALAYA GACHIBOWLI,GPRA CAMPUS,HYD-32
SAMPLE PAPER TEST 10 FOR BOARD EXAM 2024

SUBJECT: MATHEMATICS MAX. MARKS : 80


CLASS : X DURATION : 3 HRS
General Instruction:
1. This Question Paper has 5 Sections A-E.
2. Section A has 20 MCQs carrying 1 mark each.
3. Section B has 5 questions carrying 02 marks each.
4. Section C has 6 questions carrying 03 marks each.
5. Section D has 4 questions carrying 05 marks each.
6. Section E has 3 case based integrated units of assessment (04 marks each) with sub-parts of the
values of 1, 1 and 2 marks each respectively.
7. All Questions are compulsory. However, an internal choice in 2 Qs of 5 marks, 2 Qs of 3 marks and
2 Questions of 2 marks has been provided. An internal choice has been provided in the 2marks
questions of Section E
8. Draw neat figures wherever required. Take π =22/7 wherever required if not stated.
SECTION – A
Questions 1 to 20 carry 1 mark each.

1. The LCM of two numbers is 182 and their HCF is 13. If one of the numbers is 26, the other
number is
(a) 31 (b) 71 (c) 61 (d) 91

2. If p and q are positive integers such that p = a3b2 and q = a2b, where ‘a’ and ‘b’ are prime numbers,
then the HCF ( p, q) is …..
(a) a2b (b) a2b2 (c) a3b2 (d) a3b3

3. The quadratic equations x2 – 4x + k = 0 has distinct real roots if


(a) k = 4 (b) k > 4 (c) k = 16 (d) k < 4

4. The number of polynomials having zeroes as -2 and 5 is


(a) 1 (b) 2 (c) 3 (d) more than 3

5. The pair of equations y = 0 and y = -7 has


(a) one solution (b) two solutions (c) infinitely many solutions (d) no solution

6. The line segment joining the points A (5, 3) and B (-3, 11) is divided by the point C (3,5) in the
ratio
(a) 1:3 (b) 3:1 (c) 2:3 (d) 3:2

7. ∆ABC is such that AB = 3 cm, BC = 2 cm, CA = 2.5 cm. If ∆ABC~∆DEF and EF=4cm, then
perimeter of ∆DEF is
(a) 7.5 cm (b) 15cm (c) 22.5 cm (d) 30 cm

8. The value of sin 30° cos 60° + sin 60° cos 30° is:
(a) 0 (b) 1 (c) 2 (d) 4

9. If 2 sin 2θ = √3, then find the value of θ.


(a) 30° (b) 60° (c) 90° (d) 45°

Prepared by: M. S. KumarSwamy, TGT(Maths) Page - 1 -


10. A girl walks 200m towards East and then 150m towards North. The distance of the girl from the
starting point is
(a) 350m (b) 250m (c) 300 m (d) 325 m

11. Consider the data:


Class 65-85 85-105 105-125 125-145 145-165 165-185 185-205
Frequency 4 5 13 20 14 7 4
The difference of the upper limit of the median class and the lower limit of the modal class is
(a) 0 (b) 19 (c) 20 (d) 38

12. From an external point Q, the length of the tangents to a circle is 5 cm and the distance of Q from
the centre is 8 cm. The radius of the circle is
(a) 39 cm (b) 3 cm (c) √39 cm (d) 7 cm

13. If the sum of the areas of two circles with radii R1 and R2 is equal to the area of a circle of radius R,
then:
(a) R1 + R2 = R (b) R12 + R22 = R2 (c) R1 + R2 < R (d) R1 + R2 < R2

14. The base radii of a cone and a cylinder are equal. If their curved surface areas are also equal, then
the ratio of the slant height of the cone to the height of the cylinder is:
(a) 2 : 1 (b) 1 : 2 (c) 1 : 3 (d) 3 : 1

15. For the following distribution:


Marks Below Below Below Below Below Below
10 20 30 40 50 60
No. of Students 3 12 27 57 75 80
the modal class is
(a) 10 – 20 (b) 20 – 30 (c) 30 – 40 (d) 50 – 60

16. The area of a circle that can be inscribed in a square of side 6 cm is:
(a) 36 π cm2 (b) 18 π cm2 (c) 12 π cm2 (d) 9 π cm2

17. A girl calculates that the probability of her winning the first prize in a lottery is 0.08. If 6000
tickets are sold, how many tickets has she bought?
(a) 40 (b) 240 (c) 480 (d) 750

18. If sin A = 1/2, cos B = 1, 0 < A, B ≤ π/2, then the value of cot (A + B) is:
(a) √3 /2 (b) 1/2 (c) 0 (d) √3

Direction: In the question number 19 & 20, A statement of Assertion (A) is followed by a
statement of Reason(R). Choose the correct option
19. Assertion (A): The value of y is 3, if the distance between the points P(2, -3) and Q(10, y) is 10.
Reason (R): Distance between two points is given by ( x2  x1 )2  ( y2  y1 )2
(a) Both Assertion (A) and Reason (R) are true and Reason (R) is the correct explanation of
Assertion (A)
(b) Both assertion (A) and reason (R) are true and reason (R) is not the correct explanation of
Assertion (A)
(c) Assertion (A) is true but reason(R) is false.
(d) Assertion (A) is false but reason(R) is true.

20. Assertion (A): If HCF ( 90, 144) = 18, then LCM (90, 144) = 720
Reason (R): HCF (a, b) x LCM (a, b) = a x b

Prepared by: M. S. KumarSwamy, TGT(Maths) Page - 2 -


(a) Both Assertion (A) and Reason (R) are true and Reason (R) is the correct explanation of
Assertion (A).
(b) Both Assertion (A) and Reason (R) are true but Reason (R) is not the correct explanation of
Assertion (A).
(c) Assertion (A) is true but Reason (R) is false.
(d) Assertion (A) is false but Reason (R) is true.

SECTION-B
Questions 21 to 25 carry 2 marks each

21. For what value of p will the following pair of linear equations have infinitely many solutions?
(p – 3)x + 3y = p; px + py = 12

KP 4
22. In Figure, PQ is parallel to MN. If = and KN = 20.4 cm. Find KQ.
PM 13

OR
In the below figure, if ST || QR. Find PS.

1
23. If tan (A + B) = 3 and tan (A – B) = ; 0° < A+B ≤ 90°; A > B, find A and B.
3

24. XY and MN are the tangents drawn at the end points of the diameter DE of the circle with centre
O. Prove that XY || MN.

25. In the given figure, sectors of two concentric circles of radii 7 cm and 3.5 cm are given. Find the
area of the shaded region. (Use π = )

OR

Prepared by: M. S. KumarSwamy, TGT(Maths) Page - 3 -


A horse is placed for grazing inside a rectangular field 70 m by 52 m and is tethered to one corner
by a rope 21 m long. On how much area can it graze?

SECTION-C
Questions 26 to 31 carry 3 marks each

26. Given that √3 is irrational, prove that (2 + 5√3) is an irrational number.

27. Find the zeroes of the polynomial x2 + x – 2, and verify the relation between the coefficients and
the zeroes of the polynomial.

28. A number consists of two digits. Where the number is divided by the sum of its digits, the quotient
is 7. If 27 is subtracted from the number, the digits interchange their places, find the number.
OR
Students of a class are made to stand in rows. If 4 students are extra in a row, there would be two
rows less. If 4 students are less in a row, there would be four more rows. Find the number of
students in the class.

29. Prove that : sin θ (1 + tan θ) + cos θ (1 + cot θ) = sec θ + cosec θ.

30. A circle is inscribed in a ∆ABC having sides 16 cm, 20 cm and 24 cm as shown in figure. Find AD,
BE and CF.

OR
In the figure XY and X'Y' are two parallel tangents to a circle with centre O and another tangent AB
with point of contact C interesting XY at A and X'Y' at B, prove that ∠AOB is a right angle.

31. Two dice are thrown at the same time. What is the probability that the sum of the two numbers
appearing on the top of the dice is
(i) at least 9? (ii) 7? (iii) less than or equal to 6?

SECTION-D
Questions 32 to 35 carry 5 marks each

32. State and Prove Basic Proportionality Theorem.

Prepared by: M. S. KumarSwamy, TGT(Maths) Page - 4 -


33. The median of the following data is 137. Find the values of x and y, If the total frequency is 68.
Class 105 – 125 – 145 – 165 – 185 –
65 – 85 85 – 105
intervals 125 145 165 185 205
Frequency 4 x 13 20 14 y 4
34. A juice seller serves his customers using a glass as shown in figure. The inner diameter of the
cylindrical glass is 5 cm, but the bottom of the glass has a hemispherical portion raised which
reduces the capacity of the glass. If the height of the glass is 10 cm, find the apparent capacity of
the glass and its actual capacity. [ π = 3.14]

OR
A rectangular sheet of paper 30 cm × 18 cm can be transformed into the curved surface of a right
circular cylinder in two ways either by rolling the paper along its length or by rolling it along its
breadth. Find the ratio of the volumes of the two cylinders thus formed.
35. A person on tour has Rs.360 for his expenses. If he extends his tour for 4 days, he has to cut down
his daily expenses by Rs.3. Find the original duration of the tour.
OR
Rs.6500 were divided equally among a certain number of persons. Had there been 15 more persons,
each would have got Rs.30 less. Find the original number of persons.

SECTION-E (Case Study Based Questions)


Questions 36 to 38 carry 4 marks each

36. Case Study – 1:


In the month of April to June 2022, the exports of passenger cars from India increased by 26% in
the corresponding quarter of 2021–22, as per a report. A car manufacturing company planned to
produce 1800 cars in 4th year and 2600 cars in 8th year. Assuming that the production increases
uniformly by a fixed number every year.

Based on the above information answer the following questions.


(i) Find the production in the 1st year. (1)
(ii) Find the production in the 12th year. (1)
(iii) Find the total production in first 10 years. (2)
OR
(iii) In how many years will the total production reach 31200 cars? (2)

Prepared by: M. S. KumarSwamy, TGT(Maths) Page - 5 -


37. Case Study – 2:
In order to conduct sports day activities in your school, lines have been drawn with chalk powder at
a distance of 1 m each in a rectangular shaped ground ABCD. 100 flower pots have been placed at
the distance of 1 m from each other along AD, as shown in the following figure. Niharika runs
1 1
( )th distance AD on the 2nd line and posts a green Flag. Preet runs ( ) th distance AD on the
4 5
eighth line and posts are red flags. Taking A as the origin AB along x-axis and AD along y-axis,
answer the following questions:
(i) Find the coordinates of the green flag. (1)
(ii) Find the distance between the two flags. (1)
(iii) If Rashmi has to post a blue flag exactly halfway between the line segment joining the two
flags, where should she post her flag? (2)
OR
(iii) If Joy has to post a flag at one fourth distance from the green flag, in the line segment joining
the green and red flags, then where should he post his flag? (2)

38. Case Study – 3:


A lighthouse is a tall tower with light near the top. These are often built on islands, coasts or on cliffs.
Lighthouses on water surface act as a navigational aid to the mariners and send warning to boats and ships
for dangers. Initially wood, coal would be used as illuminators. Gradually it was replaced by candles,
lanterns, electric lights. Nowadays they are run by machines and remote monitoring. Prongs Reef lighthouse
of Mumbai was constructed in 1874-75. It is approximately 40 meters high and its beam can be seen at a
distance of 30 kilometres. A ship and a boat are coming towards the lighthouse from opposite directions.
Angles of depression of flash light from the lighthouse to the boat and the ship are 300 and 600 respectively.

(i) Which of the two, boat or the ship is nearer to the light house. Find its distance from the lighthouse? (2)
(ii) Find the time taken by the boat to reach the light house if it is moving at the rate of 2 km per hour. (2)
OR
(ii) The ratio of the height of a light house and the length of its shadow on the ground is √3 : 1 . What is the
angle of elevation of the sun?

Prepared by: M. S. KumarSwamy, TGT(Maths) Page - 6 -


PM SHRI KENDRIYA VIDYALAYA GACHIBOWLI,GPRA CAMPUS,HYD-32
SAMPLE PAPER TEST 11 FOR BOARD EXAM 2024

SUBJECT: MATHEMATICS MAX. MARKS : 80


CLASS : X DURATION : 3 HRS
General Instruction:
1. This Question Paper has 5 Sections A-E.
2. Section A has 20 MCQs carrying 1 mark each.
3. Section B has 5 questions carrying 02 marks each.
4. Section C has 6 questions carrying 03 marks each.
5. Section D has 4 questions carrying 05 marks each.
6. Section E has 3 case based integrated units of assessment (04 marks each) with sub-parts of the
values of 1, 1 and 2 marks each respectively.
7. All Questions are compulsory. However, an internal choice in 2 Qs of 5 marks, 2 Qs of 3 marks and
2 Questions of 2 marks has been provided. An internal choice has been provided in the 2marks
questions of Section E
8. Draw neat figures wherever required. Take π =22/7 wherever required if not stated.
SECTION – A
Questions 1 to 20 carry 1 mark each.

1. A ticket is drawn at random from a bag containing tickets numbered from 1 to 40. The probability
that the selected ticket has a number which is a multiple of 5 is
(a) 1/5 (b) 3/5 (c) 4/5 (d) 1

2. If two positive integers p and q can be expressed as p = ab3 and q = a3 b; a, b being prime numbers,
then HCF (p, q) is
(a) ab (b) a2 b2 (c) a3 b2 (d) a3 b3

3. If triangles ABC and DEF are similar and AB=4 cm, DE=6 cm, EF=9 cm and FD=12 cm, the perimeter of
triangle ABC is:
(a) 22 cm (b) 20 cm (c) 21 cm (d) 18 cm

4. If r = 3 is a root of quadratic equation kr2 – kr – 3 = 0, then the value of k is:


(a) 3/2 (b) 1/2 (c) 2 (d) 5/2

5. In the below figure, AD = 3 cm, AE = 5 cm, BD = 4 cm, CE = 4 cm, CF = 2 cm, BF = 2.5 cm, then
(a) DE || BC (b) DF || AC (c) EF || AB (d) none of these

1
6. If for some angle θ, cot 2θ = , then the value of cos3θ, where 3θ ≤ 90⁰, is
3
1 3
(a) (b) 1 (c) 0 (d)
2 2

Prepared by: M. S. KumarSwamy, TGT(Maths) Page - 1-


7. In ∆ABC, right-angled at C, if tan A=1, then the value of 2sin A cos A is
1 3
(a) 1 (b) (c) 2 (d)
2 2

8. Volumes of two spheres are in the ratio 64:27. The ratio of their surface areas is
(a) 3:4 (b) 4:3 (c) 9:16 (d) 16:9

9. The LCM of smallest two digit composite number and smallest composite number is:
(a) 12 (b) 4 (c) 20 (d) 44

10. Find the value of k so that the following system of equations has no solution:
3x – y – 5 = 0, 6x – 2y + k = 0
(a) k ≠ 10 (b) k ≠ -10 (c) k ≠ 12 (d) k ≠ -12

11. The mean and median of a distribution are 14 and 15, respectively. The value of the mode is:
(a) 16 (b) 17 (c) 18 (d) 13

12. If the sum of the circumferences of two circles with radii R1 and R2 is equal to the circumference of
a circle of radius R, then:
(a) R1 + R2 = R
(b) R1 + R2 > R
(c) R1 + R2 < R
(d) Nothing definite can be said about the relation among R1, R2 and R.

13. In figure AT is a tangent to the circle with centre O such that OT = 4 cm and OTA = 30°. Then AT
is equal to

(a) 4 cm (b) 2 cm (c) 2√3 cm (d) 4√3 cm

14. Mode and mean of a data are 12k and 15k. Median of the data is
(a) 12k (b) 14k (c) 15k (d) 16k

15. 4 tan2 A – 4 sec2 A is equal to:


(a) 2 (b) 3 (c) 4 (d) –4
16. Which of the following equations has 2 as a root?
(a) x2 – 4x + 5 = 0 (b) x2 + 3x – 12 = 0
(c) 2x2 – 7x + 6 = 0 (d) 3x2 – 6x – 2 = 0

17. The radii of two concentric circles are 4 cm and 5 cm. The difference in the areas of these two
circles is:
(a) π (b) 7π (c) 9π (d) 13π

18. If the distance between the points (x, –1) and (3, 2) is 5, then the value of x is
(a) –7 or –1 (b) –7 or 1 (c) 7 or 1 (d) 7 or –1

Prepared by: M. S. KumarSwamy, TGT(Maths) Page - 2-


Direction : In the question number 19 & 20 , A statement of Assertion (A) is followed by a
statement of Reason(R) . Choose the correct option
(a) Both Assertion (A) and Reason (R) are true and Reason (R) is the correct explanation of Assertion
(A).
(b) Both Assertion (A) and Reason (R) are true but Reason (R) is not the correct explanation of
Assertion (A).
(c) Assertion (A) is true but Reason (R) is false.
(d) Assertion (A) is false but Reason (R) is true.
19. Assertion (A): The number 6n, n being a natural number, ends with the digit 5.
Reason (R): The number 9n cannot end with digit 0 for any natural number n.

20. Assertion (A): The point (3, 0) lies on x -axis.


Reason (R): The x co-ordinate on the point on y -axis is zero.

SECTION-B
Questions 21 to 25 carry 2M each

21. If sin (A + B) = 1 and sin (A – B) = , 0 ≤ A + B ≤ 90° and A > B, then find A and B.
OR
1  tan 2 A
Prove that: 2
 tan 2 A
1  cot A

22. The perimeter of a sector of a circle of radius 5.2 cm is 16.4 cm. Find the area of the sector.
OR
If the perimeter of a semi-circular protractor is 108 cm, find the diameter of the protractor. (Take
  22 / 7) )

23. In the below figure, ΔABC is circumscribing a circle. Find the length of BC.

24. Determine the values of a and b for which the following system of linear equations has infinite
solutions: 2x – (a – 4) y = 2b + 1; 4x – (a – 1) y = 5b – 1

25. In ABC, DE || AB. If AD = 2x, DC = x + 3 , BE = 2x − 1 and CE = x, then find the value of ‘x’

SECTION-C
Questions 26 to 31 carry 3 marks each

26. A man wished to give Rs. 12 to each person and found that he fell short of Rs. 6 when he wanted to
give to all the persons present. He, therefore, distributed Rs. 9 to each person and found that Rs. 9
were left over. How much money did he have and how many persons were there?
OR
A father’s age is three times the sum of the ages of his children. After 5 years, his age will be two
times the sum of their ages. Find the present age of the father.

Prepared by: M. S. KumarSwamy, TGT(Maths) Page - 3-


sin   cos   1 1
27. Prove that 
cos   sin   1 sec   tan 

28. Find the zeroes of the quadratic polynomial 2x2 – x – 6 and verify the relationship between the
zeroes and the coefficients of the polynomial.

29. Given that √3 is irrational, prove that 5 + 2√3 is irrational.

30. Cards numbered 1 to 30 are put in a bag. A card is drawn at random from this bag. Find the
probability that the number on the drawn card is
(i) not divisible by 3.
(ii) a prime number greater than 7.
(iii) not a perfect square number.

31. Two tangents PA and PB are drawn to a circle with centre O from an external point P. Prove that
APB = 2OAB.

OR
Prove that opposite sides of a quadrilateral circumscribing a circle subtend supplementary angles at
the centre of the circle.

SECTION-D
Questions 32 to 35 carry 5M each
32. If a line is drawn parallel to one side of a triangle, prove that the other two sides are divided in the
same ratio. Using this theorem, find x in below figure, if MN || QR, PM = x cm, MQ = 10 cm, PN
= (x – 2) cm, NR = 6 cm

33. A train travels at a certain average speed for a distance of 63 km and then travels at a distance of 72
km at an average speed of 6 km/hr more than its original speed. If it takes 3 hours to complete total
journey, what is the original average speed?
OR
In a flight of 600 km, an aircraft was slowed due to bad weather. Its average speed for the trip was
reduced by 200 km/hr and time of flight increased by 30 minutes. Find the original duration of
flight.

34. If the median of the following distribution is 46, find the missing frequencies p and q if the total
frequency is 230.
Marks 10 – 20 20 – 30 30 – 40 40 – 50 50 – 60 60 – 70 70 – 80
Frequency 12 30 p 65 q 25 18

Prepared by: M. S. KumarSwamy, TGT(Maths) Page - 4-


35. Rasheed got a playing top (lattu) as his birthday present, which surprisingly had no colour on it. He
wanted to colour it with his crayons. The top is shaped like a cone surmounted by a hemisphere (see
below figure). The entire top is 5 cm in height and the diameter of the top is 3.5 cm. Find the area
he has to colour. (Take π = 22/7)
OR
A solid toy is in the form of a hemisphere surmounted by a right circular cone. The height of the
cone is 2 cm and the diameter of the base is 4 cm. Determine the volume of the toy. If a right
circular cylinder circumscribes the toy, find the difference of the volumes of the cylinder and the
toy. (Take π = 3.14)

SECTION-E (Case Study Based Questions)


Questions 36 to 38 carry 4M each

36. India is competitive manufacturing location due to the low cost of manpower and strong technical
and engineering capabilities contributing to higher quality production runs. The production of TV
sets in a factory increases uniformly by a fixed number every year. It produced 16000 sets in 6th
year and 22600 in 9th year.

On the basis of the above information, answer any four of the following questions:
(i) What is the production of first year? (1)
(ii) What is the production of 8th year? (1)
(iii) What is the production during first three years? (2)
OR
(iii) In which year, the production is 29,200? (2)

37. Raj is an electrician in a village. One day power was not there in entire village and villagers called
Raj to repair the fault. After thorough inspection he found an electric fault in one of the electric pole
of height 5 m and he has to repair it. He needs to reach a point 1.3m below the top of the pole to
undertake the repair work.

Prepared by: M. S. KumarSwamy, TGT(Maths) Page - 5-


Based on the above information answer the following questions.
(i) When the ladder is inclined at an angle of α such that √3 tan α + 2 = 5 to the horizontal, find the
angle α. (1)
(ii) In the above situation if BD = 3 cm and BC = 6 cm. Find α (1)
(iii) How far from the foot of the pole should he place the foot of the ladder? (Use √3 = 1.73) (2)
OR
(iii) Given 15 cot α = 8, find sin α. (2)

38. Aditya, Ritesh and Damodar are fast friend since childhood. They always want to sit in a row in the
classroom . But teacher doesn’t allow them and rotate the seats row-wise everyday. Ritesh is very
good in maths and he does distance calculation everyday. He consider the centre of class as origin
and marks their position on a paper in a co-ordinate system. One day Ritesh make the following
diagram of their seating position marked Aditya as A, Ritesh as B and Damodar as C.

(i) What is the distance between A and B ? [1]


(ii) What is the distance between B and C ? [1]
(iii) A point D lies on the line segment between points A and B such that AD :DB = 4 : 3 . What are
the the coordinates of point D ? [2]
OR
(iii) If the point P(k, 0) divides the line segment joining the points A(2, –2) and B(–7, 4) in the ratio
1 : 2, then find the value of k [2]

Prepared by: M. S. KumarSwamy, TGT(Maths) Page - 6-


Maths 11 Sample paperS
FOR CLASS X (Answers)
PREPARED BY: M. S. KUMAR SWAMY, TGT(MATHS)
KENDRIYA VIDYALAYA GACHIBOWLI, GPRA CAMPUS, HYD-32
SAMPLE PAPER TEST 01 FOR BOARD EXAM 2024
(ANSWERS)
SUBJECT: MATHEMATICS MAX. MARKS : 80
CLASS : X DURATION : 3 HRS
General Instruction:
1. This Question Paper has 5 Sections A-E.
2. Section A has 20 MCQs carrying 1 mark each.
3. Section B has 5 questions carrying 02 marks each.
4. Section C has 6 questions carrying 03 marks each.
5. Section D has 4 questions carrying 05 marks each.
6. Section E has 3 case based integrated units of assessment (04 marks each) with sub-parts of the
values of 1, 1 and 2 marks each respectively.
7. All Questions are compulsory. However, an internal choice in 2 Qs of 5 marks, 2 Qs of 3 marks and
2 Questions of 2 marks has been provided. An internal choice has been provided in the 2marks
questions of Section E
8. Draw neat figures wherever required. Take π =22/7 wherever required if not stated.
SECTION – A
Questions 1 to 20 carry 1 mark each.

1. In a formula racing competition, the time taken by two racing cars A and B to complete 1 round of
the track is 30 minutes and p minutes respectively. If the cars meet again at the starting point for the
first time after 90 minutes and the HCF (30, p) = 15, then the value of p is
(a)45 minutes (b)60 minutes (c)75 minutes (d)180 minutes
Ans: (a)45 minutes
LCM = 90 and HCF = 15
We know that HCF x LCM = Product of two numbers
⇒ 15 x 90 = 30 x p
⇒ p = 45

2. The solution of the following pair of equation is:


x – 3y = 2, 3x – y = 14
(a) x = 5, y = 1 (b) x = 2, y = 3 (c) x = 1, y = 2 (d) x = 1, y = 4
Ans: (a) x = 5, y = 1
Given, equations are x – 3y = 2 …(i)
and 3x – y = 14 …(ii)
Solving equations (i) and (ii), we get y = 1
x = 2 + 3y = 2 + 3 × 1 = 5
Hence, x = 5 and y = 1

3. If two positive integers a and b are written as a = x3y2 and b = xy3, where x and y are prime
numbers, then the HCF (a, b) is:
(a) xy (b) xy2 (c) x3y3 (d) x2y2
3 2 3
Ans: (b) Here, a = x y and b = xy
⇒ a = x × x × x × y × y and b = xy × y × y
∴ HCF(a, b) = x × y × y = x × y2 = xy2

4. The ratio in which x-axis divides the join of (2, -3) and (5, 6) is:
(a) 1: 2 (b) 3 : 4 (c) 1: 3 (d) 1: 5
Ans: (a) 1 : 2

Prepared by: M. S. KumarSwamy, TGT(Maths) Page - 1-


Let P(x, 0) be the point on x-axis which divides the join of (2, -3) and (5, 6) in the ratio k : 1.
∴ By section formula,
 5k  2 6k  3 
P(x, 0) =  , 
 k 1 k 1 
6k  3 1
 y0  0  6k  3  0  k 
k 1 2

5. The 11th and 13th terms of an AP are 35 and 41 respectively, its common difference is
(a) 38 (b) 32 (c) 6 (d) 3
Ans: (d) 3
Given, a11 = 35 ⇒ a + 10d = 35 ...(i)
and, a13 = 41 ⇒ a + 12d = 41 ...(ii)
Subtracting (i) from (ii), we get
2d = 6 ⇒ d = 3

6. A medicine-capsule is in the shape of a cylinder of radius 0.25 cm with two hemispheres stuck to
each of its ends. The length of the entire capsule is 2 cm. What is the total surface area of the
capsule? (Take π as 3.14)

(a) 0.785 cm2 (b) 0.98125 cm2 (c) 2.7475 cm2 (d) 3.14 cm2
Ans: (d) 3.14 cm2
Total surface area of the capsule = curved surface area of cylindrical portion
+ 2 × curved surface area of hemispherical portion
= 2πrh + 2 × 2πr2 = 2πrh + 4πr2
= 2πr(h + 2r)
= 2πr(1.50 + 0.50) = 4πr
= 4 × 3.14 × 0.25 = 3.14 cm2

7. A 1.6 m tall girl stands at distance of 3.2 m from a lamp post and casts shadow of 4.8 m on the
ground, then the height of the lamp post is
(a) 8 m (b) 4 m (c) 6 m (d) 8/3 m
Ans: (d) 8/3 m
Let AB be the position of the give and PQ be the lamp post.

Now, Δ OAB ∼ ΔOPQ (by AA similarity)


OA AB 4.8 1.6 4.8 1.6 16  8 8
       PQ   m
OP PQ 4.8  3.2 PQ 8 PQ 48 3

Prepared by: M. S. KumarSwamy, TGT(Maths) Page - 2-


8. A tangent is drawn from a point at a distance of 17 cm of circle (O, r) of radius 8 cm. The length of
tangent is
(a) 5 cm (b) 9 cm (c) 15 cm (d) 23 cm
Ans: (c) 15 cm

9. The runs scored by a batsman in 35 different matches are given below:


Runs Scored 0-15 15-30 30-45 45-60 60-75 75-90
Frequency 5 7 4 8 8 3
The lower limit of the median class is
(a) 15 (b) 30 (c) 45 (d) 60
Ans: (c) 45
Runs Scored 0-15 15-30 30-45 45-60 60-75 75-90
Frequency 5 7 4 8 8 3
cf 5 12 16 24 32 35
Here, n = 35 ⇒ n/2 = 17.5
Median class is 45 – 60
Hence, lower limit is 45

10. If in two triangles, DEF and PQR, ∠ =∠ and ∠ =∠ , then which of the following is not true?
EF DF EF DE DE DF EF DE
(a)  (b)  (c)  (d) 
PR PQ RP PQ QR PQ RP QR
EF DE
Ans: (b) 
RP PQ

11. In the given figure, if AB = 14 cm, then the value of tan B is:

4 14 5 13
(a) (b) (c) (d)
3 3 3 3
4
Ans: (a)
3

12. Two cubes each with 6 cm edge are joined end to end. The surface area of the resulting cuboid is
(a) 180 ² (b) 360 ² (c) 300 ² (d) 260 ²
Ans: (b) 360 ²
If two cubes of edges 6 cm are joined face to face it will take the shape of a cuboid whose length,
breadth and height are (6 + 6) cm, 6cm and 6cm i.e. 12 cm, 6cm and 6cm respectively.
Thus, total surface area of the cuboid = 2 (lb + bh + lh)
= 2 (12× 6 + 6× 6 + 12× 6)
= 2 (72 + 36 + 72)
= 2 ×180 cm2 = 360 cm2

Prepared by: M. S. KumarSwamy, TGT(Maths) Page - 3-


13. A cone, a hemisphere and cylinder are of the same base and of the same height. The ratio of their
volumes is
(a) 1 : 2 : 3 (b) 2 : 1 : 3 (c) 3 : 1 : 2 (d) 3 : 2 : 1
Ans: (a) 1 : 2 : 3
Let the base radii of them be r and height h.
Then ratio of volumes
cone : hemisphere : cylinder
1 2 1 2
  r 2 h :  r 3 :  r 2 h   r 2 r :  r 3 :  r 2 r (∵ r = h)
3 3 3 3
1 3 2 3 1 2
  r :  r :  r 3  : :1  1: 2 : 3
3 3 3 3

14. The probability of getting a bad egg in a lot of 400 is 0.035. The number of bad eggs in the lot is
(a) 7 (b) 14 (c) 21 (d) 28
Ans: (b) 14
Total number of eggs = 400
Probability of getting a bad egg P(E) = 0.035
Consider x as the number of bad eggs
P(E) = Number of bad eggs/ Total number of eggs
Substituting the values
0.035 = x/400 ⇒ 35/1000 = x/400 ⇒ x = 35/1000 x 400
⇒ x = 140/10 ⇒ x = 14

15. If 3 sin θ – cos θ = 0 and 0° < θ < 90°, find the value of θ.
(a) 30° (b) 45° (c) 60° (d) 90°
Ans: (a) 30°
3 sin θ – cos θ = 0 ⇒ 3 sin θ = cos θ
cos 
⇒ 3 ⇒ cot   3  cot 300 ⇒ θ = 30°
sin 

16. Find the value of k for which the equation x2 + k(2x + k − 1)+ 2 = 0 has real and equal roots.
(a) 2 (b) 3 (c) 4 (d) 5
Ans: (a) 2
Given quadratic equation: x2 + k(2x + k – 1) + 2 = 0
⇒ x2 + 2kx + (k2 – k + 2) = 0
For equal roots, b2 – 4ac = 0
⇒ 4k2 – 4k2 + 4k – 8 = 0
⇒ 4k = 8 ⇒ k = 2

17. In the below figure, the pair of tangents AP and AQ drawn from an external point A to a circle with
centre O are perpendicular to each other and length of each tangent is 5 cm. Then radius of the
circle is
(a) 10 cm (b) 7.5 cm (c) 5 cm (d) 2.5 cm

Prepared by: M. S. KumarSwamy, TGT(Maths) Page - 4-


Ans: (c) 5 cm
we know that radius of a circle is perpendicular to the tangent at the point of contact
⇒ OP ⊥ AP and OQ ⊥ AQ
Also sum of all angles of a quadrilateral is 360°
⇒∠O + ∠P + ∠A + ∠Q = 360°
⇒∠O + 90° + 90° + 90° = 360°
⇒∠O = 360° – 270° = 90°
Thus ∠O = ∠P = ∠A = ∠Q = 90° and OP = OQ (radii)
⇒ OPAQ is a square
⇒ radius = OP = OQ = AP = AQ = 5 cm

18. The radii of two cylinders are in the ratio 5 : 7 and their heights are in the ratio 3 : 5. The ratio of
their curved surface area is
(a) 3 : 7 (b) 7 : 3 (c) 5 : 7 (d) 3 : 5
Ans: (a) 3 : 7
2 r1 h1 r1 h1 5 3 3
Ratio of their curved surface area      
2 r2 h2 r2 h2 7 5 7

Direction : In the question number 19 & 20 , A statement of Assertion (A) is followed by a


statement of Reason(R) . Choose the correct option

19. Assertion (A): If x = 2 sin2θ and y = 2 cos2θ + 1 then the value of x + y = 3.


Reason (R): For any value of θ, sin2θ + cos2θ = 1
Ans: We know that for any value of θ, sin2θ + cos2θ = 1
So, Reason is correct.
For assertion: We have x = 2 sin2θ and y = 2 cos2θ + 1
Then, x + y = 2 sin2θ + 2 cos2θ + 1
= 2(sin2θ + cos2θ) + 1
= 2 x 1 + 1 [∵ sin2θ + cos2θ = 1]
= 2 + 1 = 3.
Hence, Assertion is also correct.
Correct option is (a) Both assertion (A) and reason (R) are true and reason (R) is the correct
explanation of assertion (A).

20. Assertion (A): The length of the minute hand of a clock is 7 cm. Then the area swept by the
minute hand in 5 minute is 77/6 cm2.

Reason (R): The length of an arc of a sector of angle q and radius r is given by l   2 r
3600
Ans: (b) Both assertion (A) and reason (R) are true and reason (R) is not the correct explanation of
Assertion (A)

SECTION-B
Questions 21 to 25 carry 2M each

21. Find the point on y-axis which is equidistant from the points (5, – 2) and (–3, 2).
Ans: Let point on y-axis be (0, a)
Now distance of this point from (5, –2) is equal to distance from point (–3, 2)
i.e., 52  (2  a )2  32  (a  2) 2
Squaring and simplifying, we get
25 + 4 + a2 + 4a = 9 + a2 + 4 – 4a 8a = –16 a = –2
Hence, the required point is (0, –2)

Prepared by: M. S. KumarSwamy, TGT(Maths) Page - 5-


22. X is a point on the side BC of ∆ABC. XM and XN are drawn parallel to AB and AC respectively
meeting AB in N and AC in M. MN produced meets CB produced at T. Prove that TX2 = TB × TC.
TB TN
Ans: In △TXM, XM  BN   ......(1)
TX TM

TX TN
In △TMC,  .......(2)
TC TM
TB TX
From 1 and 2, we have   TX 2  TB  TC
TX TC

23. The probability of selecting a blue marble at random from a jar that contains only blue, black and
green marbles is 1/5. The probability of selecting a black marble at random from the same jar is 1/4.
If the jar contains 11 green marbles, find the total number of marbles in the jar.
Ans: Let A be the event of getting blue marbles, B be the event of getting black marbles and C be
the event of getting green marbles.
P(A) = 1/5 , P(B) = 1/4
1 1 45 9 11
P(C) = 1 – [P(A) + P(B)] = 1      1   1 
5 4 20 20 20
11 11
⇒  (Assume x be the total number of marbles)
x 20
⇒ x = 20
Hence, Total number of marbles in the jar = 20.

24. In figure PA and PB are tangents to the circle drawn from an external point P. CD is the third
tangent touching the circle at Q. If PA = 15 cm, find the perimeter of ΔPCD.

Ans: Since, PA and PB are tangent from same external point.


∴ PA = PB = 15 cm
Now, perimeter of ∆PCD = PC + CD + DP
= PC + CQ + QD+ DP
= PC + CA + DB + DP
= PA + PB = 15 + 15 = 30 cm

OR

Prepared by: M. S. KumarSwamy, TGT(Maths) Page - 6-


Two concentric circles are of radii 8 cm and 5 cm. Find the length of the chord of the larger circle
which touches the smaller circle.
Ans: Let O be the center of the concentric circle.

Let AB be the chord of larger circle touching the smaller circle at P.


Here, OA = 8 cm, OP = 5 cm
Since AB is tangent of P to the smaller circle and OP is the radius of the smaller circle.
∴ OP ⊥ AB.
In right triangle APO, we have
OA2 = AP2 + OP2 ⇒ (8)2 = AP2 + (5)2 ⇒ AP2 = 64 – 25 = 39
⇒ AP = √39 cm
Hence, Length of the chord of largest circle AB = 2× AP = 2√39 cm

25. For what value of k, the following system of equations have infinite solutions:
2x – 3y = 7, (k + 2)x – (2k + 1)y = 3 (2k – 1)?
Ans: Here, a1 = 2, b1 = -3, c1 = -7
a2 = (k + 2), b2 = -(2k + 1), c2 = -3(2k – 1)
a b c
We know that the condition of infinite solution, 1  1  1
a2 b2 c2
2 3 7
    4k  2  3k  6  k  4
k  2 2k  1 3(2k  1)
OR
Sumit is 3 times as old as his son. Five years later, he shall be two and a half time as old as his son.
How old is Sumit at present?
Ans: Let present age of Sumit be x years and present age of his son be y years.
∴ x = 3y ⇒ x – 3y = 0 ...(i)
After 5 years, Age of Sumit = (x + 5) years
Age of his son = (y + 5) years
According to the question,
1
(x + 5) = 2 (y + 5)
2
5
⇒ (x + 5) = (y + 5)
2
⇒ 2x + 10 = 5y + 25 ⇒ 2x – 5y = 15 ...(ii)
From equation (i) and (ii), we get y = 15
Putting y = 15 in equation (i), we get x = 3 × 15 = 45 years
∴ Sumit is 45 years old at present.

SECTION-C
Questions 26 to 31 carry 3 marks each

26. Find the coordinates of the points which divide the line segment joining A (–2, 2) and B (2, 8) into
four equal parts.
Ans: Let P, Q, R be the points that divide the line segment joining A(–2, 2) and B(2, 8) into four
equal parts.

Prepared by: M. S. KumarSwamy, TGT(Maths) Page - 7-


Since, Q divides the line segment AB into two equal parts, i.e., Q is the mid-point of AB.
 2  2 2  8 
∴ Coordinates of Q are  ,   (0,5)
 2 2 
Now, P divides AQ into two equal parts i.e., P is the mid-point of AQ.
 2  0 2  5   7
∴ Coordinates of P are  ,    1, 
 2 2   2
Again, R is the mid-point of QB.
 0  2 5  8   13 
Coordinates of R are  ,    1, 
 2 2   2

27. If PQ is a tangent drawn from an external point P to a circle with centre O and QOR is a diameter
where length of QOR is 8 cm such that ∠POR = 120°, then find OP and PQ.
Ans: Let O be the centre and QOR = 8 cm is diameter of a circle.
PQ is tangent such that ∠POR = 120°.

Now, OQ = OR = 8/2 = 4 cm
∠POQ = 180° – 120° = 60° (Linear pair)
Also OQ ⊥ PQ
Now, in right ΔPOQ,
cos 60° = OQ/PO ⇒ 1/2 = OQ/PO
⇒ 1/2 = 4/PO ⇒ PO = 8 cm
Again, tan 60° = PQ/OQ ⇒ √3 = PQ/4
⇒ PQ = 4√3 cm.

p2 1
28. If sec θ + tan θ = p, prove that sin θ = .
p2 1
p 2  1 (sec   tan  )2  1
Ans: 2 
p  1 (sec   tan  )2  1
(sec2   1)  tan 2   2sec  tan  tan 2   tan 2   2sec  tan 
 
sec 2   (tan 2   1)  2sec  tan  sec 2   sec2   2sec  tan 
2 tan 2   2sec  tan  2 tan  (tan   sec  )
 
2sec 2   2sec  tan  2sec  (sec   tan  )
2 tan  sin 
  tan   cos    cos   sin 
2sec  cos 

OR
If sin θ + cos θ = √3 , then prove that tan θ + cot θ = 1.
Ans: sin   cos   3  (sin   cos  ) 2  3

Prepared by: M. S. KumarSwamy, TGT(Maths) Page - 8-


 sin 2   cos2   2sin  cos   3
 1  2 sin  cos   3  2sin  cos   2
 sin  cos   1  sin 2   cos 2 
sin 2   cos2  sin  cos 
1     tan   cot 
sin  cos  cos  sin 
 tan   cot   1

29. Daily wages of 110 workers, obtained in a survey, are tabulated below:
Daily Wages (in Rs. ) 100-120 120-140 140-160 160-180 180-200 200-220 220-240
Number of Workers 10 15 20 22 18 12 13
Compute the mean daily wages and modal daily wages of these workers.
Ans:

30. Solve the following linear equations:


152x – 378y = –74 and –378x + 152y = –604
Ans: We have, 152x – 378y = –74 ...(i)
–378x + 152y = –604 ...(ii)
Adding equation (i) and (ii), we get
⇒ –226(x +y) = –678
⇒ x + y = 3 …(iii)
Subtracting equation (ii) from (i), we get
530x – 530y = 530
⇒ x – y = 1 …(iv)
Adding equations (iii) and (iv), we get 2x = 4 ⇒ x = 2
Putting the value of x in (iii), we get
2+y=3⇒y=1
Hence, the solution of given system of equations is x = 2, y = 1.

31. The sum of the 5th and the 9th terms of an AP is 30. If its 25th term is three times its 8th term, find
the AP.
Ans: According to question, a5 + a9 = 30
⇒ (a + 4d) + (a + 8d) = 30
⇒ 2a + 12d = 30 ⇒ a + 6d = 15
⇒ a = 15 – 6d ...(i)
Also, a25 = 3a8 ⇒ a + 24d = 3(a + 7d)
⇒ a + 24d = 3a + 21d ⇒ 2a = 3d
Putting the value of a form (i), we have
2 (15 – 6d) = 3d ⇒ 30 – 12d = 3d

Prepared by: M. S. KumarSwamy, TGT(Maths) Page - 9-


⇒ 15d = 30 ⇒ d = 2
So, a = 15 – 6 × 2 = 15 – 12 [From equation (i)]
⇒a=3
The AP will be 3, 5, 7, 9....
OR
If the ratio of the sum of first n terms of two AP’s is (7n + 1) : (4n + 27), find the ratio of their mth
terms.
Ans:

SECTION-D
Questions 32 to 35 carry 5M each

32. From a solid cylinder whose height is 2.4 cm and diameter 1.4 cm, a conical cavity of the same
height and same diameter is hollowed out. Find the total surface area of the remaining solid to the
nearest cm2.
Ans: We have, Radius of the cylinder = 1.4/2 = 0.7 cm
Height of the cylinder = 2.4 cm
Also, radius of the cone = 0.7 cm
and height of the cone = 2.4 cm

Now, slant height of the cone = l


Now, l2 = (0.7)2+(2.4)2
⇒ l2 = 0.49 + 5.76 = 6.25 ⇒ l = 2.5 cm
∴ Total surface area of the remaining solid
= CSA of cylinder + CSA of the cone + area of upper circular base of cylinder
= 2πrh + πrl + πr2 = πr(2h + l + r)
22
= × 0.7 × [2 × 2.4 + 2.5 + 0.7] = 22 × 0.1 × (4.8 + 2.5 + 0.7)
7
= 2.2 × 8.0 = 17.6 cm2 = 18 cm2 (approx.)
OR
Rasheed got a playing top (lattu) as his birthday present, which surprisingly had no colour on it. He
wanted to colour it with his crayons. The top is shaped like a cone surmounted by a hemisphere.

Prepared by: M. S. KumarSwamy, TGT(Maths) Page - 10-


The entire top is 5 cm in height and the diameter of the top is 3.5 cm. Find the area he has to
colour.
3.5 7
Ans: Radius of hemispherical portion of the lattu = Radius of the conical portion, r   cm
2 4

 3.5  13
Height of the conical portion, h   5    cm
 2  4
Slant height of the conical part, l  r 2  h 2
2 2
 7   13  218
l        3.69 cm  3.7 cm
4  4  4
22 7  7 
Total surface area of the top = 2 r 2   rl   r (2r  l )    2   3.7   39.6 cm 2
7 4 4 

33. A motor boat whose speed is 15 km/hr in still water goes 30 km downstream and comes back in 4
hours 30 minutes. Find the speed of the stream.
Ans: Let the speed of the stream be x km/hr
Then Speed of boat downstream = (15 + x) km/hr
Speed of boat upstream = (15 − x) km/hr
30 30 1 9
According to the question,  4 
15  x 15  x 2 2
30(15  x)  30(15  x ) 9
 
(15  x )(15  x) 2
900 9
 2
  200  225  x 2  x 2  25
225  x 2
 x  5 (x is the speed of the stream and thus cannot have negative value)
Thus, the speed of the stream is 5 km/hr.

34. State and prove Basic Proportional Theorem.


Ans: Statement – 1 mark
Given, To Prove, Construction and Figure – 2 marks
Correct Proof – 2 marks

35. The lower window of a house is at a height of 2 m above the ground and its upper window is 4 m
vertically above the lower window. At certain instant, the angles of elevation of a balloon from
these windows are observed to be 60° and 30°, respectively. Find the height of the balloon above
the ground.
Ans: Let P and Q be the position of windows respectively.
Let AB = x m and CQ = y m

Prepared by: M. S. KumarSwamy, TGT(Maths) Page - 11-


CQ ' 1 y
Now, in ∆QQ'C, we have tan 300   
QQ ' 3 x
 x  3 y ...(i)
CP ' y4
Also, in ∆CP'P we have tan 600   3
PP ' 3y
⇒ 3y = y+ 4 & 2y = 4
⇒ y = 2m
⇒ Height of the balloon = (y + 4 + 2) m = (2 + 4 + 2) m = 8m
OR
From the top of a 60 m high building, the angles of depression of the top and the bottom of a tower
are 45° and 60° respectively. Find the height of the tower. [Take √3 = 1.73]
Ans: Let the height of the building be AE = 60 m, the height of the tower is ‘h’. The distance
between the base of the building and the tower be ‘d’.

AE 60 60
In ∆ADE, tan 600   3 d   20 3
DE d 3
BC = 20√3 = 20 × 1.73 = 34.60 m
AC AC
In ∆ABC, tan 450  1
BC 34.60
⇒ AC = 34.60 m
Now, height of tower = AE – AC = 60 – 34.60 = 25.4 m

Prepared by: M. S. KumarSwamy, TGT(Maths) Page - 12-


SECTION-E (Case Study Based Questions)
Questions 36 to 38 carry 4M each

36. Shivani took a pack of 52 cards. She kept aside all the black face cards and shuffled the remaining
cards well.

Based on the above information answer the following questions.


(i) Write the number of total possible outcomes.
(ii) She draws a card from the well-shuffled pack of remaining cards. What is the probability that
the card is a face card?
(iii) Write the probability of drawing a black card.
OR
(iii) What is the probability of getting neither a black card nor an ace card?
Ans: (i) Total possible outcomes = 52 – 6 = 46
(ii) Number of favourable outcomes = 6
P(face card) = 6/46 = 3/23
(iii) Number of black cards in the shuffled cards = 13 + 7 = 20
P(black card) = 20/46 = 10/23
OR
Number of black cards and ace = 20 + 2 = 22
∴ Number of favourable outcomes = 46 – 22 = 24
P(neither a black card nor an ace) = 24/46 = 12/23

37. In the month of April to June 2022, the exports of passenger cars from India increased by 26% in
the corresponding quarter of 2021–22, as per a report. A car manufacturing company planned to
produce 1800 cars in 4th year and 2600 cars in 8th year. Assuming that the production increases
uniformly by a fixed number every year.

Based on the above information answer the following questions.


(i) Find the production in the 1st year. (1)
(ii) Find the production in the 12th year. (1)

Prepared by: M. S. KumarSwamy, TGT(Maths) Page - 13-


(iii) Find the total production in first 10 years. (2)
OR
(iii) In how many years will the total production reach 31200 cars? (2)
Ans: (i) Since the production increases uniformly by a fixed number every year, the number of Cars
manufactured in 1st, 2nd, 3rd, . . .,years will form an AP.
So, a + 3d = 1800 & a + 7d = 2600
So d = 200 & a = 1200
(ii) a12 = a + 11d ⇒ a30 = 1200 + 11 × 200
⇒ a12 = 3400
n 10
(iii) S n  [2a  (n  1) d ]  S10  [2  1200  (10  1)  200]
2 2
 S10  5[2400  1800]  5  4200  21000
OR
n
S n  [2a  (n  1) d ]  31200
2
n
 [2  1200  ( n  1)  200]  31200
2
n
  200[12  ( n  1)]  31200
2
⇒ n[12 + (n − 1) ] = 312
⇒ n2 + 11n – 312 = 0
⇒ n2 + 24n – 13n – 312 = 0
⇒ (n +24)(n – 13) = 0
⇒ n = 13 or – 24.
As n can’t be negative. So n = 13

38. Aditya plantations have two rectangular fields of the same width but different lengths. They are
required to plant 168 trees in the smaller field and 462 trees in the larger field. In both fields, the
trees will be planted in the same number of rows but in different number of columns.

(i) What is the maximum number of rows in which the trees can be planted in each of the fields? (2)
(ii) If the trees are planted in the number of rows obtained in part (i), how many columns will each
field have?
(iii) If total cost of planted trees in one column is Rs. 500, then find the cost to plant the trees in
smaller field.
OR
If the total cost of planted trees in one column is Rs. 500, the find the cost to plant the trees in larger
field.
Ans: (i) The maximum number of rows for two field is HCF of 168 and 462.
Now 168 = 23 × 3 × 7
462 = 2 × 3 × 7 × 11
∴ HCF (168, 462) = 2 × 3 × 7 = 42
∴ Number of rows = 42

Prepared by: M. S. KumarSwamy, TGT(Maths) Page - 14-


168
(ii) Number of columns in smaller field = =4
42
462
Number of columns in larger field = = 11
42
(iii) Number of columns in smaller field = 4
∴ cost = Rs. 4 × 500 = Rs. 2000.
OR
Number of columns in larger field = 11
∴ Required cost = 11 × 500 = Rs. 5500

Prepared by: M. S. KumarSwamy, TGT(Maths) Page - 15-


PM SHRI KENDRIYA VIDYALAYA GACHIBOWLI, GPRA CAMPUS, HYD-32
SAMPLE PAPER TEST 02 FOR BOARD EXAM 2024
(ANSWERS)
SUBJECT: MATHEMATICS MAX. MARKS : 80
CLASS : X DURATION : 3 HRS
General Instruction:
1. This Question Paper has 5 Sections A-E.
2. Section A has 20 MCQs carrying 1 mark each.
3. Section B has 5 questions carrying 02 marks each.
4. Section C has 6 questions carrying 03 marks each.
5. Section D has 4 questions carrying 05 marks each.
6. Section E has 3 case based integrated units of assessment (04 marks each) with sub-parts of the
values of 1, 1 and 2 marks each respectively.
7. All Questions are compulsory. However, an internal choice in 2 Qs of 5 marks, 2 Qs of 3 marks and
2 Questions of 2 marks has been provided. An internal choice has been provided in the 2marks
questions of Section E
8. Draw neat figures wherever required. Take π =22/7 wherever required if not stated.
SECTION – A
Questions 1 to 20 carry 1 mark each.

1. If two positive integers p and q can be expressed as p = ab2 and q = a3b; a, b being prime numbers,
then LCM (p, q) is
(a) ab (b) a2b2 (c) a3b2 (d) a3b3
3 2
Ans: (c) a b

2. The perimeter of a triangle with vertices (0, 4), (0, 0) and (3, 0) is
(a) 5 units (b) 12 units (c) 11 units (d) (7 + √5) units
Ans: (b) 12 units

3. The zeroes of the polynomial x2 – 3x – m(m + 3) are


(a) m, m + 3 (b) –m, m + 3 (c) m, – (m + 3) (d) –m, – (m + 3)
Ans: (b) –m, m + 3
Let p(x) = x2 – 3x – m (m + 3)
⇒ p(x) = x2 – (m + 3) x +mx – m (m + 3)
= x{x – (m + 3)} + m {x – (m + 3)}
For zeros of p(x)
⇒ p(x) = (x + m) {(x – (m + 3)} = 0
⇒ x = – m, m + 3
∴ Its zeros are – m, m + 3.

4. The area of a quadrant of a circle, whose circumference is 22 cm, is


11 77 77 77
(a) cm2 (b) cm2 (c) cm2 (d) cm2
8 8 2 4
77
Ans: (b) cm2
8
5. The pair of linear equations 2x + 3y = 5 and 4x + 6y = 10 is
(a) inconsistent (b) consistent (c) dependent consistent (d) none of these
Ans: (c) dependent consistent

6. If the circumference of a circle and the perimeter of a square are equal, then

Prepared by: M. S. KumarSwamy, TGT(Maths) Page - 1-


(a) Area of the circle = Area of the square
(b) Area of the circle > Area of the square
(c) Area of the circle < Area of the square
(d) Nothing definite can be said about the relation between the areas of the circle and square.
Ans: (b) Area of the circle > Area of the square

7. The sum of the lower limit of median class and the upper limit of the modal class of the following
data is:
Marks 0 – 10 10 – 20 20 – 30 30 – 40 40 – 50 50 – 60
No. of students 8 10 12 22 30 18
(a) 70 (b) 80 (c) 90 (d) 100
Ans: (b) 80
8. A card is selected at random from a well shuffled deck of 52 cards. The probability of its being a
face card is
(a) 3/26 (b) 3/13 (c) 2/13 (d) 1/2
Ans: (b) 3/13
7
9. In ABC right angled at B, sin A = , then the value of cos C is ………….
25
7 24 7 24
(a) (b) (c) (d)
25 25 24 7
7
Ans: (a)
25
10. The radius of the largest right circular cone that can be cut out from a cube of edge 4.2 cm is
(a) 2.1 cm (b) 4.2 cm (c) 3.1 cm (d) 2.2 cm
Ans: (a) 2.1 cm
The diameter of the largest right circular cone that can be cut out from a cube of edge 4.2 cm.
∴ 2r = 4.2 cm ⇒ r = 2.1 cm
⇒ Radius of the largest right circular cone = 2.1 cm.

11. Volume and surface area of a solid hemisphere are numerically equal. What is the diameter of
hemisphere?
(a) 9 units (b) 6 units (c) 4.5 units (d) 18 units
Ans: (a) 9 units
Volume of hemisphere = Surface area of hemisphere
2 9
  r 3  3 r 2  r  units
3 2
∴ d = 9 units

12. In the ∆ABC, DE ∥ BC and AD = 3x − 2, AE = 5x − 4, BD = 7x − 5, CE = 5x − 3, then find the


value of x
(a) 1 (b) 7/10 (c) both (a) & (b) (d) none of these
Ans: (c) both (a) & (b)

Prepared by: M. S. KumarSwamy, TGT(Maths) Page - 2-


Given that, AD = 3x − 2, AE = 5x − 4, BD = 7x − 5, CE = 5x − 3
AD AE
By Basic Proportionality theorem, we have 
BD EC
3x  2 5 x  4
   (3 x  2)(5 x  3)  (5 x  4)(7 x  5)
7 x  5 5x  3
 15 x 2  19 x  6  35 x 2  53x  20
 20 x 2  34 x  14  0  10 x 2  17 x  7  0
7
 ( x  1)(10 x  7)  0  x  1, x 
10

13. Two circles touch each other externally at C and AB is common tangent of circles, then ∠ACB is
(a) 70° (b) 60° (c) 100° (d) 90°
Ans: (d) 90°
Draw CM perpendicular to AB.

Now, AM = MC and MB = MC (tangents drawn from external point are equal).


⇒ AM = MC
⇒ ∠MAC = ∠MCA = 45°
(Since Δ AMC is right triangle)
∴ Also, MB = MC ⇒ ∠MBC = ∠MCB = 45° (Since Δ MBC is right angle triangle)
∴ ∠ACB = ∠MCA + ∠MCB = 45° + 45° = 90° ⇒ ∠ACB = 90°

5sin   3cos 
14. If 5 tan θ = 4, then the value of is
5sin   2cos 
(a) 1/6 (b) 1/7 (c) 1/4 (d) 1/5
Ans: (a) 1/6

15. Given that sin α = 1/2 and cos β = 1/2, then the value of (β – α) is
(a) 0° (b) 30° (c) 60° (d) 90°
Ans: (b) 30°

16. Two identical solid hemispheres of equal base radius are stuck along their bases. The total surface
area of the combination is
(a) πr2 (b) 2πr2 (c) 3πr2 (d) 4πr2
2
Ans: (d) 4πr
The resultant solid will be a sphere of radius r whose total surface area is 4πr2.

17. Nature of roots of quadratic equation 2x2 – 4x + 3 = 0 is


(a) real (b) equal (c) not real (d) none of them
Ans: (c) not real
D = b2 – 4ac = 42 – 4 × 2 × 3 = 16 – 24 = –8 < 0
Since D < 0

Prepared by: M. S. KumarSwamy, TGT(Maths) Page - 3-


Hence, roots are not real.

18. If ∆ABC ~ ∆EDF and ∆ABC is not similar to ∆DEF, then which of the following is not true?
(a) BC. EF = AC. FD (b) AB. EF = AC. DE
(c) BC. DE = AB. EF (d) BC. DE = AB. FD
Ans: (c) BC. DE = AB. EF
Since, ∆ABC ~ ∆EDF
Therefore, the ratio of their corresponding sides are equal.
BC AB
   BC .DE  AB.EF
EF DE

Direction : In the question number 19 & 20 , A statement of Assertion (A) is followed by a


statement of Reason(R) . Choose the correct option

19. Assertion (A): The value of y is 3, if the distance between the points P(2, -3) and Q(10, y) is 10.
Reason (R): Distance between two points is given by ( x2  x1 )2  ( y2  y1 ) 2
(a) Both Assertion (A) and Reason (R) are true and Reason (R) is the correct explanation of
Assertion (A)
(b) Both assertion (A) and reason (R) are true and reason (R) is not the correct explanation of
Assertion (A)
(c) Assertion (A) is true but reason(R) is false.
(d) Assertion (A) is false but reason(R) is true.
Ans: (a) Both Assertion (A) and Reason (R) are true and Reason (R) is the correct explanation of
Assertion (A)

20. Assertion (A): 6n never ends with the digit zero, where n is natural number.
Reason (R): Any number ends with digit zero, if its prime factor is of the form 2m × 5n, where m, n
are natural numbers.
(a) Both Assertion (A) and Reason (R) are true and Reason (R) is the correct explanation of
Assertion (A).
(b) Both Assertion (A) and Reason (R) are true but Reason (R) is not the correct explanation of
Assertion (A).
(c) Assertion (A) is true but Reason (R) is false.
(d) Assertion (A) is false but Reason (R) is true.
Ans: (a) Both Assertion (A) and Reason (R) are true and Reason (R) is the correct explanation of
Assertion (A).
6n = (2 × 3)n = 2n × 3n, Its prime factors do not contain 5 i.e., of the form 2m × 5n,
where m, n are natural numbers.

SECTION-B
Questions 21 to 25 carry 2M each

21. For what values of k will the following pair of linear equations have infinitely many solutions? kx
+ 3y – (k – 3) = 0 and 12x + ky – k = 0
Ans: Comparing with a1 x  b1 y  c1 and a2 x  b2 y  c2
a1  k , a2  12, b1  3, b2  k , c1  k  3, c2  k
a b c
For infinite solutions, 1  1  1
a2 b2 c2
k 3 k 3 k 3
      k 2  36  k  6
12 k k 12 k

Prepared by: M. S. KumarSwamy, TGT(Maths) Page - 4-


22. In the given figure, AP = 3 cm, AR = 4.5 cm, AQ = 6 cm, AB = 5 cm, AC = 10 cm. Find the length
of AD

AP 3
Ans: In ∆ABC,  ……. (i)
AB 5
AQ 6 3
  ……. (ii)
AC 10 5
AP AQ

From (i) and (ii), we get ⇒ PQ || BC
AB AC
AP AR 3 4.5
In ∆ABD, PR || BD ⇒     AD  7.5cm
AB AD 5 AD

23. Two concentric circles are of radii 5 cm and 3 cm. Find the length of the chord of the larger circle
which touches the smaller circle.
Ans: Let O be the centre of the concentric circle of radii 5 cm and 3 cm respectively. Let AB be a
chord of the larger circle touching the smaller circle at P

Then AP = PB and OP⊥AB


Applying Pythagoras theorem in △OPA, we have
OA2 = OP2 + AP2 ⇒ 25 = 9 + AP2
⇒ AP2 = 16 ⇒ AP = 4 cm
∴ AB = 2AP = 8 cm

1
24. If sin (A + B) = √3/2 and sin (A – B) = , 0 ≤ A + B ≤ 90° and A > B, then find A and B.
2
Ans: sin(A + B) = √3/2 = sin 60⁰
⇒ A + B = 60⁰ ........(i)
sin (A - B) = 1/2 = sin 30⁰
⇒ A - B = 30⁰ ........(ii)
Solving eq. (i) and (ii), A = 45⁰ and B = 15⁰
OR
(1  sin  )(1  sin  )
If tan θ =3/4, evaluate
(1  cos  )(1  cos  )
3 4
Ans: tan    cot  
4 3

Prepared by: M. S. KumarSwamy, TGT(Maths) Page - 5-


2
(1  sin  )(1  sin  ) 1  sin 2  cos 2   4  16
 2
 2
 cot 2     
(1  cos  )(1  cos  ) 1  cos  sin   3 9

25. Find the area of the sector of a circle with radius 4 cm and of angle 30°. Also, find the area of the
corresponding major sector. (Use π = 3.14)
 r 2 3.14  42  300
Ans: Area of sec tor AOB    4.19cm2
3600 3600
Area of major sector = Area of circle – Area of sector AOB
= r2 – 4.19 = 3.14 × 16 – 4.19 = 46.1 cm2
OR
What is the angle subtended at the centre of a circle of radius 10 cm by an arc of length 5π cm?

Ans: Arc length of a circle of radius r   2 r
3600
  5 1
 5  0
 2 10  0
 
360 360 20 4
0
   90

SECTION-C
Questions 26 to 31 carry 3 marks each

26. Four bells toll at an interval of 8, 12, 15 and 18 seconds respectively. All the four begin to toll
together. Find the number of times they toll together in one hour excluding the one at the start.
Ans: Prime factorisation of the given numbers are:
8 = 2 × 2 × 2 = 23
12 = 2 × 2 × 3 = 22 × 31
15 = 3 × 5 = 31 × 51
18 = 2 × 3 × 3 = 21 × 32
LCM (8, 12, 15 and 18) = 23 × 32 × 51 = 8 × 9 × 5 = 360 sec = 6 min
∴ Four bells toll together in one hour = 60 ÷ 6 = 10 times.

27. Find the zeroes of the quadratic polynomial 6x2– 3 – 7x and verify the relationship between the
zeroes and the coefficients of the polynomial.
Ans: 6x2 – 7x – 3 = 0
6x2 – 9x + 2x – 3 = 0
3x(2x – 3) + 1(2x – 3) = 0
 (3x + 1) (2x – 3) = 0
1 3
x = ,
3 2
1 3 2  9 7 b 7 b
Now,        and     
3 2 6 6 a 6 a
 1 3 1 c 1 c
    and    
3 2 2 a 2 a

OR
Find the quadratic polynomial sum and product of whose zeros are –1 and –20 respectively. Also
find the zeroes of the polynomial so obtained.
Ans: Let α and β be the zeros of the quadratic polynomial.
∴ Sum of zeros, α + β = – 1
and product of zeros, α. β = – 20
Now, quadratic polynomial be
x2 – (α + β) . x + α β = x2 –(–1) x – 20 = x2 + x – 20

Prepared by: M. S. KumarSwamy, TGT(Maths) Page - 6-


Now, for zeroes of this polynomial
x2 + x – 20 = 0 ⇒ x2 + 5x – 4x – 20 = 0
⇒ x(x + 5) – 4 (x + 5) = 0 ⇒ (x + 5) (x – 4) = 0
⇒ x = – 5, 4
∴ zeroes are – 5 and 4

28. The area of a rectangle gets reduced by 9 square units, if its length is reduced by 5 units and breadth
is increased by 3 units. If we increase the length by 3 units and the breadth by 2 units, the area
increases by 67 square units. Find the dimensions of the rectangle.
Ans: Let length and breadth be x and y , Area = xy
1st condition: ( x – 5 ) ( y + 3 ) = xy – 9
⇒ 3x – 5y = 6
2nd condition: ( x + 3 ) ( y + 2 ) = xy + 67
⇒ 2x + 3y = 61
Solve 1st and 2nd equations, we get x = 17 and y = 9
Hence, Length of rectangle = 17 units and breadth of rectangle = 9 units

29. In the below figure, XY and X′Y′ are two parallel tangents to a circle with centre O and another
tangent AB with point of contact C intersecting XY at A and X′Y′ at B. Prove that ∠AOB = 90°.

Ans: Join OC. Since, the tangents drawn to a circle from an external point are equal.
∴ AP = AC

In Δ PAO and Δ AOC, we have:


AO = AO [Common]
OP = OC [Radii of the same circle]
AP = AC
⇒ Δ PAO ≅ Δ AOC [SSS Congruency]
∴ ∠PAO = ∠CAO = ∠1
∠PAC = 2 ∠1 ...(1)
Similarly ∠CBQ = 2 ∠2 ...(2)
Again, we know that sum of internal angles on the same side of a transversal is 180°.
∴ ∠PAC + ∠CBQ = 180°
⇒ 2 ∠1 + 2 ∠2 = 180° [From (1) and (2)]
⇒ ∠1 + ∠2 = 180°/2 = 90° ...(3)

Prepared by: M. S. KumarSwamy, TGT(Maths) Page - 7-


Also ∠1 + ∠2 + ∠AOB = 180° [Sum of angles of a triangle]
⇒ 90° + ∠AOB = 180°
⇒ ∠AOB = 180° − 90° ⇒ ∠AOB = 90°.
OR
In the below figure, two equal circles, with centres O and O', touch each other at X. OO' produced
meets the circle with centre O' at A. AC is tangent to the circle with centre O, at the point C. O'D is
DO '
perpendicular to AC. Find the value of .
CO

Ans: AC is tangent to circle with centre O.


Thus ∠ACO = 90°
In ∆AO'D and ∆AOC
∠ADO' = ∠ACO = 90°
∠A = ∠A (Common)
∴ ∆AO'D ~ ∆AOC (By AA similarity)
AO ' DO '
⇒ 
AO CO
Now, AO = AO' + O' X + XO = 3r
DO ' r 1
  
CO 3r 3

sin   cos   1
30. Prove that  sec   tan 
sin   cos   1
tan   1  sec 
Ans: LHS = tan   1  sec  (Dividing numerator and denominator by cos )

tan   sec   1

tan   1  sec
tan   sec   (sec 2   tan 2  )

tan   1  sec 
(sec   tan  )(1  sec   tan  )

tan   1  sec 
 sec   tan  = RHS

31. Two dice are thrown at the same time. What is the probability that the sum of the two numbers
appearing on the top of the dice is (i) 5? (ii) 10? (iii) at least 9?
Ans: Total number of outcomes = 36
(i) Number of outcomes in which the sum of the two numbers is 5 = 4
∴ Required Probability = 4/36 = 1/9
(ii) Number of outcomes in which the sum of the two numbers is 10 = 3
∴ Required Probability = 3/36 = 1/12
(i) Number of outcomes in which the sum of the two numbers is at least 9 = 10
∴ Required Probability = 10/36 = 5/18

Prepared by: M. S. KumarSwamy, TGT(Maths) Page - 8-


SECTION-D
Questions 32 to 35 carry 5M each

32. A motor boat whose speed is 18 km/h in still water takes 1 hour more to go 24 km upstream than to
return downstream to the same spot. Find the speed of the stream.
Ans: Let the speed of the stream be x km/h.
Therefore, the speed of the boat upstream = (18 – x) km/h and the speed of the boat downstream =
(18 + x) km/h.
24
The time taken to go upstream = distance/speed =
18  x
24
Similarly, the time taken to go downstream =
18  x
24 24
According to the question,  1
18  x 18  x
24(18 + x) – 24(18 – x) = (18 – x) (18 + x)
x2 + 48x – 324 = 0
x = 6 or – 54
Since x is the speed of the stream, it cannot be negative. So, we ignore the root x = – 54. Therefore,
x = 6 gives the speed of the stream as 6 km/h.

OR
An express train takes 1 hour less than a passenger train to travel 132 km between Mysore and
Bangalore (without taking into consideration the time they stop at intermediate stations). If the
average speed of the express train is 11km/h more than that of the passenger train, find the average
speed of the two trains.
Ans: Ans: Let the average speed of the passenger train = x km/hour
And the average speed of the express train = (x + 11) km/hour.
132
The time taken by the passenger train = hour
x
132
and the time taken by the express train = hour
x  11
132 132
According to the question,  1
x  11 x
⇒ x2 + 11x −1452 = 0
⇒ (x − 33)(x + 44) = 0 ⇒ x = 33, x = − 44
The speed cannot be negative, so the speed of the passenger train is 33 km/hour and the speed of the express
train is 33 + 11 = 44 km/ hour.

33. State and prove Basic Proportional Theorem.


Ans: Statement – 1 mark
Given, To Prove, Construction and Figure – 2 marks
Correct Proof – 2 marks

34. If the median of the following distribution is 58 and sum of all the frequencies is 140. What is the
value of x and y?
Class 15 – 25 25 – 35 35 – 45 45 – 55 55 – 65 65 – 75 75 – 85 85 – 95
Frequency 8 10 x 25 40 y 15 7
Ans:

Prepared by: M. S. KumarSwamy, TGT(Maths) Page - 9-


35. A toy is in the form of a hemisphere surmounted by a right circular cone of the same base radius as
that of the hemisphere. If the radius of the base of the cone is 21 cm and its volume is 2/3 of the
volume of the hemisphere, calculate the height of the cone and the surface area of the toy.
Ans: We have, Radius of cone = Radius of hemisphere = 21 cm
⇒ r = 21 cm

2
According to question, Volume of cone = × volume of hemisphere
3
1 2 2 4 4
  r 2 h    r 3  h  r   21  28 cm
3 3 3 3 3
Slant height, l  r 2  h 2  212  282  35 cm
Total surface area = CSAcone  CSAhemisphere   rl  2 r 2   r (l  2r)
22
  21  (42  35)  22  3  77  5082 cm 2
7
OR
A vessel full of water is in the form of an inverted cone of height 8 cm and the radius of its top,
which is open, is 5 cm. 100 spherical lead balls are dropped into the vessel. One fourth of the water
flows out of the vessel. Find the radius of a spherical ball.
Ans: Height (h) of the cone = 8 cm and radius (r) of the cone = 5 cm
1
∴ Volume of water flows out = × volume of cone
4
1 1 2 1
   r h     25  8
4 3 12
∴ Volume of water flows out =100× volume of spherical ball

Prepared by: M. S. KumarSwamy, TGT(Maths) Page - 10-


1 4
    25  8  100   R 3
12 3
3 1 1
 R   R  cm  0.5cm
8 2

SECTION-E (Case Study Based Questions)


Questions 36 to 38 carry 4M each

36. The top of a table is shown in the figure given below:

On the basis of above information answer the following questions.


(i) Find the distance between points A and B.
(ii) Write the co-ordinates of the mid point of line segment joining points M and Q.
(iii) If G is taken as the origin, and x, y axis put along GF and GB, then find the point denoted by
coordinates (4, 2) and (8, 4).

Prepared by: M. S. KumarSwamy, TGT(Maths) Page - 11-


OR
Find the coordinates of H, G and also find the distance between them.
Ans: (i) Distance between A(1, 9) and B(5, 13) is
 (5  1)2  (13  9) 2  16  16  32  4 2units
(ii) Midpoint of the line segment joining M(5, 11) and Q(9, 3) is given by
 5  9 11  3   14 14 
 ,    ,   (7, 7)
 2 2  2 2
(iii) If G is (0, 0) then Q is (4, 2) and E is (8, 4).
OR
As per graph the coordinate of H is (1, 5) and of G is (5, 1).
Distance HG  (5  1)2  (1  5)2  16  16  32  4 2units

37. Ananya saves Rs. 24 during the first month Rs. 30 in the second month and Rs. 36 in the third
month. She continues to save in this manner.

On the basis of above information answer the following questions.


(i) Whether the monthly savings of Ananya form an AP or not? If yes then write the first term and
common difference.
(ii) What is the amount that she will save in 15th month?
(iii) In which month, will she save Rs. 66?
OR
What is the common difference of an AP whose nth term is 8 – 5n?
Ans: (i) Savings of Ananya are Rs. 24, Rs. 30, Rs. 36, ...
Since it is uniformly increasing by Rs. 6, therefore it forms an AP.
Here, a = 24, d = 30 – 24 = 6
(ii) a15 = a + 14d = 24 +14 × 6 = 24 + 84 = Rs. 108
(iii) an = 66 ⇒ a + (n – 1)d = 66
⇒ 24 + (n – 1)6 = 66 ⇒ n – 1 = 42/6 = 7 ⇒ n = 8
OR
an = 8 – 5n
a1 = 8 – 5 = 3
a2 = 8 – 10 = –2 ⇒ d = a2 – a1 = –2 – 3 = – 5

38. A person/observer on the sea coast observes two ships in the sea, both the ships are in same straight
path one behind the other.
If the observer is on his building of height 20 meters (including observer) and he observes the angle
of depression of two ships as 45° and 60° respectively.

Prepared by: M. S. KumarSwamy, TGT(Maths) Page - 12-


On the basis of above information answer the following questions.
(i) If a person observes a ship whose angle of depression is 60° then how much distance is the ship
away from the building?
(ii) If a person observes another ship whose angle of depression is 45° then how much distance that
ship is away from the building?
(iii) If a person observes the ship whose angle of depression changes from 60° to 30° then how far
be ship from the building if the observer is at 20 m of height (including him)?
OR
At a time when a person observes two ships whose angle of depressions are 60° and 45° the
distance between the ships is (in meter).
OC 20 20 20 3
Ans: (i) tan 600   3  AC   m  11.55m
AC AC 3 3

OC 20
(ii) tan 450  1   BC  20m
BC BC

OB 1 20
(iii) tan 300     OA  20 3m
OA 3 OA

OR
Distance between two ships 20 m = BC – AC = 20 – 11.55 = 8.45 m

Prepared by: M. S. KumarSwamy, TGT(Maths) Page - 13-


KENDRIYA VIDYALAYA GACHIBOWLI, GPRA CAMPUS, HYD-32
SAMPLE PAPER TEST 03 FOR BOARD EXAM 2024
(ANSWERS)
SUBJECT: MATHEMATICS MAX. MARKS : 80
CLASS : X DURATION : 3 HRS
General Instruction:
1. This Question Paper has 5 Sections A-E.
2. Section A has 20 MCQs carrying 1 mark each.
3. Section B has 5 questions carrying 02 marks each.
4. Section C has 6 questions carrying 03 marks each.
5. Section D has 4 questions carrying 05 marks each.
6. Section E has 3 case based integrated units of assessment (04 marks each) with sub-parts of the
values of 1, 1 and 2 marks each respectively.
7. All Questions are compulsory. However, an internal choice in 2 Qs of 5 marks, 2 Qs of 3 marks and
2 Questions of 2 marks has been provided. An internal choice has been provided in the 2marks
questions of Section E
8. Draw neat figures wherever required. Take π =22/7 wherever required if not stated.
SECTION – A
Questions 1 to 20 carry 1 mark each.

1. A card is selected from a deck of 52 cards. The probability of being a red face card is
(a) 3/26 (b) 3/13 (c) 2/13 (d) 1/2
Ans: (a) 3/26
Total number of red face cards = 6
∴ Probability of being a red face card = 6/52 = 3/26
2. If two tangents inclined at an angle of 60ᵒ are drawn to a circle of radius 3cm, then the length of
each tangent is equal to
3 3
(a) cm (b) 3 cm (c) 6 cm (d) 3 3
2
Ans: (d) 3 3

3. If the mean of a frequency distribution is 8.1 and fi = 20, fixi = 132 + 5k, then k =
(a) 3 (b) 4 (c) 5 (d) 6
Ans: (d) 6
4. If the radii of two circles are in the ratio of 4 : 3, then their areas are in the ratio of :
(a) 4 : 3 (b) 8 : 3 (c) 16 : 9 (d) 9 : 16
Ans: (c) 16 : 9
5. If one zero of the quadratic polynomial x2 + 3x + k is 2, then the value of k is
(a) 10 (b) –10 (c) 5 (d) –5
Ans: (b) –10
6. If two positive integers a and b are written as a = x3y2 and b = xy3; x, y are prime numbers, then
HCF (a, b) is
(a) xy (b) xy2 (c) x3y3 (d) x2y2
Ans: (b) xy2
7. When 2120 is expressed as the product of its prime factors we get
(a) 2 × 5³ × 53 (b) 2³ × 5 × 53 (c) 5 × 7² × 31 (d) 5² × 7 × 33
Ans: (b) 2³ × 5 × 53

Prepared by: M. S. KumarSwamy, TGT(Maths) Page - 1-


8. In the ∆ABC, D and E are points on side AB and AC respectively such that DE || BC.
If AE = 2 cm, AD = 3 cm and BD = 4.5 cm, then CE equals
(a) 1 cm (b) 2 cm (c) 3 cm (d) 4 cm
Ans: (c) 3 cm

9. If the distance between the points (2, –2) and (–1, x) is 5, one of the values of x is
(a) –2 (b) 2 (c) –1 (d) 1
Ans: (b) 2
Let us consider the points as A = (2, -2) and B = (-1, x)
AB = 5 units
⇒ 52 = (-1 - 2)2 + (x + 2)2 ⇒ 25 = (-3)2 + (x + 2)2
⇒ 25 = 9 + x2 + 4 + 4x ⇒ 25 = x2 + 4x + 13
⇒ x2 + 4x + 13 - 25 = 0 ⇒ x2 + 4x - 12 = 0
⇒ x2 + 6x - 2x - 12 = 0 ⇒ x(x + 6) - 2(x + 6) = 0
⇒ (x + 6)(x - 2) = 0 ⇒ x + 6 = 0 ⇒ x = -6
And x - 2 = 0 ⇒ x = 2
Therefore, one of the values of x is 2.
10. The value of k for which the pair of equation kx – y = 2 and 6x – 2y = 3 has unique solution
(a) k = 3 (b) k ≠ 3 (c) k ≠ 0 (d) k = 0
Ans: (b) k ≠ 3
For unique solution, we have
a1 b1 k 1 k 1
     k 3
a2 b2 6 2 6 2

11. The median class of the following data is:


Marks 0 – 10 10 – 20 20 – 30 30 – 40 40 – 50 50 – 60
No. of students 8 10 12 22 30 18
(a) 20 – 30 (b) 30 – 40 (c) 40 – 50 (d) 50 – 60
Ans: (b) 30 – 40
12. The ratio of outer and inner perimeters of circular path is 23:22. If the path is 5 m wide, the
diameter of the inner circle is
(a) 55 m (b) 110 m (c) 220 m (d) 230 m
Ans: (c) 220 m
Let the radius of inner perimeter be r1 and the outer perimeter be r2

2r2 23 23
Now,   r2  r1
2r1 22 22
According to the question, r2  r1  5
23
 r1  r1  5  r1  110 & r2  115
22
∴ diameter of the inner circle = 2 x 110 = 220 m

Prepared by: M. S. KumarSwamy, TGT(Maths) Page - 2-


13. In ΔABC, right angled at B, AB = 5 cm and sin C = 1/2. Determine the length of side AC.
(a) 10 cm (b) 15 cm (c) 20 cm (d) none of these
Ans: (a) 10 cm
14. If x2 + k (4x + k -1) + 2 = 0 has equal roots, then k = ………
2 2 3 1 3 1
(a)  ,1 (b) , 1 (c) , (d) ,
3 3 2 3 2 3
2
Ans: (b) , 1
3
15. If x = a cos θ and y = b sin θ, then the value of b2x2 + a2y2 is
(a) a2 + b2 (b) a2/b2 (c) a2b2 (d) None of these
2 2
Ans: (c) a b
We have, b2x2 + a2y2 = b2 ×a2cos2θ + a2×b2sin2θ
= a2b2 (cos2θ + sin2θ) = a2b2 × 1 = a2b2

16. ABCD is a trapezium with AD ∥ BC and AD = 4cm. If the diagonals AC and BD intersect each
other at O such that AO/OC = DO/OB =1/2, then BC =
(a) 6cm (b) 7cm (c) 8cm (d) 9cm
Ans: (c) 8cm
17. The value of (sin 45° + cos 45°) is
(a) 1√2 (b) √2 (c) √3/2 (d) 1
Ans: (b) √2

18. Volumes of two spheres are in the ratio 64:27. The ratio of their surface areas is
(a) 3:4 (b) 4:3 (c) 9:16 (d) 16:9
Ans. (d) 16:9

Direction : In the question number 19 & 20 , A statement of Assertion (A) is followed by a


statement of Reason(R) . Choose the correct option

19. Statement A (Assertion): If product of two numbers is 5780 and their HCF is 17, then their LCM
is 340
Statement R( Reason) : HCF is always a factor of LCM
(a) Both Assertion (A) and Reason (R) are true and Reason (R) is the correct explanation of
Assertion (A).
(b) Both Assertion (A) and Reason (R) are true but Reason (R) is not the correct explanation of
Assertion (A).
(c) Assertion (A) is true but Reason (R) is false.
(d) Assertion (A) is false but Reason (R) is true.
Ans: (b) Both Assertion (A) and Reason (R) are true but Reason (R) is not the correct explanation
of Assertion (A).

20. Assertion (A): The point (0, 4) lies on y-axis.


Reason (R): The y co-ordinate of the point on x-axis is zero.
(a) Both Assertion (A) and Reason (R) are true and Reason (R) is the correct explanation of
Assertion (A)
(b) Both assertion (A) and reason (R) are true and reason (R) is not the correct explanation of
Assertion (A)
(c) Assertion (A) is true but reason(R) is false.
(d) Assertion (A) is false but reason(R) is true.
Ans: (b) Both assertion (A) and reason (R) are true and reason (R) is not the correct explanation of
Assertion (A)

Prepared by: M. S. KumarSwamy, TGT(Maths) Page - 3-


The x co-ordinate of the point (0, 4) is zero and y co-ordinate of the point on x-axis is zero.
∴ Point (0, 4) lies on y-axis.

SECTION-B
Questions 21 to 25 carry 2M each

21. If the system of equations 2x + 3y = 7 and (a + b)x + (2a – b)y = 21 has infinitely many solutions,
then find a and b.
Ans: Given system of equations
2x + 3y = 7 ...(i)
(a + b)x + (2 a – b) y = 21 ...(ii)
2 3 7 1 2 1
Equations have infinitely many solutions, if     
a  b 2a  b 21 3 a b 3
⇒ 6 = a + b ⇒ a + b = 6 ...(i)
3 1
and  ⇒ 2a – b = 9 ...(ii)
2a  b 3
On solving equation (i) and (ii), we get a = 5, b = 1

tan 2  cot 2 
22. Simplify: 
1  tan 2  1  cot 2 
tan 2  cot 2  tan 2  cot 2 
Ans:   
1  tan 2  1  cot 2  sec 2  cos ec 2
sin 2  cos2  cos 2  sin 2 
     sin 2   cos2   1
cos2  1 sin 2  1
OR
If 7 sin2A + 3 cos2A = 4, then find tan A
Ans: Given, 7sin2A + 3cos2A = 4
Dividing both sides by cos2A, we get
7 tan2A + 3 = 4 sec2A [∵ sec2θ = 1 + tan2θ ]
⇒ 7 tan2A + 3 = 4(1 + tan2A)
⇒ 7 tan2A + 3 = 4 + 4 tan2A
⇒ 3tan2A = 1 ⇒ tan2A = 1/3 ⇒ tan A = 1/√3
22
23. If the perimeter of a protractor is 72 cm, calculate its area. (Use π = )
7
36r 36r
Ans: ( r  2r )    72  r  14
7 7
 r 2 11
Now area of the protractor is  14 14  22  14  308cm 2
2 7
OR
Two circular pieces of equal radii and maximum area, touching each other are cut out from a
Rectangular card board of dimensions 14 cm × 7 cm. Find the area of the remaining card board.
[Use π = 22/7 ]
Ans: Here, r = 7/2 cm , l = 14 cm, b = 7 cm
Area of the remaining card board
= area of rectangle – 2 ( area of circle)
= l x b – 2 r2
= 14 x 7 – 2 x x x = 98 – 77 = 21 cm2

24. In the given figure below, AD/AE=AC/BD and ∠1=∠2. Show that Δ BAE~ ΔCAD .

Prepared by: M. S. KumarSwamy, TGT(Maths) Page - 4-


Ans: In ΔABC, ∠1 = ∠2
∴ AB = BD ………(i)
AD AC
Given, 
AE BD
AD AC
Using equation (i), we get  ……….(ii)
AE AB
AC AD
In ΔBAE and ΔCAD, by equation (ii), 
AB AE
and ∠A= ∠A (common)
∴ ΔBAE ~ ΔCAD [By SAS similarity criterion]

25. Find the length of the tangent from an external point P at a distance of 20 cm from the centre of a
circle of radius 12 cm.
Ans: Let PA be the tangent to the circle of point A.

Here, OP = 20 cm and OA = 12 cm
Since PA is tangent of A, therefore OA = PA.
In right angled triangle ∆OAB, OP2 = PA2 + OA2 (Using Pythagoras Theorem)
⇒ (20)2 = PA2 + (12)2 ⇒ PA2 = 400 –144 = 256
⇒ PA = 16 cm
Hence, Length of tangent be 16 cm.

SECTION-C
Questions 26 to 31 carry 3 marks each

26. Two numbers are in the ratio of 1 : 3. If 5 is added to both the numbers, the ratio becomes 1 : 2.
Find the numbers.
Ans: Let two numbers are x and y respectively such that its fraction = x/y.
x 1
According to the question, 
y 3
⇒ 3x = y ⇒ y = 3x...(i)
x 5 1
Also, 
y 5 2
⇒ 2x + 10 = y + 5 ⇒ 2x – y = –5 ...(ii)
Putting the value of y=3x in (ii), we get
2x – 3x = – 5 ⇒ – x = – 5 ⇒ x = 5

Prepared by: M. S. KumarSwamy, TGT(Maths) Page - 5-


Putting the value of x = 5 in (i), we get y = 3×5 = 15
Hence, Numbers are 5 and 15.

OR
A train covered a certain distance at a uniform speed. If the train would have been 6 km/h faster, it
would have taken 4 hours less than the scheduled time. And, if the train were slower by 6 km/hr; it
would have taken 6 hours more than the scheduled time. Find the length of the journey.
Ans: Let the actual speed of the train be x km/hr and let the actual time taken be y hours.
Distance covered is xy km If the speed is increased by 6 km/hr, then time of journey is reduced by 4
hours i.e., when speed is (x+6)km/hr, time of journey is (y−4) hours.
∴ Distance covered = (x + 6)(y − 4)
⇒ xy = (x + 6)(y − 4) ⇒ −4x + 6y − 24 = 0 ⇒ −2x + 3y −12 = 0 ………….(i)
Similarly xy = (x − 6)(y + 6) ⇒ 6x − 6y − 36 = 0 ⇒ x − y − 6 = 0 …………(ii)
Solving (i) and (ii) we get x=30 and y=24
Putting the values of x and y in equation (i), we obtain
Distance = (30 × 24)km =720km.
Hence, the length of the journey is 720km.

27. Given that √3 is irrational, prove that 2 + 5√3 is irrational.


Ans: Let us assume 2 + 5√3 is rational, then it must be in the form of p/q where p and q are co-
prime integers and q ≠0
i.e 2 + 5√3 = p/q
p  2q
So √3 = …(i)
5q
Since p, q, 5 and 2 are integers and q ≠ 0, RHS of equation (i) is rational.
But LHS of (i) is √3 which is irrational. This is not possible. This contradiction has arisen due to
our wrong assumption that 2 + 5√3 is
rational
So, 2 + 5√3 is irrational.

28. Two dice are thrown at the same time. What is the probability that the sum of the two numbers
appearing on the top of the dice is (i) 7? (ii) 14? (iii) equal to 12?
Ans: (i) P(sum of the numbers is 7) = 6/36 = 1/6
(ii) P(sum of the numbers is 14) = 0/36 = 0
(iii) P(sum of the numbers is 12) = 1/36

a a sin   b cos  a 2  b 2
29. If tan   , prove that 
b a sin   b cos  a 2  b 2
sin 
a b
a sin   b cos  cos  a tan   b
Ans: LHS   
a sin   b cos  a sin   b a tan   b
cos 
2 2
a a b
a b
a tan   b b a 2  b2
   2b 2  2  RHS
a tan   b a  a  b a  b a  b2
b b

30. If the zeroes of the polynomial x2 + px + q are double in value to the zeroes of 2x2 – 5x – 3, then
find the values of p and q
Ans:

Prepared by: M. S. KumarSwamy, TGT(Maths) Page - 6-


31. In the given figure, AB is a chord of length 8 cm of a circle of radius 5 cm. The tangents to the
circle at A and B intersect at P. Find the length of AP.

Ans: AB = 8 cm ⇒ AM = 4 cm
∴ OM = √(52 – 42) = 3 cm

Let AP = y cm, PM = x cm
∴ ∆OPA is a right angle triangle
∴ OP2 = OA2 + AP2
(x + 3)2 = y2 + 25
⇒ x2 + 9 + 6x = y2 + 25 ...(i)
Also, x2 + 42 = y2 ...(ii)
⇒ x2 + 6x + 9 = x2 + 16 + 25
⇒ 6x = 32
⇒ x = 32/6 = 16/3 cm
∴ y2 = x2 + 16 = 256/9 + 16 = 400/9
⇒ y = 20/3 cm
OR
Prove that opposite sides of a quadrilateral circumscribing a circle subtend supplementary angles at
the centre of the circle.
Ans: Let ABCD be the quadrilateral circumscribing a circle at the center O such that it touches the
circle at the point P,Q,R,S. Let join the vertices of the quadrilateral ABCD to the center of the circle

Prepared by: M. S. KumarSwamy, TGT(Maths) Page - 7-


In ΔOAP and ΔOAS
AP=AS ( Tangents from to same point A)
PO=OS ( Radii of the same circle)
OA=OA ( Common side)
so, ΔOAP=ΔOAS (SSS congruence criterion)
∴ ∠POA=∠AOS (CPCT)
⇒ ∠1=∠8
Similarly, ∠2=∠3, ∠4=∠5 and ∠6=∠7
∠1+∠2 +∠3+∠4+∠5+∠6+∠7+∠8 = 360⁰
⇒ (∠1 +∠8) +(∠2 +∠3) + (∠4 +∠5) + (∠6 +∠7) = 360⁰
⇒ 2(∠1) + 2(∠2) + 2(∠5) + 2(∠6) = 360⁰
⇒ (∠1) + (∠2) + (∠5) + (∠6) = 180⁰
∴ ∠AOD + ∠COD=180⁰
Similarly, ∠BOC + ∠DOA = 180⁰

SECTION-D
Questions 32 to 35 carry 5M each

32. A motorboat whose speed in still water is 9 km/h, goes 15km downstream and comes back to the
same spot, in a total time of 3 hours 45 minutes. Find the speed of the stream.
Ans: Let speed of stream be x km/h.
Given, Speed of boat = 9 km/h
Distance covered upstream = 15 km
Distance covered downstream = 15 km
Total time taken = 3 hours 45 minutes = 15/4 hours
Now, Speed of boat upstream = 9 − x km/h
Speed of boat downstream = 9 + x km/h
15 15 15
According to the question,  
9 x 9 x 4
1 1 1 9 x9 x 1 18 1
      
9 x 9 x 4 (9  x )(9  x) 4 (9  x )(9  x) 4
18 1
 2

81  x 4
⇒ 81 - x2 = 72
⇒ x2 = 9
⇒ x = 3 (x is the speed of the stream and thus cannot have negative value)
Thus, the speed of the stream is 3 km/hr.
OR
A takes 6 days less than the time taken by B to finish a piece of work. If both A And B together
can finish it in 4 days, find the time taken by B to finish the work.
Ans: Let B takes a total of x days to complete the work alone.
So as know that A takes 6 days less than B we can write that A takes
x - 6 days to complete the work alone.

Prepared by: M. S. KumarSwamy, TGT(Maths) Page - 8-


1
Work done by B in a day =
x
1
Work done by A in a day =
x6
1 1 1 x6 x 1 2x  6 1
According to the question,      
x x6 4 x ( x  6) 4 x( x  6) 4
⇒ 8x − 24 = x² −6x
⇒ x² −14x + 24 = 0
⇒ x² − 12x − 2x + 24 = 0
⇒ x(x − 12) – 2(x − 12) = 0
⇒ x = 2, 12
We will reject x = 2 as x − 6 will become negative.
Hence, B takes 12 days to complete the work alone.

33. Prove that if a line is drawn parallel to one side of a triangle intersecting the other two sides in
distinct points, then the other two sides are divided in the same ratio.
Using the above theorem prove that a line through the point of intersection of the diagonals and
parallel to the base of the trapezium divides the non parallel sides in the same ratio.
Ans: For the Theorem :
Given, To prove, Construction and figure of 1½ marks
Proof of 1½ marks
Let ABCD be a trapezium DC ∥ AB and EF is a line parallel to AB and hence to DC.
Join AC, meeting EF in G.

34. Ramesh made a bird-bath for his garden in the shape of a cylinder with a hemispherical depression
at one end. The height of the cylinder is 1.45 m and its radius is 30 cm. Find the total surface area
of the bird-bath.

Ans: Let h be height of the cylinder, and r the common radius of the cylinder and hemisphere.
Then, the total surface area = CSA of cylinder + CSA of hemisphere
= 2 rh + 2 r2 = 2 r (h + r)

Prepared by: M. S. KumarSwamy, TGT(Maths) Page - 9-


22
=2x x 30 (145 + 30) cm2
7
22
=2x x 30 x 175 cm2
7
= 33000 cm2 = 3.3 m2
OR
A tent is in shape of a cylinder surmounted by a conical top. If the height and diameter of the
cylindrical part are 2.1m and 4m respectively and the slant height of the top is 2.8m. Find the area
of canvas used for making the tent. Also find the cost of canvas of the tent at the rate of 500 per m2.
Ans: Radius = 2m, Slant height l= 2.8m , height h= 2.1m
Cost of canvas per m2= Rs.500
Area of canvas used = CSA of cone + CSA of cylinder
= πrl + 2πrh
=22/7 x 2 x 2.8 + 2 x 22/7 x 2 x 2.1
=17.6 + 26.4
=44m2
Cost of the canvas of tent =44 x 500
=Rs.22,000

35. A life insurance agent found the following data for distribution of ages of 100 policy holders.
Calculate the median age, if policies are given only to persons having age 18 years onwards but
less than 60 years.
Age (in years) Number of policy holders
Below 20 2
20 – 25 4
25 – 30 18
30 – 35 21
35 – 40 33
40 – 45 11
45 – 50 3
50 – 55 6
55 – 60 2
Ans:
Age (in years) Number of policy holders cf
Below 20 2 2
20 – 25 4 6
25 – 30 18 24
30 – 35 21 45
35 – 40 33 78
40 – 45 11 89
45 – 50 3 92
50 – 55 6 98
55 – 60 2 100
Here, n = 100 ⇒ n/2 = 50, therefore median class is 35 – 40
So, l = 35, cf = 45, f = 33, h = 5
n 
 2  cf   50  45   5 
Now, Median  l    h   35    5   35    5 
 f   33   33 
 
25
 35   35  0.76  35.76
33
Hence, median age is 35.76 years

Prepared by: M. S. KumarSwamy, TGT(Maths) Page - 10-


SECTION-E (Case Study Based Questions)
Questions 36 to 38 carry 4M each

36. Aditya is a fitness freak and great athlete. He always wants to make his nation proud by winning
medals and prizes in the athletic activities.

An upcoming activity for athletes was going to be organised by Railways. Aditya wants to
participate in 200 m race. He can currently run that distance in 51 seconds. But he wants to increase
his speed, so to do it in 31 seconds. With each day of practice, it takes him 2 seconds less.
(i) He wants to makes his best time as 31 sec. In how many days will be able to achieve his target?
(ii) What will be the difference between the time taken on 5th day and 7th day.
OR
(ii) Which term of the arithmetic progression 3, 15, 27, 39 .... will be 120 more than its 21st term?
Ans: Ans:
(i) Let, the number of days taken to achieve the target be n.
In the given A.P., a = 51, d = – 2
Since an = a + (n – 1)d ⇒ 31 = 51 + (n – 1)(– 2)
⇒ 31 – 51 = (n – 1)(– 2) ⇒ – 20 = (n – 1) (– 2)
⇒ (n – 1) = 10 ⇒ n = 11
Hence, 11 days are needed to achieve the target.
(ii) a5 = a + 4d = 51 + 4(– 2) = 51 – 8 = 43 sec
a7 = a + 6d = 51 + 6(– 2) = 51 – 12 = 39 sec
Now, time difference = 43 – 39 = 4 sec.
OR
(ii) We have, a = 3 and d = 12
∴ a21 = a + 20d = 3 + 20 × 12 = 243
Let nth term of the given AP be 120 more than its 21st term. Then, an = 120 + a21
⇒ 3 + (n – 1)d = 120 + 243
⇒ 3 + 12(n – 1) = 363 ⇒ 12(n – 1) = 360
⇒ n – 1 = 30 ⇒ n = 31
Hence, 31st term of the given AP is 120 more than its 21st term.

37. A 1.2 m tall girl spots a balloon moving with the wind in a horizontal line at a height of 88.2 m
from the ground. The angle of elevation of the balloon from the eyes of the girl at any instant is
60°. After 30 seconds, the angle of elevation reduces to 30° (see the below figure).

Prepared by: M. S. KumarSwamy, TGT(Maths) Page - 11-


Based on the above information, answer the following questions. (Take √3 =1.732)
(i) Find the distance travelled by the balloon during the interval. (2)
(ii) Find the speed of the balloon. (2)
OR
(ii) If the elevation of the sun at a given time is 30°, then find the length of the shadow cast by a
tower of 150 feet height at that time. (2)
Ans: (i) In the figure, let C be the position of the observer (the girl).
A and P are two positions of the balloon.
CD is the horizontal line from the eyes of the (observer) girl.
Here PD = AB = 88.2 m − 1.2 m = 87 m

Thus, the required distance between the two positions of the balloon = 58 √3 m
= 58 x 1.732 = 100.46 m (approx.)
(ii) Speed of the balloon = Distance/time = 100.46/30 = 3.35 m/s (approx.)

OR

Prepared by: M. S. KumarSwamy, TGT(Maths) Page - 12-


In right ∆ABC
AB 150 1
= tan 30°    BC  150 3 feet
BC BC 3
38. Resident Welfare Association (RWA) of a Gulmohar Society in Delhi have installed three electric
poles A, B and C in a society’s common park. Despite these three poles, some parts of the park are
still in dark. So, RWA decides to have one more electric pole D in the park. The park can be
modelled as a coordinate systems given below.

On the basis of the above information, answer any four of the following questions:
(i) What is the position of the pole C? (1)
(ii) What is the distance of the pole B from the corner O of the park? (1)
(iii) Find the position of the fourth pole D so that four points A, B, C and D form a parallelogram.
(2)
OR
(iii) What is the distance between poles A and C? (2)
Ans: (i) From the given diagram we can easily get that position of the pole C (5, 4).
(ii) Coordinates of B is (6, 6).
Distance from origin = (6  0) 2  (6  0) 2  36  36  72  6 2
(iii) If ABCD is a parallelogram, the diagonals bisects each other. Here AC and BD are diagonals.
 25 7 4
Mid-point of AC =  ,  = (3.5, 5.5)
 2 2 
Now, mid-point of diagonal, BD will be (3.5, 5.5) also.
Let, the coordinates of D be (x, y)
6 x 6 y
Now,  3.5,  5.5 ⇒ x = 1 and y = 5
2 2
OR
(iii) Coordinates of A are (2, 7) and coordinates of C are (5, 4).
Distance between pole A and C , AC = (5  2) 2  (4  7) 2  9  9  18  3 2

Prepared by: M. S. KumarSwamy, TGT(Maths) Page - 13-


PM SHRI KENDRIYA VIDYALAYA GACHIBOWLI, GPRA CAMPUS, HYD-32
SAMPLE PAPER TEST 04 FOR BOARD EXAM 2024
(ANSWERS)
SUBJECT: MATHEMATICS MAX. MARKS : 80
CLASS : X DURATION : 3 HRS
General Instruction:
1. This Question Paper has 5 Sections A-E.
2. Section A has 20 MCQs carrying 1 mark each.
3. Section B has 5 questions carrying 02 marks each.
4. Section C has 6 questions carrying 03 marks each.
5. Section D has 4 questions carrying 05 marks each.
6. Section E has 3 case based integrated units of assessment (04 marks each) with sub-parts of the
values of 1, 1 and 2 marks each respectively.
7. All Questions are compulsory. However, an internal choice in 2 Qs of 5 marks, 2 Qs of 3 marks and
2 Questions of 2 marks has been provided. An internal choice has been provided in the 2marks
questions of Section E
8. Draw neat figures wherever required. Take π =22/7 wherever required if not stated.
SECTION – A
Questions 1 to 20 carry 1 mark each.

1. The point on the x-axis which is equidistant from (– 4, 0) and (10, 0) is:
(a) (7, 0) (b) (5, 0) (c) (0, 0) (d) (3, 0)
Ans. (d) (3, 0)

2. If a cylinder is covered by two hemispheres shaped lid of equal shape, then the total curved surface
area of the new object will be
(a) 4πrh + 2πr2 (b) 4πrh – 2πr2 (c) 2πrh + 4πr2 (d) 2πrh + 4πr
Ans: (c) 2πrh + 4πr2
Curved surface area of cylinder = 2πrh
The curved surface area of hemisphere = 2πr2
Here, we have two hemispheres.
So, total curved surface area = 2πrh + 2(2πr2) = 2πrh + 4πr2

3. If the LCM of a and 18 is 36 and the HCF of a and 18 is 2, then a =


(a) 1 (b) 2 (c) 3 (d) 4
Ans. (d) 4

4. The sum of exponents of prime factors in the prime-factorisation of 196 is:


(a) 3 (b) 4 (c) 5 (d) 6
Ans. (b) 4

5. The values of k for which the quadratic equation 2x2 – kx + k = 0 has equal roots is
(a) 0 only (b) 8 only (c) 0,8 (d) 4
Ans: (c) 0,8
For equal roots, D = b2 – 4ac = 0
⇒ (-k)2 – 4(2)(k) = 0
⇒ k2 – 8k = 0 ⇒ k(k – 8) = 0 ⇒ k= 0,8

6. A number x is chosen at random from the numbers -3, -2, -1, 0, 1, 2, 3 the probability that |x| < 2 is
(a) 1/7 (b) 2/7 (c) 3/7 (d) 5/7
Ans: (c) 3/7

Prepared by: M. S. KumarSwamy, TGT(Maths) Page - 1-


Total possible number of events (n) = 7
Now for |x| < 2, possible values of x = -1, 0, 1
∴ Required probability = 3/7

7. If x = 2sin2θ and y = 2cos2θ + 1 then x + y is:


(a) 3 (b) 2 (c) 1 (d) 1/2
Ans: (a) 3

8. If ½ is a root of the equation x2 + kx – 5/4 = 0, then the value of k is


(a) 2 (b) – 2 (c) ¼ (d) ½
Ans: (a) 2
If ½ is a root of the equation
x2 + kx – 5/4 = 0 then, substituting the value of ½ in place of x should give us the value of k.
Given, x2 + kx – 5/4 = 0 where, x = ½
(½)2 + k (½) – (5/4) = 0
⇒ (k/2) = (5/4) – ¼ ⇒ k = 2

9. The pair of equations x + 2y + 5 = 0 and –3x – 6y + 1 = 0 have


(a) a unique solution (b) exactly two solutions
(c) infinitely many solutions (d) no solution
Ans: (d) no solution
a1 = 1; b1 = 2; c1 = 5
a2 = -3; b2 = -6; c2 = 1
a1/a2 = -1/3
b1/b2 = -2/6 = -1/3
c1/c2 = 5/1 = 5
Here, a1/a2 = b1/b2 ≠ c1/c2
Therefore, the pair of equation has no solution.

10. The point which lies on the perpendicular bisector of the line segment joining point A (–2, –5) and
B (2, 5) is:
(a) (0, 0) (b) (0, –1) (c) (–1, 0) (d) (1, 0)
Ans. (a) (0, 0)

11. A card is selected at random from a well shuffled deck of 52 playing cards. The probability of its
being a face card is
(a) 3/13 (b) 4/13 (c) 6/13 (d) 9/13
Ans: (a) 3/13
Total number of outcomes = 52
Number of face cards = 12
The probability of its being a face card = 12/52 = 3/13

12. The ratio in which the line segment joining the points P(-3, 10) and Q(6, –8) is divided by O(-1, 6)
is:
(a) 1:3 (b) 3:4 (c) 2:7 (d) 2:5
Ans: (c) 2:7
Let k :1 be the ratio in which the line segment joining P( -3, 10) and Q(6, -8) is divided by point O(-
1, 6).
By the section formula, we have -1 = ( 6k – 3)/(k + 1)
⇒ –k – 1 = 6k – 3
⇒ 7k = 2 ⇒ k = 2/7
Hence, the required ratio is 2:7.

Prepared by: M. S. KumarSwamy, TGT(Maths) Page - 2-


13. A box contains cards numbered 6 to 50. A card is drawn at random from the box. The probability
that the drawn card has a number which is a perfect square is :
(a) 1/45 (b) 2/15 (c) 4/45 (d) 1/9
Ans. (d) 1/9
P(perfect Square)= 5/45 = 1/9

14. In a circle of diameter 42cm, if an arc subtends an angle of 60º at the centre, then the length of the
arc is:
(a) 22/7 cm (b) 11cm (c) 22 cm (d) 44 cm
Ans: (c) 22 cm

15. If the lines 3x + 2ky – 2 = 0 and 2x + 5y + 1 = 0 are parallel, then what is the value of k?
(a) 4/15 (b) 15/4 (c) ⅘ (d) 5/4
Ans: (b) 15/4
The condition for parallel lines is a1/a2 = b1/b2 ≠ c1/c2
Hence, 3/2 = 2k/5
⇒ k = 15/4

16. For the following distribution:


Marks Below 10 Below 20 Below 30 Below 40 Below 50 Below 60
No. of students 3 12 27 57 75 80
the modal class is
(a) 10-20 (b) 20-30 (c) 30-40 (d) 50-60
Ans: (c) 30-40

17. The distance of the point P (2, 3) from the x-axis is


(a) 2 (b) 3 (c) 1 (d) 5
Ans: (b) 3
We know that, (x, y) is a point on the Cartesian plane in first quadrant.
Then, x = Perpendicular distance from Y – axis and
y = Perpendicular distance from X – axis
Therefore, the perpendicular distance from X-axis = y coordinate = 3

18. A circus artist is climbing a 30 m long rope, which is tightly stretched and tied from the top of a
vertical pole to the ground. Find the distance of the pole to the peg in the ground, if the angle made
by the rope with the ground level is 30⁰.
(a) 20√3 m (b)15√3 m (c)10√3 m (d) 20 m
Ans: (b)15√3 m

BC 3 AC 30 3
cos 30   BC  cos 30  AC    15 3 m
AC 2 2

Direction : In the question number 19 & 20 , A statement of Assertion (A) is followed by a


statement of Reason(R) . Choose the correct option

19. Assertion (A): The largest number that divide 70 and125 which leaves remainder 5 and 8 is 13
Reason (R): HCF (65,117) =13
(a) Both A and R are true and R is the correct explanation of A

Prepared by: M. S. KumarSwamy, TGT(Maths) Page - 3-


(b) Both A and R are true but R is not the correct explanation of A
(c) A is true and R is false
(d) A is false and R is true
Ans: (a) Both A and R are true and R is the correct explanation of A

20. Assertion (A): In ∆ABC, DE || BC such that AD = (7x - 4) cm, AE = (5x - 2) cm, DB = (3x + 4)
cm and EC = 3x cm than x equal to 5.
Reason (R): If a line is drawn parallel to one side of a triangle to intersect the other two sides in
distant point, than the other two sides are divided in the same ratio.
(a) Both Assertion (A) and Reason (R) are true and Reason (R) is the correct explanation of
Assertion (A).
(b) Both Assertion (A) and Reason (R) are true but Reason (R) is not the correct explanation of
Assertion (A).
(c) Assertion (A) is true but Reason (R) is false.
(d) Assertion (A) is false but Reason (R) is true.
Ans: (d) Assertion (A) is false but Reason (R) is true.
AD AE 7 x  4 5x  2
  
DB EC 3x  4 3x
⇒ 21x2 - 12x = 15x2 + 20x – 6x – 8
⇒ 6x2 – 26 + 8x = 0 ⇒ 3x2 – 13x + 4 =0
⇒ 3x2 – 12x - x + 4 = 0 ⇒ 3x(x – 4) – 1(x – 4) = 0
⇒ (x – 4)(3x – 1) = 0 ⇒ x = 4, 1/3
Neglecting x = 1/3 as AD will become negative, we have x = 4
So, A is false but R is true.

SECTION-B
Questions 21 to 25 carry 2M each

21. Find the value of m for which the pair of linear equations:
2x + 3y – 7 = 0 and (m – 1) x + (m + 1) y = (3m – 1) has infinitely many solutions
Ans: For infinitely many solutions the condition is
a1 b1 c1 2 3 7
    
a2 b2 c2 m  1 m  1 3m  1
Now, 2(m + 1) = 3(m – 1) ⇒ m = 5
and 3(3m – 1) = 7(m + 1) ⇒ m = 5
Hence, for m = 5, the system has infinitely many solutions.

22. Find the zeroes of the quadratic polynomials p(t) = 5t2 + 12t + 7 and verify the relationship between
the zeroes and the coefficients.
Ans: 5t2 + 12t + 7 = 0 ⇒ 5t2 + 5t + 7t + 7 = 0
⇒ 5t (t + 1) + 7(t + 1) = 0 ⇒ (t + 1)(5t + 7) = 0
⇒ t + 1 = 0 ⇒ t = -1
5t + 7 = 0 ⇒ 5t = -7 ⇒ t = -7/5
Therefore, zeroes are (-7/5) and -1
Now, Sum of the zeroes = – (coefficient of x) ÷ coefficient of x2
α + β = – b/a
⇒ (- 1) + (- 7/5) = – (12)/5
⇒ – 12/5 = – 12/5
Product of the zeroes = constant term ÷ coefficient of x2
α β = c/a
⇒ (- 1)(- 7/5) = 7/5
⇒ 7/5 = 7/5

Prepared by: M. S. KumarSwamy, TGT(Maths) Page - 4-


23. Two dice are thrown at the same time. Find the probability of getting (i) same number on both dice
(ii) different numbers on both dice.
Ans: Total number of possible outcomes = 36
(i) Same number on both dice.
Number of possible outcomes = 6
Therefore, the probability of getting same number on both dice = 6/36 = 1/6
(ii) Different number on both dice.
Number of possible outcomes = 36 – 6 = 30
Therefore, the probability of getting different number on both dice = 30/36 = 5/6
OR
Cards marked with number 3, 4, 5, …, 50 are placed in a box and mixed thoroughly. A card is
drawn at random from the box. Find the probability that the selected card bears (i) a perfect square
number (ii) a single digit number
Ans: Total number of cards = 48
(i) Total number of perfect squares = 6
∴ Required Probability = 6/48 = 1/8
(ii) Total single digit numbers = 7
∴ Required Probability = 7/48

24. A quadrilateral ABCD is drawn to circumscribe a circle. Prove that AB + CD = AD + BC.


Ans: We know that the lengths of tangents drawn from an exterior point to a circle are equal.

AP = AS ... (i) [tangents from A]


BP = BQ ... (ii) [tangents from B]
CR = CQ ... (iii) [tangents from C]
DR = DS ... (iv) [tangents from D]
AB + CD = (AP + BP) + (CR + DR)
= (AS + BQ) + (CQ + DS) [using (1), (ii), (iii), (’v)]
= (AS + DS) + (BQ + CQ)
=AD+ BC.
Hence, AB + CD = AD + BC.

25. Find the points on the x–axis which are at a distance of 2√5 from the point (7, –4). How many such
points are there?
Ans: Let coordinates of the point=(x, 0)(given that the point lies on x axis)
x1 = 7, y1=-4and x2 = x, y2=0
Distance =√(x2 – x1)2 + (y2 – y1)2
According to the question, 2√5=√(x – 7)2+ (0 – 4(-4))2
Squaring L.H.S and R.H.S, we get 20 = x2 + 49 – 14x + 16
⇒ 20=x2 + 65 – 14x ⇒ x2 – 14x + 45 = 0 ⇒ x2 – 9x – 5x + 45 = 0
⇒ x(x – 9) – 5(x – 9) = 0 ⇒ (x – 9) (x – 5) = 0 ⇒ x – 9 = 0, x – 5 = 0
⇒ x = 9 or x = 5
Therefore, coordinates of points (9, 0) or (5, 0)
OR

Prepared by: M. S. KumarSwamy, TGT(Maths) Page - 5-


3
If A and B are (-2, -2) and (2, -4) respectively, find the coordinates of P such that AP = AB and P
7
lies on the line segment AB.
3 4
Ans: Given that AP = AB ⇒ PB = AB
7 7
Therefore, Point P divides AB internally in the ratio 3 : 4
Using section formula, we get
 3  (2)  4  (2) 3  (4)  4  (2)   2 20 
Coordinates of P   ,  , 
 3 4 3 4   7 7 

SECTION-C
Questions 26 to 31 carry 3 marks each

26. On a morning walk, three persons step off together and their steps measure 40 cm, 42 cm and 45
cm, respectively. Find the minimum distance each should walk so that each can cover the same
distance in complete steps.
Ans: Step measures of three persons are 40 cm, 42 cm and 45 cm.
The minimum distance each should walk so that each can cover the same distance in complete steps
is the LCM of 40 cm, 42 cm and 45 cm.
Prime factorisation of 40, 42 and 45 gives
40 = 23 × 5, 42 = 2 × 3 × 7, 45 = 32 × 5
LCM (40, 42, 45) = Product of the greatest power of each prime factor involved in the numbers
= 23 × 32 × 5 × 7 = 8 × 9 × 35 = 72 × 35 = 2520 cm.
OR
Show that 5 + 2√7 is an irrational number, where √7 is given to be an irrational number.
Ans: Let 5 + 2√7 is a rational number such that
5 + 2√7 = a, where a is a rational number
a 5
⇒ 2√7 = a – 5 ⇒ 7 
2
a 5
Since a is a rational number and 2, 5 are integers, therefore is a rational number
2
⇒ √7 is a rational number which contradicts the fact that √7 is an irrational number
Therefore, our assumption is wrong
Hence, 5 + 2√7 is an irrational number

27. From a point on a ground, the angle of elevation of bottom and top of a transmission tower fixed on
the top of a 20 m high building are 45° and 60° respectively. Find the height of the tower.
Ans: Let the height of the building is BC, the height of the transmission tower which is fixed at the
top of the building be AB.

Prepared by: M. S. KumarSwamy, TGT(Maths) Page - 6-


D is the point on the ground from where the angles of elevation of the bottom B and the top A of
the transmission tower AB are 45° and 60° respectively.
The distance of the point of observation D from the base of the building C is CD.
Combined height of the building and tower = AC = AB + BC
In ΔBCD, tan 45° = BC/CD
⇒ 1 = 20/CD
⇒ CD = 20
In ΔACD, tan 60° = AC/CD
⇒ √3 = AC/20
⇒ AC = 20√3
Height of the tower, AB = AC - BC
⇒ AB = 20√3 - 20 m = 20 (√3 - 1) m

28. In the below figure, if ∠1 =∠2 and ΔNSQ = ΔMTR, then prove that ΔPTS ~ ΔPRQ.

Ans: According to the question, ∆ NSQ ≅ ∆MTR and ∠1 = ∠2


Since, ∆NSQ = ∆MTR
So, SQ = TR ….(i)
Also, ∠1 = ∠2 ⇒ PT = PS….(ii) [sides opposite to equal angles]
From Equation (i) and (ii),
PS/SQ = PT/TR
⇒ ST || QR (By converse of basic proportionality theorem)
∴ ∠1 = ∠PQR and ∠2 = ∠PRQ (corresponding angles)
In ∆PTS and ∆PRQ.
∠P = ∠P [Common angles]
∠1 = ∠PQR (proved)
∠2 = ∠PRQ (proved)
∴ ∆PTS – ∆PRQ [By AAA similarity criteria]
Hence proved

p2 1
29. If cosecθ + cotθ = p, then prove that cos  
p2 1
Ans: Given cosecθ + cotθ = p ….. (1)
 (cosec  cot  )(cosec  cot  )  1  (cosec  cot  ) p  1
1
 cos ec  cot   ……. (2)
p
Adding (1) and (2), we get
1 1
p 2 p
p p 1 p p2 1
cosec   ;cot   
2 2p 2 2p

Prepared by: M. S. KumarSwamy, TGT(Maths) Page - 7-


p2 1
cot  2p p2 1
Now, cos    2 
cos ec p  1 p2  1
2p

30. If 2x + y = 23 and 4x – y = 19, find the values of 5y – 2x and y/x – 2.


Ans: Given equations are 2x + y = 23 …(i)
4x – y = 19 …(ii)
On adding both equations, we get 6x = 42
⇒x=7
Put the value of x in Eq. (i), we get
2(7) + y = 23
⇒ y = 23 – 14
⇒y=9
Hence 5y – 2x = 5(9) – 2(7) = 45 – 14 = 31
y/x – 2 = 9/7 -2 = -5/7

31. In the given figure, OP is equal to diameter of the circle. Prove that ABP is an equilateral triangle.

Ans: Join OP and let it meets the circle at point Q.


Since OP = 2r (Diameter of the circle)
⇒ OQ = QP = r
Consider ΔAOP in which OA ⊥ AP and OP is the hypotenuse.
∴ OQ = AQ = OA
(Mid-point of the hypotenuse is equidistant from the vertices)
⇒ OAQ is an equilateral triangle.
⇒ ∠AOQ = 60° (Each angle of an equilateral triangle is 60°)

Consider right-angled triangle OAP.


∠AOQ = 60° (Proved above)
∠OAP = 90° ⇒ ∠APO = 30°
∠APB = 2∠APO = 2 × 30° = 60°
Also PA = PB (Tangents to a circle from an external point are equal.)
⇒ ∠PAB = ∠PBA (Angles opposite to equal sides in ΔPAB)
In ΔABP, ∠APB = 60°
1800  600
⇒ ∠PAB = ∠PBA =  600
2
⇒ Each angle of DPAB is 60°

Prepared by: M. S. KumarSwamy, TGT(Maths) Page - 8-


⇒ PAB is an equilateral triangle.
OR
A circle is inscribed in a ΔABC having sides 8 cm, 10 cm and 12 cm as shown in the following
figure. Find AD, BE and CF.

Ans: Let AD = x1, BE = x2 and CF = x3;


then AF = AD = x1, BD = BE = x2
and CE = CF = x3.
∴ x1 + x2 = 12; x2 + x3 = 8; x1 + x3 = 10 (1)
Adding,
2(x1 + x2 + x3) = 30
⇒ x1 + x2 + x3 = 15
Solve for x1, x2 and x3 to get
AD = 7 cm, BE = 5 cm, CF = 3 cm

SECTION-D
Questions 32 to 35 carry 5M each

1
32. Two pipes running together can fill a cistern in 3 hours. If one pipe takes 3 hours more than the
13
other to fill it, find the time in which each pipe would fill the cistern.
Ans: Let time taken by faster pipe to fill the cistern be x hrs.
Therefore, time taken by slower pipe to fill the cistern = (x + 3) hrs
Since the faster pipe takes x minutes to fill the cistern.
1
∴ Portion of the cistern filled by the faster pipe in one hour =
x
1
Portion of the cistern filled by the slower pipe in one hour =
x3
1 13
Portion of the cistern filled by the two pipes together in one hour = 
40 40
13
1 1 13 x  3  x 13
According to question,    
x x  3 40 x( x  3) 40
⇒ 40 (2x + 3) = 13x (x + 3) ⇒ 80x + 120 = 13x2 + 39x
⇒ 13x2 – 41x – 120 = 0 ⇒ 13x2 – 65x + 24x – 120 = 0
⇒ 13x (x – 5) + 24 (x – 5) = 0 ⇒ (x – 5) (13x + 24) = 0
Either x – 5 = 0 or 13x + 24 = 0
⇒ x = 5 as x = −24/13 not possible.
Hence, the time taken by the two pipes is 5 hours and 8 hours respectively.
OR
If Zeba was younger by 5 years than what she really is, then the square of her age (in years) would
have been 11 more than five times her actual age. What is her age now? [NCERT Exemplar]
Ans: Let the present age of Zeba be x years.
Age before 5 years = (x – 5) years
According to given condition, (x – 5)2 = 5x + 11

Prepared by: M. S. KumarSwamy, TGT(Maths) Page - 9-


⇒ x2 + 25 – 10x = 5x + 11 ⇒ x2 – 10x – 5x + 25 – 11 = 0
⇒ x2 – 15x + 14 = 0 ⇒ x2 – 14x – x + 14 = 0
⇒ x (x – 14) – 1 (x – 14) = 0 ⇒ (x – 1) (x – 14) = 0
⇒ x – 1 = 0 or x – 14 = 0
⇒ x = 1 or x = 14
But present age cannot be 1 year.
Hence, Present age of Zeba is 14 years.

33. State and prove Basic Proportional Theorem.


Ans: Statement – 1 mark
Given, To Prove, Construction and Figure – 2 marks
Correct Proof – 2 marks

34. A survey regarding the heights (in cm) of 50 girls of class Xth of a school was conducted and the
following data was obtained. Find the mean, median and mode of the given data.
Heights (in cm) 120 – 130 130 – 140 140 – 150 150 – 160 160 – 170
No. of Girls 2 8 12 20 8
Ans:

35. A chord of a circle of radius 15 cm subtends an angle of 60° at the centre. Find the areas of the
corresponding minor and major segments of the circle. (Use π = 3.14 and √3 = 1.73)
Ans: Here, O is the centre of circle, AB is a chord
AXB is a major arc, OA = OB = radius = 15 cm
Arc AXB subtends an angle 60⁰ at O.
60 60
Area of sector AOB    r2   3.14  (15) 2  117.75 cm 2
360 360

Prepared by: M. S. KumarSwamy, TGT(Maths) Page - 10-


Area of minor segment (Area of Shaded region) = Area of sector AOB − Area of △ AOB
By trigonometry, AC = 15sin30⁰ and OC = 15cos30⁰
Also, AB = 2AC
∴ AB = 2 × 15sin30⁰ = 15 cm
3 1.73
 OC  15cos 300  15  15   12.975
2 2
∴ Area of △AOB = 0.5×15×12.975 = 97.3125cm2
∴ Area of minor segment (Area of Shaded region) =117.75 − 97.3125 = 20.4375 cm2
Area of major segment = Area of circle − Area of minor segment
= (3.14×15×15) − 20.4375 = 686.0625 cm2
OR
PQRS is a diameter of a circle of radius 6 cm. The lengths PQ, QR and RS are equal. Semi-circles
are drawn on PQ and QS as diameters as shown in below figure. Find the perimeter and area of the
shaded region

Ans: Here, PS = 12 cm
1 1
as PQ = QR = RS = × PS = ×12 = 4 cm
3 3
and QS = 2PQ ⇒ QS = 2 × 4 = 8 cm
Area of shaded region: A = area of a semicircle with PS as diameter + area of a semicircle with PQ
as diameter – the area of a semicircle with QS as diameter;
1
= [ 3.14 × 6² + 3.14 × 2² - 3.14 × 4² ]
2
1
= [ 3.14 ×36 + 3.14 ×4 – 3.14 ×16 ]
2
1
= [ 3.14 ( 36 + 4 – 16)]
2
1 1
= ( 3.14 × 24 ) = × 75.36 = 37.68 cm²
2 2
The area of shaded region = 37.68 cm².
The perimeter of the shaded region = Arc of the semicircle of radius 6 +Arc of the semicircle of
radius 4 + Arc of the semicircle of radius 2
= (6π + 4π + 2π) = 12π
22 264
= 12 × = = 37.71 cm
7 7

SECTION-E (Case Study Based Questions)


Questions 36 to 38 carry 4M each

36. In a toys manufacturing company, wooden parts are assembled and painted to prepare a toy. One
specific toy is in the shape of a cone mounted on a cylinder. For the wood processing activity
center, the wood is taken out of storage to be sawed, after which it undergoes rough polishing, then

Prepared by: M. S. KumarSwamy, TGT(Maths) Page - 11-


is cut, drilled and has holes punched in it. It is then fine polished using sandpaper. For the retail
packaging and delivery activity center, the polished wood sub-parts are assembled together, then
decorated using paint. The total height of the toy is 26 cm and the height of its conical part is 6 cm.
The diameters of the base of the conical part is 5 cm and that of the cylindrical part is 3 cm. On the
basis of the above information, answer the following questions:

(a) If its cylindrical part is to be painted yellow, find the surface area need to be painted. [1]
(b) If its conical part is to be painted green, find the surface area need to be painted. [2]
OR
(b) Find the volume of the wood used in making this toy. [2]
(c) If the cost of painting the toy is 3 paise per sq cm, then find the cost of painting the toy. (Use π =
3.14) [1]
Ans: Let the radius of cone be r, slant height of cone be l, height of cone be h, radius of cylinder be
r′ and height of cylinder be h′.
Then r = 2.5 cm, h = 6 cm, r′ = 1.5 cm, h′ = 26 – 6 = 20 cm and
Slant height, l  r 2  h 2  2.52  62  6.25  36  42.25  6.5cm
(a) Area to be painted yellow = CSA of the cylinder + area of one base of the cylinder
= 2πr′h′ + π(r′)2 = πr′ (2h′ + r′) = (3.14 × 1.5) (2 × 20 + 1.5) cm2
= 4.71 × 41.5 cm2
= 195.465 cm2
(b) Area to be painted green = CSA of the cone + base area of the cone – base area of the cylinder
= πrl + πr2 – π(r′)2 = π[(2.5 × 6.5) + (2.5)2 – (1.5)2] cm2
= π[20.25] cm2 = 3.14 × 20.25 cm2
= 63.585 cm2
OR
Volume of wood used in making the toy = Volume of cone + Volume of cylinder
1 1  1 
=  r 2 h   r '2 h '    r 2 h  r '2 h '  3.14   2.5  2.5  6  1.5  1.5  20 
3 3  3 
3
 3.14(12.5  45)  180.55cm
(c) Total area of painting = 195.465 + 63.585 = 259.05 cm2
Cost of painting 1 cm2 = Re. 0.03
Total cost of painting = Rs. 0.03 x 256.05
= Rs. 7.77

37. Radio towers are used for transmitting a range of communication services including radio and
television. The tower will either act as an antenna itself or support one or more antennas on its
structure, including microwave dishes. They are among the tallest human-made structures. There

Prepared by: M. S. KumarSwamy, TGT(Maths) Page - 12-


are 2 main types: guyed and self-supporting structures. On a similar concept, a radio station tower
was built in two sections A and B.
Tower is supported by wires from a point O. Distance between the base of the tower and point O is
36 m. From point O, the angle of elevation of the top of section B is 30° and the angle of elevation
of the top of section A is 45°.

(i) What is the height of the section B? (1)


(ii) What is the height of the section A? (1)
(iii) What is the length of the wire structure from the point O to the top of section A? (2)
OR
(iii) What is the length of the wire structure from the point O to the top of section B? (2)
Ans: Given, that the distance between the base of the tower and point O = 36 m
BC
(i)Consider ΔOCB, tan 300 = ⇒ =
OC
Hence, BC= 12√3 = 20.78 m
AB  BC
(ii)In ΔOAC, tan 450 = ⇒ = 1 ⇒ AC = 36m
OC
∴ Height of section A = 36 – 12√3 = 12(3 – √3) m
(iii) length of the wire structure from the point O to the top of the section
cos 450 = ⇒ OA = 36√2M
OR
length of the wire structure from the point O to the top of the section
cos 300 = ⇒ = ⇒ OB = 72 / √3 = 24√3m

38. Mohan is an auto driver. His autorickshaw was too old and he had to spend a lot of money on repair
and maintenance every now and then. One day he got to know about the EV scheme of the
Government of India where he can not only get a good exchange bonus but also avail heavy
discounts on the purchase of an electric vehicle. So, he took a loan of 71,18,000 from a reputed
bank and purchased a new autorickshaw.
Mohan repays his total loan of 118000 rupees by paying every month starting with the first
instalment of 1000 rupees.

Prepared by: M. S. KumarSwamy, TGT(Maths) Page - 13-


(i) If he increases the instalment by 100 rupees every month, then what amount will be paid by him
in the 30th instalment? [1]
(ii) If he increases the instalment by 100 rupees every month, then what amount of loan does he still
have to pay after 30th instalment? [2]
OR
(ii)If he increases the instalment by 200 rupees every month, then what amount would he pay in
40th instalment? [2]
(iii) If he increases the instalment by 100 rupees every month, then what amount will be paid by
him in the 100th instalment [1]
Ans: (i) Clearly, the amount of installment in the first month = Rs. 1000, which increases by Rs.
100 every month
therefore, installment amount in second month = Rs. 1100, third month = Rs. 1200, fourth month =
Rs. 1300 .....
which forms an AP, with first term, a = 1000 and common difference, d = 1100 - 1000 = 100
Now, amount paid in the 30th installment,
a30 = 1000 + (30 - 1)100 = 3900 [∵ an = a + (n - 1)d]
(ii) Amount paid in 30 instalments,
30
S30 = [2 x 1000 + (30 - 1)100] = 73500
2
Hence, remaining amount of loan that he has to pay = 118000 - 73500 = Rs. 44500
OR
If he increases the instalment by 200 rupees every month, amount would he pay in 40th instalment
Then a = 1000, d = 200 and n= 40
a40 = a + 39d
⇒ a40 = 1000 + 39(200)
⇒ a40 = 880
(iii) Here, a = 1000 and common difference, d = Rs. 100
Amount paid in the 100th instalments
a100 = 1000 + (100 - 1)100 = 10900 [∵ an = a + (n - 1)d]

Prepared by: M. S. KumarSwamy, TGT(Maths) Page - 14-


PM SHRI KENDRIYA VIDYALAYA GACHIBOWLI, GPRA CAMPUS, HYD-32
SAMPLE PAPER TEST 05 FOR BOARD EXAM 2024
(ANSWERS)
SUBJECT: MATHEMATICS MAX. MARKS : 80
CLASS : X DURATION : 3 HRS
General Instruction:
1. This Question Paper has 5 Sections A-E.
2. Section A has 20 MCQs carrying 1 mark each.
3. Section B has 5 questions carrying 02 marks each.
4. Section C has 6 questions carrying 03 marks each.
5. Section D has 4 questions carrying 05 marks each.
6. Section E has 3 case based integrated units of assessment (04 marks each) with sub-parts of the
values of 1, 1 and 2 marks each respectively.
7. All Questions are compulsory. However, an internal choice in 2 Qs of 5 marks, 2 Qs of 3 marks
and 2 Questions of 2 marks has been provided. An internal choice has been provided in the 2marks
questions of Section E
8. Draw neat figures wherever required. Take π =22/7 wherever required if not stated.
SECTION – A
Questions 1 to 20 carry 1 mark each.

1. Three cubes each of side 15 cm are joined end to end. The total surface area of the cuboid is:
(a) 3150 cm2 (b) 1575 cm2 (c) 1012.5 cm2 (d) 576.4 cm2
2
Ans. (a) 3150 cm

2. The midpoint of a line segment joining two points A(2, 4) and B(-2, -4) is
(a) (-2, 4) (b) (2, -4) (c) (0, 0) (d) (-2, -4)
Ans: (c) (0, 0)
As per midpoint formula, we know;
x-coordinate of the midpoint = [2 + (-2)]/2 = 0/2 = 0
y-coordinate of the midpoint = [4 + (-4)]/2=0/2=0
Hence, (0, 0) is the midpoint of AB.

3. If the distance between the points A(2, -2) and B(-1, x) is equal to 5, then the value of x is:
(a) 2 (b) -2 (c) 1 (d) -1
Ans: (a) 2
( x2  x1 ) 2  ( y2  y1 ) 2  5  (1  2) 2  ( x  2) 2  5  9  ( x  2) 2  5
⇒ 9 + (x + 2)2 = 25
⇒ (2 + x)2 = 16 ⇒ 2 + x = 4 ⇒ x = 2

4. If cos A = 4/5, then the value of tan A is


(a) 3/5 (b) 3/4 (c) 4/3 (d) 5/3
Ans: (b) 3/4

5. If cos θ + cos2 θ = 1, the value of sin2 θ + sin4 θ is :


(a) –1 (b) 0 (c) 1 (d) 2
Ans: (c) 1

6. The HCF and the LCM of 12, 21, 15 respectively are


(a) 3, 140 (b) 12, 420 (c) 3, 420 (d) 420, 3
Ans: (c) 3, 420
Here, 12 = 22 × 3

Prepared by: M. S. KumarSwamy, TGT(Maths) Page - 1 -


21 = 3 × 7
15 = 3 × 5
So, HCF = 3; LCM = 22 × 3 × 7 i.e., 420

7. If the sum of LCM and HCF of two numbers is 1260 and their LCM is 900 more than their HCF,
then the product of two numbers is
(a) 205400 (b) 203400 (c) 194400 (d) 198400
Ans: (c) 194400
Let the HCF of the numbers be x and their LCM be y.
It is given that the sum of the HCF and LCM is 1260, therefore
x + y = 1260 ....(i)
And, LCM is 900 more than HCF.
y = x + 900..... (ii)
Substituting (ii) in (i), we get x + x + 900 =1260
⇒ 2x + 900 = 1260
⇒ 2x = 1260 - 900 ⇒ 2x = 360 ⇒ x = 180
Substituting x = 180 in (1), we get:
y = 180 + 900 ⇒ y = 1080
We also know that the product the two numbers is equal to the product of their LCM and HCF
Thus, product of the numbers = 1080(180) = 194400

8. If the zeroes of the quadratic polynomial x2 + (a + 1)x + b are 2 and -3, then
(a) a = -7, b = -1 (b) a = 5, b = -1 (c) a = 2, b = –6 (d) a = 0, b = –6
Ans. (d) a = 0, b = –6

9. In the given figure, from an external point P, two tangents PQ and PR are drawn to a circle of
radius 4 cm with centre O. If ∠QPR = 90°, then length of PQ is

(a) 3 cm (b) 4 cm (c) 2 cm (d) 2.2 cm


Ans: (b) 4 cm
10. In the given figure, quadrilateral ABCD is circumscribed, touching the circle at P, Q, R and S
such that ∠DAB = 90°, If CR = 23 cm and CB = 39 cm and the radius of the circle is 14 cm, then
the measure of AB is

(a) 37 cm (b) 16cm (c) 30 cm (d) 39 cm


Ans: (c) 30 cm
∵ Tangent is perpendicular to the radius through the point of contact.
∠OQA = ∠OPA = 90° and OQ = OP [Radii]
∴ OQAP is a square.

Prepared by: M. S. KumarSwamy, TGT(Maths) Page - 2 -


⇒ AP = 14cm
Now, CR = CS = 23 cm [Tangents from an external point to a circle are equal]
∴ BS = 39 - 23 = 16cm
And BS = BP = 16 cm [Tangents from an external point to a circle are equal]
Now, AB = AP + BP = 14 + 16 = 30cm

11. If the circumference of a circle increases from 2π to 4π then its area ......the original area :
(a) Half (b) Double (c) Three times (d) Four times
Ans: (d) Four times

12. In the figure given below, AD = 4 cm, BD = 3 cm and CB = 12 cm, then cot θ equals :

(a) 3/4 (b) 5/12 (c) 4/3 (d) 12/5


Ans: (d) 12/5

13. The perimeters of two similar triangles are 26 cm and 39 cm. The ratio of their areas will be :
(a) 2 : 3 (b) 6 : 9 (c) 4 : 6 (d) 4 : 9
Ans: (d) 4 : 9

14. If ∆ABC ~ ∆EDF and ∆ABC is not similar to ∆DEF, then which of the following is not true?
(a) BC.EF = AC.FD (b) AB.EF = AC.DE (c) BC.DE = AB.EF (d) BC.DE = AB.FD
Ans. (c) BC.DE = AB.EF

15. The radii of 2 cylinders are in the ratio 2 : 3 and their heights are in the ratio 5 : 3. Then, the ratio
of their volumes is:
(a) 19 : 20 (b) 20 : 27 (c) 18:25 (d) 17:23
Ans: (b) 20 : 27

16. Consider the following frequency distribution


Class 0–5 6 – 11 12 – 17 18 – 23 24 – 29
Frequency 13 10 15 8 11
The upper limit of the median class is
(a) 7 (b) 17.5 (c) 18 (d) 18.5
Ans: (b) 17.5

17. Consider the following distribution:


Marks obtained Number of students
More than or equal to 0 63
More than or equal to 10 58
More than or equal to 20 55
More than or equal to 30 51
More than or equal to 40 48
More than or equal to 50 42
the frequency of the class 30-40 is
(a) 4 (b) 48 (c) 51 (d) 3
Ans: (d) 3

Prepared by: M. S. KumarSwamy, TGT(Maths) Page - 3 -


18. Two dice are thrown simultaneously. The probability that the product of the numbers appearing
on the dice is 7 is
(a) 7/36 (b) 2/36 (c) 0 (d) 1/36
Ans: (c) 0

Direction : In the question number 19 & 20 , A statement of Assertion (A) is followed by a


statement of Reason(R) . Choose the correct option
19. Assertion (A): The mid-point of the line segment joining the points A (3, 4) and B (k, 6) is P (x,
y) and x + y – 10 = 0, the value of k is 7
x x y y 
Reason (R): Midpoint of line segment is  1 2 , 1 2 
 2 2 
(a) Both A and R are true and R is the correct explanation of A
(b) Both A and R are true but R is not the correct explanation of A
(c) A is true and R is false
(d) A is false and R is true
Ans: (b) Both A and R are true but R is not the correct explanation of A
20. Assertion (A): For any two positive integers a and b, HCF(a, b) x LCM(a, b) = a x b
Reason (R): The HCF of two numbers is 5 and their product is 150. Then their LCM is 40.
(a) Both Assertion (A) and Reason (R) are true and Reason (R) is the correct explanation of
Assertion (A)
(b) Both assertion (A) and reason (R) are true and reason (R) is not the correct explanation of
Assertion (A)
(c) Assertion (A) is true but reason(R) is false.
(d) Assertion (A) is false but reason(R) is true.
Ans: (c) Assertion (A) is true but reason(R) is false.
LCM(a, b) x HCF(a, b) =a x b
⇒ LCM x 5 = 150
⇒ LCM = 150/5 = 30 ⇒ LCM = 30

SECTION-B
Questions 21 to 25 carry 2M each

21. A quadrilateral ABCD is drawn to circumscribe a circle. Prove that AB + CD = AD + BC.

Ans: We know that the lengths of the tangents drawn from an external point to the circle
are equal.
DR = DS ...... (i)
BP = BQ ...... (ii)
AP = AS ...... (iii)
CR = CQ ...... (iv)
Adding (i), (ii), (iii), (iv), we get DR + BP + AP + CR = DS + BQ + AS + CQ
By rearranging the terms we get,
(DR + CR) + (BP + AP) = (CQ + BQ) + (DS + AS)
⇒ CD + AB = BC + AD
Hence it is proved AB + CD = AD + BC.

Prepared by: M. S. KumarSwamy, TGT(Maths) Page - 4 -


QR QT
22. In the figure,  and ∠1 = ∠2, Show that ∆PQS ∼ ∆TQR.
QS PR

Ans: In △PQR,
Since, ∠1=∠2
∴ PR=PQ (Opposite sides of equal angles are equal) .....(1)
QR QT
In △PQS and △TQR,  . (Given )
QS PR
QR QT
i.e.,  .( From 1)
QS PQ
Also, ∠Q is common
∴ By SAS criterion of similarity, △PQS∼△TQR.

OR
ABCD is a trapezium in which AB || CD and its diagonals intersect each other at the point O.
Using a similarity criterion of two triangles, show that =
Ans: ABCD is a trapezium with AB∥CD and diagonals AB and CD intersecting at O.

In △OAB and △OCD


∠AOB =∠DOC [ Vertically opposite angles ]
∠ABO = ∠CDO [ Alternate angles ]
∠BAO = ∠OCD [ Alternate angles ]
∴ △OAB ∼ △OCD [ AAA similarity ]
We know that if triangles are similar, their corresponding sides are in proportional
OA OB
 
OC OD

1 1
23. If sin(A – B) = , cos(A + B) = , 00< A + B ≤900 , A > B. Find A and B.
2 2
1  1
Ans: sin( A  B )   sin( A  B )  30  sin 30  
2  2
On equating both sides
A  B  30  (1)
1  1
cos( A  B)   cos( A  B)  cos(60 )  cos(60 )  
2  2
On equating both sides
A  B  60..(2)

Prepared by: M. S. KumarSwamy, TGT(Maths) Page - 5 -


Adding (1) and (2),we get 2A = 900 ⇒ A = 450
Putting value of A in (i)
450 + B = 600 ⇒ B = 150

24. Find the value of p if the pair of equations 2x + 3y – 5 = 0 and px – 6y – 8 = 0 has a unique
solution.
Ans: Given, pair of linear equations is 2x + 3y – 5 = 0 and px – 6y – 8 = 0
On comparing with ax + by + c = 0 we get
Here, a1 = 2, b1 = 3, c1 = – 5;
And a2 = p, b2 = – 6, c2 = – 8;
a1 /a2 = 2/p, b1 /b2 = – 3/6 = – ½, c1 /c2 = 5/8
Since, the pair of linear equations has a unique solution,
∴ a1/a2 ≠ b1/b2
⇒ 2/p ≠ – ½
⇒p≠–4
Hence, the pair of linear equations has a unique solution for all values of p except – 4.

25. The short and long hands of a clock are 4 cm and 6 cm long respectively. Find the sum of
distances travelled by their tips in 2 days
Ans: In 2 days, the short hand will complete 4 rounds.
∴ Distance moved by its tip = 4(circumference of a circle of radius 4 cm)
 22  704
 4   2   4  cm  cm
 7  7
In 2 days, the long hand will complete 48 rounds.
∴ Distance moved by its tip = 48(circumference of a circle of radius 6 cm)
 22  12.672
 48   2   6  cm  cm
 7  7
 704 12672 
Hence, sum of distances moved by the tips of two hands of the clock     cm
 7 7 
= 1910.85 cm
OR
A car has two wipers which do not overlap. Each wiper has a blade of length 21 cm sweeping
through an angle of 120°. Find the total area cleaned at each sweep of the blades
Ans: Here, r = 21 cm, θ = 120°
 2 1200 22
Area of a sector =   r    21 21
3600 3600 7
= 462 cm2
∴ Total area cleaned by two wipers
= 2 × 462 = 924 cm2
SECTION-C
Questions 26 to 31 carry 3 marks each

26. 4 Bells toll together at 9.00 am. They toll after 7, 8, 11 and 12 seconds respectively. How many
times will they toll together again in the next 3 hours?
Ans: 7 = 7 × 1
8=2×2×2
11 = 11 × 1
12 = 2 × 2 × 3
∴ LCM of 7, 8, 11, 12 = 2 × 2 × 2 × 3 × 7 × 11 = 1848
∴ Bells will toll together after every 1848 sec.

Prepared by: M. S. KumarSwamy, TGT(Maths) Page - 6 -


3  3600
∴ In next 3 hrs, number of times the bells will toll together = = 5.84 = 5 times.
1848
OR
Given that √3 is irrational, prove that (2 + 5√3) is an irrational number.
Ans: Let 2 + 5√3 be a rational number such that
2 + 5√3 = a, where a is a non-zero rational number.
a 2
5 3  a 2  3 
5
a2
Since 5 and 2 are integers and a is a rational number, therefore is a rational number
5
⇒ √3 is a rational number which contradicts the fact that √3 is an irrational number.
Therefore, our assumption is wrong.
Hence 2 + 5√3 is an irrational number

27. Find the ratio in which the line 2x + y – 4 = 0 divides the line segment joining the points A (2, –
2) and B (3, 7)
Ans: Let P(x, y) be the point on the line 2x + y – 4 = 0 dividing the line segment joining the
points A(2, –2) and B(3, 7) in the ratio k : 1.
 3k  2 7 k  2 
∴ The coordinate of P are  , 
 k  1 k 1 
Since, point (x, y) lies on the line 2x + y = 4.
 3k  2   7 k  2  6k  4  7k  2
 2  4  4
 k 1   k 1  k 1
⇒ 13k + 2 = 4k + 4 ⇒ 9k = 2 ⇒ k = 2/9
Thus, required ratio is 2 : 9.

28. In the given figure, PA and PB are the tangent segments to a circle with centre O. Show that the
points A, O, B and P are concyclic.

Ans: Here, OA = OB
And OA ⊥ AP, OB ⊥ BP (tangent ⊥ radius)
∴ ∠OAP = 90°,
∠OBP = 90°
∴ ∠OAP + ∠OBP = 90° + 90° = 180°
∴ ∠AOB + ∠APB = 180°
(Since, ∠AOB + ∠OAP + ∠OBP + ∠APB = 360°)
Thus, sum of opposite angle of a quadrilateral is 180°.
Hence, A, O, B and P are concyclic.
OR
In the given figure, ABC is a triangle in which ∠B = 90°, BC = 48 cm and AB = 14 cm. A circle
is inscribed in the triangle, whose centre is O. Find radius r of in-circle.

Prepared by: M. S. KumarSwamy, TGT(Maths) Page - 7 -


Ans: In ∆ABC, AC 2  AB 2  BC 2  (14) 2  (48)2  50 cm
∠OQB = 90°⇒ OPBQ is a square ⇒ BQ = r
⇒ QA = 14 - r = AR (tangents from a external point are equal in length)
Again PB = r ⇒ PC = 48 - r
⇒ RC = 48 - r (tangents from a external point are equal in length)
⇒ AR + RC = AC
⇒ 14 - r + 48 - r = 50
⇒ r = 6cm.

29. From a pack of 52 playing cards, jacks, queens, kings and aces of red colour are removed. From
the remaining a card is drawn at random. Find the probability that the card drawn is (i) a black
queen (ii) a red card (iii) a face card.
Ans: From the total playing 52 cards, red coloured jacks, queen, kings and aces are removed(i.e.,
2 jacks, 2 queens, 2 kings, 2 aces) ∴ Remaining cards = 52 – 8 = 44
(i) Favourable cases for a black queen are 2 (i.e., queen of club or spade)
∴ Probability of drawing a black queen = 2/44 = 1/22
(ii) Favourable cases for red cards are 26 – 8 = 18 (as 8 cards have been removed) (i.e.9 diamonds
+ 9 hearts)
∴ Probability of drawing a red card = 18/44 = 9/22
(iii) Favourable cases for a face card are 6 (i.e. 2 black jacks, queens and kings each)
∴ Probability of drawing a face card = 6/44 = 3/22

30. If a, b are the zeroes of the polynomial 2x2 – 5x + 7, then find a polynomial whose zeroes are 2a
+ 3b, 3a + 2b
Ans: Since a, b are the zeroes of 2x2 – 5x + 7
(5) 5 7
∴a+b= = and ab =
2 2 2
The given zeroes of required polynomial are 2a + 3b and 3a + 2b
5 25
Sum of the zeroes = 2a + 3b + 3a + 2b = 5a + 5b = 5(a + b) = 5 × =
2 2
Again, product of the zeroes = (2a + 3b) (3a + 2b) = 6 (a2 + b2) + 13ab
= 6 [(a + b)2 – 2ab)] + 13 ab = 6(a + b)2 + ab
= 6(5/2)2 + 7/2 = 75/2 + 7/2 = 82/2 = 41
Now, required polynomial is given by
25
k [x2 – (Sum of the zeroes) x + Product of the zeroes] = k [ x2 – x + 41]
2
k
= [2 x 2  25 x  82] , where k is any non-zero real number.
2
Hence the required polynomial is 2 x 2  25 x  82 .

cos A 1  sin A
31. Prove that   2sec A
1  sin A cos A

Prepared by: M. S. KumarSwamy, TGT(Maths) Page - 8 -


cos A 1  sin A
Ans: LHS  
1  sin A cos A
cos 2 A  (1  sin A) 2 cos 2 A  1  sin 2 A  2sin A
 
cos A(1  sin A) cos A(1  sin A)
2  2sin A 2
   2sec A  RHS
cos A(1  sin A) cos A

SECTION-D
Questions 32 to 35 carry 5M each

32. The mean of the following frequency distribution is 62.8 and the sum of all the frequencies is 50.
Compute the missing frequencies f1 and f2.
Class 0-20 20-40 40-60 60-80 80-100 100-120
Frequency 5 f1 10 f2 7 8
Ans:
Class Frequency Class mark (x) u fu
0-20 5 10 -1 -5
20-40 f1 30 0 0
40-60 10 50 1 10
60-80 f2 70 2 2f2
80-100 7 90 3 21
100-120 8 110 4 32
Total f1 + f2 + 30 2f2 + 58
f1 + f2 + 30 = 50 ⇒ f1 + f2 = 20
Σfu = 2f2 + 58, h = 20, A = 30
  fu   2 f  58 
Mean  A    h   62.8  30   2  20 
f   50 
 
2 f  58
 62.8  30  2  2  32.8  5  2(2 f 2  58)
5
 4 f 2  116  164  4 f 2  164  116  48  f 2  12
 f1  20  12  8

33. A train, travelling at a uniform speed for 360 km, would have taken 48 minutes less to travel the
same distance if its speed were 5 km/h more. Find the original speed of the train.
Ans: Let original speed of train = x km/h
We know, Time = distance/speed
According to the question, we have Time taken by train = 360/x hour
And, Time taken by train its speed increase 5 km/h = 360/(x + 5)
It is given that,
Time taken by train in first – time taken by train in 2nd case = 48 min = 48/60 hour
360 360 48 4 1 1  4
    360   
x x  5 60 5  x x5 5
 x5 x  4  5  4
 360     360  
 x( x  5)  5  x( x  5)  5
5 5  2
 360   2   1  450  5  x  5 x
4  x  5x 
⇒ x² + 5x – 2250 = 0 ⇒ x² + 50x – 45x – 2250 = 0
⇒ x(x + 50) – 45(x + 50) = 0 ⇒ (x + 50)(x – 45) = 0

Prepared by: M. S. KumarSwamy, TGT(Maths) Page - 9 -


⇒ x = -50, 45
But x ≠ -50 because speed cannot be negative
So, x = 45 km/h
Hence, original speed of train = 45 km/h
OR
Two water taps together can fill a tank in 6 hours. The tap of larger diameter takes 9 hours less
than the smaller one to fill the tank separately. Find the time in which each tap can separately fill
the tank.
Ans: Let the time taken by the smaller tap to fill the tank = x hours and
time taken by larger tap = x – 9
1 1
In 1 hour, the smaller tap will fill of tank and the larger tap will fill of tank.
x x 9
1 1
In 1 hour both the tank will fill the tank = 
x x 9
1 1 1 x 9 x 1 2x  9 1
According to the question,      2 
x x 9 6 x( x  9) 6 x  9x 6
Solving by cross multiplication, 6(2x − 9) = x2 – 9x
⇒ 12x − 54 = x2 − 9x ⇒ x2 − 9x − 12x + 54 = 0 ⇒ x2 −21x + 54 = 0
⇒ x2−18x−3x+54=0
⇒ x(x − 18) − 3(x − 18)=0
⇒ (x − 18)(x − 3) = 0
⇒ x = 18, x = 3
Neglecting x = 3 as x – 9 can’t negative, therefore x = 18
If we take x = 18
Smaller tap = (x) = 18 hrs
Larger tap = (x – 9) = 18 – 9 = 9hrs
Hence, the time taken by the smaller tap to fill the tank = 18 hrs & the time taken by the larger
tap to fill the tank = 9 hrs

34. 200 logs are stacked in the following manner: 20 logs in the bottom row, 19 in the next row, 18 in
the row next to it and so on (see below figure). In how may rows are the 200 logs placed and how
many logs are in the top row?

Ans: Here, a is the first term, d is a common difference and n is the number of terms.
It can be observed that the number of logs in rows are forming an A.P. 20, 19, 18, ...
n
We know that sum of n terms of AP is given by the formula Sₙ = [2a + (n - 1) d]
2
n
⇒ 200 = [2 × 20 + (n - 1)(- 1)] ⇒ 400 = n [40 - n + 1] ⇒ 400 = n [41 - n]
2
⇒ 400 = 41n - n² ⇒ n² - 41n + 400 = 0 ⇒ n² - 16n - 25n + 400 = 0
⇒ n(n - 16) - 25(n - 16) = 0 ⇒ (n - 16)(n - 25) = 0
⇒ Either (n -16) = 0 or (n - 25) = 0
∴ n = 16 or n = 25
The number of logs in nth row will be aₙ = a + (n - 1) d
⇒ a₁₆ = a + 15d ⇒ a₁₆ = 20 + 15 × (- 1) ⇒ a₁₆ = 20 – 15 ⇒ a₁₆ = 5
Similarly, a₂₅ = 20 + 24 × (- 1)
⇒ a₂₅ = 20 – 24 ⇒ a₂₅ = - 4

Prepared by: M. S. KumarSwamy, TGT(Maths) Page - 10 -


Clearly, the number of logs in the 16th row is 5. However, the number of logs in the 25th row is
negative 4, which is not possible.
Therefore, 200 logs can be placed in 16 rows. The number of logs in the top (16th) row is 5.
OR
The sum of the third and the seventh terms of an AP is 6 and their product is 8. Find the sum of
first sixteen terms of the AP.
Ans: Here, a is the first term, d is the common difference and n is the number of terms.
Given: a₃ + a₇ = 6 ----- (1)
a₃ × a₇ = 8 ----- (2)
We know that nth term of AP is aₙ = a + (n - 1)d
Third term, a₃ = a + 2d ----- (3)
Seventh term, a₇ = a + 6d ----- (4)
Using equation (3) and equation (4) in equation (1) to find the sum of the terms,
(a + 2d) + (a + 6d) = 6
⇒ 2a + 8d = 6 ⇒ a + 4d = 3 ⇒ a = 3 - 4d ----- (5)
Using equation (3) and equation (4) in equation (2) to find the product of the terms,
(a + 2d ) × (a + 6d ) = 8
Substituting the value of a from equation (5) above,
(3 - 4d + 2d) × (3 - 4d + 6d) = 8
⇒ (3 - 2d) × (3 + 2d) = 8
⇒ (3)² - (2d)² = 8 [Since (a + b)(a - b) = a² - b² ]
⇒ 9 - 4d² = 8 ⇒ 4d² = 1 ⇒ d² = ¼ ⇒ d = ½, -½
Case 1: When d = ½
a = 3 - 4d = 3 - 4 × ½ = 3 - 2 = 1
n 16
Sₙ = [2a + (n - 1) d] ⇒ S₁₆ = [ 2 × 1 + (16 - 1) × ½ ] = 8 × 19/2 = 76
2 2
Case 2: When d = - ½
a = 3 - 4d = 3 - 4 × (- ½) = 3 + 2 = 5
n 16
Sₙ = [2a + (n – 1) d] ⇒ S₁₆ = [2 × 5 + (16 - 1) × (- ½)] = 8 [10 - 15/2] = 8 × 5/2 = 20
2 2

35. Prove that if a line is a drawn parallel to one side of a triangle intersecting the other two sides in
distinct points, then the other two sides are divided in the same ratio. Using the above theorem.
Prove that = if LM || CB and LN || CD as shown in the figure.

Ans: Given, To Prove, Figure and Construction – 1 ½ marks


Proof - 1 ½ mark
In △ABC, LM ∥ BC
AM AL
∴ By Basic proportionality theorem,  .(1)
AB AC
Similarly, In △ADC, LN ∥ CD
AN AL
∴ By Basic proportionality theorem,   (2)
AD AC
AM AN
∴ from (1) and (2), 
AB AD

Prepared by: M. S. KumarSwamy, TGT(Maths) Page - 11 -


SECTION-E (Case Study Based Questions)
Questions 36 to 38 carry 4M each

36. Mayank a student of class 7th loves watching and playing with birds of different kinds. One day
he had an idea in his mind to make a bird-bath on his garden. His brother who is studying in class
10th helped him to choose the material and shape of the birdbath. They made it in the shape of a
cylinder with a hemispherical depression at one end as shown in the Figure below. They opted for
the height of the hollow cylinder as 1.45 m and its radius is 30 cm. The cost of material used for
making bird bath is Rs. 40 per square meter.

(i) Find the curved surface area of the hemisphere. (Take π = 3.14)
(ii) Find the total surface area of the bird-bath. (Take π = 22/7)
(iii) What is total cost for making the bird bath?
OR
(iii) Mayank and his brother thought of increasing the radius of hemisphere to 35 cm with same
material so that birds get more space, then what is the new height of cylinder?
Ans: (i) Let r be the common radius of the cylinder and hemisphere and h be the height of the
hollow cylinder.
Then, r = 30 cm and h = 1.45 m = 145 cm.
Curved surface area of the hemisphere = 2πr2
=2 x 3.14 x 302 = 0.56 m2
(ii) Let S be the total surface area of the birdbath.
S = Curved surface area of the cylinder + Curved surface area of the hemisphere
⇒ S = 2πrh+ 2πr2 = 2πr(h + r)
22
⇒S=2x x 30(145 + 30) = 33000 cm2 = 3.3 m2
7
(iii) Total Cost of material = Total surface area x cost per sq m2
= 3.3 x 40 = Rs. 132
OR
We know that S = 3.3 m2

S  2 r (r  h)
22 35  35 
 3.3  2     h
7 100  100 
22  35  33 35
 3.3    h   h
10  100  22 100
3 7 23
h    1.15m
2 20 20

37. Tower Bridge is a Grade I listed combined bascule and suspension bridge in London, built
between 1886 and 1894, designed by Horace Jones and engineered by John Wolfe Barry. The

Prepared by: M. S. KumarSwamy, TGT(Maths) Page - 12 -


bridge is 800 feet (240 m) in length and consists of two bridge towers connected at the upper
level by two horizontal walkways, and a central pair of bascules that can open to allow shipping.
In this bridge, two towers of equal heights are standing opposite each other on either side of the
road, which is 80 m wide. During summer holidays, Neeta visited the tower bridge. She stood at
some point on the road between these towers. From that point between the towers on the road, the
angles of elevation of the top of the towers was 60° and 30° respectively.

(i) Find the distances of the point from the base of the towers where Neeta was standing while
measuring the height. [2]
(ii) Neeta used some applications of trigonometry she learned in her class to find the height of the
towers without actually measuring them. What would be the height of the towers she would have
calculated? [2]
OR
(ii) Find the distance between Neeta and top of tower AB? Also, Find the distance between Neeta
and top tower CD? [2]
Ans: (i) Suppose AB and CD are the two towers of equal height h m. BC be the 80 m wide road.
P is any point on the road. Let CP be x m, therefore BP = (80 - x).
Also, ∠APB = 60° and ∠DPC = 30°
CD 1 h x
In right angled triangle DCP, tan 300    h …… (1)
CP 3 x 3

AB h
In right angled triangle ABP, tan 600    3  h  3(80  x)
BP 80  x
x
  3(80  x) ⇒ x = 3(80 – x) ⇒ x = 240 -3x
3
⇒ x + 3x = 240 ⇒ 4x = 240 ⇒ x=60
Thus, the position of the point P is 60 m from C.
x 60
(ii) Height of the tower, h =  = 20√3
3 3
The height of each tower is 20√3 m.
OR
(ii) The distance between Neeta and top of tower AB.

Prepared by: M. S. KumarSwamy, TGT(Maths) Page - 13 -


AB 3 20 3
In ∆ABP, sin 600     AP  40m
AP 2 AP
Similarly, the distance between Neeta and top of tower CD.
CD 1 20 3
In ∆CDP, sin 300     PD  40 3m
PD 2 PD
38. On the roadway, Points A and B, which stand in for Chandigarh and Kurukshetra, respectively,
are located nearly 90 kilometres apart. At the same time, a car departs from Kurukshetra and one
from Chandigarh. These cars will collide in 9 hours if they are travelling in the same direction,
and in 9/7 hours if they are travelling in the other direction. Let X and Y be two cars that are
travelling at x and y kilometres per hour from places A and B, respectively. On the basis of the
above information, answer the following questions:

(a) When both cars move in the same direction, then find the situation can be represented
algebraically. [2]
OR
(a) When both cars move in the opposite direction, then find the situation can be represented
algebraically. [2]
(b) Find the speed of car x. [1]
(c) Find the speed of car y. [1]
Ans: (a) Suppose two cars meet at point Q. Then, Distance travelled by car X = AQ, Distance
travelled by car Y = BQ. It is given that two cars meet in 9 hours.
∴ Distance travelled by car X in 9 hours = 9x km = AQ = 9x
Distance travelled by car Y in 9 hours = 9y km = BQ = 9y
Clearly, AQ - BQ = AB = 9x - 9y = 90 = x - y = 10
OR
Suppose two cars meet at point P. Then Distance travelled by car X = AP and Distance travelled
by car Y = BP.
9 9
In this case, two cars meet in hours. Distance travelled by car X in hours
7 7
9 9
= x km ⇒ AP = x
7 7
9
Distance travelled by car Y in hours
7
9 9
= y km ⇒ BP = y
7 7
Clearly, AP + BP = AB
9 9 9
⇒ x + y = 90 ⇒ (x + y) = 90 ⇒ x + y = 70
7 7 7
(b) We have x - y = 10 and x + y = 70
Adding equations (i) and (ii), we get 2x = 80 ⇒ x = 40
Hence, speed of car X is 40 km/hr.
(c) We have x - y = 10 ⇒ 40 - y = 10 ⇒ y = 30
Hence, speed of car y is 30 km/hr

Prepared by: M. S. KumarSwamy, TGT(Maths) Page - 14 -


PM SHRI KENDRIYA VIDYALAYA GACHIBOWLI, GPRA CAMPUS, HYD-32
SAMPLE PAPER TEST 06 FOR BOARD EXAM 2024
(ANWERS)

SUBJECT: MATHEMATICS MAX. MARKS : 80


CLASS : X DURATION : 3 HRS
General Instruction:
1. This Question Paper has 5 Sections A-E.
2. Section A has 20 MCQs carrying 1 mark each.
3. Section B has 5 questions carrying 02 marks each.
4. Section C has 6 questions carrying 03 marks each.
5. Section D has 4 questions carrying 05 marks each.
6. Section E has 3 case based integrated units of assessment (04 marks each) with sub-parts of the
values of 1, 1 and 2 marks each respectively.
7. All Questions are compulsory. However, an internal choice in 2 Qs of 5 marks, 2 Qs of 3 marks
and 2 Questions of 2 marks has been provided. An internal choice has been provided in the
2marks questions of Section E
8. Draw neat figures wherever required. Take π =22/7 wherever required if not stated.
SECTION – A
Questions 1 to 20 carry 1 mark each.
1. The solution of the following pair of equation is:
x – 3y = 2, 3x – y = 14
(a) x = 5, y = 1 (b) x = 2, y = 3 (c) x = 1, y = 2 (d) x = 1, y = 4
Ans: (a) x = 5, y = 1
Given, equations are x – 3y = 2 …(i)
and 3x – y = 14 …(ii)
Solving equations (i) and (ii), we get y = 1
x = 2 + 3y = 2 + 3 × 1 = 5
Hence, x = 5 and y = 1

2. What is the positive real root of 64x2 – 1 = 0?


(a) 1/8 (b) 1/4 (c) 1/2 (d) 1/6
Answer: (a) 1/8,−1/8
Given, 64x2 – 1 = 0
⇒ (8x + 1) (8x – 1) = 0
⇒ 8x = – 1, 8x = 1
⇒ x = 1/8,−1/8
Thus, positive root is 1/8

3. In ∆ABC and ∆DEF, ∠B = ∠E, ∠F = ∠C and AB = 3DE. Then, the two triangles are
(a) congruent but not similar (b) similar but not congruent
(c) neither congruent nor similar (d) congruent as well as similar
Ans: (b) similar but not congruent

4. The LCM of smallest two-digit composite number and smallest composite number is:
(a) 12 (b) 4 (c) 20 (d) 44
Ans: (c) 20
Smallest 2-digit number = 10 = 2 x 5
Smallest composite number = 4 = 22
LCM = 22 x 5 = 20

Prepared by: M. S. KumarSwamy, TGT(Maths) Page - 1 -


2sin A  3cos A
5. If cosec A = 13/12, then the value of
4sin A  9cos A
(a) 4 (b) 5 (c) 6 (d) 3
Ans: (d) 3
Given cosec A = 13/12,
12 5
sin A  , cos A 
13 13
 12   5 
2    3 
2sin A  3cos A 13 13 24  15 9
Now,       3
4sin A  9cos A  12   5  48  45 3
4   9 
 13   13 

OA OC
6. In the figure, if  , then
OD OB

which pair of angles are equal? [1]


(a) ∠A = ∠C, ∠B = ∠D (b) ∠A = ∠B, ∠C = ∠D
(c) ∠C = ∠B, ∠A = ∠D (d) None of these
Ans: (a) ∠A = ∠C, ∠B = ∠D
OA OC OA OD
In given figure,   
OD OB OC OB
Then, in ΔAOD and ΔCOB
OA OD
 (proved above)
OC OB
and ∠AOD = ∠COB (vertically opposite angles)
∴ ΔAOD ~ ΔCOB by SAS Similarity
Hence, ∠A = ∠C and ∠B = ∠D. (Corresponding angles of similar triangles)

7. The value of ‘a’, if HCF (a, 18) = 2 and LCM (a, 18) = 36, is: (1)
(a) 2 (b) 5 (c) 7 (d) 4
Ans: (d) 4
Given, HCF (a, 18) = 2 and LCM (a, 18) = 36
HCF (a, b) × LCM (a, b) = a × b
⇒ 2 × 36 = a × 18 ⇒ a = 4
Hence, value of ‘a’ is 4.

8. If r = 3 is a root of quadratic equation kr2 – kr – 3 = 0, then the value of k is:


(a) 1/2 (b) 3 (c) 1/3 (d) 1/4
Ans: (a) 1/2
Given, equation is kr2 – kr – 3 = 0
If, r = 3, then, k(3)2 – k(3) – 3 = 0
⇒ 9k – 3k – 3 = 0 ⇒ 6k = 3 ⇒k=½

9. The ratio in which x-axis divides the join of (2, -3) and (5, 6) is:
(a) 1: 2 (b) 3 : 4 (c) 1: 3 (d) 1: 5
Ans: (a) 1 : 2

Prepared by: M. S. KumarSwamy, TGT(Maths) Page - 2 -


Let P(x, 0) be the point on x-axis which divides the join of (2, -3) and (5, 6) in the ratio k : 1.
∴ By section formula,
 5k  2 6k  3 
P(x, 0) =  , 
 k 1 k 1 
6k  3 1
 y0  0  6k  3  0  k 
k 1 2
10. If tan θ = 1, then the value of sec θ + cosec θ is:
(a) 3√2 (b) 4√2 (c) 2√2 (d) √2
Ans: (c) 2√2
Given, tan θ = 1, we have θ = 45°
So, sec θ + cosec θ = √2 + √2 = 2√2.

11. If the area of circle is numerically equal to twice its circumference, then the diameter of the
circle is
(a) 4 units (b) 6 units (c) 8 units (d) 12 units
Ans: (c) 8 units

12. In the given figure, if TP and TQ are tangents to a circle with centre O, so that ∠POQ = 110°,
then ∠PTQ is

(a) 110° (b) 90° (c) 80° (d) 70°


Ans: (d) 70°

13. If the angle of elevation of the top of a tower from a point of observation at a distance of 100 m
from its base is 45°, then the height of the tower is:
(a) 160 m (b) 100 m (c) 200 m (d) 150 m
Ans: (b) 100 m
Here, PQ is the tower and A is a point of observation at a distance of 100 m from PQ.
PQ h
From right ΔAPQ,   tan 450  1  h  100m
AP 100

Thus, the height of tower is 100 metre.

14. If the perimeter of a circle is equal to that of a square, then the ratio of the area of circle to the
area of the square is
(a) 14: 11 (b) 12: 13 (c) 11:14 (d) 13:12
Ans: (a) 14: 11
r
Here, it is given that 4s = 2πr  s 
2

Prepared by: M. S. KumarSwamy, TGT(Maths) Page - 3 -


Area of Circle  r 2  r 2 4 4  7 14
Now,  2  2 2   
Area of Square s  r  22 11
4
So, the ratio of the area of the circle to the area of square is 14 : 11.

15. For the following distribution:


Class 0-5 6-11 12-17 18-23 24-29
Frequency 13 10 15 8 11
the upper limit of the median class is
(a) 18.5 (b) 20.5 (c) 25.5 (d) 17.5
Ans:
Class Frequency Cf
– 0.5 – 5.5 13 13
5.5 – 11.5 10 23
11.5 – 17.5 15 38
17.5 – 23.5 8 46
23.5 – 29.5 11 57
n
Here, n = 57 So, = 28.5
2
The cumulative frequency, just greater than 28.5, is 38 which belongs to class 11.5 – 17.5.
So, the median class is 11.5 – 17.5 Its upper limit is 17.5

16. If the mean of the following distribution is 2.6, then the value of y is
Variable (x) 1 2 3 4 5
Frequency 4 5 y 1 2
(a) 3 (b) 8 (c) 13 (d) 24
Ans: (b) 8
Variable (x) 1 2 3 4 5 Total
Frequency (f) 4 5 y 1 2 y + 12
fx 4 10 3y 4 10 3y + 28
Here, ∑f = y + 12 and ∑fx = 3y + 28

Mean, x 
 f x  2.6  3 y  28  3 y  28  2.6 y  31.2
f y  12
 0.4 y  3.2  y  8

17. Two different dice are thrown together. The probability of getting the sum of the two numbers
less than 7 is:
(a) 5/12 (b) 7/12 (c) 12/5 (d) 3/11
Ans: (a) 5/12
Total outcomes = 36
Number of outcomes in which sum of two numbers is less than 7 = 15
∴ Required probability = 15/36 = 5/12

18. The radii of 2 cylinders are in the ratio 2 : 3 and their heights are in the ratio 5 : 3. Then, the
ratio of their volumes is:
(a) 19 : 20 (b) 20 : 27 (c) 18:25 (d) 17:23
Ans: (b) 20 : 27
Let r1 and r2 be the two radii and h1 and h2 be the corresponding two heights of the two
cylinders. Then
r1 L h 5
 and 1  (Given)
r2 3 h2 3

Prepared by: M. S. KumarSwamy, TGT(Maths) Page - 4 -


V1  r12 h1 4 5 20
∴    
V2  r22 h2 9 3 27

DIRECTION: In the question number 19 and 20, a statement of Assertion (A) is followed by a
statement of Reason (R).
Choose the correct option

19. Assertion (A): The value of y is 3, if the distance between the points P(2, -3) and Q(10, y) is 10.
Reason (R): Distance between two points is given by ( x2  x1 ) 2  ( y2  y1 )2
(a) Both Assertion (A) and Reason (R) are true and Reason (R) is the correct explanation of
Assertion (A).
(b) Both Assertion (A) and Reason (R) are true but Reason (R) is not the correct explanation of
Assertion (A).
(c) Assertion (A) is true but Reason (R) is false.
(d) Assertion (A) is false but Reason (R) is true.
Ans: (a) Both assertion (A) and reason (R) are true and reason (R) is the correct explanation of
assertion (A)
For y = 3
Distance PQ  (10  2)2  ( y  3)2  82  62  64  36  100  10 units

20. Assertion: The HCF of two numbers is 9 and their LCM is 2016. If the one number is 54, then
the other number is 336.
Reason: Relation between numbers and their HCF and LCM is product of two numbers a, b =
HCF (a, b) × LCM (a, b).
(a) Both Assertion (A) and Reason (R) are true and Reason (R) is the correct explanation of
Assertion (A).
(b) Both Assertion (A) and Reason (R) are true but Reason (R) is not the correct explanation of
Assertion (A).
(c) Assertion (A) is true but Reason (R) is false.
(d) Assertion (A) is false but Reason (R) is true.
Ans: (a) Both assertion (A) and reason (R) are true and reason (R) is the correct explanation of
assertion (A)
Let the other number be x.
9 × 2016 = 54 × x
⇒ x = 336

SECTION – B
Questions 21 to 25 carry 2 marks each.

21. Two concentric circles are of radii 5 cm and 3 cm. Find the length of the chord of the larger
circle which touches the smaller circle.
Ans: Let O be the centre of the concentric circle of radii 5 cm and 3 cm respectively. Let AB be
a chord of the larger circle touching the smaller circle at P

Prepared by: M. S. KumarSwamy, TGT(Maths) Page - 5 -


Then AP = PB and OP⊥AB
Applying Pythagoras theorem in △OPA, we have
OA2 = OP2 + AP2 ⇒ 25 = 9 + AP2
⇒ AP2 = 16 ⇒ AP = 4 cm
∴ AB = 2AP = 8 cm

22. Evaluate: 3 cos2 60° sec2 30° – 2 sin2 30° tan2 60°.
Ans: 3 cos2 60° sec2 30° – 2 sin2 30° tan2 60°
2 2 2
1  2  1 2 3 4 1 3 1
= 3  
2  3
  2 
2
 3    2   3  1  
4 3 4 2 2

23. The length of the minute hand of a clock is 14 cm. Find the area swept by the minute hand in 5
minutes.
Ans: Area swept by the minute hand in 5 minutes will be the area of a sector of 30⁰ in a circle of
14 cm radius.

Area of sector of angle θ = πr²
3600
300 22 1 22 154
Area of sector of 30° = 0
 14 14   14  14 
360 7 12 7 3
= 51.33 cm²
Therefore, the area swept by the minute hand in 5 minutes is 51.33 cm²

OR
A horse is tied to a peg at one corner of a square shaped grass field of side 15 m by means of a 5
m long rope. Find the area of that part of the field in which the horse can graze (use π = 3.14)
Ans: Length of the rope = 5 m
Area of the field the horse can graze = Area of the sector with θ = 90° and r = 5
= θ/360° × πr2 = 90°/360° × πr2 = 1/4 × π × (5 m)2 = 25/4 × 3.14 m2 = 19.625 m2

24. For what value of k for which the following pair of linear equations have infinitely many
solutions: 2x + 3y = 7, (k – 1)x + (k + 2)y = 3k is
a b c
Ans: For a pair of linear equations to have infinitely many solutions, 1  1  1
a2 b2 c2
2 3 7 2 3
⇒    
k  1 k  2 3k k 1 k  2
Now, 2k + 4 = 3k – 3 ⇒ k = 7
and 9k = 7k + 14 ⇒ k = 7
Hence, the value of k is 7.

25. In the below left figure, two chords AB and CD intersect each other at the point P.

Prove that (i) ΔAPC ~ ΔDPB (ii) AP. PB = CP. DP


Ans: (i) Consider Δs APC and DPB
∠APC = ∠DPB (Vertically opposite angles)
Prepared by: M. S. KumarSwamy, TGT(Maths) Page - 6 -
Also, ∠CAP = ∠PDB (Angles made by the same arc CB)
So, by AA similarity criteria,
ΔAPC ~ ΔDPB.
(ii) Corresponding sides of similar triangles are proportional
AP DP
∴ 
PC PB
⇒ AP.PB = CP.DP
OR
If in the given above right sided figure, AB || DE and BD || EF, then prove that DC2 = CF x AC

Ans: ΔABC in which DE || AB and BD || EF.


In ∆ABC, DE || AB
CD CE
⇒  ...(i) [Basic proportionality theorem]
AC BC
Again in ∆CDB, EF || BD
CF CE
⇒  ...(ii) [BPT]
CD BC
From (i) and (ii), we get
CD CF
  CD 2  AC  CF
AC CD

SECTION – C
Questions 26 to 31 carry 3 marks each.

26. A part of monthly hostel charges in a college is fixed and the remaining depends on the number
of days one has taken food in the mess. When a student ‘A’ takes food for 22 days, he has to
pay Rs. 1380 as hostel charges; whereas a student ‘B’, who takes food for 28 days, pays Rs.
1680 as hostel charges. Find the fixed charges and the cost of food per day.
Ans: Let the fixed hostel charges be Rs. x and the cost of food per day be Rs. y.
According to the question, we get
x + 22y = 1380 ...(i)
and x + 28y = 1680 ...(ii)
Subtracting (i) from (ii), we get
6y = 300 ⇒ y = 300 ÷ 6 = 50
Putting y = 50 in (i), we get
x + 22(50) = 1380 ⇒ x + 1100 = 1380 ⇒ x = 280
∴ Fixed hostel charges = Rs. 280 and cost of the food per day = Rs. 50.
OR
Meena went to a bank to withdraw Rs 2,000. She asked the cashier to give her Rs. 50 and Rs.
100 notes only. Meena got 25 notes in all. How many notes of Rs. 50 and Rs. 100 she received?
Ans: Let Meena has received x no. of Rs. 50 notes and y no. of Rs. 100 notes.
So, 50x + 100y = 2000 ... (i)
x + y = 25 ... (ii)
Solving (i) and (ii), we get y = 15
Putting y = 15 in equation (ii), we get
x + 15 = 25
⇒ x = 10
Meena has received 10 pieces of Rs. 50 notes and 15 pieces of Rs. 100 notes.

27. Prove that √5 is an irrational number.


p
Ans: Let 5 is a rational number then we have 5 , where p and q are co-primes.
q

Prepared by: M. S. KumarSwamy, TGT(Maths) Page - 7 -


 p  5q
Squaring both sides, we get p 2  5q 2
 p2 is divisible by 5  p is also divisible by 5
So, assume p = 5m where m is any integer.
Squaring both sides, we get p2 = 25m2
But p 2  5q 2
Therefore, 5q2 = 25m2  q2 = 5m2
 q2 is divisible by 5  q is also divisible by 5
From above we conclude that p and q have one common factor i.e. 5 which contradicts that p
and q are co-primes.
Therefore, our assumption is wrong.
Hence, 5 is an irrational number.

28. What number should be added to the polynomial x2 – 5x + 4 so that 3 is the zero of the
polynomial?
Ans: Let k be the number to be added to the given polynomial.
Then the polynomial becomes x2 – 5x + (4 + k)
As 3 is the zero of this polynomial, we get (3)2 – 5(3) + (4 + k) = 0
⇒ (4 + k) = 15 – 9 ⇒ 4 + k = 6 ⇒ k = 2
Thus, 2 is to be added to the given polynomial.

cos2  sin 2 
29. Prove that:   1  sin  cos 
1  tan  1  cot 
cos2  sin 2 
Ans: LHS  
1  tan  1  cot 
3
cos  sin 3 
 
cos   sin  cos   sin 
cos3   sin 3  (cos   sin  )(cos2   sin 2   cos  sin  )
 
cos   sin  cos   sin 
2 2
 cos   sin   cos sin 
 1  sin  cos   RHS
OR
If cos θ + sin θ = √2 cos θ, show that cos θ – sin θ = √2 sin θ.
Ans: Given, cos θ + sin θ = √2 cos θ
Squaring both sides, we get
(cos θ + sin θ)2 = (√2 cos θ)2
⇒ cos2θ + sin2θ + 2sin θ cos θ
⇒ 2sin θ cos θ = cos2θ – sin2θ
⇒ 2sin θ cos θ = (cos θ – sin θ)(cos θ + sin θ)
⇒ 2sin θ cos θ = (cos θ – sin θ) (√2 cos θ)
⇒ √2 sin θ = cos θ – sin θ ⇒ cos θ – sin θ = √2 sin θ

30. All the black face cards are removed from a pack of 52 playing cards. The reaming cards are
well shuffled and then a card is drawn at random. Find the probability of getting (i) face card (ii)
red card (iii) black card.
Ans: When all the black face cards are removed,
Remaining number of cards = 52 – 6 = 46
(i) Number of face cards in the remaining deck = 6
∴ P(getting a face card) = 6/46 = 3/23
(ii) Number of red cards in the remaining deck = 26

Prepared by: M. S. KumarSwamy, TGT(Maths) Page - 8 -


∴ P (getting a red card) = 26/46 = 13/23
(iii) Number of black cards in the remaining deck = 20
∴ P (getting a black card) = 20/46 = 10/23

31. In the given figure, OP is equal to diameter of the circle. Prove that ABP is an equilateral
triangle.

Ans: Join OP and let it meets the circle at point Q.


Since OP = 2r (Diameter of the circle)
Consider ΔAOP in which OA ⊥ AP and OP is the hypotenuse.

OA r 1
In ∆AOP, sin OPA     sin 300
OP 2r 2
 OPA  300
 APB  2OPA  600 (Centre lies on the bisector of the angle formed between two
tangents)
Now, PA = PB (Tangents from P)
⇒ ∠PAB = ∠PBA (∠s opposite to equal sides)
Using ∠APB and angle sum property of triangle, we have
∠PAB = ∠PBA = ∠APB
⇒∆APB is an equilateral triangle.

SECTION – D
Questions 32 to 35 carry 5 marks each.
32. Some students planned a picnic. The total budget for food was Rs. 2,000. But 5 students failed
to attend the picnic and thus the cost of food for each member increased by Rs. 20. How many
students attended the picnic and how much did each student pay for the food?
Ans: Case I. Let number of students = x
and cost of food for each member = Rs. y
Then x × y = 2,000 ...(i)
Case II. New number of students = x – 5
New cost of food for each member = Rs. (y + 20)
Then (x – 5)(y + 20) = 2,000
⇒ xy + 20x – 5y – 100 = 2,000 ...(ii)
Solving (i) and (ii), we get x = –20, 25
x = –20 is rejected because number of students can’t be negative.
So, x = 25 ⇒ y = 80
Number of students = 25
Cost of food for each student = Rs. 80.
Prepared by: M. S. KumarSwamy, TGT(Maths) Page - 9 -
OR
If Zeba was younger by 5 years than what she really is, then the square of her age (in years)
would have been 11 more than five times her actual age. What is her age now?
Ans: Let the present age of Zeba be x years.
Age before 5 years = (x – 5) years
According to given condition, (x – 5)2 = 5x + 11
⇒ x2 + 25 – 10x = 5x + 11 ⇒ x2 – 10x – 5x + 25 – 11 = 0
⇒ x2 – 15x + 14 = 0 ⇒ x2 – 14x – x + 14 = 0
⇒ x (x – 14) – 1 (x – 14) = 0 ⇒ (x – 1) (x – 14) = 0
⇒ x – 1 = 0 or x – 14 = 0 ⇒ x = 1 or x = 14
But present age cannot be 1 year.
Hence, Present age of Zeba is 14 years.

33. A tent is in the shape of a cylinder surmounted by a conical top. If the height and diameter of
the cylindrical part are 2.1 m and 4 m respectively, and the slant height of the top is 2.8 m, find
the area of the canvas used for making the tent. Also, find the cost of the canvas of the tent at
the rate of Rs. 500 per m2.
Ans: Radius (r) of the cylindrical part = Radius of the conical part = 4/2 m = 2 m

Area of the canvas used = CSA of the cylindrical part + CSA of the conical part
= 2πrh + πrl
22 22
= πr (2h + l)   2  (4.2  2.8)   2  7
7 7
2
= 44 m
Cost of the canvas = Rate per m2 × area of canvas
= Rs 500 × 44 = Rs. 22,000
OR
A rectangular metal block has length 15 cm, breadth 10 cm and height 5 cm. From this block, a
circular hole of diameter 7 cm is drilled out. Find: (i) the volume of the remaining solid (ii) the
surface area of the remaining solid.
Ans: (i) The volume of the remaining solid
= Volume of rectangular block – Volume of the circular hole

Prepared by: M. S. KumarSwamy, TGT(Maths) Page - 10 -


2
22  7 
= (15  10  5)      5 cm³
7 2
= 750 cm³ – 192.5 cm³ = 557.5 cm³
(ii) The surface area of the remaining solid
= Total surface area of the block – 2 (area of circle of the hole) + curved surface of circular hole
(cylinder)
= 2(l × b + b × h + h × l) – 2(πr2) + 2πrh
2
22  7  22 7
= 2(15  10  10  5  5 15)  2      5  2    5
7 2 7 2
= 550 cm² – 77 cm² + 110 cm² = 583 cm².
34. Prove that “If a line is drawn parallel to one side of a triangle to intersect the other two sides in
distinct points, the other two sides are divided in the same ratio.”
In the figure, find EC if AD/DB = AE/EC using the above theorem.

Ans: For Theorem:


Figure, Given, To Prove and Construction - 2 marks
Proof - 2 marks
To find the value of EC = 9 cm – 1 mark

35. The distribution below gives the makes of 100 students of a class, if the median marks are 24,
find the frequencies f1 and f2
Marks 0-5 5-10 10-15 15-20 20-25 25-30 30-35 35-40
No. of students 4 6 10 f1 25 f2 18 5
Ans:
Class Frequency cf
0–5 4 4
5 – 10 6 10
10 – 15 10 20
15 – 20 f1 20 + f1
20 – 25 25 45 + f1
25 – 30 f2 45 + f1 + f2
30 – 35 18 63 + f1 + f2
35 – 40 5 68 + f1 + f2
Now, Median = 24 (Given)
So, median class = 20 – 25
For this class,
I = 20. h = 5, N2 = 50, cf = 20 + f1, f= 25
n 
 2  cf 
We know, Median = l    h
 f 
 
50  (20  f1 ) 30  f1
 24  20  5  4   30  f1  20  f1  10
25 5

Prepared by: M. S. KumarSwamy, TGT(Maths) Page - 11 -


Also, sum of frequencies = 100
⇒ 68 + f1 + f2 = 100 ⇒ f1 + f2 = 32 ⇒ 10 + f2 = 32 ⇒ f2 = 22
∴ f1 = 10, f2 = 22.

SECTION – E(Case Study Based Questions)


Questions 36 to 38 carry 4 marks each.

36. Case Study – 1


Saving money is a good habit and it should be inculcated in children from the beginning. A
father brought a piggy bank for his son Aditya. He puts one five-rupee coin of his savings in the
piggy bank on the first day. He increases his savings by one five-rupee coin daily.

(i) If the piggy bank can hold 190 coins of five rupees in all, find the number of days he can
contribute to put the five-rupee coins into it
(ii) Find the total money he saved.
OR
If 6 times the 6th term of an A.P., is equal to 9 times the 9th term, find its 15th term.
Ans: Child's savings day wise are 5, 10, 15, 20, 25, ....... to n days
We can have at most 190 coins
i.e., 1 + 2 + 3 + 4 + 5 + .... to n term = 190
n
⇒ [2  1  ( n  1)1]  190
2
⇒ n(n + 1) = 380 ⇒ n2 + n - 380 = 0
⇒ (n + 20)(n−19) = 0 ⇒ (n + 20)(n - 19) = 0
⇒ n = −20 or n = 19
But number of coins cannot be negative
n = 19 (rejecting n = -20)
So, number of days = 19
Total money she saved = 5 + 10 + 15 + 20 + ...
= 5 + 10 + 15 + 20 + ... upto 19 terms
19 19 19
= [2  5  (19  1)5]  [10  90]  [100]  19  50  950
2 2 2
OR
Let, the first term of A.P. be ‘a’ and its common difference be ‘d’
Given, 6a6 = 9a9
⇒ 6(a + 5d) = 9(a + 8d)
⇒ 6a + 30d = 9a + 72d
⇒ -3a = 42d ⇒ a = -14d …(i)
Then, 15th term i.e. a15 = a + 14d = -14d + 14d [from (i)]
=0
Hence, 15th term of A.P. is 0.

Prepared by: M. S. KumarSwamy, TGT(Maths) Page - 12 -


37. Case Study – 2
In a GPS, The lines that run east-west are known as lines of latitude, and the lines running
north-south are known as lines of longitude. The latitude and the longitude of a place are its
coordinates and the distance formula is used to find the distance between two places. The
distance between two parallel lines is approximately 150 km. A family from Uttar Pradesh
planned a round trip from Lucknow (L) to Puri (P) via Bhuj (B) and Nashik (N) as shown in the
given figure below.

Based on the above information answer the following questions using the coordinate geometry.
(i) Find the distance between Lucknow (L) to Bhuj(B).
(ii) If Kota (K), internally divide the line segment joining Lucknow (L) to Bhuj (B) into 3 : 2
then find the coordinate of Kota (K).
(iii) Name the type of triangle formed by the places Lucknow (L), Nashik (N) and Puri (P)
OR
Find a place (point) on the longitude (y-axis) which is equidistant from the points Lucknow (L)
and Puri (P).
(i)

 3  0  2  5 3  7  2  10   10 41   41 
(ii) Coordinates of Kota (K)=  ,    ,    2, 
 3 2 3 2  5 5   5 
(iii)

Prepared by: M. S. KumarSwamy, TGT(Maths) Page - 13 -


OR

38. Case Study – 3


Ram is watching the top and bottom of a lighthouse from the top of the building. The angles of
elevation and depression of the top and bottom of a lighthouse from the top of a 60 m high
building are 30° and 60° respectively.

Find (i) the difference between the heights of the lighthouse and the building.
(ii) the distance between the lighthouse and the building.
OR
The ratio of the height of a light house and the length of its shadow on the ground is √3 : 1 What
is the angle of elevation?
Ans: In right ∆ABD,
AB 60 60
tan 600   3  BD   20 3m
BD BD 3
∴ AE = 20√3 m (∵ BD = AE)
Now in right ∆AEC
CE 1 CE
tan 300     CE  20m
AE 3 20 3
(i) Difference between the heights of the lighthouse and the building = CE = 20 m
(ii) The distance between the lighthouse and the building = BD = 20√3 m.
OR
Let AB be the light house, BC be its shadow and θ be the angle of elevation of the sun at that
instant

Prepared by: M. S. KumarSwamy, TGT(Maths) Page - 14 -


AB
Then, in triangle ABC, we have, tan θ =
BC
3
tan θ =  tan 600 ⇒ θ = 60°
1
Hence, angle of elevation of the sun is 60°.

Prepared by: M. S. KumarSwamy, TGT(Maths) Page - 15 -


PM SHRI KENDRIYA VIDYALAYA GACHIBOWLI,GPRA CAMPUS, HYD-32
SAMPLE PAPER TEST 07 FOR BOARD EXAM 2024
(ANSWERS)
SUBJECT: MATHEMATICS MAX. MARKS : 80
CLASS : X DURATION : 3 HRS
General Instruction:
1. This Question Paper has 5 Sections A-E.
2. Section A has 20 MCQs carrying 1 mark each.
3. Section B has 5 questions carrying 02 marks each.
4. Section C has 6 questions carrying 03 marks each.
5. Section D has 4 questions carrying 05 marks each.
6. Section E has 3 case based integrated units of assessment (04 marks each) with sub-parts of the
values of 1, 1 and 2 marks each respectively.
7. All Questions are compulsory. However, an internal choice in 2 Qs of 5 marks, 2 Qs of 3 marks
and 2 Questions of 2 marks has been provided. An internal choice has been provided in the 2marks
questions of Section E
8. Draw neat figures wherever required. Take π =22/7 wherever required if not stated.
SECTION – A
Questions 1 to 20 carry 1 mark each.
1. The pair of equations x + 2y + 5 = 0 and –3x – 6y + 1 = 0 have
(a) a unique solution (b) exactly two solutions
(c) Infinitely many solutions (d) no solution
Ans: (d) no solution
The given equations are:
x + 2y + 5 = 0
−3x − 6y + 1 = 0
From the given equations we have:
a1 1 b 2 1 c 5
 ; 1   ; 1 
a2 3 b2 6 3 c2 1
a b c
 1  1  1
a2 b2 c2
Hence the given pair of equations have no solution.

2. If p and q are the zeroes of the quadratic polynomial f(x) = 2x2 – 7x + 3, find the value of p + q –
pq is
(a) 1 (b) 2 (c) 3 (d) None of these
Ans: (b) 2
Here, a = 2, b = -7 and c = 3
p + q = -b/a = 7/2 and pq = c/a = 3/2
7 3 73 4
p  q  pq     2
2 2 2 2

1
3. If sin 2A = tan² 45° where A is an acute angle, then the value of A is
2
(a) 60° (b) 45° (c) 30° (d) 15°
Ans: (d) 15°
1 1 1
sin 2 A  tan 2 450  12   sin 300  2 A  300  A  150
2 2 2

Prepared by: M. S. KumarSwamy, TGT(Maths) Page - 1 -


4. A girl walks 200m towards East and then 150m towards North. The distance of the girl from the
starting point is
(a) 350m (b) 250m (c) 300 m (d) 325 m
Ans: (b) 250m
Let girl starting point is A and she goes to B towards east covering 200 m distance .
Now from B she moves 150 m towards north let at point C.

Then, using Pythagoras theorem we get,


⇒ AC = √(AB² + BC²) ⇒ AC = √(200² + 150²) ⇒ AC = √(40000 + 22500)
⇒ AC = √62500 ⇒ AC = 250 m

5. In ΔABC right angled at B, if cotC = √3 , then then cosAsinC + sinAcosC =


(a) –1 (b) 0 (c) 1 (d) √3 / 2
Ans: (c) 1
cot C = √3 = cot 30°, so, ∠C = 30°,
Hence, ∠A = 60°.
1 1 3 3 1 3 1 3 4
So, cos A sin C + sin A cos C         1
2 2 2 2 4 4 4 4

6. If is an acute angle and tan + cot = 2, then the value of sin3 + cos3 is
1 2
(a) 1 (b) (c) (d) 2
2 2
2
Ans: (c)
2
1
tan   cot   2  tan    2  tan 2   2 tan   1  0
tan 
 (tan   1)  0  tan   1  tan 450    450
2

3 3
3 3 3 0  1   1 
3 0 1 1 1 2
Now,sin   cos   sin 45  cos 45         
 2  2 2 2 2 2 2 2

7. If the distance between the points (4,p) and (1,0) is 5, then value of p is
(a) 4 only (b) ±4 (c) -4 only (d) 0
Ans: (b) ±4
The points given are (4, p) and (1, 0)
By distance formula, 5 = √(4 − 1)2 + p2
⇒ 25 = (4 − 1)2 + p2 ⇒ 25 = 32 + p2 ⇒ 25 = 9 + p2
⇒ p2 = 25 – 9 = 16 ⇒ p = ±4
8. If p and q are positive integers such that p = a3b2 and q = a2b, where ‘a’ and ‘b’ are prime
numbers, then the LCM ( p, q) is …..
(a) ab (b) a2b2 (c) a3b2 (d) a3b3

Prepared by: M. S. KumarSwamy, TGT(Maths) Page - 2 -


Ans: (c) a3b2

9. 108 can be expressed as a product of its primes as ……………..


(a) 23 x 32 (b) 23 x 33 (c) 22 x 32 (d) 22 x 33
Ans: (d) 22 x 33
108 = 2 x 2 x 3 x 3 x 3 = 22 x 33

10. If angle between two radii of a circle is 130⁰, the angle between the tangents at the ends of the
radii is :
(a) 90⁰ (b) 50⁰ (c) 70⁰ (d) 40⁰
Ans: (b) 50⁰

11. The relationship between mean, median and mode for a moderately skewed distribution is
(a) mode = median – 2 mean (b) mode = 3 median – 2 mean
(c) mode = 2 median – 3 mean (d) mode = median – mean
Ans: (b) mode = 3 median – 2 mean

12. For the following distribution:


Marks Below Below Below Below Below Below
10 20 30 40 50 60
No. of Students 3 12 27 57 75 80
the modal class is
(a) 10 – 20 (b) 20 – 30 (c) 30 – 40 (d) 50 – 60
Ans: (c) 30 – 40
Marks 0 – 10 10 – 20 20 – 30 30 – 40 40 – 50 50 – 60
No. of Students 3 9 15 30 18 5
Highest frequency is 30 which belong to 30 – 40. Hence, Modal class is 30 – 40

13. The area of a quadrant of a circle, whose circumference is 22 cm, is


11 2 77 77 77
(a) cm (b) cm2 (c) cm2 (d) cm2
8 8 2 4
77
Ans: (b) cm2
8
Let the radius of the circle be 'r'
Circumference (C) = 22 cm
⇒ radius (r) = C/2π = 22/(2 × 22/7) = (22 × 7)/(2 × 22) = 7/2 cm
Therefore, the area of a quadrant = 1/4 × πr2
= 1/4 × 22/7 × 7/2 × 7/2
= 77/8 cm2

14. If the quadratic equation x2 + 4x + k = 0 has real and equal roots, then
(a) k < 4 (b) k > 4 (c) k = 4 (d) k ≥ 4
Ans: (c) k = 4
For a quadratic equation to have equal and real roots the discriminant should be equal to zero.
D = 0. Now, D = b2 – 4ac
⇒ 0 = (4)2 – 4 (1)(k) ⇒ 0 = 16 – 4k ⇒ 4k = 16 ⇒ k = 16/4 ⇒ k = 4

15. Volumes of two spheres are in the ratio 64 : 27. The ratio of their surface areas is
(a) 3 : 4 (b) 4 : 3 (c) 9 : 16 (d) 16 : 9
Ans: (d) 16 : 9
Let the radius of two spheres be r1 and r2.
Given, the ratio of the volume of two spheres = 64 : 27

Prepared by: M. S. KumarSwamy, TGT(Maths) Page - 3 -


4 3
r
V1 3 1 r13 64 r 4
  3  1 
V2 4  r 3 r2 27 r2 3
2
3
Let the surface areas of the two spheres be S1 and S2.
2
S1 4 r12 r12  4  16
∴     
S 2 4 r2 2 r2 2  3  9

16. The area of the square that can be inscribed in a circle of radius 8 cm is
(a) 256 cm² (b) 128 cm² (c) 64√2 cm² (d) 64 cm²
Ans: (b) 128 cm²
Radius of circle =8 cm
⇒ Diameter = 8 × 2=16 cm
The diameter of circle = diagonal of square =16 cm = a√2
⇒ a = 16/√2 = 8√2
Area of square = (side)2 = (8√2)2 = 128 cm2

17. Two dice are thrown at the same time and the product of numbers appearing on them is noted.
The probability that the product is a prime number is
(a) 1/3 (b) 1/6 (c) 1/5 (d) 5/6
Ans: (b) 1/6
Total number of possible outcomes = 36
Now for the product of the numbers on the dice is prime number can be have in these possible
ways = (1, 2), (2, 1), (1, 3), (3, 1), (5, 1), (1, 5)
So, number of possible ways = 6
∴ Required probability = 6/36 = 1/6

18. In ABC, DE || AB, If CD = 3 cm, EC = 4 cm, BE = 6 cm, then DA is equal to


(a) 7.5 cm (b) 3 cm (c) 4.5 cm (d) 6 cm
Ans: (c) 4.5 cm
CD CE 3 4 9
DE || AB      DA   4.5cm
DA EB DA 6 2

DIRECTION: In the question number 19 and 20, a statement of Assertion (A) is followed by a
statement of Reason (R).
Choose the correct option

19. Assertion (A): The point (0, 4) lies on y -axis.


Reason (R): The x co-ordinate on the point on y -axis is zero.
(a) Both Assertion (A) and Reason (R) are true and Reason (R) is the correct explanation of
Assertion (A)
(b) Both assertion (A) and reason (R) are true and reason (R) is not the correct explanation of
Assertion (A)
(c) Assertion (A) is true but reason(R) is false.
(d) Assertion (A) is false but reason(R) is true.
Ans: (a) Both Assertion (A) and Reason (R) are true and Reason (R) is the correct explanation of
Assertion (A)

20. Assertion (A): If HCF ( 90, 144) = 18, then LCM (90, 144) = 720
Reason (R): HCF (a, b) x LCM (a, b) = a x b
(a) Both Assertion (A) and Reason (R) are true and Reason (R) is the correct explanation of
Assertion (A).

Prepared by: M. S. KumarSwamy, TGT(Maths) Page - 4 -


(b) Both Assertion (A) and Reason (R) are true but Reason (R) is not the correct explanation of
Assertion (A).
(c) Assertion (A) is true but Reason (R) is false.
(d) Assertion (A) is false but Reason (R) is true.
Ans: (a) Both Assertion (A) and Reason (R) are true and Reason (R) is the correct
explanation of Assertion (A).
HCF x LCM = Product of two numbers
90  144
 LCM   5  144  720
18

SECTION – B
Questions 21 to 25 carry 2 marks each.

21. The length of the minute hand of a clock is 14 cm. Find the area swept by the minute hand in 5
minutes.
Ans: We know that the minute hand completes one rotation in 1 hour or 60 minutes.
Length of the minute hand (r) = 14 cm
Area swept by minute hand in 1 minute = πr2/60
Thus, area swept by minute hand in 5 minutes = (πr2/60) × 5 = πr2/12
= 1/12 × 22/7 × 14 × 14 cm2
= 154/3 cm2
OR
In a circle of radius 21 cm, an arc subtends an angle of 60° at the centre. Find (i) the length of the
arc (ii) area of the sector formed by the arc
Ans: Here, r = 21 cm, θ = 60°

Area of the segment APB = Area of sector AOPB - Area of ΔAOB


(i) Length of the Arc, APB = θ/360° × 2πr
= 60°/360° × 2 × 22/7 × 21 cm
= 22 cm
(ii) Area of the sector, AOBP = θ/360° x πr2
= 60°/360° × 22/7 × 21 × 21 cm2 = 231 cm2

22. For what value of k will the following system of linear equations have no solution?
3x + y = 1; (2k – 1) x + (k – 1) y = 2k + 1
Ans: 3x + y – 1 = 0
(2k – 1)x + (k – 1)y – 2k – 1 = 0
a1 3 b1 1 c1 1 1
 ;  ;  
a2 2k  1 b2 k  1 c2 2k  1 2k  1
a b c
For no solutions, 1  1  1
a2 b2 c2
3 1 1 3 1
     ⇒ 3k – 3 = 2k – 1 ⇒ k = 2
2 k  1 k  1 2k  1 2k  1 k  1

Prepared by: M. S. KumarSwamy, TGT(Maths) Page - 5 -


23. From a point P, two tangents PA and PB are drawn to a circle C(0, r). If OP = 2r, then find ∠APB.
Prove that triangle APB is an equilateral triangle.

Ans: Let APO  


OA 1
 sin     sin 300    300
OP 2
 APO  2  2(300 )  600
Also, PAB  PBA  600 ( PA  PB)
⇒ ∆APB is an equilateral triangle.

1
24. If tan (A + B) = 3 and tan (A – B) = ; 0° < A+B ≤ 90°; A > B, find A and B.
3
Ans: tan(A + B) = √3 = tan 60°
⇒ A + B = 60° ……(i)
tan(A – B) = 1/√3 = tan 30° ⇒ A – B = 30° …… (ii)
Adding equation (i) and (ii),
2A = 90° ⇒ A = 45°
Putting the value of A in equation (i), 45° + B = 60°
⇒ B = 60°- 45° ⇒ B = 15°
OR
If xsin3θ + ycos3θ = sinθ cosθ and xsinθ = ysinθ then find x2 + y2.
Ans:

25. ABCD is a trapezium in which AB || CD and its diagonals intersect each other at the point O.
OA OB
Using a similarity criterion of two triangles, show that 
OC OD
Ans: Given : ABCD is a trapezium, AB II CD

Prepared by: M. S. KumarSwamy, TGT(Maths) Page - 6 -


In ∆AOB and ∆COD
∠OBA = ∠ODC ----eq.1 (alt.angles are equal)
∠OAB = ∠OCD ----eq.2 (alt.angles are equal)
Therefore, ∆AOB ∼ ∆COD( A.A Similarity)
OA OB
Hence, 
OC OD

SECTION – C
Questions 26 to 31 carry 3 marks each.
26. The sum of the digits of a two-digit number is 9. Also 9 times this number is twice the number
obtained by reversing the order of the digits. Find the number.
Ans: Let the tens digits and unit digit of the number be x and y respectively. Then, the number
will be 10x + y
Number after reversing the digits is 10y + x
According to the question,
x + y = 9... (i)
9(10x + y) = 2(10y + x)
⇒ 88x – 11y = 0 ⇒ 8x + y = 0... (ii)
Adding equation (i) and (ii), we get
9x = 9 ⇒ x = 1
Putting the value in equation (i), we get y = 8
Hence, the number is 18.
OR
Yash scored 40 marks in a test, getting 3 marks for each right answer and losing 1 mark for each
wrong answer. Had 4 marks been awarded for each correct answer and 2 marks been deducted for
each incorrect answer, then Yash would have scored 50 marks. How many questions were there in
the test?
Ans: Let x be the number of right answers and y be the number of wrong answers.
According to the question,
3x – y = 40 ….(i)
and, 2x – y = 25 ….(ii)
On subtraction, we get: x = 15
putting the value of x in (i), we get 3 (15)-y = 40 y = 5
Number of right answers= 15 answers
Number of wrong answers= 5 answers.
Total Number of questions = 5 + 15 = 20

27. Prove that (sinA + cosecA)2 + (cosA + secA)2 = 7 + tan2A + cot2A


Ans: L.H.S = (sinA + cosecA)2 + (cosA + secA)2
=sin2 A + cosec2 A +2sinAcosecA + cos2 A + sec2 A +2cosAsecA
= sin2 A + cos2 A +cosec2 A+sec2 A +2sinA×1/sinA + 2cosA×1/cosA
Since, (sin2 A + cos2 A =1)
(sec2 A =1 + tan2A, cosec2 A = 1 + cot2A)
= 1 + 1 + cot2A +1 + tan2A +2 +2
= 7 + tan2A +cot2A = RHS

28. Prove that √5 is an irrational number.


p
Ans: Let 5 is a rational number then we have 5 , where p and q are co-primes.
q
 p  5q
Squaring both sides, we get p 2  5q 2
 p2 is divisible by 5  p is also divisible by 5
Prepared by: M. S. KumarSwamy, TGT(Maths) Page - 7 -
So, assume p = 5m where m is any integer.
Squaring both sides, we get p2 = 25m2
But p 2  5q 2
Therefore, 5q2 = 25m2  q2 = 5m2
 q2 is divisible by 5  q is also divisible by 5
From above we conclude that p and q have one common factor i.e. 5 which contradicts that p and
q are co-primes.
Therefore, our assumption is wrong.
Hence, 5 is an irrational number.

29. Find the zeroes of the quadratic polynomial x2 – 2x – 8 and verify the relationship between the
zeroes and the coefficients of the polynomial.
Ans: Given, f(x)=x2 − 2x – 8
The zeroes of f(x) are given by, f(x) = 0
⇒ x2 + 2x − 4x – 8 = 0 ⇒ x (x + 2) − 4(x + 2) = 0 ⇒ (x + 2) (x − 4) = 0
⇒ x = −2 (or) x = 4
Hence, the zeros of f(x) = x2 − 2x − 8 are α = −2 and β = 4
b
α + β = -2 + 4 = 2 = =2
a
c
αβ = -2 x 4= -8 = =-8
a

30. Prove that the lengths of the tangents drawn from an external point to a circle are equal.
Ans: Given, To Prove, Constructions and Figure – 1½ marks
Correct Proof – 1½ marks
OR
In the figure XY and X'Y' are two parallel tangents to a circle with centre O and another tangent
AB with point of contact C interesting XY at A and X'Y' at B, what is the measure of ∠AOB.

Ans: Join OC. Since, the tangents drawn to a circle from an external point are equal.
∴ AP = AC

In Δ PAO and Δ AOC, we have:


AO = AO [Common]

Prepared by: M. S. KumarSwamy, TGT(Maths) Page - 8 -


OP = OC [Radii of the same circle]
AP = AC
⇒ Δ PAO ≅ Δ AOC [SSS Congruency]
∴ ∠PAO = ∠CAO = ∠1
∠PAC = 2 ∠1 ...(1)
Similarly ∠CBQ = 2 ∠2 ...(2)
Again, we know that sum of internal angles on the same side of a transversal is 180°.
∴ ∠PAC + ∠CBQ = 180°
⇒ 2 ∠1 + 2 ∠2 = 180° [From (1) and (2)]
⇒ ∠1 + ∠2 = 180°/2 = 90° ...(3)
Also ∠1 + ∠2 + ∠AOB = 180° [Sum of angles of a triangle]
⇒ 90° + ∠AOB = 180°
⇒ ∠AOB = 180° − 90°
⇒ ∠AOB = 90°.

31. Two dice are thrown at the same time. What is the probability that the sum of the two numbers
appearing on the top of the dice is
(i) 8? (ii) 7? (iii) less than or equal to 12?
Ans: (i) Number of outcomes with sum of the numbers 8 = 5
∴ Required Probability = 5/36
(ii) Number of outcomes with sum of the numbers 7 = 6
∴ Required Probability = 6/36 = 1/6
(iii) Number of outcomes with sum of the numbers less than or equal to 12 = 36
∴ Required Probability = 36/36 = 1

SECTION – D
Questions 32 to 35 carry 5 marks each.

32. State and Prove Basic Proportionality Theorem.


Ans: Statement – 1 mark
Given, To prove, Construction and figure of 2 marks
Proof of 2 marks

33. A person on tour has Rs.360 for his expenses. If he extends his tour for 4 days, he has to cut down
his daily expenses by Rs.3. Find the original duration of the tour.
Ans: Let days be the original duration of the tour.
Total expenditure on tour ₹ 360
Expenditure per day ₹ 360/x
Duration of the extended tour (x + 4) days
Expenditure per day according to the new schedule ₹ 360/(x + 4)
Given that daily expenses are cut down by ₹ 3
360 360
As per the given condition,  3
x x4
1 1 
 360   3
 x x4
1 1  3 1
   
 x x  4  360 120
x4 x 1 4 1
   
x( x  4) 120 x( x  4) 120
⇒ x(x + 4) = 480
⇒ x² + 4x = 480
⇒ x² + 4x – 480 = 0
Prepared by: M. S. KumarSwamy, TGT(Maths) Page - 9 -
⇒ x² + 24x – 20x – 480 = 0
⇒ x(x + 24) - 20(x + 24) = 0
⇒ x – 20 = 0 or x + 24 = 0
⇒ x = 20 or x = -24
Since the number of days cannot be negative. So, x = 20
Therefore, the original duration of the tour was 20 days

OR
Rs.6500 were divided equally among a certain number of persons. Had there been 15 more
persons, each would have got Rs.30 less. Find the original number of persons.
Ans: Let the original number of persons be x
Total money which was divided = Rs. 6500
Each person share = Rs. 6500/x
6500 6500
According to the question,   30
x x  15
6500 x  97500  6500 x
  30
x ( x  15)
97500 3250
  30  1
x ( x  15) x( x  15)
⇒ x² + 15x – 3250 = 0
⇒ x² + 65x – 50x – 3250 = 0
⇒ x(x + 65) – 50(x + 65) = 0
⇒ (x + 65)(x – 50) = 0
⇒ x = -65, 50
Since the number of persons cannot be negative, hence the original numbers of person is 50

34. Ramesh made a bird-bath for his garden in the shape of a cylinder with a hemispherical
depression at one end. The height of the cylinder is 1.45 m and its radius is 30 cm. Find the total
surface area of the bird-bath.

Ans: Let h be height of the cylinder, and r the common radius of the cylinder and hemisphere.
Then, the total surface area = CSA of cylinder + CSA of hemisphere
= 2 rh + 2 r2 = 2 r (h + r)
22
=2x x 30 (145 + 30) cm2
7
22
=2x x 30 x 175 cm2
7
= 33000 cm2 = 3.3 m2
OR
A tent is in shape of a cylinder surmounted by a conical top. If the height and diameter of the
cylindrical part are 2.1m and 4m respectively and the slant height of the top is 2.8m. Find the area

Prepared by: M. S. KumarSwamy, TGT(Maths) Page - 10 -


of canvas used for making the tent. Also find the cost of canvas of the tent at the rate of 500 per
m2 .
Ans: Radius = 2m, Slant height l= 2.8m , height h= 2.1m
Cost of canvas per m2= Rs.500
Area of canvas used = CSA of cone + CSA of cylinder
= πrl + 2πrh
=22/7 x 2 x 2.8 + 2 x 22/7 x 2 x 2.1
=17.6 + 26.4
=44m2
Cost of the canvas of tent =44 x 500
=Rs.22,000

35. The following frequency distribution gives the monthly consumption of 68 consumers of a
locality. Find median, mean and mode of the data and compare them.
Monthly consumption of Number of consumers
electricity (in units)
65-85 4
85-105 5
105-125 13
125-145 20
145-165 14
165-185 8
185-205 4
Ans: For mean , median , mode
To calculate xi , cumulative frequency , identifying highest frequency
Formulae for mean , median, mode

Mean = a   i i  135 
fd 140
 fi 68
=137.05
n 
 2  cf   34  22 
Median = l    h  = 125    20   125  12  137
 f   20 
 
 f1  f 0   20  13 
Mode = l    h   125    20 
 2 f1  f 0  f 2   40  13  14 
7 
 125    20   125  10.77  135.77
 13 

SECTION – E(Case Study Based Questions)


Questions 36 to 38 carry 4 marks each.

36. Case Study-2


In order to conduct sports day activities in your school, lines have been drawn with chalk powder
at a distance of 1 m each in a rectangular shaped ground ABCD. 100 flower pots have been
placed at the distance of 1 m from each other along AD, as shown in the following figure.
1 1
Niharika runs ( )th distance AD on the 2nd line and posts a green Flag. Preet runs ( ) th
4 5
distance AD on the eighth line and posts are red flags. Taking A as the origin AB along x-axis
and AD along y-axis, answer the following questions:
(i) Find the coordinates of the green flag. (1)
Prepared by: M. S. KumarSwamy, TGT(Maths) Page - 11 -
(ii) Find the distance between the two flags. (1)
(iii) If Rashmi has to post a blue flag exactly halfway between the line segment joining the two
flags, where should she post her flag? (2)
OR
(iii) If Joy has to post a flag at one fourth distance from the green flag, in the line segment joining
the green and red flags, then where should he post his flag? (2)

 1 
Ans: (i) Position of the red flag is  2, 100   (2, 25)
 4 
(ii) Distance between the two flags = (36  25) = 61cm
 2  8 25  20 
(iii) Position of the blue flag =  , 
 2 2 
  5, 22.5 
OR

Required point is (3.5, 23.75)

37. Case Study – 3


Lakshaman Jhula is located 5 kilometers north-east of the city of Rishikesh in the Indian state of
Uttarakhand. The bridge connects the villages of Tapovan to Jonk. Tapovan is in Tehri Garhwal
district, on the west bank of the river, while Jonk is in Pauri Garhwal district, on the east bank.
Lakshman Jhula is a pedestrian bridge also used by motorbikes. It is a landmark of Rishikesh. A
group of Class X students visited Rishikesh in Uttarakhand on a trip. They observed from a point
(P) on a river bridge that the angles of depression of opposite banks of the river are 60° and 30°
respectively. The height of the bridge is about 18 meters from the river.

Prepared by: M. S. KumarSwamy, TGT(Maths) Page - 12 -


Based on the above information answer the following questions.
(i) Find the distance PA. (1)
(ii) Find the distance PB (1)
(iii) Find the width AB of the river. (2)
OR
(iii) Find the height BQ if the angle of the elevation from P to Q be 30°. (2)
PC
Ans: (i) sin 600 
PA
3 18 36
   PA   12 3m
2 PA 3
PC
(ii) sin 300 
PB
1 18
   PB  36 m
2 PB
PC 18
(iii) tan 60 0   3  AC  6 3m
AC AC
PC 1 18
tan 300     CB  18 3m
CB 3 CB
Width AB = AC + CB = 6 3m  18 3m  24 3m

OR

RB = PC = 18 m & PR = CB = 18√3m
QR 1 QR
tan 300     QR  18m
PR 3 18 3
QB = QR + RB = 18 + 18 = 36 m.
Hence height BQ is 36m.

38. Case Study-1


Mohan takes a loan from a bank for his car. Mohan replays his total loan of Rs.118000 by paying
every month starting with the first instalment of Rs.1000. If he increases the instalment by Rs.100
every month.

Prepared by: M. S. KumarSwamy, TGT(Maths) Page - 13 -


(i) What is the first term and common difference of given question. (1)
(ii) The amount paid buy him in 30th instalment. (1)
(iii) The amount paid by him in the 30 instalments is (2)
(OR)
(iii) What amount does he still have to pay after 30th instalment? (2)
Ans: (i) Given , a1 =1000
Common difference, d =100
Total loan= Rs.1,18,000
(ii)a30 = a + 29d
= 1000 + 29 x 100
= 3900
Amount paid in 30th installment is Rs.3900
30
(iii) S30 = [2x 1000 +(30 -1) x 100]
2
=15 x 4900
=73,500
Amount paid in 30 installments is Rs.73,500
(OR)
The amount he still have to pay after 30 installments=Rs.118000 – Rs. 73,500
=Rs.44,500

Prepared by: M. S. KumarSwamy, TGT(Maths) Page - 14 -


PM SHRI KENDRIYA VIDYALAYA GACHIBOWLI,GPRA CAMPUS,HYD-32
SAMPLE PAPER TEST 08 FOR BOARD EXAM 2024
(ANWERS)
SUBJECT: MATHEMATICS MAX. MARKS : 80
CLASS : X DURATION : 3 HRS
General Instruction:
1. This Question Paper has 5 Sections A-E.
2. Section A has 20 MCQs carrying 1 mark each.
3. Section B has 5 questions carrying 02 marks each.
4. Section C has 6 questions carrying 03 marks each.
5. Section D has 4 questions carrying 05 marks each.
6. Section E has 3 case based integrated units of assessment (04 marks each) with sub-parts of the
values of 1, 1 and 2 marks each respectively.
7. All Questions are compulsory. However, an internal choice in 2 Qs of 5 marks, 2 Qs of 3 marks
and 2 Questions of 2 marks has been provided. An internal choice has been provided in the
2marks questions of Section E
8. Draw neat figures wherever required. Take π =22/7 wherever required if not stated.
SECTION – A
Questions 1 to 20 carry 1 mark each.
1. The pair of linear equations 2x + 3y = 5 and 4x + 6y = 10 is
(a) inconsistent (b) consistent (c) dependent consistent (d) none of these
Ans: (c)
a1 2 1 b1 3 1 c1 5 1
  ,   ,  
a2 4 2 b2 6 2 c2 10 2
a1 b1 c1
  
a2 b2 c2
Therefore, the pair of linear equations has infinity many solutions and hence dependent
consistent

2. Points A(3, 1), B(5, 1), C(a, b) and D(4, 3) are vertices of a parallelogram ABCD. The values of
a and b are respectively
(a) a = 6, b = 3 (b) a = 2, b = 1 (c) a = 4, b = 2 (d) None of these
Ans: (a) a = 6, b = 3

3. If ΔABC ~ ΔEDF and ΔABC is not similar to ΔDEF, then which of the following is not true?
(a) BC · EF = AC · FD (b) AB · EF = AC · DE
(c) BC · DE = AB · EF (d) BC · DE = AB · FD
Ans: (c) BC · DE = AB · EF

4. If sec A = 15/7 and A + B = 90°, find the value of cosec B.


(a) 8/7 (b) 12/7 (c) 7/15 (d) 15/7
Prepared by: M. S. KumarSwamy, TGT(Maths) Page - 1 -
Ans: (d)

5. The LCM of two numbers is 14 times their HCF. The sum of LCM and HCF is 600. If one
number is 280, then the other number is
(a) 20 (b) 28 (c) 60 (d) 80
Ans: (d) According to the question, LCM + HCF = 600
Since LCM = 14 × HCF
⇒ 14 × HCF + HCF = 600 ⇒ 15 × HCF = 600 ⇒ HCF = 600 ÷ 15 = 40
⇒ LCM = 600 – HCF = 600 – 40 = 560
We know that HCF (a, b) × LCM (a, b) = a × b
⇒ Other number = 40 x 560 /280 = 80

6. When 2120 is expressed as the product of its prime factors we get


(a) 2 × 5³ × 53 (b) 2³ × 5 × 53 (c) 5 × 7² × 31 (d) 5² × 7 × 33
3
Ans: (b) 2120 = 2 × 2 × 2 × 5 × 53 = 2 × 5 × 53

7. If p and q are the zeroes of the quadratic polynomial f(x) = 2x2 – 7x + 3, find the value of p + q
– pq is
(a) 1 (b) 2 (c) 3 (d) None of these
Ans: (b) 2

8. ABCD is a trapezium with AD ∥ BC and AD = 4cm. If the diagonals AC and BD intersect each
other at O such that AO/OC = DO/OB =1/2, then BC =
(a) 6cm (b) 7cm (c) 8cm (d) 9cm
Ans: (c) 8cm

9. If the angle between two radii of a circle is 140°, then the angle between the tangents at the ends
of the radii is
(a) 90° (b) 50° (c) 70° (d) 40°
Ans: (d) 40°

10. The number of revolutions made by a circular wheel of radius 0.7 m in rolling a distance of 176
m is
(a) 22 (b) 24 (c) 75 (d) 40
Ans: (d) 40

11. In ΔABC, right angled at B, AB = 5 cm and sin C = 1/2. Determine the length of side AC.
(a) 10 cm (b) 15 cm (c) 20 cm (d) none of these
Ans: (a)

Prepared by: M. S. KumarSwamy, TGT(Maths) Page - 2 -


AB 1 5
sin C     AC  10cm
AC 2 AC

12. In the ∆ABC, D and E are points on side AB and AC respectively such that DE || BC. If AE = 2
cm, AD = 3 cm and BD = 4.5 cm, then CE equals

(a) 1 cm (b) 2 cm (c) 3 cm (d) 4 cm


Ans: (c) 3 cm
AD AE 3 2
    CE  3cm
BD CE 4.5 CE

13. The median class of the following data is:


Marks 0 – 10 10 – 20 20 – 30 30 – 40 40 – 50 50 – 60
No. of students 8 10 12 22 30 18
(a) 20 – 30 (b) 30 – 40 (c) 40 – 50 (d) 50 – 60
Ans: (b)

14. Two dice are thrown simultaneously. What is the probability of getting doublet?
(a) 1/36 (b) 1/6 (c) 5/6 (d) 11/36
Ans: (b) 1/6
Number of Possible outcomes are 36
Number of favourable outcomes = 6
Probability = 6/36 = 1/6

4sin   cos 
15. If 4 tan  = 3, then the value of is
4sin   cos 
(a) 1/2 (b) 1/3 (c) 1/4 (d) 1/5
Ans: (a) 1/2
Dividing Numerator and Denominator by cos, we get
3
4sin   cos  4 tan   1 4  4  1 3  1 2 1
    
4sin   cos  4 tan   1 4  3  1 3  1 4 2
4
16. The area of the square that can be inscribed in a circle of radius 8 cm is
(a) 256 cm² (b) 128 cm² (c) 64√2 cm² (d) 64 cm²
Ans: (b) 128 cm²

17. The ratio of the total surface area to the lateral surface area of a cylinder with base radius 80 cm
and height 20 cm is
(a) 1 : 2 (b) 2 : 1 (c) 3 : 1 (d) 5 : 1
Ans: (d) 5 : 1

18. The mean and mode of a frequency distribution are 28 and 16 respectively. The median is
(a) 22 (b) 23.5 (c) 24 (d) 24.5
Ans: (c) 24
We know that, Mode = 3 Median – 2 Mean
⇒ 3 Median = Mode + 2 Mean
⇒ 3 Median = 16 + 2 × 28 ⇒ Median = 72/3 = 24

DIRECTION: In the question number 19 and 20, a statement of Assertion (A) is followed by a
statement of Reason (R).
Choose the correct option
Prepared by: M. S. KumarSwamy, TGT(Maths) Page - 3 -
19. Statement A (Assertion): The value of y is –6, for which the distance between the points P(2, –
3) and Q(10, y) is 10.
Statement R( Reason) : Distance between two given points A (x1, y1) and B (x2, y2) is given by
AB = ( x2  x1 ) 2  ( y2  y1 )2
(a) Both assertion (A) and reason (R) are true and reason (R) is the correct explanation of
assertion (A)
(b) Both assertion (A) and reason (R) are true and reason (R) is not the correct explanation of
assertion (A)
(c) Assertion (A) is true but reason (R) is false.
(d) Assertion (A) is false but reason (R) is true.
Ans: (d) Assertion (A) is false but reason (R) is true.

20. Statement A (Assertion): The number 6n never end with digit 0 for any natural number n..
Statement R( Reason) : The number 9n never end with digit 0 for any natural number n.
(a) Both assertion (A) and reason (R) are true and reason (R) is the correct explanation of
assertion (A)
(b) Both assertion (A) and reason (R) are true and reason (R) is not the correct explanation of
assertion (A)
(c) Assertion (A) is true but reason (R) is false.
(d) Assertion (A) is false but reason (R) is true.
Ans: (b) Both assertion (A) and reason (R) are true and reason (R) is not the correct explanation
of assertion (A)

SECTION – B
Questions 21 to 25 carry 2 marks each.

21. In the given figure, find the value of x in terms of a, b and c.

Ans: In ∆s LMK and PNK, we have


∠M = ∠N = 50⁰ and ∠K = ∠K
So, by AA similarity criterion, ∆LMK ~ ∆PNK
LM KM a bc ac
Thus,    x
PN KN x c bc

22. XY and MN are the tangents drawn at the end points of the
diameter DE of the circle with centre O. Prove that XY ||
MN.
Ans: Since, XY is the tangent to the circle at the point D.
⇒ OD ⊥ XY ⇒ ∠EDX = 90°
Also, MN is the tangent to the circle at E.
⇒ OE ⊥ MN ⇒ ∠ DEN = 90°
As, ∠EDX = ∠DEN (each 90°)
which are alternate interior angles.
⇒ XY ||MN
23. A rope by which a cow is tethered is increased from 16mto 23m. How much additional ground
does it have now to graze?
Ans: Given : length of rope (r) = 16 m
Increased length of rope (R) = 23 m
Prepared by: M. S. KumarSwamy, TGT(Maths) Page - 4 -
Hence the additional area cow can graze =  R 2   r 2   ( R 2  r 2 )
22 22
 (232  162 )  (529  256)
7 7
22
  273  858m 2
7
OR
In the below figure, OACB is a quadrant of a circle with centre O and radius 3.5 cm. If OD = 2
cm, find the area of the (i) quadrant OACB, (ii) shaded region.

1 2 1 22 7 7 77
Ans: (i) Area of the quadrant OACB =  r =    = =9.625 cm2
4 4 7 2 2 8
(ii) Area of the BOD = (1/2) x OB x OD = (1/2) x 3.5 x 2 = 3.5 cm2
Area of the shaded region = Area of the quadrant OACB – Area of the BOD
= 9.625 – 3.5 = 6.125 cm2

24. In figure, ABCD is a rectangle. Find the values of x and y.

Ans: AB = DC and BC = AD
⇒ x + y = 30 ...(i)
and x – y = 14 ...(ii)
Adding (i) and (ii), we get 2x = 44 ⇒ x = 22
⇒ y = 30 - 22 = 8
Thus, x = 22 and y = 8

25. Find A and B, if sin (A + 2B) = √3/2 and cos (A + B) = 1/2.


Ans: Given : sin (A + 2B) = sin 60°
⇒ A + 2B = 60° ...(i)
cos (A + B) = cos 60°
⇒ A + B = 60° ...(ii)
Subtracting equation (i) and (ii), we get B = 0°
Putting the value of B in equation (ii), we get,
A = 60° – 0° = 60°
So, A = 60° and B = 0°.
OR
If (1 + cos A) (1 – cos A) = 3/4 , find the value of tan A.
Ans: (1 + cos A) (1 – cos A) = 3/4
⇒ 1 – cos²A = 3/4 ⇒ cos²A = 1 – 3/4 = 1/4 ⇒ cosA = ±1/2
Also, 1 – cos²A = 3/4 ⇒ sin²A = 3/4 ⇒ sinA = ±√3/2
⇒ tanA = sinA/cosA = ±√3

Prepared by: M. S. KumarSwamy, TGT(Maths) Page - 5 -


SECTION – C
Questions 13 to 22 carry 3 marks each.
26. A part of monthly hostel charges in a college is fixed and the remaining depends on the number
of days one has taken food in the mess. When a student ‘A’ takes food for 22 days, he has to
pay Rs. 1380 as hostel charges; whereas a student ‘B’, who takes food for 28 days, pays Rs.
1680 as hostel charges. Find the fixed charges and the cost of food per day.
Ans: Let the fixed hostel charges be Rs. x and the cost of food per day be Rs. y.
According to the question, we get
x + 22y = 1380 ...(i)
and x + 28y = 1680 ...(ii)
Subtracting (i) from (ii), we get
6y = 300 ⇒ y = 300 ÷ 6 = 50
Putting y = 50 in (i), we get
x + 22(50) = 1380 ⇒ x + 1100 = 1380 ⇒ x = 280
∴ Fixed hostel charges = Rs. 280 and cost of the food per day = Rs. 50.
OR
The ratio of income of two persons is 9 : 7 and the ratio of their expenditure is 4 : 3, if each of
them manage to save Rs. 2000/month. Find their monthly incomes.
Ans: Let the income of first person be 9x and the income of second person be 7x. Further, let
the expenditures of first and second persons be 4y and 3y respectively. Then, Saving of the first
person = 9x – 4y
Saving of the second person = 7x – 3y
According to question,
9x – 4y = 2000 or 9x – 4y – 2000 = 0 ...(i)
and 7x – 3y = 2000 or 7x – 3y – 2000 = 0 ...(ii)
Solving (i) and (ii), we get x = 2000 and y = 4000
Thus, monthly income of first person = 9 × 2000 = Rs. 18000
Monthly income of second person = 7 × 4000 = Rs. 28000

sin   cos   1
27. Prove that:  sec   tan 
sin   cos   1
tan   1  sec 
And: LHS = tan   1  sec  (Dividing numerator and denominator by cos)

tan   sec   1

tan   1  sec 
tan   sec   (sec 2   tan 2  )

tan   1  sec 
(sec   tan  )(1  sec   tan  )

tan   1  sec 
 sec   tan  = RHS

28. Prove that √5 is and irrational number.


p
Ans: Let 5 is a rational number then we have 5 , where p and q are co-primes.
q
 p  5q
Squaring both sides, we get p 2  5q 2
 p2 is divisible by 5  p is also divisible by 5
So, assume p = 5m where m is any integer.
Squaring both sides, we get p2 = 25m2
But p 2  5q 2
Therefore, 5q2 = 25m2  q2 = 5m2

Prepared by: M. S. KumarSwamy, TGT(Maths) Page - 6 -


 q2 is divisible by 5  q is also divisible by 5
From above we conclude that p and q have one common factor i.e. 5 which contradicts that p
and q are co-primes.
Therefore, our assumption is wrong.
Hence, 5 is an irrational number.

29. Find the zeroes of the quadratic polynomial 6x2 – 7x – 3 and verify the relationship between the
zeroes and the coefficients of the polynomial.
Ans: 6x2 – 7x – 3 = 0
6x2 – 9x + 2x – 3 = 0
3x(2x – 3) + 1(2x – 3) = 0
 (3x + 1) (2x – 3) = 0
1 3
x = ,
3 2
1 3 2  9 7 b 7 b
Now,        and     
3 2 6 6 a 6 a
1 3 1 c 1 c
    and    
3 2 2 a 2 a
30. Prove that the intercept of a tangent between two parallel tangents to a circle subtends a right
angle at the center
Ans: Given: XY and X'Y' are two parallel tangents to the circle wth centre O and AB is the
tangent at the point C, which intersects XY at A and X'Y' at B.

In ΔOAP and ΔOAC


AP = AC ( Tangents from to same point A)
PO = OC ( Radii of the same circle)
OA = OA ( Common side)
so, ΔOAP = ΔOAC (SSS congruence criterion)
∴ ∠AOP =∠AOC = ∠1 (CPCT)
Similarly, ∠BOQ =∠BOC = ∠2
Now, POQ is a diameter of the circle.
Hence, it is a straight line.
∴ ∠1 + ∠1 + ∠2 + ∠2 = 180°
2(∠1 + ∠2) = 180°
∴ ∠1 + ∠2 = 90°
∴ ∠AOB = 90°.
OR
Prove that opposite sides of a quadrilateral circumscribing a circle subtend supplementary
angles at the centre of the circle.
Ans: Let ABCD be the quadrilateral circumscribing a circle at the center O such that it touches
the circle at the point P,Q,R,S. Let join the vertices of the quadrilateral ABCD to the center of
the circle

Prepared by: M. S. KumarSwamy, TGT(Maths) Page - 7 -


In ΔOAP and ΔOAS
AP=AS ( Tangents from to same point A)
PO=OS ( Radii of the same circle)
OA=OA ( Common side)
so, ΔOAP=ΔOAS (SSS congruence criterion)
∴ ∠POA=∠AOS (CPCT)
⇒ ∠1=∠8
Similarly, ∠2=∠3, ∠4=∠5 and ∠6=∠7
∠1+∠2 +∠3+∠4+∠5+∠6+∠7+∠8 = 360⁰
⇒ (∠1 +∠8) +(∠2 +∠3) + (∠4 +∠5) + (∠6 +∠7) = 360⁰
⇒ 2(∠1) + 2(∠2) + 2(∠5) + 2(∠6) = 360⁰
⇒ (∠1) + (∠2) + (∠5) + (∠6) = 180⁰
∴ ∠AOD + ∠COD=180⁰
Similarly, ∠BOC + ∠DOA = 180⁰

31. One card is drawn at random from a well-shuffled deck of 52 playing cards. Find the probability
that the card drawn is (i) either a red card or a king, (ii) neither a red card nor a queen.
Ans: Total number of cards = 52
(i) Number of either red card or a king card = 28
28 7
Required Probability = 
52 13
(ii) Number of cards neither a red card or a queen card = 52 – 28 = 24
24 6
Required Probability = 
52 13

SECTION – D
Questions 32 to 35 carry 5 marks each.

32. A vessel is in the form of an inverted cone. Its height is 8 cm and the radius of its top which is
open, is 5 cm. It is filled with water upto the brim. When lead shots, each of which is a sphere of
radius 0.5 cm are dropped into the vessel, one-fourth of water flows out. Find the number of
lead shots dropped into the vessel.
Ans: We have, height of the conical vessel, h = 8 cm
and radius of the conical vessel, r = 5 cm
1 1 200
∴ Volume of water filled in the vessel cone =  r 2 h    52  8   cm3
3 3 3
Also, we have radius of a spherical lead shot = 0.5 cm
4 4
∴ Volume of each lead shot =  r 3    (0.5)3 cm3
3 3
1 200 50
∴ Volume of lead shots dropped = Volume of water that overflows =   cm3   cm3
4 3 3

Prepared by: M. S. KumarSwamy, TGT(Maths) Page - 8 -


50

∴ Number of lead shots dropped = 3  100
4
  0.5  0.5  0.5
3
Hence required number of lead shots is 100.
OR
A copper wire of diameter 8 mm is evenly wrapped on a cylinder of length 24 cm and diameter
49 cm to cover the whole surface. Find (i) the length of the wire (ii) the volume of the wire.
Ans: The thickness of wire = its diameter = 8 mm = 0.8 cm.
And, the length of the cylinder = 24 cm
∴ Number of turns of the wire required to cover the whole surface of the cylinder
= Length of the cylinder/Diameter of the wire = 24/0.8 = 30
Since, diameter of the cylinder = 49 cm
∴ Radius of the cylinder, r = 49/2 cm
(i) Length of wire wrapped in 1 round = Circumference of the cylinder
22 49
= 2 r  2    154cm
7 2
Length of wire wrapped in 30 rounds = 30 × 154 cm = 4620 cm
0.8
(ii) Since radius (r) of wire =  0.4cm and its length or height (h) = 4620 cm
2
22
∴ Volume of the wire =  r 2 h   (0.4)2  4620  2323.2cm3
7
33. Prove that if a line is drawn parallel to one side of a triangle intersecting the other two sides in
distinct points, then the other two sides are divided in the same ratio.
Using the above theorem prove that a line through the point of intersection of the diagonals and
parallel to the base of the trapezium divides the non parallel sides in the same ratio.

Ans: For the Theorem :


Given, To prove, Construction and figure of 1½ marks
Proof of 1½ marks
Let ABCD be a trapezium DC ∥ AB and EF is a line parallel to AB and hence to DC.
Join AC, meeting EF in G.

3
34. Two water taps together can fill a tank in 9 hours. The tap of larger diameter takes 10 hours
8
less than the smaller one to fill the tank separately. Find the time in which each tap can
separately fill the tank. (NCERT Exercise 4.3 Q9)

OR
A rectangular park is to be designed whose breadth is 3 m less than its length. Its area is to be 4
square metres more than the area of a park that has already been made in the shape of an
isosceles triangle with its base as the breadth of the rectangular park and of altitude 12 m. Find
its length and breadth. NCERT Quadratic Equations Example-12, page no. 84

Prepared by: M. S. KumarSwamy, TGT(Maths) Page - 9 -


35. If the median of the distribution given below is 28.5, find the values of x and y.
Class 0-10 10-20 20-30 30-40 40-50 50-60 Total
Frequency 5 x 20 15 y 5 60

Ans: Here, median = 28.5, n = 60

SECTION – E(Case Study Based Questions)


Questions 35 to 37 carry 4 marks each.
36. Anita’s mother start a new shoe shop. To display the shoes, she put 3 pairs of shoes in 1st row, 5
pairs in 2nd row, 7 pairs in 3rd row and so on.

On the basis of above information, answer the following questions.


(i) If she puts a total of 120 pairs of shoes, then find the number of rows required.
(ii) What is the difference of pairs of shoes in 17th row and 10th row.
Ans: Number of pairs of shoes in 1st, 2nd, 3rd row, ... are 3, 5, 7, ...
So, it forms an A.P. with first term a = 3, d = 5 – 3 = 2
(i) Let n be the number of rows required.
∴ Sn = 120 ⇒ (n/2) [2(3) + (n − 1)2] = 120
⇒ n2 + 2n – 120 = 0 ⇒ n2 + 12n – 10n – 120 = 0
⇒ (n + 12) (n – 10) = 0 ⇒ n = 10

Prepared by: M. S. KumarSwamy, TGT(Maths) Page - 10 -


So, 10 rows required to put 120 pairs.
(ii) No. of pairs in 17th row = a17 = 3 + 16(2) = 35
No. of pairs in 10th row = a10 = 3 + 9(2) = 21
∴ Required difference = 35 – 21 = 14

37. A 1.2 m tall girl spots a balloon moving with the wind in a horizontal line at a height of 88.2 m
from the ground. The angle of elevation of the balloon from the eyes of the girl at any instant is
60°. After 30 seconds, the angle of elevation reduces to 30° (see the below figure).

Based on the above information, answer the following questions. (Take √3 =1.732)
(i) Find the distance travelled by the balloon during the interval.
(ii) Find the speed of the balloon.
Ans: (i) In the figure, let C be the position of the observer (the girl).
A and P are two positions of the balloon.
CD is the horizontal line from the eyes of the (observer) girl.
Here PD = AB = 88.2 m − 1.2 m = 87 m

Thus, the required distance between the two positions of the balloon = 58 √3 m
= 58 x 1.73 = 100.46 m (approx.)
(ii) Speed of the balloon = Distance/time = 100.46/30 = 3.35 m/s (approx.)

38. In the sport of cricket the Captain sets the field according to a plan. He instructs the players to
take a position at a particular place. There are two reasons to set a cricket field—to take wickets
and to stop runs being scored.
The following graph shows the position of players during a cricket match.
(i) Find the coordinate of the point on y-axis which are equidistant from the points representing
the players at Cover P(2, –5) and Mid-wicket Q(–2, 9)
(ii) Find the ratio in which x-axis divides the line segment joining the points Extra Cover S(3, –
3) and Fine Leg (–2, 7).

Prepared by: M. S. KumarSwamy, TGT(Maths) Page - 11 -


Ans: (i) Let A (0, y) be any point on the y-axis.
Since A (0, y) is equidistant from P (2, –5) and Q (–2, 9)
So AP = AQ ⇒ AP2 = AQ2
⇒ (2)2 + (y + 5)2 = (2)2 + (y – 9)2 ⇒ y2 + 10 y + 25 = y2 – 18y + 81
⇒ 28y = 81 – 25 ⇒ 28y = 56
⇒ y = 28/56 = 2
So, the point is (0, 2)
(ii) Let point P(x, 0) divides the line segment joining the points A and B in the ratio k : 1

Hence, the point P divides the line segment in the ratio 3 : 7.

Prepared by: M. S. KumarSwamy, TGT(Maths) Page - 12 -


PM SHRI KENDRIYA VIDYALAYA GACHIBOWLI,GPRA CAMPUS,HYD-32
SAMPLE PAPER TEST 09 FOR BOARD EXAM 2024
(ANSWERS)
SUBJECT: MATHEMATICS MAX. MARKS : 80
CLASS : X DURATION : 3 HRS
General Instruction:
1. This Question Paper has 5 Sections A-E.
2. Section A has 20 MCQs carrying 1 mark each.
3. Section B has 5 questions carrying 02 marks each.
4. Section C has 6 questions carrying 03 marks each.
5. Section D has 4 questions carrying 05 marks each.
6. Section E has 3 case based integrated units of assessment (04 marks each) with sub-parts of the
values of 1, 1 and 2 marks each respectively.
7. All Questions are compulsory. However, an internal choice in 2 Qs of 5 marks, 2 Qs of 3 marks
and 2 Questions of 2 marks has been provided. An internal choice has been provided in the
2marks questions of Section E
8. Draw neat figures wherever required. Take π =22/7 wherever required if not stated.
SECTION – A
Questions 1 to 20 carry 1 mark each.
1. If LCM(x, 18) = 36 and HCF(x, 18) = 2, then x is:
(a) 2 (b) 3 (c) 4 (d) 5 1
Ans. (c) 4
LCM × HCF = Product of two numbers
⇒ 36 × 2 = 18 × x
⇒x=4

2. In ΔABC right angled at B, if tanA = √3 , then then cosA cosC – sinAsinC =


(a) –1 (b) 0 (c) 1 (d) √3 / 2
Ans: (b) 0
tan A = √3 = tan 60°, so, ∠A = 60°,
Hence, ∠C = 30°.
1 3 3 1
So, cos A cos C – sin A sin C     0
2 2 2 2

3. If 2sin2 β – cos2 β = 2, then β is:


(a) 0° (b) 90° (c) 45° (d) 30° 1
Ans. (b) 90°
2 sin2 β – cos2 β = 2
Then, 2 sin2 β – (1 – sin2 β) = 2
⇒ 3sin2 β = 3 or sin2 β = 1 ⇒ β is 90°.

4. The ratio of LCM and HCF of the least composite and the least prime numbers is:
(a) 1: 2 (b) 2: 1 (c) 1: 1 (d) 1: 3 1
Ans. (b) 2: 1
Least composite number is 4 and the least prime number is 2
LCM(4, 2): HCF (4, 2) = 4: 2 = 2: 1

5. The value of k for which the lines 5x + 7y = 3 and 15x + 21y = k coincide is:
(a) 9 (b) 5 (c) 7 (d) 18
Ans: (a) 9

Prepared by: M. S. KumarSwamy, TGT(Maths) Page - 1 -


a1 b1 c1
For coincident lines we have  
a2 b2 c2
5 7 3 1 3
     k 9
15 21  k 3 k

1 1
6. Write a quadratic polynomial whose sum of zeroes is and product of zeroes is .
4 4
(a) 4x2 + x + 1 (b) x2 + 4x – 1
(c) 2x2 + 3x – 1 (d) x2 – 2x + 1
2
Ans: (a) 4x + x + 1
1 1
Sum of zeroes = and Product of zeroes =
4 4
2
∴ Quadratic Polynomial is p(x) = x – (sum of zeroes) x + product of zeroes
 1  1 1
∴ p(x) = x2 –   x + = 0 ⇒ p(x) = (4x2 + x + 1)
 4 4 4
Hence, required quadratic polynomial is 4x2 + x + 1

7. The vertices of a parallelogram in order are A(1, 2), B(4, y), C(x, 6) and D(3, 5). Then (x, y) is:
(a) (6, 3) (b) (3, 6) (c) (5, 6) (d) (1, 4)
Ans: (a) (6, 3)

8. A horse is tied to a pole with 28 m long rope. The perimeter of the field where the horse can
graze is (Take π = 22/7)
(a) 60 cm (b) 85 cm (c) 124 cm (d) 176 cm
Ans: (d) 176 cm
Horse can graze in the field which is a circle of radius 28 cm.
So, required perimeter = 2πr = 2π (28) cm = 2 × 22/7 × 28 cm = 176 cm

9. Two dice are thrown at the same time. The probability of getting not doublet is
(a) 1/3 (b) 1/6 (c) 1/5 (d) 5/6
Ans: (d) 5/6
Total number of possible outcomes = 36
Number of outcomes with doublets = 6 i.e. (1, 1), (2, 2), (3, 3), (4, 4), (5, 5), (6, 6)
So, number of favourable outcomes = 36 – 6 = 30
∴ Required probability = 30/36 = 5/6

10. ΔABC~ΔPQR. If AM and PN are altitudes of ΔABC and ΔPQR respectively and AB2: PQ2 = 4:
9, then AM: PN =
(a) 16: 81 (b) 4: 9 (c) 3: 2 (d) 2: 3
Ans: (d) 2 : 3

Prepared by: M. S. KumarSwamy, TGT(Maths) Page - 2 -


11. In the given figure, PA and PB are tangents to the circle with centre O. If ∠APB = 60°, then
∠OAB is

(a) 30° (b) 60° (c) 90° (d) 15°


Ans: (a) 30°
PA = PB ( Tangents drawn from external point are equal)
⇒ ∠ABP = ∠BAP = x ( Angles opposite to equal sides are equal)
In ∆APB, 60° + x + x = 180°
⇒ 2x = 120° ⇒ x = 60°
Now, ∠OAP = 90° (∵ Tangent is perpendicular to the radius through the point of contact)
∴ ∠OAB = 90° – 60° = 30°

12. If the difference of Mode and Median of a data is 24, then the difference of median and mean is
(a) 8 (b) 12 (c) 24 (d) 36
Ans: (b) 12

13. For the following distribution:


Class 0-5 5-10 10-15 15-20 20-25
Frequency 10 15 12 20 9
the sum of lower limits of the median class and modal class is
(a) 15 (b) 25 (c) 30 (d) 35
Ans: (b) 25

Prepared by: M. S. KumarSwamy, TGT(Maths) Page - 3 -


5sin   3cos 
14. If 5 tan θ = 4, then the value of is
5sin   2cos 
(a) 1/6 (b) 1/7 (c) 1/4 (d) 1/5
Ans: (a) 1/6

5sin   3cos  5 tan   3


 [Dividing numerator and denominator by cos θ]
5sin   2cos  5 tan   2
4
5  3
5 43 1
  
4
5  2 4  2 6
5

15. The ratio of the volumes of two spheres is 8 : 27. The ratio between their surface areas is
(a) 2 : 3 (b) 4 : 27 (c) 8 : 9 (d) 4 : 9
Ans: (d) 4 : 9

16. The area of the circle that can be inscribed in a square of 6cm is
(a) 36π cm2 (b) 18π cm2 (c) 12 π cm2 (d) 9π cm2
Ans: (d) 9π cm2

17. In the figure, if DE || BC, AD = 3 cm, BD = 4 cm and BC = 14 cm, then DE equals :


(a) 7 cm (b) 6 cm (c) 4 cm (d) 3 cm

Prepared by: M. S. KumarSwamy, TGT(Maths) Page - 4 -


Ans: (b) 6 cm
∵ DE || BC
∴ ∠ADE = ∠ABC [corresonding angles] ....(i)
Now, in ΔADE and ΔABC,
∠ADE = ∠ABC [Proved in (i)]
∠A = ∠A [Common angle]
∴ ΔADE ~ ΔABC [By AA similarity axiom]
AD DE
  [∵ Corresponding sides of similar triangles are proportional]
AB BC
AD DE 3 DE 3 DE
       DE  6
AD  BD BC 3  4 14 7 14
18. ABCD is a trapezium with AD ∥ BC and AD = 4cm. If the diagonals AC and BD intersect each
other at O such that AO/OC = DO/OB =1/2, then BC =
(a) 6cm (b) 7cm (c) 8cm (d) 9cm
Ans: (c) 8cm

DIRECTION: In the question number 19 and 20, a statement of Assertion (A) is followed by a
statement of Reason (R).
Choose the correct option

19. Assertion (A): The number 6n, n being a natural number, ends with the digit 5.
Reason (R): The number 9n cannot end with digit 0 for any natural number n.
(a) Both assertion (A) and reason (R) are true and reason (R) is the correct explanation of
assertion (A)
(b) Both assertion (A) and reason (R) are true and reason (R) is not the correct explanation of
assertion (A)
(c) Assertion (A) is true but reason (R) is false.
(d) Assertion (A) is false but reason (R) is true.
Ans: (d) Assertion (A) is false but reason (R) is true.

20. Assertion (A): The point (–1, 6) divides the line segment joining the points (–3, 10) and (6, –8)
in the ratio 2 : 7 internally.
Reason (R): Given three points, i.e. A, B, C form an equilateral triangle, then AB = BC = AC.
(a) Both assertion (A) and reason (R) are true and reason (R) is the correct explanation of
assertion (A)
(b) Both assertion (A) and reason (R) are true and reason (R) is not the correct explanation of
assertion (A)
(c) Assertion (A) is true but reason (R) is false.
(d) Assertion (A) is false but reason (R) is true.
Ans: (b) Both assertion (A) and reason (R) are true and reason (R) is not the correct explanation
of assertion (A)

SECTION – B
Questions 21 to 25 carry 2 marks each.

21. If sin(A + B) = 1 and cos(A – B) = √3/2, 0°< A + B ≤ 90° and A > B, then find the measures of
angles A and B.
Prepared by: M. S. KumarSwamy, TGT(Maths) Page - 5 -
Ans: sin(A + B) = 1 = sin 900, so A + B = 900 ……….(i)
cos(A – B) = √3/2 = cos 300, so A – B = 300 ………(ii)
From (i) & (ii) ∠A = 60° and ∠B = 30°
OR
cos   sin  1  3
Find an acute angle θ when 
cos   sin  1  3
cos   sin  1  3
Ans: 
cos   sin  1  3

22. In the given figure below, AD/AE=AC/BD and ∠1=∠2. Show that Δ BAE~ ΔCAD .

Ans: In ΔABC, ∠1 = ∠2
∴ AB = BD ………(i)
AD AC
Given, 
AE BD
AD AC
Using equation (i), we get  ……….(ii)
AE AB
AC AD
In ΔBAE and ΔCAD, by equation (ii), 
AB AE
and ∠A= ∠A (common)
∴ ΔBAE ~ ΔCAD [By SAS similarity criterion]

23. If 217x + 131y = 913, 131x + 217y = 827, then find the value of x and y
Ans: Adding the two equations and dividing by 348, we get : x + y = 5
Subtracting the two equations and dividing by 86, we get : x – y = 1
Solving these two new equations, we get, x = 3 and y = 2

24. A circle is inscribed in a ΔABC having AB= 10cm, BC = 12cm and CA = 8cm and touching
these sides at D, E, F respectively. Find the lengths of AD, BE and CF

Ans: Let AD = AF = x cm,


BD = BE = y cm
Prepared by: M. S. KumarSwamy, TGT(Maths) Page - 6 -
and CE = CF = z cm

Now, x + y = AB = 10 cm
y + z = BC = 12 cm
z + x = CA = 8 cm
Adding all we get 2(x+ y +z) = 30 ⇒ x + y + z = 15
Subtracting, we get z = 5 cm, x = 3 cm and y = 7 cm
Hence, AD = 3 cm, BE = 7 cm and CF = 5 cm.

25. The length of the minute hand of a clock is 6cm. Find the area swept by it when it moves from
5:25 pm to 6:00 pm.
Ans: We know that, in 60 minutes, the tip of minute hand moves 360°
In 1 minute, it will move = 360°/60 = 6°
∴ From 5:25 pm to 6:00 pm i.e. 35 min, it will move through = 35 × 6° = 210°
∴ Area of swept by the minute hand in 35 min = Area of sector with sectorial angle θ of 210°
and radius of 6 cm

OR
In the given figure, the shape of the top of a table is that of a sector of a circle with centre O and
∠AOB = 90°. If AO = OB = 42 cm, then find the perimeter of the top of the table is [Take π =
22/7]

Ans: Perimeter = length of major arc + 2r

SECTION – C
Questions 26 to 31 carry 3 marks each.
26. A train covered a certain distance at a uniform speed. If the train would have been 6 km/h faster,
it would have taken 4 hours less than the scheduled time. And, if the train were slower by 6

Prepared by: M. S. KumarSwamy, TGT(Maths) Page - 7 -


km/hr; it would have taken 6 hours more than the scheduled time. Find the length of the
journey.
Ans: Let the actual speed of the train be x km/hr and let the actual time taken be y hours.
Distance covered is xy km If the speed is increased by 6 km/hr, then time of journey is reduced
by 4 hours i.e., when speed is (x+6)km/hr, time of journey is (y−4) hours.
∴ Distance covered = (x + 6)(y − 4)
⇒ xy = (x + 6)(y − 4) ⇒ −4x + 6y − 24 = 0 ⇒ −2x + 3y −12 = 0 ………….(i)
Similarly xy = (x − 6)(y + 6) ⇒ 6x − 6y − 36 = 0 ⇒ x − y − 6 = 0 …………(ii)
Solving (i) and (ii) we get x=30 and y=24
Putting the values of x and y in equation (i), we obtain
Distance = (30 × 24)km =720km.
Hence, the length of the journey is 720km.
OR
Anuj had some chocolates, and he divided them into two lots A and B. He sold the first lot at the
rate of ₹2 for 3 chocolates and the second lot at the rate of ₹1 per chocolate, and got a total of
₹400. If he had sold the first lot at the rate of ₹1 per chocolate, and the second lot at the rate of
₹4 for 5 chocolates, his total collection would have been ₹460. Find the total number of
chocolates he had.
Ans: Let the number of chocolates in lot A be x
And let the number of chocolates in lot B be y
∴ total number of chocolates =x+y
2
Price of 1 chocolate = ₹ 2/3 , so for x chocolates = x
3
and price of y chocolates at the rate of ₹ 1 per chocolate =y.
2
∴ by the given condition x + y = 400 ⇒ 2x + 3y = 1200 ..............(i)
3
4
Similarly, x + y = 460 ⇒ 5x + 4y = 2300 ........ (ii)
5
Solving (i) and (ii) we get x = 300 and y = 200
∴ x + y = 300 + 200 = 500
So, Anuj had 500 chocolates.

sin   cos   1
27. Prove that:  sec   tan 
sin   cos   1
tan   1  sec 
Ans: LHS = tan   1  sec  (Dividing numerator and denominator by cos)

tan   sec   1

tan   1  sec 
tan   sec   (sec 2   tan 2  )

tan   1  sec 
(sec   tan  )(1  sec   tan  )

tan   1  sec 
 sec   tan  = RHS

28. Given that √5 is irrational, prove that 2 + 3√5 is irrational.


Ans: Let us assume 2 + 3√5 is rational, then it must be in the form of p/q where p and q are co-
prime integers and q ≠0
p
i.e. 2 + 3√5 =
q

Prepared by: M. S. KumarSwamy, TGT(Maths) Page - 8 -


p  2q
So 5 …(i)
3q
Since p, q, 5 and 2 are integers and q ≠ 0,
RHS of equation (i) is rational.
But LHS of (i) is √5 which is irrational. This is not possible.
This contradiction has arisen due to our wrong assumption that 2 + 3√5 is rational.
So, 2 + 3√5 is irrational.

29. Find the zeroes of the polynomial x2 + x – 2, and verify the relation between the coefficients
and the zeroes of the polynomial.
1
Ans: Now we have given the polynomial: x2 + x–2=0
6
Simplifying it, we get 6x2 + x – 12 = 0
⇒ 6x2 – 8x + 9x – 12 = 0
⇒ (6x2 – 8x) + (9x – 12) = 0
⇒ 2x(3x – 4) + 3(3x – 4) = 0
⇒ (3x – 4)(2x + 3) = 0
⇒ x = 4/3 or x = -3/2
Here, a = 6, b = 1, c = –12
4  3  8  9 1 b
Sum of zeroes =       
3  2 6 6 a
4  3  12 c
Product of zeroes =      
3  2 6 a

30. Two coins are tossed simultaneously. What is the probability of getting
(i) At least one head? (ii) At most one tail? (iii) A head and a tail?
Ans: Total number of outcomes = 4
(i) Number of outcomes with at least one head = 3
∴ Required probability = 3/4
(ii) Number of outcomes with at most one tail = 3
∴ Required probability = 3/4
(iii)Number of outcomes with a head and a tail = 2
∴ Required probability = 2/4 = 1/2

31. Prove that a parallelogram circumscribing a circle is a rhombus


Ans: We have ABCD, a parallelogram which circumscribes a circle (i.e., its sides touch the
circle) with centre O.
Since tangents to a circle from an external point are equal in length,
∴ AP = AS, BP = BQ, CR = CQ and DR = DS
Adding, we get
(AP + BP) + (CR + DR) = (AS + DS) + (BQ + CQ)
⇒ AB + CD = AD + BC
But AB = CD [opposite sides of ABCD]
and BC = AD
∴ AB + CD = AD + BC ⇒ 2 AB = 2 BC
⇒ AB = BC
Similarly AB = DA and DA = CD
Thus, AB = BC = CD = AD
Hence ABCD is a rhombus.

OR

Prepared by: M. S. KumarSwamy, TGT(Maths) Page - 9 -


In the figure XY and X'Y' are two parallel tangents to a circle with centre O and another tangent
AB with point of contact C interesting XY at A and X'Y' at B, what is the measure of ∠AOB.

Ans: Join OC. Since, the tangents drawn to a circle from an external point are equal.
∴ AP = AC

In Δ PAO and Δ AOC, we have:


AO = AO [Common]
OP = OC [Radii of the same circle]
AP = AC
⇒ Δ PAO ≅ Δ AOC [SSS Congruency]
∴ ∠PAO = ∠CAO = ∠1
∠PAC = 2 ∠1 ...(1)
Similarly ∠CBQ = 2 ∠2 ...(2)
Again, we know that sum of internal angles on the same side of a transversal is 180°.
∴ ∠PAC + ∠CBQ = 180°
⇒ 2 ∠1 + 2 ∠2 = 180° [From (1) and (2)]
⇒ ∠1 + ∠2 = 180°/2 = 90° ...(3)
Also ∠1 + ∠2 + ∠AOB = 180° [Sum of angles of a triangle]
⇒ 90° + ∠AOB = 180°
⇒ ∠AOB = 180° − 90° ⇒ ∠AOB = 90°.

SECTION – D
Questions 32 to 35 carry 5 marks each.
32. Prove that if a line is drawn parallel to one side of a triangle intersecting the other two sides in
distinct points, then the other two sides are divided in the same ratio.
In the figure, find EC if AD/DB = AE/EC using the above theorem.

Prepared by: M. S. KumarSwamy, TGT(Maths) Page - 10 -


Ans: For the Theorem :
Given, To prove, Construction and figure of 2 marks
Proof of 2 marks
AD AE 2 3 1 3
Using Thales theorem, we get       EC  9cm 1 mark
DB EC 6 EC 3 EC

33. A cubical block of side 10 cm is surmounted by a hemisphere. What is the largest diameter that
the hemisphere can have? Find the cost of painting the total surface area of the solid so formed,
at the rate of Rs. 5 per 100 sq. cm. [Use π = 3.14]
Ans: Side of the cubical block (l) = 10 cm.

The hemisphere is surmounted on it.


The largest diameter the hemisphere can have =
side of the cubical block
Diameter of the hemisphere = 10 cm
Radius of the hemisphere (r) = 5 cm
Total surface area of the solid formed
= TSA of the cubical + CSA of the hemisphere - Area of the base of the hemisphere
= 6l2 + 2πr2 – πr2 = 6l2 + πr2
= 6 × (10)2 + 3.14 × (5)2
= 6 × 100 + 3.14 × 25
= 600 + 78.50
= 678.5 cm2
Rate of painting = Rs. 5 per 100 cm2
Cost pf painting the solid formed = Rs. 5/100 × 678.5
= Rs. 33.925 = Rs. 33.93 (approx)

OR
Due to heavy floods in a state, thousands were rendered homeless. 50 schools collectively
decided to provide place and the canvas for 1500 tents and share the whole expenditure equally.
The lower part of each tent is cylindrical with base radius 2.8 m and height 3.5 m and the upper
part is conical with the same base radius, but of height 2.1 m. If the canvas used to make the
tents costs ₹120 per m2, find the amount shared by each school to set up the tents.
Ans: Radius of the base of cylinder (r) = 2.8 m = Radius of the base of the cone (r)
Height of the cylinder (h)=3.5 m
Prepared by: M. S. KumarSwamy, TGT(Maths) Page - 11 -
Height of the cone (H)=2.1 m.
Slant height of conical part (l)=√(r2 + H2) = √[(2.8)2 + (2.1)2] = √(7.84 + 4.41) = √12.25 = 3.5 m
Area of canvas used to make tent = CSA of cylinder + CSA of cone = 2πrh + πrl
22
= πr(2h + l) =  2.8  (7  3.5)  22  0.4  10.5  92.4m2
7
Cost of 1500 tents at ₹120 per sq.m = 1500 × 120 × 92.4 = 1,66,32,000
Share of each school to set up the tents = 16632000/50 = ₹3,32,640

34. The median of the following data is 868. Find the values of x and y, if the total frequency is 100
Class Frequency
800 – 820 7
820 – 840 14
840 – 860 x
860 – 880 25
880 – 900 y
900 – 920 10
920 – 940 5
Ans:
Class Frequency Frequency
800 – 820 7 7
820 – 840 14 21
840 – 860 x x + 21
860 – 880 25 x + 46
880 – 900 y x + y + 46
900 – 920 10 x + y + 56
920 – 940 5 x + y + 61

From table, we have x + y + 61 = 100 ⇒ x + y = 100 – 61 ⇒ x + y = 39


Here, median = 868, therefore median class is 860 – 880
So, l = 860, cf = x + 21, f = 25, h = 20, n/2 = 50
n 
 2  cf   50  ( x  21) 
Now, Median  l    h   868  860    20 
 f   25 
 
 50  x  21)  29  x
 868  860    4  8  4
 5  5
 40  (29  x )4  29  x  10  x  29  10  19
 y  39  19  20

1
35. Two pipes running together can fill a cistern in 3 hours. If one pipe takes 3 hours more than
13
the other to fill it, find the time in which each pipe would fill the cistern.
Ans: Let time taken by faster pipe to fill the cistern be x hrs.
Therefore, time taken by slower pipe to fill the cistern = (x + 3) hrs
Since the faster pipe takes x minutes to fill the cistern.
1
∴ Portion of the cistern filled by the faster pipe in one hour =
x
1
Portion of the cistern filled by the slower pipe in one hour =
x3

Prepared by: M. S. KumarSwamy, TGT(Maths) Page - 12 -


13
Portion of the cistern filled by the two pipes together in one hour =
40
1 1 13 x  3  x 13
According to the question,    
x x  3 40 x( x  3) 40
⇒ 40 (2x + 3) = 13x (x + 3) ⇒ 80x + 120 = 13x2 + 39x
⇒ 13x2 – 41x – 120 = 0 ⇒ 13x2 – 65x + 24x – 120 = 0
⇒ 13x (x – 5) + 24 (x – 5) = 0 ⇒ (x – 5) (13x + 24) = 0
⇒ Either x – 5 = 0 or 13x + 24 = 0 ⇒ x = 5 as x = −24/13 not possible.
Hence, the time taken by the two pipes is 5 hours and 8 hours respectively.
OR
In a flight of 600km, an aircraft was slowed down due to bad weather. Its average speed for the
trip was reduced by 200 km/hr from its usual speed and the time of the flight increased by 30
min. Find the scheduled duration of the flight.
Ans: Let the usual speed of plane be x km/hr
and the reduced speed of the plane be (x – 200) km/hr
Distance =600 km [Given]
According to the question,
(time taken at reduced speed) – (Schedule time) = 30 minutes = 0.5 hours.
600 600 1
⇒  
x  200 x 2
Which on simplification gives: x2 – 200x − 240000 = 0
⇒ x2 – 600x + 400x −240000 = 0
⇒ x(x – 600) + 400( x – 600) = 0 ⇒ (x – 600)(x + 400) =0 ⇒ x = 600 or x = −400
But speed cannot be negative.
∴ The usual speed is 600 km/hr and
the scheduled duration of the flight is 600/600 =1hour

SECTION – E(Case Study Based Questions)


Questions 36 to 38 carry 4 marks each.

36. Case Study – 1


Anita purchased a new building for her business. Being in the prime location, she decided to
make some more money by putting up an advertisement sign for a rental ad income on the roof
of the building.

Prepared by: M. S. KumarSwamy, TGT(Maths) Page - 13 -


From a point P on the ground level, the angle of elevation of the roof of the building is 30° and
the angle of elevation of the top of the sign board is 45°. The point P is at a distance of 24 m
from the base of the building.

On the basis of the above information, answer the following questions:


(i) Find the height of the building (without the sign board). (2)
OR
Find the height of the building (with the sign board) (2)
(ii) Find the height of the sign board. (1)
(iii) Find the distance of the point P from the top of the sign board. (1)
Ans: (i) In ∆APC,

tan 30° = AB/AP


⇒ 1/√3 = AB/24
⇒ AB = 24/√3 m = 13.85 m = 14 m (approx)
OR
Considering, the diagram in the above question, AC as the new height of the shop including the
sign-baard.
In ∆APC,
tan 45° = AC/AP
⇒ 1 = AC/24
⇒ AC = 24 m

(ii) From Q (i) and Q (ii).


Length of sign board, BC = AC – AB
= 24 – 14
= 10 m

(iii) In ∆APC,
cos 45° = AP/AC
⇒ 1/√2 = 24/AC
⇒ PC = 24√2 m
Prepared by: M. S. KumarSwamy, TGT(Maths) Page - 14 -
37. Case Study-2
The school auditorium was to be constructed to accommodate at least 1500 people. The chairs
are to be placed in concentric circular arrangement in such a way that each succeeding circular
row has 10 seats more than the previous one.

(i) If the first circular row has 30 seats, how many seats will be there in the 10th row? (1)
(ii) For 1500 seats in the auditorium, how many rows need to be there? (2)
OR
If 1500 seats are to be arranged in the auditorium, how many seats are still left to be put after
10th row? (2)
(iii) If there were 17 rows in the auditorium, how many seats will be there in the middle row?(1)

Ans: (i) Since each row is increasing by 10 seats, so it is an AP with first term a = 30, and
common difference d =10. So number of seats in 10th row = 10 = a + 9d
= 30 + 9 ×10 = 120
n n
(ii) Sn= ( 2a + (n – 1)d) ⇒ 1500 = ( 2 × 30 + (n-1)10)
2 2
2 2
⇒ 3000 = 50n + 10n ⇒ n + 5n – 300 = 0
⇒ n2 + 20n – 15n – 300 = 0 ⇒ (n + 20) (n – 15) =0
Rejecting the negative value, n = 15
OR
No. of seats already put up to the 10th row = S10
10
S10 = {2 × 30 + (10-1)10)} = 5(60 + 90) = 750
2
So, the number of seats still required to be put are 1500 – 750 = 750
(iii) If no. of rows =17
then the middle row is the 9th row
9= a + 8d = 30 + 80 = 110 seats

38. Case Study-3


The diagrams show the plans for a sun room. It will be built onto the wall of a house. The four
walls of the sunroom are square clear glass panels. The roof is made using
• Four clear glass panels, trapezium in shape, all the same size
• One tinted glass panel, half a regular octagon in shape

Prepared by: M. S. KumarSwamy, TGT(Maths) Page - 15 -


(i) Find the mid-point of the segment joining the points J (6, 17) and I (9, 16). (1)
(ii) Find the distance between the points A and S. (1)
(iii) Find the co-ordinates of the point which divides the line segment joining the points A and B
in the ratio 1:3 internally. (2)
OR
(iii) If a point (x,y) is equidistant from the Q(9,8) and S(17,8),then find the relation between x
and y. (2)
 6  9 17  16   15 33 
Ans: (i) Mid-point of JI =  ,  , 
 2 2   2 2
(ii) Distance between A and S = 16 boxes.
(iii) Coordinates of A and B are (1, 8) and (5, 10) respectively.
Coordinates of point dividing AB in the ratio 1 : 3 internally are:
1  5  3 1 1 10  3  8 8 34
x ,y  x   2, y   8.5
1 3 1 3 4 4
Co-ordinates of required points be (2, 8.5)
OR
(iii) Let P (x,y) is equidistant from the Q(9,8) and S(17,8) then we have
PQ = PS ⇒ PQ2 = PS2
⇒ (x – 9)2 + (y – 8)2 = (x – 17)2 + (y – 8)2
⇒ (x – 9)2 = (x – 17)2
⇒ x2 – 18x + 81 = x2 – 34x + 289
⇒ 34x – 18x + 81 – 289 = 0
⇒ 16x – 208 = 0
⇒ x – 13 = 0

Prepared by: M. S. KumarSwamy, TGT(Maths) Page - 16 -


PM SHRI KENDRIYA VIDYALAYA GACHIBOWLI,GPRA CAMPUS,HYD-32
SAMPLE PAPER TEST 10 FOR BOARD EXAM 2024
(ANSWERS)
SUBJECT: MATHEMATICS MAX. MARKS : 80
CLASS : X DURATION : 3 HRS
General Instruction:
1. This Question Paper has 5 Sections A-E.
2. Section A has 20 MCQs carrying 1 mark each.
3. Section B has 5 questions carrying 02 marks each.
4. Section C has 6 questions carrying 03 marks each.
5. Section D has 4 questions carrying 05 marks each.
6. Section E has 3 case based integrated units of assessment (04 marks each) with sub-parts of the
values of 1, 1 and 2 marks each respectively.
7. All Questions are compulsory. However, an internal choice in 2 Qs of 5 marks, 2 Qs of 3 marks and
2 Questions of 2 marks has been provided. An internal choice has been provided in the 2marks
questions of Section E
8. Draw neat figures wherever required. Take π =22/7 wherever required if not stated.
SECTION – A
Questions 1 to 20 carry 1 mark each.

1. The LCM of two numbers is 182 and their HCF is 13. If one of the numbers is 26, the other
number is
(a) 31 (b) 71 (c) 61 (d) 91
Ans: (d) 91
a × b = HCF (a, b) × LCM (a, b) ⇒ 26 × b = 13 × 182 ⇒ b = 13 × 182 / 26 = 91

2. If p and q are positive integers such that p = a3b2 and q = a2b, where ‘a’ and ‘b’ are prime numbers,
then the HCF ( p, q) is …..
(a) a2b (b) a2b2 (c) a3b2 (d) a3b3
2
Ans: (a) a b

3. The quadratic equations x2 – 4x + k = 0 has distinct real roots if


(a) k = 4 (b) k > 4 (c) k = 16 (d) k < 4
Ans. (d) k < 4

4. The number of polynomials having zeroes as -2 and 5 is


(a) 1 (b) 2 (c) 3 (d) more than 3
Ans: (d) more than 3

5. The pair of equations y = 0 and y = -7 has


(a) one solution (b) two solutions (c) infinitely many solutions (d) no solution
Ans. (d) no solution

6. The line segment joining the points A (5, 3) and B (-3, 11) is divided by the point C (3,5) in the
ratio
(a) 1:3 (b) 3:1 (c) 2:3 (d) 3:2
Ans: (a) 1:3

7. ∆ABC is such that AB = 3 cm, BC = 2 cm, CA = 2.5 cm. If ∆ABC~∆DEF and EF=4cm, then
perimeter of ∆DEF is
(a) 7.5 cm (b) 15cm (c) 22.5 cm (d) 30 cm
Ans: (b) 15cm

Prepared by: M. S. KumarSwamy, TGT(Maths) Page - 1 -


8. The value of sin 30° cos 60° + sin 60° cos 30° is:
(a) 0 (b) 1 (c) 2 (d) 4
Ans: (b) 1
sin 30° cos 60° + sin 60° cos 30°
1 1  3 3  1 3 1 3 4
           1
2 2  2 2  4 4 4 4

9. If 2 sin 2θ = √3, then find the value of θ.


(a) 30° (b) 60° (c) 90° (d) 45°
Ans: (a) 30°
2 sin 2θ = √3 ⇒ sin 2θ = √3/2 = sin600
⇒ 2θ = 600 ⇒ θ = 300

10. A girl walks 200m towards East and then 150m towards North. The distance of the girl from the
starting point is
(a) 350m (b) 250m (c) 300 m (d) 325 m
Ans: (b) 250 m

11. Consider the data:


Class 65-85 85-105 105-125 125-145 145-165 165-185 185-205
Frequency 4 5 13 20 14 7 4
The difference of the upper limit of the median class and the lower limit of the modal class is
(a) 0 (b) 19 (c) 20 (d) 38
Ans: (c) 20

12. From an external point Q, the length of the tangents to a circle is 5 cm and the distance of Q from
the centre is 8 cm. The radius of the circle is
(a) 39 cm (b) 3 cm (c) √39 cm (d) 7 cm
Ans: (c) √39 cm

13. If the sum of the areas of two circles with radii R1 and R2 is equal to the area of a circle of radius R,
then:
(a) R1 + R2 = R (b) R12 + R22 = R2 (c) R1 + R2 < R (d) R1 + R2 < R2
Ans. (b) R12 + R22 = R2
According to the given condition,
Area of circle with radius R = Area of circle with radius R1 + Area of circle with radius R2
⇒ πR2 = πR12 + πR22
⇒ R2 = R12 + R22

14. The base radii of a cone and a cylinder are equal. If their curved surface areas are also equal, then
the ratio of the slant height of the cone to the height of the cylinder is:
(a) 2 : 1 (b) 1 : 2 (c) 1 : 3 (d) 3 : 1
Ans. (a) 2 : 1

15. For the following distribution:


Marks Below Below Below Below Below Below
10 20 30 40 50 60
No. of Students 3 12 27 57 75 80
the modal class is
(a) 10 – 20 (b) 20 – 30 (c) 30 – 40 (d) 50 – 60
Ans: (c) 30 – 40

Prepared by: M. S. KumarSwamy, TGT(Maths) Page - 2 -


16. The area of a circle that can be inscribed in a square of side 6 cm is:
(a) 36 π cm2 (b) 18 π cm2 (c) 12 π cm2 (d) 9 π cm2
2
Ans. (d) 9 π cm

17. A girl calculates that the probability of her winning the first prize in a lottery is 0.08. If 6000
tickets are sold, how many tickets has she bought?
(a) 40 (b) 240 (c) 480 (d) 750
Ans. (c) 480

18. If sin A = 1/2, cos B = 1, 0 < A, B ≤ π/2, then the value of cot (A + B) is:
(a) √3 /2 (b) 1/2 (c) 0 (d) √3
Ans: (d) √3
sin A = 1/2 = sin300 ⇒ A = 300
cos B = 1 = cos00 ⇒ B = 00
Now, cot(A+ B) = cot(300 + 00) = cot300 = √3

Direction: In the question number 19 & 20, A statement of Assertion (A) is followed by a
statement of Reason(R). Choose the correct option
19. Assertion (A): The value of y is 3, if the distance between the points P(2, -3) and Q(10, y) is 10.
Reason (R): Distance between two points is given by ( x2  x1 )2  ( y2  y1 )2
(a) Both Assertion (A) and Reason (R) are true and Reason (R) is the correct explanation of
Assertion (A)
(b) Both assertion (A) and reason (R) are true and reason (R) is not the correct explanation of
Assertion (A)
(c) Assertion (A) is true but reason(R) is false.
(d) Assertion (A) is false but reason(R) is true.
Ans: (a) Both Assertion (A) and Reason (R) are true and Reason (R) is the correct explanation of
Assertion (A)

20. Assertion (A): If HCF ( 90, 144) = 18, then LCM (90, 144) = 720
Reason (R): HCF (a, b) x LCM (a, b) = a x b
(a) Both Assertion (A) and Reason (R) are true and Reason (R) is the correct explanation of
Assertion (A).
(b) Both Assertion (A) and Reason (R) are true but Reason (R) is not the correct explanation of
Assertion (A).
(c) Assertion (A) is true but Reason (R) is false.
(d) Assertion (A) is false but Reason (R) is true.
Ans: (a) Both Assertion (A) and Reason (R) are true and Reason (R) is the correct explanation of
Assertion (A).

SECTION-B
Questions 21 to 25 carry 2 marks each

21. For what value of p will the following pair of linear equations have infinitely many solutions?
(p – 3)x + 3y = p; px + py = 12
Ans: Consider equations (p – 3)x + 3y = p
and px + py = 12
For infinitely many solutions,
p3 3 p
  ...(i)
p p 12

Prepared by: M. S. KumarSwamy, TGT(Maths) Page - 3 -


3 p
Consider,   p 2  36  p  6
p 12
p3 3 p
For p = 6, from (i),   is true
p p 12
p3 3 p
For p = – 6, from (i),   is false.
p p 12
Hence, for p = 6, pair of linear equations has infinitely many solutions.

KP 4
22. In Figure, PQ is parallel to MN. If = and KN = 20.4 cm. Find KQ.
PM 13

Ans: In ΔKMN, we have PQ ∥ MN


KP KQ
  [Basic proportionality Theorem]
PM QN
KP KQ 4 KQ
   
PM KN  KQ 13 20.4  KQ
⇒ 4(20.4 − KQ) = 13KQ
⇒ 81.6 − 4KQ = 13KQ
⇒ 17KQ = 81.6
81.6
⇒ KQ   4.8cm
17

OR
In the below figure, if ST || QR. Find PS.

PS PT
Ans: By Basic proportionality theorem, 
QS RT
PS 3 9
   PS  cm
3 2 2

1
23. If tan (A + B) = 3 and tan (A – B) = ; 0° < A+B ≤ 90°; A > B, find A and B.
3
Ans: tan(A + B) = √3 = tan 60°

Prepared by: M. S. KumarSwamy, TGT(Maths) Page - 4 -


⇒ A + B = 60° ⇢ (i)
tan(A – B) = 1/√3 = tan 30°
⇒ A – B = 30° ⇢ (ii)
Adding equation (i) and (ii),
2A = 90°
⇒ A = 45°
Putting the value of A in equation (i),
45° + B = 60°
⇒ B = 60°- 45°
⇒ B = 15°

24. XY and MN are the tangents drawn at the end points of the diameter DE of the circle with centre
O. Prove that XY || MN.
Ans: Since, XY is the tangent to the circle at the point D.
⇒ OD ⊥ XY ⇒ ∠EDX = 90°
Also, MN is the tangent to the circle at E.
⇒ OE ⊥ MN ⇒ ∠ DEN = 90°
As, ∠EDX = ∠DEN (each 90°)
which are alternate interior angles.
⇒ XY ||MN

25. In the given figure, sectors of two concentric circles of radii 7 cm and 3.5 cm are given. Find the
area of the shaded region. (Use π = )

Ans: Area of the shaded region = area of the sector of 30° with radius 7 cm – area of the sector 30°
with radius 3.5 cm

OR
A horse is placed for grazing inside a rectangular field 70 m by 52 m and is tethered to one corner
by a rope 21 m long. On how much area can it graze?
Ans : Area of the portion that horse can graze = area of the shaded portion.

Shaded portion is a sector of radius 21 m = length of the rope


Angle of this sector = angle of the corners of the rectangle = 90°
Area of the shaded portion that horse can graze

Prepared by: M. S. KumarSwamy, TGT(Maths) Page - 5 -


SECTION-C
Questions 26 to 31 carry 3 marks each

26. Given that √3 is irrational, prove that (2 + 5√3) is an irrational number.


Ans: Let 2 + 5√3 be a rational number such that
2 + 5√3 = a, where a is a non-zero rational number.
a2
5 3  a2  3 
5
a2
Since 5 and 2 are integers and a is a rational number, therefore is a rational number
5
⇒ √3 is a rational number which contradicts the fact that √3 is an irrational number.
Therefore, our assumption is wrong.
Hence 2 + 5√3 is an irrational number

27. Find the zeroes of the polynomial x2 + x – 2, and verify the relation between the coefficients and
the zeroes of the polynomial.
1
Ans: The polynomial can be rewritten as (6x2 + x − 12)
6
On factoring, 6x² + x – 12 = 6x² + 9x – 8x – 12
= 3x(2x + 3) - 4(2x + 3)
= (3x - 4)(2x + 3)
So, 1/6 (6x² + 6x - 12) = 0
(3x - 4)(2x + 3) = 0
3x - 4 = 0 ⇒ 3x = 4 ⇒ x = 4/3
2x + 3 = 0 ⇒ 2x = -3 ⇒ x = -3/2
Therefore, the zeros of the polynomial are 4/3 and -3/2.
Sum of the roots:
LHS: + ꞵ =4/3 + (-3/2) = 8 - 9/6 = -1/6
RHS: -coefficient of x/coefficient of x² = -1/6
Product of the roots
LHS: ꞵ = (4/3)(-3/2) = -12/6 = -2
RHS: constant term/coefficient of x² = -12/6 = -2.

28. A number consists of two digits. Where the number is divided by the sum of its digits, the quotient
is 7. If 27 is subtracted from the number, the digits interchange their places, find the number.
Ans : Let digit at unit place be x, and at tenth place be y.
∴ Number = 10y + x
10 y  x
According to the question,  7 ⇒ 6x – 3y = 0
yx
⇒ 2x – y = 0 ...(i)
Again according to the question, (10y + x) – 27 = 10x + y
⇒ 9x – 9y = – 27 ⇒ x – y = – 3 ...(ii)
Solving for x and y, we get
x = 3 and y = 6
∴ Number is 63.
OR

Prepared by: M. S. KumarSwamy, TGT(Maths) Page - 6 -


Students of a class are made to stand in rows. If 4 students are extra in a row, there would be two
rows less. If 4 students are less in a row, there would be four more rows. Find the number of
students in the class.
Ans : Let number of students in a row be x and number of rows be y.
⇒ Total number of students = x . y
From condition 1 : (x + 4) (y – 2) = xy
⇒ xy – 2x + 4y – 8 = xy – 2x + 4y = 8 ...(i)
From condition 2 : (x – 4) (y + 4) = xy xy + 4x – 4y – 16 = xy
⇒ 4x – 4y = 16 ...(ii)
Adding (i) and (ii), we get 2x = 24 ⇒ x = 12
Substituting in (i), we get y = 8.
∴ Total number of students = xy = 12 × 8 = 96.

29. Prove that : sin θ (1 + tan θ) + cos θ (1 + cot θ) = sec θ + cosec θ.


Ans: LHS = sin θ (1 + tan θ) + cos θ (1 + cot θ)
= sin θ + sin θ . tan θ + cos θ + cos θ . cot θ
sin  cos  cos2  sin 2 
= sin θ + sin θ .  cos   cos  .  sin     cos 
cos sin  sin  cos 
sin 2   cos 2  sin 2   cos 2  1 1
=   
sin  cos  sin  cos 

= cosec θ + sec θ = sec θ + cosec θ = RHS

30. A circle is inscribed in a ∆ABC having sides 16 cm, 20 cm and 24 cm as shown in figure. Find AD,
BE and CF.

Ans: Since, tangents drawn from an external point to a circle are equal.
∴ AD = AF = x (say)
BD = BE = y (say)
CE = CF = z (say)
According to the question,
AB = x + y = 24 cm ...(i)
BC = y + z = 16 cm ...(ii)
AC = x + z = 20 cm ...(iii)
Subtracting (iii) from (i), we get
y – z = 4 ...(iv)
Adding (ii) and (iv), we get
2y = 20 ⇒ y = 10 cm
Substituting the value of y in (i) and (ii), we get
x = 14 cm, z = 6 cm
Hence, AD = 14 cm, BE = 10 cm and CF = 6 cm.
OR
In the figure XY and X'Y' are two parallel tangents to a circle with centre O and another tangent AB
with point of contact C interesting XY at A and X'Y' at B, prove that ∠AOB is a right angle.

Prepared by: M. S. KumarSwamy, TGT(Maths) Page - 7 -


Ans: Join OC. Since, the tangents drawn to a circle from an external point are equal.
∴ AP = AC
In Δ PAO and Δ AOC, we have:
AO = AO [Common]
OP = OC [Radii of the same circle]
AP = AC
⇒ Δ PAO ≅ Δ AOC [SSS Congruency]
∴ ∠PAO = ∠CAO = ∠1
∠PAC = 2 ∠1 ...(1)
Similarly ∠CBQ = 2 ∠2 ...(2)

Again, we know that sum of internal angles on the same side of a transversal is 180°.
∴ ∠PAC + ∠CBQ = 180°
⇒ 2 ∠1 + 2 ∠2 = 180° [From (1) and (2)]
⇒ ∠1 + ∠2 = 180°/2 = 90° ...(3)
Also ∠1 + ∠2 + ∠AOB = 180° [Sum of angles of a triangle]
⇒ 90° + ∠AOB = 180°
⇒ ∠AOB = 180° − 90° ⇒ ∠AOB = 90°.

31. Two dice are thrown at the same time. What is the probability that the sum of the two numbers
appearing on the top of the dice is
(i) at least 9? (ii) 7? (iii) less than or equal to 6?
Ans: (i) Number of outcomes with sum of the numbers is at least 9 = 10
∴ Required Probability = 10/36 = 5/18
(ii) Number of outcomes with sum of the numbers 7 = 6
∴ Required Probability = 6/36 = 1/6
(iii) Number of outcomes with sum of the numbers less than or equal to 6 = 36
∴ Required Probability = 15/36 = 5/12

SECTION-D
Questions 32 to 35 carry 5 marks each

32. State and Prove Basic Proportionality Theorem.

Prepared by: M. S. KumarSwamy, TGT(Maths) Page - 8 -


Ans: Statement - 1 mark
Given, To Prove, Construction, Figure – 2 marks
Proof – 2 marks
33. The median of the following data is 137. Find the values of x and y, If the total frequency is 68.
Class 105 – 125 – 145 – 165 – 185 –
65 – 85 85 – 105
intervals 125 145 165 185 205
Frequency 4 x 13 20 14 y 4
Ans:
Class Frequency Cumulative Frequency
65-85 4 4
85-105 x 4+x
105-125 13 17+ x
125-145 20 37+ x
145-165 14 51+ x
165-185 y 51 + x + y
185-205 4 55 + x + y
∴ Median class is "125−145." cf = 17 + x , l = 125 , f = 20 h = 20, N = 68
Median = l + ( )xh
⇒ 137 = 125 + ( ) x 20
⇒ 137 – 125 = 17 – x
⇒ x = 17 – 12 = 5 ⇒ 55 + 5 + y = 68 ⇒ y = 8

34. A juice seller serves his customers using a glass as shown in figure. The inner diameter of the
cylindrical glass is 5 cm, but the bottom of the glass has a hemispherical portion raised which
reduces the capacity of the glass. If the height of the glass is 10 cm, find the apparent capacity of
the glass and its actual capacity. [ π = 3.14]

Ans:

OR
A rectangular sheet of paper 30 cm × 18 cm can be transformed into the curved surface of a right
circular cylinder in two ways either by rolling the paper along its length or by rolling it along its
breadth. Find the ratio of the volumes of the two cylinders thus formed.

Prepared by: M. S. KumarSwamy, TGT(Maths) Page - 9 -


Ans:

35. A person on tour has Rs.360 for his expenses. If he extends his tour for 4 days, he has to cut down
his daily expenses by Rs.3. Find the original duration of the tour.
Ans: Let days be the original duration of the tour.
Total expenditure on tour ₹ 360
Expenditure per day ₹ 360/x
Duration of the extended tour (x + 4) days
Expenditure per day according to the new schedule ₹ 360/(x + 4)
Given that daily expenses are cut down by ₹ 3
360 360 1 1 
As per the given condition,   3  360   3
x x4  x x4
1 1  3 1 x4 x 1 4 1
       
 x x  4  360 120 x( x  4) 120 x( x  4) 120
⇒ x(x + 4) = 480 ⇒ x² + 4x = 480 ⇒ x² + 4x – 480 = 0
⇒ x² + 24x – 20x – 480 = 0 ⇒ x(x + 24) - 20(x + 24) = 0
⇒ x – 20 = 0 or x + 24 = 0 ⇒ x = 20 or x = -24
Since the number of days cannot be negative. So, x = 20
Therefore, the original duration of the tour was 20 days
OR
Rs.6500 were divided equally among a certain number of persons. Had there been 15 more persons,
each would have got Rs.30 less. Find the original number of persons.
Ans: Let the original number of persons be x
Total money which was divided = Rs. 6500
Each person share = Rs. 6500/x
6500 6500 6500 x  97500  6500 x
According to the question,   30   30
x x  15 x ( x  15)
97500 3250
  30   1 ⇒ x² + 15x – 3250 = 0
x ( x  15) x( x  15)
⇒ x² + 65x – 50x – 3250 = 0 ⇒ x(x + 65) – 50(x + 65) = 0
⇒ (x + 65)(x – 50) = 0 ⇒ x = -65, 50
Since the number of persons cannot be negative, hence the original numbers of person is 50

SECTION-E (Case Study Based Questions)


Questions 36 to 38 carry 4 marks each

36. Case Study – 1:


In the month of April to June 2022, the exports of passenger cars from India increased by 26% in
the corresponding quarter of 2021–22, as per a report. A car manufacturing company planned to

Prepared by: M. S. KumarSwamy, TGT(Maths) Page - 10 -


produce 1800 cars in 4th year and 2600 cars in 8th year. Assuming that the production increases
uniformly by a fixed number every year.

Based on the above information answer the following questions.


(i) Find the production in the 1st year. (1)
(ii) Find the production in the 12th year. (1)
(iii) Find the total production in first 10 years. (2)
OR
(iii) In how many years will the total production reach 31200 cars? (2)

Ans: (i) Since the production increases uniformly by a fixed number every year, the number of Cars
manufactured in 1st, 2nd, 3rd, . . .,years will form an AP.
So, a + 3d = 1800 & a + 7d = 2600
So d = 200 & a = 1200
(ii) a12 = a + 11d ⇒ a30 = 1200 + 11 × 200
⇒ a12 = 3400
n 10
(iii) S n  [2a  (n  1) d ]  S10  [2  1200  (10  1)  200]
2 2
 S10  5[2400  1800]  5  4200  21000
OR
n
S n  [2a  (n  1) d ]  31200
2
n
 [2  1200  ( n  1)  200]  31200
2
n
  200[12  ( n  1)]  31200
2
⇒ n[12 + (n − 1) ] = 312
⇒ n2 + 11n – 312 = 0
⇒ n2 + 24n – 13n – 312 = 0
⇒ (n +24)(n – 13) = 0
⇒ n = 13 or – 24.
As n can’t be negative. So n = 13

37. Case Study – 2:


In order to conduct sports day activities in your school, lines have been drawn with chalk powder at
a distance of 1 m each in a rectangular shaped ground ABCD. 100 flower pots have been placed at
the distance of 1 m from each other along AD, as shown in the following figure. Niharika runs
1 1
( )th distance AD on the 2nd line and posts a green Flag. Preet runs ( ) th distance AD on the
4 5
eighth line and posts are red flags. Taking A as the origin AB along x-axis and AD along y-axis,
answer the following questions:

Prepared by: M. S. KumarSwamy, TGT(Maths) Page - 11 -


(i) Find the coordinates of the green flag. (1)
(ii) Find the distance between the two flags. (1)
(iii) If Rashmi has to post a blue flag exactly halfway between the line segment joining the two
flags, where should she post her flag? (2)
OR
(iii) If Joy has to post a flag at one fourth distance from the green flag, in the line segment joining
the green and red flags, then where should he post his flag? (2)

 1 
Ans: (i) Position of the green flag is  2, 100   (2, 25)
 4 
(ii) Distance between the two flags = (36  25) = 61cm
 2  8 25  20 
(iii) Position of the blue flag =  ,    5, 22.5 
 2 2 
OR

Required point is (3.5, 23.75)

38. Case Study – 3:


A lighthouse is a tall tower with light near the top. These are often built on islands, coasts or on cliffs.
Lighthouses on water surface act as a navigational aid to the mariners and send warning to boats and ships
for dangers. Initially wood, coal would be used as illuminators. Gradually it was replaced by candles,
lanterns, electric lights. Nowadays they are run by machines and remote monitoring. Prongs Reef lighthouse
of Mumbai was constructed in 1874-75. It is approximately 40 meters high and its beam can be seen at a
distance of 30 kilometres. A ship and a boat are coming towards the lighthouse from opposite directions.
Angles of depression of flash light from the lighthouse to the boat and the ship are 300 and 600 respectively.

Prepared by: M. S. KumarSwamy, TGT(Maths) Page - 12 -


(i) Which of the two, boat or the ship is nearer to the light house. Find its distance from the lighthouse? (2)
(ii) Find the time taken by the boat to reach the light house if it is moving at the rate of 2 km per hour. (2)
OR
(ii) The ratio of the height of a light house and the length of its shadow on the ground is √3 : 1 . What is the
angle of elevation of the sun?
Ans:

(iii) Let height of light house be AB and its shadow be BC.


AB
In ∆ABC, tan  
AC
AB 3
But   3  tan   3  tan 600    600
AC 1

Prepared by: M. S. KumarSwamy, TGT(Maths) Page - 13 -


PM SHRI KENDRIYA VIDYALAYA GACHIBOWLI,GPRA CAMPUS,HYD-32
SAMPLE PAPER TEST 11 FOR BOARD EXAM 2024
(ANSWERS)
SUBJECT: MATHEMATICS MAX. MARKS : 80
CLASS : X DURATION : 3 HRS
General Instruction:
1. This Question Paper has 5 Sections A-E.
2. Section A has 20 MCQs carrying 1 mark each.
3. Section B has 5 questions carrying 02 marks each.
4. Section C has 6 questions carrying 03 marks each.
5. Section D has 4 questions carrying 05 marks each.
6. Section E has 3 case based integrated units of assessment (04 marks each) with sub-parts of the
values of 1, 1 and 2 marks each respectively.
7. All Questions are compulsory. However, an internal choice in 2 Qs of 5 marks, 2 Qs of 3 marks and
2 Questions of 2 marks has been provided. An internal choice has been provided in the 2marks
questions of Section E
8. Draw neat figures wherever required. Take π =22/7 wherever required if not stated.
SECTION – A
Questions 1 to 20 carry 1 mark each.

1. A ticket is drawn at random from a bag containing tickets numbered from 1 to 40. The probability
that the selected ticket has a number which is a multiple of 5 is
(a) 1/5 (b) 3/5 (c) 4/5 (d) 1
Ans: (a) 1/5

2. If two positive integers p and q can be expressed as p = ab3 and q = a3 b; a, b being prime numbers,
then HCF (p, q) is
(a) ab (b) a2 b2 (c) a3 b2 (d) a3 b3
Ans: (a) ab

3. If triangles ABC and DEF are similar and AB=4 cm, DE=6 cm, EF=9 cm and FD=12 cm, the perimeter of
triangle ABC is:
(a) 22 cm (b) 20 cm (c) 21 cm (d) 18 cm
Ans: (d) 18 cm
∆ABC ~ ∆DEF
AB=4 cm, DE=6 cm, EF=9 cm and FD=12 cm
⇒ AB/DE = BC/EF = AC/DF
⇒ 4/6 = BC/9 = AC/12
⇒ BC = (4.9)/6 = 6 cm
⇒ AC = (12.4)/6 = 8 cm
⇒ Perimeter of triangle ABC = AB + BC + AC = 4 + 6 + 8 =18 cm

4. If r = 3 is a root of quadratic equation kr2 – kr – 3 = 0, then the value of k is:


(a) 3/2 (b) 1/2 (c) 2 (d) 5/2
Ans: (b) 1/2
As r = 3 is a root of kr2 – kr – 3 = 0, we have:
9k – 3k – 3 = 0
⇒ 6k − 3 = 0 ⇒ k = 3/6 = 1/2

5. In the below figure, AD = 3 cm, AE = 5 cm, BD = 4 cm, CE = 4 cm, CF = 2 cm, BF = 2.5 cm, then
(a) DE || BC (b) DF || AC (c) EF || AB (d) none of these

Prepared by: M. S. KumarSwamy, TGT(Maths) Page - 1-


Ans: (c) EF || AB

1
6. If for some angle θ, cot 2θ = , then the value of cos3θ, where 3θ ≤ 90⁰, is
3
1 3
(a) (b) 1 (c) 0 (d)
2 2
Ans: (c) 0

7. In ∆ABC, right-angled at C, if tan A=1, then the value of 2sin A cos A is


1 3
(a) 1 (b) (c) 2 (d)
2 2
Ans: (a) 1

8. Volumes of two spheres are in the ratio 64:27. The ratio of their surface areas is
(a) 3:4 (b) 4:3 (c) 9:16 (d) 16:9
Ans. (d) 16:9

9. The LCM of smallest two digit composite number and smallest composite number is:
(a) 12 (b) 4 (c) 20 (d) 44
Ans: (c) 20

10. Find the value of k so that the following system of equations has no solution:
3x – y – 5 = 0, 6x – 2y + k = 0
(a) k ≠ 10 (b) k ≠ -10 (c) k ≠ 12 (d) k ≠ -12
Ans: (b) -10
Here a1 = 3, b1 = –1, c1 = –5,
and a2 = 6, b2 = –2, c2 = k.
For no solution,

11. The mean and median of a distribution are 14 and 15, respectively. The value of the mode is:
(a) 16 (b) 17 (c) 18 (d) 13
Ans. (b) 17

12. If the sum of the circumferences of two circles with radii R1 and R2 is equal to the circumference of
a circle of radius R, then:
(a) R1 + R2 = R
(b) R1 + R2 > R
(c) R1 + R2 < R
(d) Nothing definite can be said about the relation among R1, R2 and R.

Prepared by: M. S. KumarSwamy, TGT(Maths) Page - 2-


Ans. (a) R1 + R2 = R

13. In figure AT is a tangent to the circle with centre O such that OT = 4 cm and OTA = 30°. Then AT
is equal to

(a) 4 cm (b) 2 cm (c) 2√3 cm (d) 4√3 cm


Ans: (c) 2√3 cm

14. Mode and mean of a data are 12k and 15k. Median of the data is
(a) 12k (b) 14k (c) 15k (d) 16k
Ans: (b) 14k
∵ Mode = 3 median – 2 mean
⇒ 12k = 3 median – 2 × 15k
⇒ 42k = 3 median ⇒ Median = 14k.

15. 4 tan2 A – 4 sec2 A is equal to:


(a) 2 (b) 3 (c) 4 (d) –4
Ans: (d) –4
16. Which of the following equations has 2 as a root?
(a) x2 – 4x + 5 = 0 (b) x2 + 3x – 12 = 0
(c) 2x2 – 7x + 6 = 0 (d) 3x2 – 6x – 2 = 0
2
Ans: (c) 2x – 7x + 6 = 0

17. The radii of two concentric circles are 4 cm and 5 cm. The difference in the areas of these two
circles is:
(a) π (b) 7π (c) 9π (d) 13π
Ans. (c) 9π
Required difference = π(52 – 42) = 9π

18. If the distance between the points (x, –1) and (3, 2) is 5, then the value of x is
(a) –7 or –1 (b) –7 or 1 (c) 7 or 1 (d) 7 or –1
Ans: (d) 7 or –1

Direction : In the question number 19 & 20 , A statement of Assertion (A) is followed by a


statement of Reason(R) . Choose the correct option
(a) Both Assertion (A) and Reason (R) are true and Reason (R) is the correct explanation of Assertion
(A).
(b) Both Assertion (A) and Reason (R) are true but Reason (R) is not the correct explanation of
Assertion (A).
(c) Assertion (A) is true but Reason (R) is false.
(d) Assertion (A) is false but Reason (R) is true.

Prepared by: M. S. KumarSwamy, TGT(Maths) Page - 3-


19. Assertion (A): The number 6n, n being a natural number, ends with the digit 5.
Reason (R): The number 9n cannot end with digit 0 for any natural number n.
Ans: (d) Assertion (A) is false but Reason (R) is true.

20. Assertion (A): The point (3, 0) lies on x -axis.


Reason (R): The x co-ordinate on the point on y -axis is zero.
Ans: (b) Both assertion (A) and reason (R) are true and reason (R) is not the correct explanation of
Assertion (A)

SECTION-B
Questions 21 to 25 carry 2M each

21. If sin (A + B) = 1 and sin (A – B) = , 0 ≤ A + B ≤ 90° and A > B, then find A and B.
Ans: sin(A + B) = 1 = sin 90⁰
⇒ A + B = 90⁰ ........(i)
sin (A - B) = 1/2 = sin 30⁰
⇒ A - B = 30⁰ ........(ii)
Solving eq. (i) and (ii), A = 60⁰ and B = 30⁰
OR
2
1  tan A
Prove that:  tan 2 A
1  cot 2 A
1
2 2 2
1  tan A sec A cos 2 A  sin A  tan 2 A
Ans:  
1  cot 2 A cos ec 2 A 1 cos2 A
sin 2 A

22. The perimeter of a sector of a circle of radius 5.2 cm is 16.4 cm. Find the area of the sector.
Ans: Let AOB be the sector with O as center.
Given: Radius = r = 5.2 cm
Perimeter of sector =16.4 cm
So, OA + AB + OB = 16.4
⇒ 5.2 + 5.2 + AB = 16.4
⇒ AB = 6 cm
1 1
Area of sector =  rl   5.2  6 =15.6 cm2
2 2
OR
If the perimeter of a semi-circular protractor is 108 cm, find the diameter of the protractor. (Take
  22 / 7) )
Ans:

23. In the below figure, ΔABC is circumscribing a circle. Find the length of BC.

Prepared by: M. S. KumarSwamy, TGT(Maths) Page - 4-


Ans: AR = 4 cm
Also, AR = AQ (Length of tangents from A)
⇒ AQ = 4 cm
Now, QC = AC – AQ = 11 cm – 4 cm = 7 cm ...(i)
Also, BP = BR
∴ BP = 3 cm and PC = QC
⇒ PC = 7 cm [From (i)]
Hence, BC = BP + PC = 3 cm + 7 cm = 10 cm

24. Determine the values of a and b for which the following system of linear equations has infinite
solutions: 2x – (a – 4) y = 2b + 1; 4x – (a – 1) y = 5b – 1
Ans:

25. In ABC, DE || AB. If AD = 2x, DC = x + 3 , BE = 2x − 1 and CE = x, then find the value of ‘x’
CD CE
Ans:  by BPT
AD BE
x 3 x
⇒ 
2x 2x 1
⇒ 2x2 + 6x – x – 3 = 2x2 ⇒ 5x – 3 = 0 ⇒ x =

SECTION-C
Questions 26 to 31 carry 3 marks each

26. A man wished to give Rs. 12 to each person and found that he fell short of Rs. 6 when he wanted to
give to all the persons present. He, therefore, distributed Rs. 9 to each person and found that Rs. 9
were left over. How much money did he have and how many persons were there?
Ans. Let, number of persons = x

Prepared by: M. S. KumarSwamy, TGT(Maths) Page - 5-


Money share per person = y
Therefore, total money = Rs. xy
According to the question, 12 × x = xy + 6 ⇒ 12x – 6 = xy ...(i)
and 9x = xy – 9
9x + 9 = xy ...(ii)
Equating (i) and (ii), we get
12x – 6 = 9x + 9 ⇒ 3x = 15
Put the value of x in equation (i). Then
12 × 5 – 6 = x × y ⇒ xy = 54
So, he have Rs. 54 and there were 5 persons.
OR
A father’s age is three times the sum of the ages of his children. After 5 years, his age will be two
times the sum of their ages. Find the present age of the father.
Ans. Let the sum of the ages of two children be 'x' years and father's age be 'y' years.
According to the given condition: y = 3x
or y – 3x = 0 ...(i)
After 5 years, Father’s age = (y + 5) years
Sum of the ages of children = (x + 5 + 5) years.
Then, y + 5 = 2(x + 10)
or y – 2x – 15 = 0 ...(ii)
On subtracting equation (i) from equation (ii), we get x = 15
If we put the value of x in equation (i), we get
y – 3 × 15 = 0
⇒ y = 45
Hence, the present age of the father is 45 years.

sin   cos   1 1
27. Prove that 
cos   sin   1 sec   tan 
sin   cos   1 tan   1  sec 
Ans: LHS   [Dividng Nr and Dr by cosθ ]
cos   sin   1 1  tan   sec 
tan   sec   1 tan   sec   (sec 2   tan 2  )
 
1  tan   sec  1  tan   sec 
tan   sec   (sec   tan  )(sec   tan  )

1  tan   sec 
(sec   tan  )(1  sec   tan  )
  sec   tan 
1  tan   sec 
sec   tan  sec 2   tan 2 
 (sec   tan  )  
sec   tan  sec   tan 
1
  RHS
sec   tan 

28. Find the zeroes of the quadratic polynomial 2x2 – x – 6 and verify the relationship between the
zeroes and the coefficients of the polynomial.
Ans: 2x2 – x – 6 = 2x2 – 4x + 3x – 6
= 2x ( x – 2) + 3 ( x – 2 )
x = 2,

Sum of zeroes = 2 + ( )= = = = =

Product of zeroes = 2 x = =

Prepared by: M. S. KumarSwamy, TGT(Maths) Page - 6-


29. Given that √3 is irrational, prove that 5 + 2√3 is irrational.
Ans: Let us assume 5 + 2√3 is rational, then it must be in the form of p/q where p and q are co-
prime integers and q ≠0
i.e 5 + 2√3 = p/q
p  5q
So √3 = …(i)
2q
Since p, q, 5 and 2 are integers and q ≠ 0,
RHS of equation (i) is rational.
But LHS of (i) is √3 which is irrational. This is not possible.
This contradiction has arisen due to our wrong assumption that 5 + 2√3 is rational
So, 5 + 2√3 is irrational.

30. Cards numbered 1 to 30 are put in a bag. A card is drawn at random from this bag. Find the
probability that the number on the drawn card is
(i) not divisible by 3.
(ii) a prime number greater than 7.
(iii) not a perfect square number.
Ans : Total possible outcomes of drawing a card from a bag out of 30 cards = 30.
(i) Favourable outcomes for a card numbered not divisible by 3 = 20 (i.e. 1, 2, 4, 5, 7, 8, 10, 11, 13,
14, 16, 17, 19, 20, 22, 23, 25, 26, 28 and 29).
Probability of drawing a card numbered not divisible by 3 = 20/30 = 2/3
(ii) Favourable outcomes for a prime numbered card greater than 7 = 6 (i.e. 11, 13, 17, 19, 23 and
29)
Probability of drawing a prime number card, greater than 7 = 6/30 = 1/5
(iii) Favourable outcomes for not a perfect square numbered card = 25 (leaving 1, 4, 9, 16 and 25)
Probability of drawing a card which is not a perfect square = 25/30 = 5/6

31. Two tangents PA and PB are drawn to a circle with centre O from an external point P. Prove that
APB = 2OAB.

PA & PB are the tangents to a circle, with Centre O from a point P outside it.
PA= PB. (The tangents to a circle from an external point are equal in length )
∠PBA = ∠PAB [Angles opposite to the equal sides of a triangle are equal.]
∠APB+ ∠PBA +∠PAB= 180° [Sum of the angles of a triangle is 180°]
x° + ∠PAB +∠PAB = 180° (∠PBA = ∠PAB)
⇒ x° + 2∠PAB = 180°
⇒ ∠PAB =½(180° - x°)
⇒ ∠PAB =90° - x°/2
⇒ ∠OAB +∠PAB=90°
⇒ ∠OAB =90° - (90° - x°/2)
⇒ ∠OAB = x°/2 ⇒ ∠OAB = ∠APB /2 ⇒ ∠APB = 2∠OAB
OR
Prove that opposite sides of a quadrilateral circumscribing a circle subtend supplementary angles at
the centre of the circle.
Ans: Let ABCD be the quadrilateral circumscribing a circle at the center O such that it touches the
circle at the point P,Q,R,S. Let join the vertices of the quadrilateral ABCD to the center of the circle

Prepared by: M. S. KumarSwamy, TGT(Maths) Page - 7-


In ΔOAP and ΔOAS
AP=AS ( Tangents from to same point A)
PO=OS ( Radii of the same circle)
OA=OA ( Common side)
so, ΔOAP=ΔOAS (SSS congruence criterion)
∴ ∠POA=∠AOS (CPCT) ⇒ ∠1=∠8
Similarly, ∠2=∠3, ∠4=∠5 and ∠6=∠7
∠1+∠2 +∠3+∠4+∠5+∠6+∠7+∠8 = 360⁰
⇒ (∠1 +∠8) +(∠2 +∠3) + (∠4 +∠5) + (∠6 +∠7) = 360⁰
⇒ 2(∠1) + 2(∠2) + 2(∠5) + 2(∠6) = 360⁰
⇒ (∠1) + (∠2) + (∠5) + (∠6) = 180⁰
∴ ∠AOD + ∠COD=180⁰
Similarly, ∠BOC + ∠DOA = 180⁰

SECTION-D
Questions 32 to 35 carry 5M each
32. If a line is drawn parallel to one side of a triangle, prove that the other two sides are divided in the
same ratio. Using this theorem, find x in below figure, if MN || QR, PM = x cm, MQ = 10 cm, PN
= (x – 2) cm, NR = 6 cm

Ans: Given, To Prove, Constructions and Figure – 1½ marks


Proof – 1½ marks
1 marks to find x = 5 cm

33. A train travels at a certain average speed for a distance of 63 km and then travels at a distance of 72
km at an average speed of 6 km/hr more than its original speed. If it takes 3 hours to complete total
journey, what is the original average speed?
Ans : Let original average speed of the train be x km/h
New average speed be (x + 6) km/h
63
Time taken for a distance of 63 km = hours
x
72
Time taken for a distance of 72 km = hours
x6

Prepared by: M. S. KumarSwamy, TGT(Maths) Page - 8-


63 72 63( x  6)  72 x
According to the question,   3 3
x x6 x( x  6)
63 x  378  72 x 135 x  378
 2
 3 3
x  6x x2  6 x
⇒ 3(x2 + 6x) = 135x + 378 = 3(45x + 126)
⇒ x2 + 6x = 45x + 126 ⇒ x2 + 6x – 45x – 126 = 0 ⇒ x2 – 39x – 126 = 0
2
⇒ x – 42x + 3x – 126 = 0 ⇒ x(x – 42) + 3(x – 42) = 0 ⇒ (x – 42)(x + 3) = 0
∴ x – 42 = 0 or x + 3 = 0
⇒ x = 42 or x = – 3 (rejected)
Therefore, original average speed of the train is 42 km/h.
OR
In a flight of 600 km, an aircraft was slowed due to bad weather. Its average speed for the trip was
reduced by 200 km/hr and time of flight increased by 30 minutes. Find the original duration of
flight.
Ans : Let original speed of the aircraft be x km/hr
Reduced speed = (x – 200) km/hr
600 600 1 600 x  600 x  120000 1
According to given condition,    
x  200 x 2 x( x  200) 2
120000 1
 2  ⇒ x2 – 200x = 240000
x  200 x 2
⇒ x2 – 200x – 240000 = 0 ⇒ x2 – 600x + 400x – 240000 = 0
⇒ x(x – 600) + 400(x – 600) = 0 ⇒ (x + 400) (x – 600) = 0
⇒ x + 400 = 0 or x – 600 = 0 ⇒ x = – 400 (rejected) or x = 600
∴ original speed = 600 km/hr
∴ original duration of flight = = 1 hour

34. If the median of the following distribution is 46, find the missing frequencies p and q if the total
frequency is 230.
Marks 10 – 20 20 – 30 30 – 40 40 – 50 50 – 60 60 – 70 70 – 80
Frequency 12 30 p 65 q 25 18
Ans:

Prepared by: M. S. KumarSwamy, TGT(Maths) Page - 9-


35. Rasheed got a playing top (lattu) as his birthday present, which surprisingly had no colour on it. He
wanted to colour it with his crayons. The top is shaped like a cone surmounted by a hemisphere (see
below figure). The entire top is 5 cm in height and the diameter of the top is 3.5 cm. Find the area
he has to colour. (Take π = 22/7)
Ans: TSA of the toy = CSA of hemisphere + CSA of cone
22 3.5 3.5 2
Now, the curved surface area of the hemisphere = 2 r 2  2    cm
7 2 2
Also, the height of the cone = height of the top – height (radius) of the hemispherical part
 3.5 
= 5   3.25cm
 2 
2
2 2  3.5  2
So, the slant height of the cone (l) = r  h     (3.25)  3.7cm(approx.)
 2 
22 3.5
Therefore, CSA of cone = πrl =   3.7 cm2
7 2

 22 3.5 3.5   22 3.5 


∴ TSA of the toy =  2       3.7 
 7 2 2   7 2 
22 3.5 11
=  (3.5  3.7)  (3.5  3.7)  39.6cm2
7 2 2
OR
A solid toy is in the form of a hemisphere surmounted by a right circular cone. The height of the
cone is 2 cm and the diameter of the base is 4 cm. Determine the volume of the toy. If a right
circular cylinder circumscribes the toy, find the difference of the volumes of the cylinder and the
toy. (Take π = 3.14)
Ans: Let BPC be the hemisphere and ABC be the cone standing on the base of the hemisphere (see
below figure).

1
The radius BO of the hemisphere (as well as of the cone) =  4  2cm
2
2 3 1 2 1
So, volume of the toy =  r   r h   r 2 (2r  h)
3 3 3
1 1
  3.14  2  2  (4  2)   3.14  2  2  6 = 25.12 cm3
3 3

Prepared by: M. S. KumarSwamy, TGT(Maths) Page - 10-


Now, let the right circular cylinder EFGH circumscribe the given solid. The radius of the base of
the right circular cylinder = HP = BO = 2 cm, and its height is
EH = AO + OP = (2 + 2) cm = 4 cm
So, the volume required = volume of the right circular cylinder – volume of the toy
= (3.14 × 22 × 4 – 25.12) cm3
= 25.12 cm3
Hence, the required difference of the two volumes = 25.12 cm3.

SECTION-E (Case Study Based Questions)


Questions 36 to 38 carry 4M each

36. India is competitive manufacturing location due to the low cost of manpower and strong technical
and engineering capabilities contributing to higher quality production runs. The production of TV
sets in a factory increases uniformly by a fixed number every year. It produced 16000 sets in 6th
year and 22600 in 9th year.

On the basis of the above information, answer any four of the following questions:
(i) What is the production of first year? (1)
(ii) What is the production of 8th year? (1)
(iii) What is the production during first three years? (2)
OR
(iii) In which year, the production is 29,200? (2)
Ans: (i) Rs 5000
(ii) Production during 8th year is (a + 7d) = 5000 + 7(2200) = 20400
(iii) Production during first 3 year = 5000 + 7200 + 9400 = 21600
OR
(iii)an = a + (n – 1)d
⇒ 29200 = 5000 + (n – 1)2200
⇒ (n – 1)2200 = 29200 – 5000 = 24200
⇒ n – 1 = 24200/2200 = 11
⇒ n = 11 + 1 = 12

37. Raj is an electrician in a village. One day power was not there in entire village and villagers called
Raj to repair the fault. After thorough inspection he found an electric fault in one of the electric pole
of height 5 m and he has to repair it. He needs to reach a point 1.3m below the top of the pole to
undertake the repair work.

Prepared by: M. S. KumarSwamy, TGT(Maths) Page - 11-


Based on the above information answer the following questions.
(i) When the ladder is inclined at an angle of α such that √3 tan α + 2 = 5 to the horizontal, find the
angle α. (1)
(ii) In the above situation if BD = 3 cm and BC = 6 cm. Find α (1)
(iii) How far from the foot of the pole should he place the foot of the ladder? (Use √3 = 1.73) (2)
OR
(iii) Given 15 cot α = 8, find sin α. (2)
Ans: (i) tan α + 2 = 5
⇒ tan α = 5 – 2 =3
⇒ tan α = = tan 600
⇒ α = 600
(ii) BD = 3 cm and BC = 6 cm
BD
In ∆BCD, sinα 
BC
3 1
⇒ sinα   = sin300 ⇒ α = 300
6 2
(iii) BD = AD – AC = 5 – 1.3 = 3.7
BD
In ∆BCD, tan600 
DC
3.7
⇒  = 1.73
DC
⇒ DC = 3.7/1.73 = 2.14 m (approx.)

OR
cot α = 8/15
⇒ DC = 8 and BD = 15
From Pythagoras theorem,
BC2 = BD2 + DC2
⇒ BC2 = 152 + 82
⇒ BC = 225 + 64 = 289
⇒ BC = 17
⇒ sin α = BD/BC = 15/17

38. Aditya, Ritesh and Damodar are fast friend since childhood. They always want to sit in a row in the
classroom . But teacher doesn’t allow them and rotate the seats row-wise everyday. Ritesh is very
good in maths and he does distance calculation everyday. He consider the centre of class as origin

Prepared by: M. S. KumarSwamy, TGT(Maths) Page - 12-


and marks their position on a paper in a co-ordinate system. One day Ritesh make the following
diagram of their seating position marked Aditya as A, Ritesh as B and Damodar as C.

(i) What is the distance between A and B ? [1]


(ii) What is the distance between B and C ? [1]
(iii) A point D lies on the line segment between points A and B such that AD :DB = 4 : 3 . What are
the the coordinates of point D ? [2]
OR
(iii) If the point P(k, 0) divides the line segment joining the points A(2, –2) and B(–7, 4) in the ratio
1 : 2, then find the value of k [2]
Ans:
(i) It may be seen easily from figure that coordinates of point A are (- 2, 2).
AB2 = (− 2 + 1)2 + (2 + 2)2 = 1 + 42 = 17
⇒ AB = √17
(ii) It may be seen easily from figure that coordinates of point C are (3, 0).
BC2 = (− 1 − 3)2 + (− 2 − 0)2 = 42 + 4 = 20
⇒ BC = 2√5
m 4
(iii) We have A(- 2, 2) and B(- 1,- 2) and 1 
m2 3
m x  m2 x1 1(4)  3(2) 10
x 1 2  
m1  m2 43 7
m y  m2 y1 2(4)  3(2) 2
y 1 2  
m1  m2 43 7
 10 2 
Coordinates of D is  , 
 7 7 
OR

Prepared by: M. S. KumarSwamy, TGT(Maths) Page - 13-


m1 x2  m2 x1 m y  m2 y1
Using Section Formula, x  and y  1 2 , we get
m1  m2 m1  m2
 2  2  1 7  4  7 3
k    1
 1  2  3 3

Prepared by: M. S. KumarSwamy, TGT(Maths) Page - 14-


Directorate of Education, GNCT of Delhi
Practice Paper (Session: 2023-24)
Class – X
Subject – Mathematics

Max. Marks: 80 Duration: 3 hours

General Instructions:

1. This Question Paper has 5 Sections ‘A’, ‘B’, ‘C’, ‘D’ and ‘E’.
2. Section A has 20 MCQs carrying 1 mark each
3. Section B has 5 questions carrying 02 marks each.
4. Section C has 6 questions carrying 03 marks each.
5. Section D has 4 questions carrying 05 marks each.
6. Section E has 3 case based questions (04 marks each) with sub parts of the values of 1, 1 and
2 marks each respectively.
7. All Questions are compulsory. However, an internal choice in 2 Questions of 5 marks, 2
Questions of 3 marks and 2 Questions of 2 marks has been provided. An internal choice has
been provided in the 2 marks questions of Section E.
8. Draw neat figures wherever required. Take π =22/7 wherever required if not stated.
9. Use of calculator is not permitted.
Please do write down the Serial Number of the question before attempting it.

SECTION A
Section A consists of 20 questions of 1 mark each.
3
1. The probability of happening of an event is . The probability of not happening of that event is :
7
1 4 2 3
(a) (b) (c) (d)
10 7 5 10

2. The value of m for which the system of equations 6x – 3y + 10 = 0 and 2x – my = – 9 has no


solution is:
(a) -1 (b) 1 (c) -3 (d) 3

3. HCF of co-prime numbers is always __________ .


(a) 0 (b) 1 (c) largest number (d) product of all numbers

4. If 𝛼 and 𝛽 are the zeroes of the polynomial p(x) = x2 – ax – b, then the value of 𝛼 2 + 𝛽 2 is:
(a) a2 – 2b (b) a2 + 2b (c) b2 – 2a (d) b2 + 2a
5. A quadratic polynomial, sum of whose zeroes is -3 and product is -10 is :
(a) x2 – 3x – 10 (b) x2 – 3x +10 (c) x2 + 3x + 10 (d) x2 + 3x – 10

6. Given that HCF (336,54) is 6 then LCM (336,54) is :


(a) 3024 (b) 3360 (c) 2688 (d) 2016

7. The prime factorization of 234 is:


(a) 2 x 2 x 3 x 13 (b) 2 x 3 x 3 x 13
(c) 2 x 3 x 13 (d) 2 x 2 x 2 x 13

22
8. The area of a quadrant of a circle having radius 14 cm is: (Take 𝜋 = )
7
(a) 145 cm2 (b) 186 cm2 (c) 168 cm2 (d) 154 cm2

9. The median of the given data with the observations in ascending order is 27.5.The value of x is:
24, 25, 26, x+2, x+3, 30, 33, 37
(a) 25 (b) 27 (c) 27.5 (d) 25.5

10. In the given figure, AP, AQ and BC are tangents to the circle. If AB = 5 cm, AC = 6 cm and BC = 4
cm, then the length of AP (in cm) is :

(a) 4.5 (b) 7.5 (c) 5 (d) 5.5

11. A bag contains 6 red balls and 5 blue balls. One ball is drawn at random. The probability of getting a
blue ball is:
6 5 6 5
(a) 11 (b) 11 (c) 5 (d) 6

12. A pair of linear equations of two variables has unique solution. What type of lines will its graph
represent?
(a) parallel (b) intersecting (c) co - incident (d) perpendicular

13. If ∆ABC ∼ ∆ DEF and ∠A=450, ∠C=550 , then the value of ∠E is:
(a) 450 (b) 750 (c) 550 (d) 800
14. A pendulum swings through an angle of 300 and describes an arc 17.6 cm in length. The length of
pendulum is:
(a) 26.2 cm (b) 33.6 cm (c) 28.5 cm (d) 32.4 cm

3
15. A surveyor wants to find out the height of a tower. He measures ∠A as tan A = 4 .What is the height
of the tower if A is 40 m from its base as shown in the figure?

B A
40 m

(a) 20 m (b) 10 m (c) 30 m (d) 40 m

16. The number of zeroes of the polynomial shown in the below graph is :

(a) 1 (b) 2 (c) 4 (d) 3

3
17. If cosec 𝜃 = 2, then 4(cosec2θ + cot2θ) is equal to :

(a) 18 (b) 10 (c) 14 (d) 6

18. In the given figure, the quadrilateral ABCD circumscribes a circle. Here PB + CR is equal to :

(a) AB (b) BC (c) CD (d) AD


Directions for Q 19 & 20 : There is one Assertion (A) and one Reason (R). Choose the correct answer of
these questions from the four options (a),(b),(c) and (d) given below :

(a) Both Assertion (A) and Reason (R) are true and Reason (R) is the correct
explanation of the assertion (A).
(b) Both Assertion (A) and Reason (R) are true but Reason (R) is not the correct
explanation of the assertion (A).
(c) Assertion (A) is true but reason (R) is false.
(d) Assertion (A) is false but reason (R) is true.

19. Assertion (A) : a,b,c are in AP if and only if 2b = a + c


Reason (R) : The sum of first n odd natural numbers is n2 .

20. Assertion (A) : If the points A(4,3) and B(x,5) lie on a circle with centre O(2,3) then
the value of x is 2.
Reason (R) : The centre of a circle is the mid-point of each chord of the circle.

SECTION –B
Section B consists of 5 questions of 2 marks each.
21. Find the area of a sector of a circle with the radius 6 cm if angle of the sector is 600 .
OR
The minute hand of a clock is 12 cm long. Find the area of the face of the clock described by the
minute hand in 35 minutes.

22. In the given figure, find the value of x which will make DE∥ BC.

7
23. If cos A = , find the value of tan A + cot A.
25
OR
5 x2 −1
If 5𝑥 = 𝑠𝑒𝑐𝜃 and = 𝑡𝑎𝑛𝜃, then find the value of 5 ( ).
𝑥 x2

24. A right triangle having sides a, b and c ,where c is the hypotenuse, is circumscribing a circle. Prove
(𝑎+𝑏−𝑐)
that the radius r of the circle is given by r = 2 .

25. How many terms of the AP: 24 , 21 , 18…… must be taken so that their sum is 78?
SECTION – C
Section C consists of 6 questions of 3 marks each.

26. Prove that √2 is an irrational number.

(𝑐𝑜𝑠𝜃−𝑠𝑖𝑛𝜃+1)
27. Prove that: = cosec θ + cot θ
(𝑐𝑜𝑠𝜃+𝑠𝑖𝑛𝜃+1)

28. In figure, two tangents TP and TQ are drawn to a circle with centre O from an external point P.
Prove that ∠PTQ=2 ∠OPQ.

29. 90 cards numbered from 1 to 90 are placed in a box. If one card is drawn at random from the box
find the probability that it is:
(i) a two-digit number
(ii) a perfect square
(iii) a number divisible by 5.
OR
Red queen and a black jack are removed from a pack of 52 playing cards. Find the probability that
the card drawn from the remaining cards is:
(i) a red card
(ii) neither a jack nor a king
(iii) either a king or a queen.

30. Find the value of p for which the quadratic equation px(x – 2) + 6 = 0 has two equal roots.
OR
One year ago, a man was 8 times as old as his son. At present, his age is equal to the square of his
son’s age in years. Find their present ages.

31. A well of diameter 3m is dug 14m deep. The earth taken out of it has been spread evenly all-around
tit in the shape of a circular ring of width 4m to form a embankment. Find the height of the
embankment.

SECTION – D
Section D consists of 4 questions of 5 marks each.

32. Find the number of terms in an A.P. 18,15,12,……., – 48 and also the sum of all its terms.

33. State and prove Basic Proportionality Theorem.


34. Prove that the points A(0, -1), B(-2, 3), C(6, 7) and D(8, 3) are the vertices of a rectangle ABCD.
OR
Points A(-1, y) and B(5, 7) lie on a circle with centre O(2, -3y). Find the values of y. Hence find the
radius of the circle.

35. A statue which is x m tall stands on the top of 100m long pedestal from a point on the ground. The
angle of elevation of the top of the statue is 600 and from the same point, the angle of elevation of the
top of the pedestal is 450. Find the height of the statue.
OR
Two poles of equal heights are standing opposite each other on either side of the road, which is
100 m wide. The angles of elevation of the top of the poles, from a point between them on the road,
are 300 and 600 respectively. Find the height of the poles and the distances of the point from the
poles. (See in figure)

300 600

100 m

SECTION – E
Section E has 3 case based questions of 04 marks each.

36. Currency is a medium of exchange for goods and services. Ashok made a payment to shopkeeper, for
the article he purchased, in the denomination of ₹500 and ₹ 20. He paid ₹ 2640 to the shopkeeper. If
all the denomination of ₹ 500 is to be changed by ₹ 100 then the boy could have paid ₹ 640 only.
Assume the number of currency notes of ₹ 500 as ‘f’ and the number of currency notes of ₹ 20 as ‘g’.

Based on the above information, answer the following questions:


i) Write a linear equation representing the amount paid by Ashok. 1
ii) If Ashok had to pay ₹ 2800 to the shopkeeper, how many ₹20 notes did he need? 1

iii) Find the number of notes in the denomination of ₹500 Ashok had? 2
OR
How many total currency notes did Ashok gave to shopkeeper?

37. All the students were very excited for the Annual Sports Day celebration in the school. In a
running competition, the time taken by students was noted down. The following data was obtained:

Time (in seconds) 0-20 20-40 40-60 60-80 80-100

No. of students 1 4 3 7 5

Based on the above information, answer the following questions:

i) Find the sum of the lower limits of the median class and modal class. 1

ii) What is the class mark of modal class? 1

iii) Find the mean of the above distribution. 2


OR
Find the median run time (in seconds).

38. An engineer is planning to make all the Pillars of the Metro green with plants to make these
beautiful and to contribute for healthy environment as shown in the picture. Observe the picture
and answer the questions if dimension of one pillar is 1.5 m X 1.5 m X 20 m.

Based on the above information, answer the following questions:

i) What is the shape of the pillars? 1

ii) How much cement is used to fill the pillar? 1

iii) Find the lateral surface area of one pillar. 2


OR
Find the cost of the plantation if it costs ₹ 50 per m2.
Directorate of Education, GNCT of Delhi
Practice Paper (Session: 2023-24)
Class – X
Subject – Mathematics

Max. Marks: 80 Duration: 3 hours

General Instructions:

1. This Question Paper has 5 Sections ‘A’, ‘B’, ‘C’, ‘D’ and ‘E’.
2. Section A has 20 MCQs carrying 1 mark each
3. Section B has 5 questions carrying 02 marks each.
4. Section C has 6 questions carrying 03 marks each.
5. Section D has 4 questions carrying 05 marks each.
6. Section E has 3 case based questions (04 marks each) with sub parts of the values of 1, 1 and 2
marks each respectively.
7. All Questions are compulsory. However, an internal choice in 2 Questions of 5 marks, 2
Questions of 3 marks and 2 Questions of 2 marks has been provided. An internal choice has
been provided in the 2 marks questions of Section E.
8. Draw neat figures wherever required. Take π =22/7 wherever required if not stated.
9. Use of calculator is not permitted.
Please do write down the Serial Number of the question before attempting it.

SECTION – A
Section A consists of 20 questions of 1 mark each.

1. HCF of a4×b3×c and a7×b2×c3 is :


(a) a4×b2×c (b) a4×b3×c (c) a7×b3×c3 (d) a4×b3×c3

2. The linear equation 3x – 7y = 10 when expressed in terms of x becomes:


(a) 10 + 7y (b) 3x – 10 (c) (d)

3. In the given figure, = and ∠PST = ∠PRQ then type P


of ∆PQR is :

(a) isosceles (b) right-angled S T

(c) scalene (d) Equilateral


Q R
4. Which of the following number is irrational?
(a) √36 (b) 0.85 (c) 31.480152…… (d) 1.434343……

5. The roots of the quadratic equation x2 + 4x + 5 = 0 are:


(a) Real (b) not real (c) real and distinct (d) real and equal

6. A die
ie is thrown once. The probability of getting an odd number is:
(a) (b) (c) (d)

7. In a ∆ABC, D and E are points on sides AC and BC respectively such that DE ∥ AB and
ADD = 2x, DC = x + 3, BE = 2x
2x-1 and CE = x. The value of x is:

(a) (b) (c) (d)

8. PQ is tangent to a circle centred at O


O, touching the circle at point P.. If the radius of the circle is 5
cm and OQ = 5√3 cm, then hen the length of the tangent PQ is:
(a) 5√2 cm (b) cm (c) 10 cm (d) cm
√ √

9. The 30thh term of the A.P. 10, 7, 4……. iis:


(a) -97 (b) -77 (c) -87 (d) -107

10. “The product of two consecutive even integers is 528”. The quadratic equation for the
above statement is:
(a) x(x+2) =528 (b) 2x(2x+1) =528 (c) 2x(x+4) =528 (d) 2x(x+1) =528

11. In the given figure, O is the centre of a circle, PQ is a chord and PT is the tangent at P.
If ∠POQ = 70°, then ∠TPQTPQ is:

(a) 700 (b) 550 (c) 350 (d) 400

12. The sum and product of the zeroes of a polynomial are and respectively then the
value of coefficient c is :
(a) 1 (b) 5 (c) 2 (d) 4

13. If ∆ABC ∼ ∆ DEF, 4AB=


=DE and BC=2.5 cm , then the value of EF is:
(a) 25 cm
m (b) 10 cm (c) 2.5 cm
m (d) 5 cm

14. The value of ‘𝑏’ if ∝ and ∝


are zeroes of polynomial ax2 + bx
x + c = 0 is:

(a) (b) (c) 1 (d)


𝐴𝑂𝐵= 1250, then value of ∠COD is:
15. In the given figure, if ∠𝐴𝑂𝐵

(a) 700 (b) 550 (c) 1050 (d) 850

16. BM is the median of △BCD


BCD.. Which is the false statement of the following:
(a) M, is the mid-point
point of CD (b) ar (△BCM)
BCM) = ar (△BDM)
(
(c) ar (△BCM) < ar (△△BDM) (d) ar (△BCM) < ar (△BCD)

17. The value of 𝑡𝑎𝑛30 𝑡𝑎𝑛60


60 − 𝑐𝑜𝑡30 𝑐𝑜𝑡60 is :

(a) √3 (b) 1 (c) 0 (d)

18. A vertical stick 15 m long casts a shadow 55√3 m long on ground. At the same time , the angle of
elevation of the sun is:
(a) 600 (b) 450 (c) 300 (d) 900

Directions for Q 19 & 20 : There is one Assertion (A) and one Reason (R). Choose the correct
answer of these questions from the four options (a),(b),(c) and (d)
given below :

(a) Both Assertion (A) and Reason (R) are true and Reason (R) is the correct
explanation of the assertion (A).
(b) Both Assertion (A) and Reason (R) are true but Reason (R) is not the correct
explanation of the assertion (A).
(c) Assertion (A) is true but reason (R) is false.
(d) Assertion (A) is fals
false but reason (R) is true.

19. Assertion (A): is a rational number.


Reason (R): The denominator of has 3 as one of its prime factor.

20. Assertion (A): The angle of the quadrant of a circle is 60°.

Reason (R): A quadrant is part of a circle.


SECTION – B
Section B consists of 5 questions of 2 marks each.

21. In the figure, the radius of the circle is 21 cm and side of the inscribed square is 13 cm.
Find the area of the shaded region.

OR
Find the area of the quadrant of a circle whose circumference is 22 cm.

22. There are 8 rotten apples in a carton and the probability of getting a rotten apple out of the
carton is . Find the total number of apples in the carton.

23. Find the value of a, if the distance between the points A (-3, -14) and B (a, -5) is 9 units.
OR
Find a relation between x and y such that the point (x, y) is equidistant from the point
(3,6) and (-3,4).

24. Find the discriminant of the quadratic equation 5x2 + 7x + 2 = 0.

∅ ∅
25. If 7 tan ∅ = 4, then find the value of .
∅ ∅

SECTION – C
Section C consists of 6 questions of 3 marks each.

26. In the given figure, △PST is an isosceles triangle having PS = ST , touching a circle at
points E ,B and N . Prove that the point N is midpoint of ST.
P

B E

S N T
27. Show that 3 + √7 is an irrational number, given that √7 is an irrational
irration number.
OR
n
Check whether 15 can ennd with the digit 0 for any natural number n.

28. How many multiples of 11 lie between 10 and 650


650?

29. From the top of a vertical tower, the angles of depression of two cars, in the same straight
line with the base of the tower, at an instant are found to be 300 and 450. If the cars are
83 m apart and on the same side of the tower, find the height of the tower.

30. A spherical glass vessel has a cylindrical neck 8 cm long and of 1 cm radius. The radius of
spherical part is 9 cm. Find the amount of water, it can hold, when filled compl
completely.
OR
A hemispherical depression is cut off from one face of a cubical wooden block such that the
diameter of the hemisphere is equal to the edge of the cube. The edge of the cube is 14 cm.
Determine thee surface area of the remaining solid.

31. Find the median weight of the 30 students as per the distribution given below.

Weight 40- 45 45- 50 50- 55 55- 60 60- 65 65-70


70 70- 75
(in kg)

No. of
2 3 8 6 6 3 2
students

SECTION – D
Section D consists of 4 questions of 5 marks each.

32. If AD and PM are medians of triangles ABC and PQR respectively where △ABC ~ △PQR.
Prove that = .
OR
In the given figure, = and ∠1 = ∠2 then show that △PQS ~ △TQR.
TQR.
° ° °
33. Evaluate ° ° °

OR

Prove that (sin θ + cosecc θ)2 + (cos θ + sec θ)2 = 7 + tan2θ + cot2θ

34. Find the missing frequencies f1 and f2, if the mean of 50 observations given below is 38.2.

Class interval Frequency


0 – 10 4
10 - 20 4
20 – 30 f1
30 - 40 10
40 - 50 f2
50 - 60 8
60 – 70 5

35. If α and β are the zeroes of the quadratic polynomial f(x) = x2 – p (x + 1) – c then show that
(α +1) (β +1) = 1 – c. If p =1 and c =5 then find α and β.

SECTION – E
Section E has 3 case based questions of 04 marks each.
36. Sunit went to India-gate
gate in one fine evening. He observed the top of the India
India-gate standing on
the ground. The position of India
India-gate
gate as AB and Anil as C is drawn in the cartesian plane as
shown in the figure.

Based on the above information, answer the following questions:

i) Find the distance betw


between Sunit and the top of India-gate.
gate. 1

ii) In which ratio point D divides the line


line-segment AC ? 1
iii) Suresh was standing exactly in between the Sunit and India-gate. 2
What are the coordinates of position of Suresh ?
OR
Find the value of sin C.

37. A company deals in casting and moulding of metal on orders received from its clients. In one
such order, company is supposed to make 50 toys in the form of a hemisphere surmounted by
a right circular cone of the same base radius as that of hemisphere. The radius of the base of the
cone is 21 cm and height is 28 cm.

Based on the above information, answer the following questions:

i) Find the surface area of the conical part. 1

ii) Find the surface area of the hemispherical part. 1

iii) Find the volume of 50 toys. 2


OR
Find the ratio of the volume of hemisphere to the volume of cone.

38. Pankaj wants to participate in the push-up challenge. He can currently make 3000 push-ups in
one hour. But he wanted to achieve a target of 3900 push-ups in one-hour. With each day of
practice, he is able to make 5 more push-ups in one hour as compared to the previous day. On
first day of practice, he do 3000 push-ups and continues to practice regularly till his target is
achieved.

Based on the above information, answer the following questions:

i) Form an AP representing the number of push-ups per day. 1


ii) Find the total number of push-ups performed by Pankaj in 1
starting first week.
iii) Find the minimum number of days Pankaj needs to practice before 2
his goal is achieved.
OR
On which day, Pankaj performed 3500 push-ups?
Marking Scheme
Class X Session 2023-24
MATHEMATICS STANDARD (Code No.041)
TIME: 3 hours MAX.MARKS: 80
SECTION A
Section A consists of 20 questions of 1 mark each.
1. (b) xy2 1
2. (b) 1 zero and the zero is ‘3’ 1
3. 1
(b) = ≠
4. (c) 2 distinct real roots 1
5. (c) 7 1
6. (a) 1:2 1
7. (d) infinitely many 1
8. 1
(b)

9. (b) 100° 1
10. (d) 11 cm 1
11. √ 1
(c)
12. (d) cos A 1
13. (a) 60° 1
14. (a) 2 units 1
15. (a) 10m 1
16. 1
(b)
17. 1
(b)
18. (d) 150 1
19. (a) Both assertion (A) and reason (R) are true and reason (R) is the correct explanation of 1
assertion (A)
20. (c) Assertion (A) is true but reason (R) is false. 1
SECTION B
Section B consists of 5 questions of 2 marks each.
21. Let us assume, to the contrary, that √2 is rational.
𝑎 ½
So, we can find integers a and b such that √2 = 𝑏 where a and b are coprime.
So, b √2 = a.
Squaring both sides,
we get 2b2 = a2. ½
Therefore, 2 divides a2 and so 2 divides a.
So, we can write a = 2c for some integer c.
Substituting for a, we get 2b2 = 4c2, that is, b2 = 2c2. ½
This means that 2 divides b2, and so 2 divides b
Therefore, a and b have at least 2 as a common factor.
But this contradicts the fact that a and b have no common factors other than 1. ½
This contradiction has arisen because of our incorrect assumption that √2 is rational.
So, we conclude that √2 is irrational.

1
22. ABCD is a parallelogram. ½
AB = DC = a
Point P divides AB in the ratio 2:3
AP = a , BP = a
point Q divides DC in the ratio 4:1. ½
DQ = a , CQ = a
∆ APO ~ ∆ CQO [AA similarity]
½
= =
½
= = OC = ½ OA
23.
PA = PB; CA = CE; DE = DB [Tangents to a circle] ½
Perimeter of ∆PCD = PC + CD + PD
= PC + CE + ED + PD
= PC + CA + BD + PD
= PA + PB 1
Perimeter of ∆PCD = PA + PA = 2PA = 2(10) = 20 ½
cm
24. ∵ tan(𝐴 + 𝐵) = √3 ∴ 𝐴 + 𝐵 = 60 …(1) ½
∵ tan(𝐴 − 𝐵) = ∴ 𝐴 − 𝐵 = 30 …(2) ½
√ ½
Adding (1) & (2), we get 2A=90 ⟹ 𝐴 = 45 ½
Also (1) –(2), we get 2𝐵 = 30 ⟹ 𝐵 = 45
[or]

2 cosec230 + x sin260 – tan230 = 10

√ 3
⇒ 2(2)2 + x −4 = 10 1

3 3 1
⇒ 2(4) + x 4 − 4 3 = 10 ½
3 1
⇒ 8 + x 4 − 4 = 10
⇒ 32 + x (3) −1 = 40 ½
⇒ 3x = 9 ⇒ x = 3
25. Total area removed =

π𝑟 +

π𝑟 +

π𝑟 ½
∠ ∠ ∠
= π𝑟
= π𝑟 ½

= X X (14) ½ ½
= 308 cm2
[or]

The side of a square = Diameter of the semi-circle = a


Area of the unshaded region ½
= Area of a square of side ‘a’ + 4(Area of a semi-circle of diameter ‘a’)
The horizontal/vertical extent of the white region = 14-3-3 = 8 cm ½
Radius of the semi-circle + side of a square + Radius of the semi-circle = 8 cm

2
2 (radius of the semi-circle) + side of a square = 8 cm
2a = 8 cm ⇒ a = 4 cm ½
Area of the unshaded region
= Area of a square of side 4 cm + 4 (Area of a semi-circle of diameter 4 cm)
= (4) + 4 X π(2) = (16 + 8π) cm2 ½
SECTION C

Section C consists of 6 questions of 3 marks each


26. Number of students in each group subject to the given condition = HCF (60,84,108) ½
HCF (60,84,108) = 12 ½
Number of groups in Music = =5
½
Number of groups in Dance = =7 ½
½
Number of groups in Handicrafts = =9 ½
Total number of rooms required = 21
27. P(x) = 5x2 + 5x + 1 ½
α+β = = = -1
½
αβ = = ½
𝛼 +𝛽 = (𝛼 + 𝛽) - 2𝛼𝛽
½
= (-1)2 – 2
½
=1- =
½
𝛼 +𝛽 = +
(𝛼+𝛽) ( )
= = = -5
28. Let the ten’s and the unit’s digits in the first number be x and y, respectively.
So, the original number = 10x + y
When the digits are reversed, x becomes the unit’s digit and y becomes the ten’s
Digit. ½
So the obtain by reversing the digits= 10y + x
According to the given condition.
(10x + y) + (10y + x) = 66
i.e., 11(x + y) = 66 ½
i.e., x + y = 6 ---- (1)
We are also given that the digits differ by 2, ½
therefore, either x – y = 2 ---- (2) ½
or y – x = 2 ---- (3)
If x – y = 2, then solving (1) and (2) by elimination, we get x = 4 and y = 2. ½
In this case, we get the number 42.
If y – x = 2, then solving (1) and (3) by elimination, we get x = 2 and y = 4. ½
In this case, we get the number 24.
Thus, there are two such numbers 42 and 24.
[or]
Let √ be ‘m’ and √ be ‘n’, ½
Then the given equations become
2m + 3n = 2
½
4m - 9n = -1
3
( 2m + 3n = 2) X- 2 ⇒ −4𝑚 − 6𝑛 = −4 …(1)
4m - 9n = -1 4𝑚 − 9𝑛 = −1 …(2)
Adding (1) and (2)
We get −15𝑛 = −5 ⇒ 𝑛 = ½

Substituting n = in 2m + 3n = 2, we get ½
2m + 1 = 2
2m = 1
m= 1
m= ⇒ √𝑥 = 2 ⇒ x = 4 and n = ⇒ 𝑦=3 ⇒ y=9
29.
∠OAB = 30°
∠OAP = 90° [Angle between the tangent and
the radius at the point of contact]
∠PAB = 90° - 30° =60° ½
AP = BP [Tangents to a circle from an external point]
∠PAB = ∠PBA [Angles opposite to equal sides of a triangle] ½
In ΔABP, ∠PAB + ∠PBA + ∠APB = 180° [Angle Sum Property]
60° + 60° + ∠APB = 180°
∠APB = 60° ½
∴ ΔABP is an equilateral triangle, where AP = BP = AB.
PA = 6 cm ½
In Right ΔOAP, ∠OPA = 30°
tan 30° =
½
=

OA = = 2√3𝑐𝑚 ½

[or]

Let ∠ TPQ = 𝜃
∠ TPO = 90° [Angle between the tangent and
the radius at the point of contact] ½
∠ OPQ = 90° - 𝜃
TP = TQ [Tangents to a circle from an external
point]
½
∠ TPQ = ∠ TQP = 𝜃 [Angles opposite to equal sides of a triangle] ½
In ΔPQT, ∠PQT + ∠QPT + ∠PTQ = 180° [Angle Sum Property] ½
𝜃 + 𝜃 + ∠PTQ = 180°
∠PTQ = 180° - 2 𝜃 ½
∠PTQ = 2 (90° - 𝜃) ½
∠PTQ = 2 ∠ OPQ [using (1)]
30. Given, 1 + sin2θ = 3 sin θ cos θ
Dividing both sides by cos2θ,
+ tan2θ = 3 tan 𝜃
½
sec2θ + tan2θ = 3 tan 𝜃
½
1 + tan2θ + tan2θ = 3 tan 𝜃
½
1 + 2 tan2θ = 3 tan 𝜃
½
2 tan θ - 3 tan 𝜃 +1 = 0
2

If tan θ = x, then the equation becomes 2x2 -3x + 1 = 0


4
⇒ (𝑥 − 1)(2𝑥 − 1) = 0 x = 1 or
tan θ = 1 or 1
31.
Length Number of
CI Mid x d fd
[in mm] leaves (f)
118 – 126 3 117.5 - 126.5 122 -27 -81

127 – 135 5 126.5 – 135.5 131 -18 -90

136 – 144 9 135.5 – 144.5 140 -9 -81

145 – 153 12 144.5 – 153.5 a = 149 0 0


153.5 – 162.5 158 9 45 2
154 – 162 5
163 – 171 4 162.5 – 171.5 167 18 72 ½
½
172 – 180 2 171.5 – 180.5 176 27 54

Mean = a + = 149 +

= 149 – 2.025 = 146.975
Average length of the leaves = 146.975
SECTION D

Section D consists of 4 questions of 5 marks each

32. Let the speed of the stream be x km/h.


The speed of the boat upstream = (18 – x) km/h and
the speed of the boat downstream = (18 + x) km/h. 1
The time taken to go upstream = = hours

the time taken to go downstream = = hours 1


According to the question,
- =1 1

24(18 + x) – 24(18 – x) = (18 – x) (18 + x)


x2 + 48x – 324 = 0
x = 6 or – 54 1
Since x is the speed of the stream, it cannot be negative.
Therefore, x = 6 gives the speed of the stream = 6 km/h.
1
[or]
Let the time taken by the smaller pipe to fill the tank = x hr.
Time taken by the larger pipe = (x – 10) hr ½
Part of the tank filled by smaller pipe in 1 hour =
1
Part of the tank filled by larger pipe in 1 hour =
3
The tank can be filled in 9 8 = hours by both the pipes together. ½

Part of the tank filled by both the pipes in 1 hour = ½

5
Therefore, + = ½
8x2 – 230x +750 = 0
1
x = 25,
Time taken by the smaller pipe cannot be 30/8 = 3.75 hours, as the time taken by ½
the larger pipe will become negative, which is logically not possible.
Therefore, the time taken individually by the smaller pipe is 25 hours and the larger ½
pipe will be 25 – 10 =15 hours.

33. (a) Statement – ½


Given and To Prove – ½
Figure and Construction ½ 3
Proof – 1 ½
G
[b] Draw DG ‖ BE
½
In Δ ABE, = [BPT]

CF = FD [F is the midpoint of DC] ---(i) ½


In Δ CDG, = = 1 [Mid point theorem] ½
GE = CE ---(ii)
∠CEF = ∠CFE [Given]
CF = CE [Sides opposite to equal angles] ---(iii) ½
From (ii) & (iii) CF = GE ---(iv)
From (i) & (iv) GE = FD
∴ = ⇒ =
34.
Length of the pond, l= 50m, width of the pond, b = 44m
Water level is to rise by, h = 21 cm = m
Volume of water in the pond = lbh = 50 x 44 x m3 =462 m3 1
Diameter of the pipe = 14 cm
Radius of the pipe, r = 7cm = m
Area of cross-section of pipe = πr²
1
= x x = m2 ½
Rate at which the water is flowing through the pipe, h = 15km/h = 15000 m/h
Volume of water flowing in 1 hour = Area of cross-section of pipe x height of water ½
coming out of pipe
= × 15000 𝑚 1
𝑉𝑜𝑙𝑢𝑚𝑒 𝑜𝑓 𝑡ℎ𝑒 𝑝𝑜𝑛𝑑
Time required to fill the pond = 𝑉𝑜𝑙𝑢𝑚𝑒 𝑜𝑓 𝑤𝑎𝑡𝑒𝑟 𝑓𝑙𝑜𝑤𝑖𝑛𝑔 𝑖𝑛 1 ℎ𝑜𝑢𝑟 1
×
= = 2 hours
×
Speed of water if the rise in water level is to be attained in 1 hour = 30km/h
[or]

6
Radius of the cylindrical tent (r) = 14 m
Total height of the tent = 13.5 m
Height of the cylinder = 3 m
Height of the Conical part = 10.5 m ½
Slant height of the cone (𝑙) = √ℎ + 𝑟
= (10.5) + (14)
= √110.25 + 196
1
= √306.25 = 17.5 m
Curved surface area of cylindrical portion
= 2πrh
= 2x ×14×3 1
= 264 m2
Curved surface area of conical portion
=πrl
= ×14×17.5 1
=770 m2 ½
Total curved surface area = 264 m2 + 770 m2 = 1034 m2
Provision for stitching and wastage = 26 m2
½
Area of canvas to be purchased = 1060 m2
Cost of canvas = Rate × Surface area ½
= 500 x 1060 = ₹ 5,30,000/-
35.
Number of Cumulative
Marks obtained
students frequency
20 – 30 p p
30 – 40 15 p + 15
40 – 50 25 p + 40 1
50 – 60 20 p + 60
60 – 70 q p + q + 60
70 – 80 8 p + q + 68 ½
80 - 90 10 p + q + 78 ½
90
p + q + 78 = 90
p + q = 12

Median =(𝑙) + .h
( ) ½
50 = 50 + . 10
( ) ½
. 10 = 0
45 − (𝑝 + 40) = 0
½
P=5
½
5 + q = 12
q=7
1
Mode = 𝑙 + .h

7
= 40 + . 10
( )
= 40 + = 40 + 6.67 = 46.67
SECTION E
36. (i) Number of throws during camp. a = 40; d = 12 1
𝑡 = 𝑎 + 10𝑑
= 40 + 10 × 12
= 160 𝑡ℎ𝑟𝑜𝑤𝑠
(ii) a = 7.56 m; d = 9cm = 0.09 m ½
n = 6 weeks ½
tn = a + (n-1) d ½
= 7.56 + 6(0.09)
= 7.56 + 0.54 ½
Sanjitha’s throw distance at the end of 6 weeks = 8.1 m
(or)
a = 7.56 m; d = 9cm = 0.09 m ½
tn =11.16 m ½
tn = a + (n-1) d
11.16 = 7.56 + (n-1) (0.09) ½
3.6 = (n-1) (0.09)
.
n-1 = = 40
.
n = 41 ½
Sanjitha’s will be able to throw 11.16 m in 41 weeks.
(iii) a = 40; d = 12; n = 15
Sn = [2a + (n-1) d] ½

Sn = [2(40) + (15-1) (12)]

= [80 + 168]
½
= [248] =1860 throws
37. (i) Let D be (a,b), then
Mid point of AC = Midpoint of BD
½
, = ,
4+a=7 3+ b = 8
a=3 b=5
Central midfielder is at (3,5) ½

8
(ii) GH = (−3 − 3) + (5 − 1) = √36 + 16 = √52 = 2√13 ½
½
GK = (0 + 3) + (3 − 5) = √9 + 4 = √13 ½
HK = (3 − 0) + (1 − 3) = √9 + 4 = √13 ½
GK +HK = GH ⇒G,H & K lie on a same straight line
[or]
CJ = (0 − 5) + (1 + 3) = √25 + 16 = √41 ½
½
CI = (0 + 4) + (1 − 6) = √16 + 25 = √41
Full-back J(5,-3) and centre-back I(-4,6) are equidistant from forward C(0,1)
½
Mid-point of IJ = , = , ½
C is NOT the mid-point of IJ

(iii) A,B and E lie on the same straight line and B is equidistant from A and E
⇒ B is the mid-point of AE
½
, = (2, −3) ½
1 + 𝑎 = 4 ; a = 3. 4+b = -6; b = -10 E is (3,-10)
38. (i) tan 45° = ⇒ CB = 80m 1
½
(ii) tan 30° =
½
½
⇒ = ½

⇒ CE = 80√3
Distance the bird flew = AD = BE = CE-CB = 80√3 – 80 = 80(√3 -1) m
½
(or) ½
tan 60° =
½
⇒ √3 = ½

⇒ CG =

Distance the ball travelled after hitting the tree =FA=GB = CB -CG

GB = 80 - = 80 (1 - )m
√ √
(√ ) ½
(iii) Speed of the bird = = m/sec
(√ ) ½
= x 60 m/min = 600(√3 + 1) m/min

9
SAMPLE QUESTION PAPER
Class X Session 2023-24
MATHEMATICS STANDARD (Code No.041)
TIME: 3 hours MAX.MARKS: 80

General Instructions:
1. This Question Paper has 5 Sections A, B, C, D and E.
2. Section A has 20 MCQs carrying 1 mark each
3. Section B has 5 questions carrying 02 marks each.
4. Section C has 6 questions carrying 03 marks each.
5. Section D has 4 questions carrying 05 marks each.
6. Section E has 3 case based integrated units of assessment (04 marks each) with sub-
parts of the values of 1, 1 and 2 marks each respectively.
7. All Questions are compulsory. However, an internal choice in 2 Qs of 5 marks, 2 Qs of 3
marks and 2 Questions of 2 marks has been provided. An internal choice has been
provided in the 2marks questions of Section E
8. Draw neat figures wherever required. Take π =22/7 wherever required if not stated.
______________________________________________________________________________________________________________
SECTION A
Section A consists of 20 questions of 1 mark each.
1. If two positive integers a and b are written as a = x3y2 and b = xy3, where x, y are prime 1
numbers, then the result obtained by dividing the product of the positive integers by the
LCM (a, b) is
(a) xy (b) xy2 (c) x3y3 (d) x2y2
2. 1
The given linear polynomial y = f(x) has
(a) 2 zeros
(b) 1 zero and the zero is ‘3’
(c) 1 zero and the zero is ‘4’
(d) No zero

Page 1 of 10
3. The lines representing the given pair of linear equations are non-intersecting. Which of the 1
following statements is true?

(a) = =

(b) = ≠

(c) ≠ =

(d) ≠ ≠

4. The nature of roots of the quadratic equation 9x2 – 6x – 2 = 0 is: 1


(a) No real roots (b) 2 equal real roots
(c) 2 distinct real roots (d) More than 2 real roots

5. Two APs have the same common difference. The first term of one of these is –1 and that of 1
the other is – 8. The difference between their 4th terms is
(a) 1 (b) -7 (c) 7 (d) 9

6. What is the ratio in which the line segment joining (2,-3) and (5, 6) is divided by x-axis? 1
(a) 1:2 (b) 2:1 (c) 2:5 (d) 5:2

7. A point (x,y) is at a distance of 5 units from the origin. How many such points lie in the third 1
quadrant?
(a) 0 (b) 1 (c) 2 (d) infinitely many

8. In 𝛥 ABC, DE ‖ AB. If AB = a, DE = x, BE = b and EC = c. 1


Then x expressed in terms of a, b and c is: A

(a) (b) D

(c) (d)
B E C

9. If O is centre of a circle and Chord PQ makes an angle 50° with the tangent PR at the point of contact 1
P, then the angle subtended by the chord at the centre is P R
(a) 130° (b) 100°
(c) 50° (d) 30° Q
O

Page 2 of 10
10. A quadrilateral PQRS is drawn to circumscribe a circle. 1
If PQ = 12 cm, QR = 15 cm and RS = 14 cm, then find the length of SP is
(a) 15 cm (b) 14 cm
(b) (c) 12 cm (d) 11 cm

11. 1
Given that sin θ = , then cos θ is.


(a) (b) (c) (d)
√ √

12. (sec A + tan A) (1 – sin A) equals: 1


(a) sec A (b) sin A (c) cosec A (d) cos A

13. If a pole 6 m high casts a shadow 2 √3m long on the ground, then the Sun’s elevation is 1

(a) 60° (b) 45° (c) 30° (d) 90°

14. If the perimeter and the area of a circle are numerically equal, then the radius of the circle 1
is
(a) 2 units (b) π units (c) 4 units (d) 7 units

15. It is proposed to build a new circular park equal in area to the sum of areas of two circular
parks of diameters 16 m and 12 m in a locality. The radius of the new park is
(a) 10m (b) 15m (c) 20m (d) 24m

16. There is a square board of side ‘2a’ units circumscribing a red circle. Jayadev is asked to 1
keep a dot on the above said board. The probability that he keeps the dot on the shaded
region is.

(a) (b) (c) (d)

17. 2 cards of hearts and 4 cards of spades are missing from a pack of 52 cards. A card is drawn at 1
random from the remaining pack. What is the probability of getting a black card?

(a) (b) (c) (d)

18. The upper limit of the modal class of the given distribution is: 1
Height
Below 140 Below 145 Below 150 Below 155 Below 160 Below 165
[in cm]
Number of
4 11 29 40 46 51
girls

Page 3 of 10
(a) 165 (b) 160 (c) 155 (d) 150
19. DIRECTION: In the question number 19 and 20, a statement of assertion (A) is followed by 1
a statement of Reason (R). Choose the correct option

Statement A (Assertion): Total Surface area of the top is the sum of the
curved surface area of the hemisphere and the curved surface area of the
cone.
Statement R( Reason) : Top is obtained by joining the plane surfaces of the
hemisphere and cone together.
(a) Both assertion (A) and reason (R) are true and reason (R) is the correct explanation
of assertion (A)
(b) Both assertion (A) and reason (R) are true and reason (R) is not the correct
explanation of assertion (A)
(c) Assertion (A) is true but reason (R) is false.
(d) Assertion (A) is false but reason (R) is true.

20. 1
Statement A (Assertion): -5, , 0, , …. is in Arithmetic Progression.
Statement R (Reason) : The terms of an Arithmetic Progression cannot have both positive
and negative rational numbers.
(a) Both assertion (A) and reason (R) are true and reason (R) is the correct explanation
of assertion (A)
(b) Both assertion (A) and reason (R) are true and reason (R) is not the correct
explanation of assertion (A)
(c) Assertion (A) is true but reason (R) is false.
(d) Assertion (A) is false but reason (R) is true.

SECTION B
Section B consists of 5 questions of 2 marks each.
21. Prove that √2 is an irrational number. 2

Page 4 of 10
22. ABCD is a parallelogram. Point P divides AB in the 2
D Q C
ratio 2:3 and point Q divides DC in the ratio 4:1.
Prove that OC is half of OA.
O

A B
P
23. From an external point P, two tangents, PA 2
and PB are drawn to a circle with centre O.
At a point E on the circle, a tangent is drawn
to intersect PA and PB at C and D,
respectively. If PA = 10 cm, find the
perimeter of ∆PCD.

24. If tan (A + B) = √3 and tan (A – B) = ; 0° < A + B < 90°; A > B, find A and B. 2

[or]
Find the value of x if

2 cosec230 + x sin260 – tan230 = 10

25. With vertices A, B and C of ΔABC as centres, arcs are drawn with radii 14 cm and the three 2
portions of the triangle so obtained are removed. Find the total area removed from the
triangle.
[or]

Find the area of the unshaded region shown in the


given figure.

SECTION C

Section C consists of 6 questions of 3 marks each

26. National Art convention got registrations from students from all parts of the country, of 3
which 60 are interested in music, 84 are interested in dance and 108 students are interested
Page 5 of 10
in handicrafts. For optimum cultural exchange, organisers wish to keep them in minimum
number of groups such that each group consists of students interested in the same artform
and the number of students in each group is the same. Find the number of students in each
group. Find the number of groups in each art form. How many rooms are required if each
group will be allotted a room?

27. If 𝛼, β are zeroes of quadratic polynomial 5x2 + 5x + 1, find the value of 3


1. 𝛼 + 𝛽
2. 𝛼 +𝛽
28. The sum of a two digit number and the number obtained by reversing the digits is 66. If the 3
digits of the number differ by 2, find the number. How many such numbers are there?
[or]

Solve : - + =2; - = -1, x, y>o


√ √ √ √
29. PA and PB are tangents drawn to a circle of centre O from an external point P. Chord AB 3
makes an angle of 30° with the radius at the point of contact.
If length of the chord is 6 cm, find the length of the tangent PA and the length of the radius
OA.

[or]

Two tangents TP and TQ are drawn to a circle with centre O from an external point T. Prove
that ∠ PTQ = 2 ∠ OPQ.
30. 3
If 1 + sin2θ = 3sinθ cosθ , then prove that tanθ = 1 or

31. The length of 40 leaves of a plant are measured correct to nearest millimetre, and the data 3
obtained is represented in the following table.
Length [in mm] Number of leaves

118 – 126 3
127 – 135 5
136 – 144 9

Page 6 of 10
145 – 153 12

154 – 162 5

163 – 171 4
172 – 180 2
Find the mean length of the leaves.
SECTION D

Section D consists of 4 questions of 5 marks each


32. A motor boat whose speed is 18 km/h in still water takes 1 hour more to go 24 km upstream 5
than to return downstream to the same spot. Find the speed of stream.
[or]

Two water taps together can fill a tank in 9 hours. The tap of larger diameter takes 10

hours less than the smaller one to fill the tank separately. Find the time in which each tap
can separately fill the tank.
33. (a) State and prove Basic Proportionality theorem. 5

(b) In the given figure ∠CEF = ∠CFE. F is the midpoint of DC.

Prove that =

34. Water is flowing at the rate of 15 km/h through a pipe of diameter 14 cm into a cuboidal 5
pond which is 50 m long and 44 m wide. In what time will the level of water in pond rise by
21 cm?
What should be the speed of water if the rise in water level is to be attained in 1 hour?
[or]
A tent is in the shape of a cylinder surmounted by a conical top. If the height and radius of
the cylindrical part are 3 m and 14 m respectively, and the total height of the tent is 13.5 m,
find the area of the canvas required for making the tent, keeping a provision of 26 m2 of
canvas for stitching and wastage. Also, find the cost of the canvas to be purchased at the rate
of ₹ 500 per m2.

Page 7 of 10
35. The median of the following data is 50. Find the values of ‘p’ and ‘q’, if the sum of all frequencies is 5
90. Also find the mode of the data.
Marks obtained Number of students
20 – 30 p
30 – 40 15
40 – 50 25
50 – 60 20
60 – 70 q
70 – 80 8
80 - 90 10

SECTION E

36. Manpreet Kaur is the national record holder for women in the shot-put discipline. Her throw of
18.86m at the Asian Grand Prix in 2017 is the
maximum distance for an Indian female athlete.
Keeping her as a role model, Sanjitha is determined
to earn gold in Olympics one day.
Initially her throw reached 7.56m only. Being an
athlete in school, she regularly practiced both in the
mornings and in the evenings and was able to
improve the distance by 9cm every week.
During the special camp for 15 days, she started with
40 throws and every day kept increasing the number
of throws by 12 to achieve this remarkable progress.

(i) How many throws Sanjitha practiced on 11th day of the camp? 1

(ii) What would be Sanjitha’s throw distance at the end of 6 weeks? 2


(or)
When will she be able to achieve a throw of 11.16 m?

(iii) How many throws did she do during the entire camp of 15 days ? 1

37. Tharunya was thrilled to know that the football tournament is fixed with a monthly timeframe from
20th July to 20th August 2023 and for the first time in the FIFA Women’s World Cup’s history, two
nations host in 10 venues. Her father felt that the game can be better understood if the position of
players is represented as points on a coordinate plane.

Page 8 of 10
(i) At an instance, the midfielders and forward formed a parallelogram. Find the 1
position of the central midfielder (D) if the position of other players who formed
the parallelogram are :- A(1,2), B(4,3) and C(6,6)

(ii) Check if the Goal keeper G(-3,5), Sweeper H(3,1) and Wing-back K(0,3) fall on a 2
same straight line.
[or]
Check if the Full-back J(5,-3) and centre-back I(-4,6) are equidistant from
forward C(0,1) and if C is the mid-point of IJ.

(iii) If Defensive midfielder A(1,4), Attacking midfielder B(2,-3) and Striker E(a,b) lie on 1
the same straight line and B is equidistant from A and E, find the position of E.
38.
One evening, Kaushik was in a park. Children were playing cricket. Birds were singing on a
nearby tree of height 80m. He observed a bird on the tree at an angle of elevation of 45°.

When a sixer was hit, a ball flew through the tree frightening the bird to fly away. In 2
seconds, he observed the bird flying at the same height at an angle of elevation of 30° and
the ball flying towards him at the same height at an angle of elevation of 60°.

Page 9 of 10
(i) At what distance from the foot of the tree was he observing the bird sitting on the 1
tree?

(ii) How far did the bird fly in the mentioned time? 2
(or)
After hitting the tree, how far did the ball travel in the sky when Kaushik saw the
ball?
(iii) What is the speed of the bird in m/min if it had flown 20(√3 + 1) m? 1

Page 10 of 10

You might also like